You are on page 1of 178

Review of

OPHTHALMOLOGY
Review of
OPHTHALMOLOGY
Quick Text Review & MCQs

Sixth Edition

AK Khurana
MS FAICO CTO (London)
Senior Professor and Head
Regional Institute of Ophthalmology
Postgraduate Institute of Medical Sciences
Rohtak, Haryana, India
Aruj K Khurana
DNB (Sankara Nethralaya, Chennai) FICO
Department of Ophthalmology
Himalayan Institute of Medical Sciences
Jolly Grant, Dehradun (UK), India
Bhawna Khurana
MS DNB FICO
Department of Ophthalmology
All India Institute of Medical Sciences (AIIMS)
Rishikesh (UK), India

The Health Sciences Publisher


New Delhi | London | Philadelphia | Panama
Jaypee Brothers Medical Publishers (P) Ltd

Headquarters
Jaypee Brothers Medical Publishers (P) Ltd
4838/24, Ansari Road, Daryaganj
New Delhi 110 002, India
Phone: +91-11-43574357
Fax: +91-11-43574314
Email: jaypee@jaypeebrothers.com

Overseas Offices
J.P. Medical Ltd Jaypee-Highlights Medical Publishers Inc Jaypee Medical Inc
83 Victoria Street, London City of Knowledge, Bld. 237, Clayton The Bourse
SW1H 0HW (UK) Panama City, Panama 111 South Independence Mall East
Phone: +44 20 3170 8910 Phone: +1 507-301-0496 Suite 835, Philadelphia, PA 19106, USA
Fax: +44 (0)20 3008 6180 Fax: +1 507-301-0499 Phone: +1 267-519-9789
Email: info@jpmedpub.com Email: cservice@jphmedical.com Email: jpmed.us@gmail.com

Jaypee Brothers Medical Publishers (P) Ltd Jaypee Brothers Medical Publishers (P) Ltd
17/1-B Babar Road, Block-B, Shaymali Bhotahity, Kathmandu, Nepal
Mohammadpur, Dhaka-1207 Phone +977-9741283608
Bangladesh Email: kathmandu@jaypeebrothers.com
Mobile: +08801912003485
Email: jaypeedhaka@gmail.com

Website: www.jaypeebrothers.com
Website: www.jaypeedigital.com

© 2015, AK Khurana
The views and opinions expressed in this book are solely those of the original contributor(s)/author(s) and do not necessarily
represent those of editor(s) of the book.

All rights reserved. No part of this publication may be reproduced, stored or transmitted in any form or by any means, electronic,
mechanical, photocopying, recording or otherwise, without the prior permission in writing of the publishers.

All brand names and product names used in this book are trade names, service marks, trademarks or registered trademarks of
their respective owners. The publisher is not associated with any product or vendor mentioned in this book.

Medical knowledge and practice change constantly. This book is designed to provide accurate, authoritative information about
the subject matter in question. However, readers are advised to check the most current information available on procedures
included and check information from the manufacturer of each product to be administered, to verify the recommended
dose, formula, method and duration of administration, adverse effects and contraindications. It is the responsibility of the
practitioner to take all appropriate safety precautions. Neither the publisher nor the author(s)/editor(s) assume any liability
for any injury and/or damage to persons or property arising from or related to use of material in this book.

This book is sold on the understanding that the publisher is not engaged in providing professional medical services. If such
advice or services are required, the services of a competent medical professional should be sought.

Every effort has been made where necessary to contact holders of copyright to obtain permission to reproduce copyright
material. If any have been inadvertently overlooked, the publisher will be pleased to make the necessary arrangements at the
first opportunity.

Inquiries for bulk sales may be solicited at: jaypee@jaypeebrothers.com


Review of Ophthalmology
First Edition: 1996
Sixth Edition: 2015
ISBN 978-93-5152-948-4

Printed at
Dedicated to
My parents and teachers for their blessings
My students for their encouragement
My children for their patience
&
My wife for her understanding
PREFACE

Sixth edition of Review of Ophthalmology has been thoroughly revised and updated with recent advances
in each aspect to keep abreast with newer concepts and principles of investigative modalities, treatment
modalities and surgical procedures evolved over the period. However, the main layout and organisation of
the book has not been changed. Twenty chapters of the book have been arranged.
Quick Text Review provides a quick reference to the main aspects of Ophthalmology in an orderly and
easily reproducible manner. Though a quick review, the subject in this heading has been covered in depth
and extensively including the recent advances. The important text on which MCQs are based has been
highlighted with under rule.
Multiple Choice Questions (MCQs) arranged in twenty chapters matching the text review. Most of the
MCQs are single best response type, baring a few extra edge questions. While framing the thought-provoking
MCQs, Particular care has been taken to include all those important MCQs which have repeatedly appeared
in various postgraduate medical entrance tests held in the last ten years up to 2014. Answers to the MCQs
have been given at the bottom of each page to facilitate easy reading.
Key features of sixth edition
■ Main feature of this book is that it is based on the Khurana’s Comprehensive Ophthalmology, a textbook,
which is used by most of the students during their graduation course.
■ Provides a means for quick text revision and self-assessment to the medical students preparing for
competitive postgraduate entrance examinations.
■ Quick review of the text given in Section I provides an opportunity for preparing for the viva questions
commonly asked in clinical/practical examinations and various interviews.
■ Includes description of femtosecond laser and its role in cataract and corneal surgery.
■ Community ophthalmology chapter has been updated in view of the objectives under Vision 2020, National
Program for Control of Blindness in India, during 12th five year plan (2012-2017).
■ Chapter on Clinical Methods in Ophthalmology includes uses of recently introduced sophisticated
equipment
■ Important feature of the sixth edition of the book is the active role of two young ophthalmologists, Aruj
K Khurana, trained at Sankara Nethralaya, Chennai and Bhawna Khurana, trained at Guru Nanak Eye
Centre, Maulana Azad Medical College, New Delhi, who have virtually revamped the text.
It is my great pleasure to convey my gratitude to all those, whose blessings and contribution has made
this venture possible. I shall remain ever indebted to my parents and teachers for their unending blessings.
I wish to express my deep sense of gratitude to Sr Prof CS Dhull, Director, PGIMS and Sr Prof SS Sangwan,
Vice-Chancellor, University of Health Sciences, Rohtak, Haryana, for providing a working environment. I
owe a lot of my students who have been a constant source of inspiration and encouragement. The sincere
help rendered by Dr Shweta Goel needs to be acknowledged.
My wife Dr Indu Khurana and my daughter Arushi Khurana, the main forces behind me, need special
mention for their endless love, patience and sacrifices.
I thank Shri Jitendar P Vij (Group Chairman), Mr Ankit Vij (Group President) and Mr Tarun Duneja
(Director–Publishing) of M/s Jaypee Brothers Medical Publishers (P) Ltd, New Delhi, India, for their
enthusiastic cooperation and accomplishing the task in a splendid manner.
Sincere efforts have been made to verify the correctness of text and the answers to the MCQs. However, in
spite of the best efforts, ventures of this kind are not likely to be free from human errors, some inaccuracies,
ambiguities and typographic mistakes. Therefore, a feedback from the users will be of utmost help in
improving future editions of the book. Endeavour of this kind shall be highly appreciated and duly
acknowledged.
AK Khurana
CONTENTS

Preface vii

1. Anatomy, Development and Physiology of Eye 1–11


2. Clinical Methods in Ophthalmology 12–22
3. Optics and Refraction 23–32
4. Diseases of Conjunctiva 33–41
5. Diseases of Cornea 42–52
6. Diseases of Sclera 53–55
7. Diseases of Uveal Tract 56–67
8. Diseases of Lens 68–76
9. Glaucoma 77–86
10. Diseases of Vitreous 87–90
11. Diseases of Retina 91-107
12. Neuro-ophthalmology 108–115
13. Disorders of Ocular Motility 116–124
14. Diseases of Eyelids 125–130
15. Diseases of Lacrimal Apparatus 131–135
16. Diseases of Orbit 136–144
17. Ocular Injuries 145–150
18. Ocular Therapeutics, Lasers and Cryotherapy in Ophthalmology 151–156
19. Systemic Ophthalmology 157-164
20. Community Ophthalmology 165–168
CHAPTER

1
Anatomy, Development and Physiology of Eye

Quick Text Review


ANATOMY OF EYE Dimensions
• Anterior vertical diameter : 11 mm
EYEBALL
• Anterior horizontal diameter : 12 mm
Dimensions of an adult eyeball • Posterior diameter : 11.5 mm
• Anteroposterior diameter : 24 mm • Radius of curvature of central part
• Vertical diameter : 23 mm - Anterior : 7.8 mm
• Horizontal diameter : 23.5 mm - Posterior : 6.5 mm
• Circumference : 75 mm • Thickness
• Volume : 6.5 cc - at the centre : 0.52 mm
• Weight : 7g - at the periphery : 1 mm
Segments and chambers of the eyeball • Refractive index : 1.33
Anterior segment. It includes crystalline lens and • Refractive power : 45 D
structures anterior to it viz. iris, cornea and two Note:
aqueous humour filled spaces, the anterior and • Pachymetry 500-600 micron, always central
posterior chamber. thickness measured because centre is the thinnest
Angle of anterior chamber from anterior to part.
posterior comprises: • Specular microscopy is a detailed microscopic
• Schwalbe’s line analysis of individual cell morphology and an
• Trabecular meshwork estimate of cellular density.
• Scleral spur
Layers of cornea
• Band of ciliary body and
• Root of iris • Epithelium: Stratified squamous type. Corneal
epithelium replaces itself about once a week.
Anterior chamber. Its depth in the centre is 2.5–3 mm,
• Bowman’s membrane: Once destroyed does not
it is comparatively shallow in very young children
regenerate.
and old people. The chamber shallows by 0.01 mm
per year . It is shallower in hypermetrops and deeper • Stroma: Constitutes 90% of total thickness. Consists
in myopes. Usually males have larger anterior of collagen fibres (lamellae arranged in many
chamber dimensions than female and volume of layers).
aqueous humour in it is 0.25 ml. Posterior chamber • Pre-Descemet’s membrane or Dua’s layer (dis­
contains about 0.06 ml of aqueous humour. covered in 2013) is about 15 micrometer thick
acellular structure which is very strong and
Posterior segment. It includes structures posterior
to the lens viz. vitreous humour, retina, optic disc, impervious to air.
choroid and pars plana (part of ciliary body). • Descemet’s membrane: Once destroyed, it
regenerates. Its prominent peripheral end forms
CORNEA Schwalbe’s line.
• It is a trasparent, dehydrated and avascular struc­ • Endothelium: Cell density in a young adult is about
ture. 6,000 cells/mm2. Metabolically, it is the most active
• Forming 1/6th of the outer fibrous coat of eyeball. layer of cornea.
2 Review of OPHTHALMOLOGY

SCLERA • Anterior limiting membrane is the anterior most


Thickness of sclera condensed part of stroma. It is deficient in the area
• At posterior pole: 1 mm (thickest) of crypts. Definitive colour of the iris depends upon
• At the insertion of extraocular muscles: 0.3 mm the amount of pigment in this layer.
(thinnest) • Anterior pigmented epithelium of ciliary body is
• Equator: 0.5 mm forward continuation of the pigment epithelium
• Lamina cribrosa is the thinnest sieve like sclera of the retina.
through which pass fibres of the optic nerve. • Posterior non-pigmented epithelium of ciliary body
is forward continuation of the sensory retina.
CRYSTALLINE LENS • Ciliary processes (About 70-80 in number) are
Dimensions white, finger-like projections from the pars plicata,
Lens is a transparent, biconvex, crystalline structure part of the ciliary body.
placed between iris and vitreous in a saucer shaped
depression called patellar fossa. VITREOUS, RETINA AND VISUAL PATHWAY
• Diameter: 9-10 mm Vitreous
• Thickness varies with age from 3.5 mm (at birth) • Volume of vitreous is approximately 4 cc (about
to 5 mm (at 60 to 70 years); in an adult average is two-third of the volume of the eye).
4.25 mm • Vitreous consists of large molecules of hyaluronic
• Thickness of the lens capsule at anterior pole is acid.
14 mm. • Subhyaloid space refers to the potential space
• Weight varies with age from about 135 mg between vitreous and retina.
(0-9 year), 175 mg (20-30 years) to 255-275 mg
• Vitreous base refers to its strongest attachment to
(60 to 80 years)
the pars plana and the retina in the region of ora-
• Radius of curvature
serrata.
- Anterior : 11 mm
- Posterior : 6 mm Retina
• Refractive index : 1.39 • Dimensions of retina
• Refractive power : 15-16D
– Optic disc : 1.5 mm
Structure – Macula lutea : 5.5 mm (15° visual field)
Lens capsule is thinnest at the posterior pole. – Fovea centralis : 1.5 mm (5° visual field)
– Foveola : 0.35 mm
Lens epithelium. It is a single layer on the anterior
– Thickness of retina : 0.5 mm near optic disc,
(front) surface. There is no epithelium on posterior
0.2 mm at equator, and
surface.
0.1 mm most anteriorly
Nucleus: It is the central part of the lens containing • Optic disc, 1.5 mm in size, is responsible for blind
oldest lens fibres. spot of Mariotte
• Embryonic nucleus: Corresponding to the lens • Rods and cones are sensory end organs of the vision
at 1-3 months of gestation. It consists the oldest • Rods are absent in the foveal region. Ganglion cell
primary fibres layer is thickest in the macular region
• Fetal nucleus (3 months gestation till birth). Its • Henle’s layer refers to thickened outer plexi­form
fibres meet around the Y-shaped sutures (anterior layer in the foveal region
Y is erect and, posterior Y is inverted). • Foveola is the most sensitive part of retina. It
• Infantile nucleus: From birth to puberty contains only cones and their nuclei covered by a
• Adults nucleus: Lens fibres formed after puberty to thin internal limiting membrane
rest of life • Major retinal vessels are present in the nerve fibre
Cortex: It is the peripheral part containing the layer
youngest fibres. • Superficial capillary network of the retina is present
at the level of nerve fibre layer
IRIS AND CILIARY BODY • Nourishment of macula-lutea is entirely dependent
• Iris is thinnest at its root. upon the choroid.
Anatomy, Development and Physiology of Eye 3
Visual pathway Actions
Visual sensation neurons Muscle Primary Secondary Tertiary
• First order—bipolar cells action action action
• Second order—ganglion cells MR Adduction — —
• Third order—neurons of lateral geniculate body. LR Abduction — —
SR Elevation Intorsion Adduction
Optic nerve
IR Depression Extorsion Adduction
■■ Fibres of optic nerve once cut, do not regenerate, SO Intorsion Depression Abduction
because they are not covered by neurilemma. IO Extorsion Elevation Abduction
■■ Length of optic nerve
APPENDAGES OF THE EYE AND ORBIT
• Total length : 47-50 mm
• Intraocular part : 1 mm Conjunctiva
• Intraorbital part : 30 mm 1. Epithelium: Stratified squamous non- keratinized
• Intracanalicular part : 6-9 mm • Marginal conjunctiva - 5 layered
• Intracranial part : 10 mm • Tarsal conjunctiva - 2 layered
■■ Diameter of optic nerve • Fornix and bulbar conjunctiva - 3 layered
• Intraocular (optic disc) : 1.5 mm • Limbal conjunctiva - 5 layered
• Intraorbital part : 3-4 mm ■■ Globlet cells are seen in nasal part of conjunctiva.
• Intracranial part : 4-7 mm Maximum density of globlet cell is present infero­
■■ Intraocular part of optic nerve is closely related to nasally and in the fornices.
ophthalmic artery, which crosses obliquely over it. 2. Adenoid layer: Consists of fine connective tissue
reticulum. It is most developed in the fornices. It is
EXTRAOCULAR MUSCLES
not present since birth but develops after 3–4 months
Origin and insertion of life (so conjunctival inflammation in infants does
• Four rectus muscles (superior, inferior, medial and not produce follicles). Limbal stem cells are present
lateral) arise from the common tendinous ring in the limbal conjunctiva.
(annulus of Zinn) at the apex of the orbit, and are
3. Fibrous layer: It contains all the blood vessels and
inserted on the sclera at following distances from nerves.
the limbus:
– Medial rectus : 5.5 mm Glands of eyelids
– Inferior rectus : 6.5 mm • Meibomian glands (Tarsal glands) are modified
– Lateral rectus : 6.9 mm sebaceous glands; about 20-30 in each lid.
– Superior rectus : 7.7 mm • Glands of Zeis are modified sebaceous glands
• Superior rectus and medial rectus have an which open into follicles of eye lashes.
attachment with the dural sheath of the optic • Glands of Moll are modified sweat glands which
nerve, this accounts for the painful movements in open into hair follicles.
retrobulbar neuritis. • Accessory lacrimal glands of Wolfring are present
• Lateral rectus muscle arises by two heads. near the superior border of upper tarsus.
• Inferior oblique (shortest extraocular muscle), is Lacrimal apparatus
the only muscle arising from the floor of the orbit. • Accessory lacrimal glands of Krause are about 42
It is inserted on the sclera posterior to the equator in the upper fornix and 6–8 in the lower fornix
in the area coincidng with the macular region. • Tears are produced after one week of life
• Superior oblique (longest extraocular muscle) • Lacrimal sac when distended, is about 15 mm in
arises from the apex of the orbit, turns around the length and 5–6 mm in breadth
trochlea and is inserted in the upper and outer part • Angular vein is situated 8 mm medial to the medial
of the sclera behind the equator. canthus.
Nerve supply Nasolacrimal duct
• 3rd cranial nerve: Superior rectus, medial rectus, • Length 12–24 mm, diameter 4-5 mm
inferior rectus and inferior oblique. • Directed downward, slightly outwards and
• 4th cranial nerve: Superior oblique. backwards
• 6th cranial nerve: Lateral rectus. Palsy produces • Opens in inferior meatus
eyeball inward, i.e. esotropia or convergent squint, • Narrowest point is near the upper end
and homonymous (uncrossed) diplopia. • Valve of Hasner is present near its lower end.
4 Review of OPHTHALMOLOGY

ORBIT • Epidermis of eyelids and its derivatives viz., cilia,


• Volume—30 cc. The eyeball occupies one-fifth of tarsal glands and conjunctival glands
the volume • Epithelium lining the lacrimal apparatus.
• Thinnest wall—medial 2. Neural ectoderm
• Thickest wall—lateral
• Retina with its pigment epithelium
• Floor is commonly involved in blow-out fractures
• Epithelial layers of ciliary body
• Surgical spaces in orbit—four (subperiosteal space,
• Epithelial layers of iris
peripheral space, central space or muscle cone and
Tenon’s space). • Sphincter and dilator pupillae muscles
• Optic nerve (neuroglia and nervous elements only)
Bones, walls and fissures of orbit • Definitive or secondary vitreous
Walls of orbit are formed by following bones: • Ciliary zonules.
1. Medial wall 3. Associated paraxial mesoderm
• Frontal process of maxilla
• Blood vessels of choroid, iris, ciliary vessels, central
• Lacrimal bone
retinal artery and other vessels
• Orbital cribiform plate of ethamoid and
• Substantia propria, Descemet’s membrane and
• Body of sphenoid
2. Lateral wall endothelium of cornea
• Zygomatic bone • The sclera
• Greater wing of sphenoid • Stroma of iris
3. Superior wall (roof ) • Ciliary muscle
• Orbital Plate of frontal bone • Sheaths of optic nerve
• Lesser wing of sphenoid • Extraocular muscles
4. Floor (commonly involved in blow out fracture) • Fat, ligaments and other connective tissue
• Orbital surface of maxillary bone structures of the orbit
• Orbital surface of zygomatic bone • Upper and medial walls of the orbit
• Palatine bone • Connective tissue of the upper eyelid.
Fissures and foramen of orbit are: 4. Visceral mesoderm of maxillary process
■■ Inferior orbital fissure below the eye
• Present between floor and lateral wall • Lower and lateral walls of orbit
■■ Superior orbital fissure • Connective tissue of the lower eyelid.
• It is at orbital apex, lateral to optic foramen
■■ Optic canal/Optic foramen IMPORTANT MILESTONES IN THE
• Formed by two wings of lesser wing of sphenoid DEVELOPMENT OF THE EYE
at orbital apex. Embryonic and fetal period
Stage of growth Development
DEVELOPMENT OF THE EYE 2.6 mm (3 weeks) Optic pits appear on either
Eyeball and its related structures are derived from side of cephalic end of
the following primordia: forebrain
• Optic vesicle (Neuroectodermal structure)—an 3.5 mm (4 weeks) Primary optic vesicle
outgrowth from the prosencephalon invaginates
• Lens placode (a specialised area of surface 5.5 to 6 mm Development of embryonic
ectoderm) and surrounding surface ectoderm fissure
• Mesoderm surrounding the optic vesicle 10 mm (6 weeks) Retinal layers differentiate,
• Visceral mesoderm of maxillary process. lens vesicle formed
20 mm (9 weeks) Sclera, cornea and extra-
STRUCTURES DERIVED FROM THE EMBRYONIC LAYERS ocular muscles differentiate
25 mm (10 weeks) Lumen of optic nerve
1. Surface ectoderm
obliterated
• The crystalline lens 50 mm (3 months) Optic tracts completed, pars
• Epithelium of the cornea ciliaris retina grows forwards,
• Epithelium of the conjunctiva pars iridica retina grows
• Lacrimal gland forward and lid folds develop
Anatomy, Development and Physiology of Eye 5
60 mm (4 months) Hyaloid vessels atrophy, iris PHYSIOLOGY OF CRYSTALLINE LENS
sphincter, dilator and ciliary Lens transparency is the result of:
muscles develop • Avascularity
230-265 mm Fetal nucleus of lens is • Tightly-packed lens fibres
(8th month) complete, all layers of retina • Arrangement of lens proteins
nearly developed and macula • Semipermeable lens capsule
starts differentiation. • Active pump mechanism in lens fibres
265-300mm Except macula, retina is fully • Auto-oxidation and high concentration of reduced
(9th month) developed, infantile nucleus glutathione
of lens begins to appear, • Aquaporin-O also help in maintaining trans­
pupillary membrane and parency.
hyaloid vessels disappear.
Medulation of optic nerve Lens metabolism
reaches lamina cribrosa. • Metabolic activity of the lens is largely limited to
Eye at birth and post-natal development
epithelium and cortex, nucleus is relatively inert
• 80% glucose is metabolised anaerobically by the
• Anterior posterior diameter about 16.5 mm (70% glycolytic pathway, 15% by HMP shunt, and a small
of adult size) which is attained by 7-8 years of age. proportion via Kreb’s cycle
• Corneal diameter is about 10 mm. Adult size
• Sorbital pathway plays role in diabetic cataract
(11.7 mm) is attained by 2 years of age.
• Respiratory quotient of lens is 1
• Lens is spherical and 3.5 mm thick at birth. Lens
• Preventive antioxidants mechanism of lens are
grow through out life and become 5 mm thick at
enzymatic (glutathione, glutathione peroxide,
60 years of age.
superoxide dismutase and catalase) and non-
• Macula differentiate 4-6 months after birth.
enzymatic (vitamin C, vitamin E and carotenoids).
• Refractive status. New born in hypermetropic by
+2 to +3 D. PHYSIOLOGY OF AQUEOUS HUMOUR
• Fixation status developing at 1 month and is
completed by 6 months. (See page 44)

PHYSIOLOGY OF VISION
PHYSIOLOGY OF EYE
Initiation, processing and transmission of
PHYSIOLOGY OF TEAR FILM visual impulse
(See page 75)
Sensory nerve endings for visual sensation are rods
PHYSIOLOGY OF CORNEA and cones. Each eye contains about 120 million rods
and 6 million cones and only 1.5 million ganglion
Corneal transparency is the result of:
cells.
• Peculiar arrangement of corneal lamellae (lattice
theory) Visual pigments (in rods rhodopsin with spectrum
• Avascularity of cornea of 500 nm and in cones erythrolabe with spectrum
• Relative dehydration maintained by epithelial and 565 nm, chlorolabe with spectrum 535 nm, and
endothelial barriers and active bicarbonate pump cyanolabe with spectrum 440 nm), absorb light
of the endothelium and trigger receptor potential which unlike other
• Swelling pressure of stroma receptor systems leads to hyperpolarization of the
• Corneal crystallines (water soluble proteins of cells and not the depolarization. This phenomenon
keratocyte) of conversion of light energy into nerve impulse is
called phototransduction.
Corneal metabolism. Epithelium is metabolically
more active than endothelium: Receptor potential generated in the photo-
• Glucose and other solutes are derived from receptors is transmitted by electronic conduction
aqueous humour and perilimbal capillaries to other retinal cells up to ganglion cell. Ganglion
• Oxygen is derived from the air through tear film cells, however, transmit by action potential to the
• Respiratory quotient of cornea is 1. LGB cells.
6 Review of OPHTHALMOLOGY

Parvocelluar system of visual pathway consisting regions which are not separated by definite borders.
of P ganglion cells and other P cells transmit Contrast sensitivity is decreased in:
information about colour and fine details of vision. • Glaucoma
• Refractive errors
Magnocellular system consisting of M ganglion
• Diabetes
cells and other M cells is concerned with contrast
• Optic nerve diseases
and motion.
• Cataractous changes.
Visual perceptions Measurement of contrast sensitivity: In clinical
1. The light sense: It is the awareness of the light. The practice, the contrast sensitivity can be measured
minimum brightness required to evoke a sensation by using any of the following charts with letters or
of light is called the light minimum. The rods are stripes represented in various shades of gray:
more sensitive to low illumination than the cones. • Arden gratings
Dark adaptation is the ability of the eye to adapt • Cambridge low-contrast gratings
itself to decreasing illumination. Dark adaptation is • Pelli-Robson contrast sensitivity chart which
delayed in: consists of low contrast letters with same size
• Vitamin A deficiency • The Visitach chart, and
• Glaucoma • Functional acuity contrast test (FACT).
• Pigmentary retinal dystrophy. 4. Colour sense vision: It is the ability of the eye
2. The form sense: It is the ability to discriminate to discriminate between different colours. It is
between the shapes of the objects. Cones play major a function of the cones. There are three primary
role in this faculty, therefore form sense is most colours—red, green and blue. All other colours
acute at the fovea where cones are most densely are produced by mixture of these primary colours
packed and highly differentiated. Visual acuity (Young Helmholtz’s trichromatic theory).
recorded by Snellen’s test chart is the measure of Note: Colour vision involves opponent colour
form sense. cells and difference between rods and cones is
3. Sense of contrast: It is the ability of the eye to all (intensity, number and colour) except signal
perceive slight changes in the luminance between transduction.
Anatomy, Development and Physiology of Eye 7

Multiple Choice Questions (MCQs)

1. Anteroposterior diameter of normal adult C. Vitreous body


eyeball is: D. Anterior surface of the lens and zonules
A. 25 mm
8. Diameter of an adult crystalline lens is:
B. 24 mm
A. 5–6 mm
C. 23.5 mm
B. 7–8 mm
D. 23 mm
C. 9–10 mm
2. Smallest diameter of the eyeball is: D. 11–12 mm
A. Vertical
9. Thickness of the adult crystalline lens is about:
B. Horizontal
A. 2.5 mm
C. Anteroposterior
B. 3.5 mm
D. More than 24 mm
C. 4.25 mm
3. Circumference of an adult eyeball is: D. 5 mm
A. 80 mm
10. Radius of curvature of the anterior surface of an
B. 65 mm
adult crystalline lens with accommodation at rest
C. 75 mm
is:
D. 70 mm
A. 7 mm
4. Volume of an adult eyeball is: B. 10 mm
A. 7.5 mL C. 8 mm
B. 6.5 mL D. 9 mm
C. 5.5 mL
D. 8 mL 11. Capsule of the crystalline lens is thinnest at:
A. Anterior pole
5. Weight of an adult eyeball is: B. Posterior pole
A. 7 g C. Equator
B. 9 g D. None of the above
C. 11 g
D. 13 g 12. Infantile nucleus of the crystalline lens refers to
the nucleus developed from:
6. Anterior segment of the eyeball includes structures A. 3 months of gestation to till birth
lying in front of the: B. Birth to one year of age
A. Iris C. Birth to puberty
B. Crystalline lens D. One year of age to 3 years of age
C. Vitreous body
13. The lens fibres meet around the Y-shaped sutures
D. Cornea
in which part of nucleus of the crystalline lens:
7. Posterior segment of the eyeball includes A. Embryonic nucleus
structures present posterior to the: B. Fetal nucleus
A. Posterior surface of the lens and zonules C. Infantile nucleus
B. Iris and pupil D. All of the above

1 : B 2 : A 3 : C 4 : B 5 : A 6 : C 8 : C 9 : D 10 : B 11 : B 12 : C 13 : A
7:A
8 Review of OPHTHALMOLOGY

14. The youngest lens fibres are present in: 22. The definitive colour of the iris depends upon the:
A. Central core of the lens nucleus A. Anterior limiting layer
B. Outer layer of the nucleus B. Stroma
C. Deeper layer of the cortex C. Anterior pigmented epithelium
D. Superficial layer of the cortex D. Posterior pigmented epithelium
23. Circulus iridis major is formed by the anastomosis
15. Schwalbe’s line forming part of the angle of
of:
anterior chamber is the prominent end of:
A. Long posterior ciliary arteries with short posterior
A. Sclera ciliary arteries
B. Descemet’s membrane of cornea B. Anterior ciliary arteries with short posterior
C. Anterior limit of trabecular meshwork ciliary arteries
D. Posterior limit of trabecular meshwork C. Long posterior ciliary arteries with anterior ciliary
arteries
16. In a normal adult person the depth of anterior D. Long posterior arteries with anterior conjunctival
chamber in the centre is about: arteries
A. 2.5 mm
B. 3 mm 24. Layer of non-pigmented epithelium of the ciliary
body is the forward continuation of the:
C. 3.5 mm
A. Pigment epithelium of the retina
D. 4 mm
B. Sensory retina
17. Is a sweat gland: C. Internal limiting membrane of the retina
A. Gland of Moll D. Bruch’s membrane of the choroid
B. Gland of Zeis 25. The number of ciliary processes is about:
C. Mebomian gland A. 20–30
D. All of the above B. 50–60
C. 70–80
18. The layer of the cornea once destroyed does not D. 90–100
regenerate is:
A. Epithelium 26. All of the following are true about circulus
B. Bowman’s membrane arteriosus minor except:
A. It receives contribution from anterior ciliary
C. Descemet’s membrane
arteries and long posterior ciliary arteries
D. All of the above
B. It is an arterial and venous plexus
19. All of the following are true about corneal endo­ C. It lies near the pupillary margin
thelium except: D. It is the seat of formation of aqueous humour
A. Cell density is about 3000 cells/mm2 at birth 27. The strongest attachment of the vitreous body to
B. Corneal decompensation occurs when cell count the surrounding structures is at the level of:
is decreased by 50 percent A. Vitreous base
C. Endothelial cells contain active pump mechanism B. Optic disc
D. Endothelium is best examined by specular C. Posterior surface of the lens
microscopy D. Foveal region
20. Adult size of the cornea is attained by the age of: 28. Diameter of the optic disc is:
A. 2 years A. 1.5 mm
B. 3 years B. 2.5 mm
C. 5 years C. 3.5 mm
D. 9 years D. 5 mm
21. Sclera is weakest at the level of: 29. Diameter of the macula lutea is:
A. Macula A. 1.5 mm
B. Equator B. 3.5 mm
C. Insertion of extraocular muscles C. 4.5 mm
D. Ora serrata D. 5.5 mm

14 : D 15 : B 16 : A 17 : A 18 : B 19 : B 22 : A 23 : C 24 : B 25 : C 26 : D 27 : A
20 : A 21 : C 28 : A 29 : D
Anatomy, Development and Physiology of Eye 9
30. Diameter of fovea centralis is: 39. The posterior end of which muscle insertion lies
A. 0.5 mm near the macula ?
B. 1.0 mm A. Inferior oblique
C 1.5 mm B. Superior oblique
D. 2.5 mm C. Superior rectus
D. Inferior rectus
31. Henle’s layer refers to the thickened outer
plexiform layer in the region of: 40. The nerve which has the longest intracranial
A. Foveola course is:
B. Foveal region A. Fourth cranial nerve
C. Parafoveal region B. Third cranial nerve
D. Paramacular region C. Sixth cranial nerve
D. Fifth cranial nerve
32. Major retinal vessels are present in:
A. Between the vitreous and internal limiting 41. Glands of Zeis are:
membrane A. Modified sebaceous glands
B. The nerve fibre layer B. Modified sweat glands
C. The inner plexiform layer C. Modified lacrimal glands
D. The inner nuclear layer D. Modified meibomian glands
33. Optic nerve consists of axons of: 42. Ducts of the main lacrimal gland open in:
A. Ganglion cells A. Superior fornix
B. Bipolar cells B. Inferior fornix
C. Rods and cones C. Both of the above
D. All of the above D. None of the above
34. Optic nerve fibres once cut, do not regenerate 43. Accessory lacrimal glands of Krause are present
because they are not covered by: in the:
A. Myelin sheath A. Upper fornix
B. Neurilemma B. Lower fornix
C. Both of the above C. Both of the above
D. None of the above D. None of the above

35. Neurons of first order for visual sensations are: 44. Length of the nasolacrimal duct is about:
A. Rods and cones A. 8–12 mm
B. Bipolar cells B. 22–34 mm
C. Ganglion cells C. 12–18 mm
D. None of the above D. 18–21 mm

36. Neurones of third order for visual sensations lie 45. Nasolacrimal duct opens into:
in: A. Superior meatus
A. Layer of bipolar cells B. Middle meatus
B. Layer of ganglion cells C. Inferior meatus
C. Lateral geniculate body D. Maxillary sinus
D. Visual cortex
46. Nasolacrimal duct is directed:
37. The longest extraocular muscle is: A. Downwards, slightly outwards and backwards
A. Superior oblique B. Downwards, slightly inwards and backwards
B. Inferior oblique C. Downwards, slightly outwards and forwards
C. Superior rectus D. Downwards, slightly inwards and forwards
D. Inferior rectus
47. In the nasolacrimal duct, valve of Hasner is present
38. The shortest extraocular muscle is: at its:
A. Superior oblique A. Upper end
B. Inferior oblique B. Lower end
C. Superior rectus C. Middle
D. Inferior rectus D. None of the above

30 : C 31 : B 32 : B 33 : A 34 : B 35 : B 39 : A 40 : A 41 : A 42 : C 43 : C 44 : C
36 : C 37 : A 38 : B 45 : C 46 : A 47 : B
10 Review of OPHTHALMOLOGY

48. Thinnest wall of the orbit is: 57. ‘Safe zone’ of the eye ball is:
A. Medial wall A. At the limbus
B. Floor B. 3-4 mm behind the limbus
C. Roof C. 8-9 mm behind the limbus
D. Lateral wall D. 12 mm behind the limbus
E. 1 mm behind the limbus
49. Thickest wall of the orbit is:
A. Medial wall 58. Yoke muscle for right superior rectus is:
B. Lateral wall A. Left superior rectus
C. Roof B. Left inferior oblique
D. Floor C. Left inferior rectus
D. Left superior oblique
50. The volume of the orbit is about:
59. The short posterior ciliary arteries are about in
A. 30 cc number:
B. 40 cc A. 10
C. 50 cc B. 20
D. 60 cc C. 30
D. 40
51. All of the following ocular structures are derived
E. 45
from the surface ectoderm except:
A. Crystalline lens 60. The canal of Schlemm possesses the following
B. Substantia propria of the cornea anatomic characteristics except:
C. Conjunctival and corneal epithelium A. Contains red cells
D. Lacrimal glands B. Contains aqueous
C. Lined by endothelium
52. Crystalline lens is derived embryologically from D. Contains partitions resembling the dural venous
the: sinuses
A. Surface ectoderm
B. Neuroectoderm 61. Muscle in the lid attached to posterior tarsal
C. Surface ectoderm and mesoderm margin is:
D. Neuroectoderm and mesoderm A. Levator palpebrae superioris
B. Superior oblique
53. Definitive or secondary vitreous is embryologically C. Muller’s muscle
derived mostly from: D. Superior rectus
A. Neuroectoderm
62. Which of the following extraocular muscle has
B. Mesoderm sympathetic innervation:
C. Surface ectoderm A. Levator palpebrae superioris
D. Surface ectoderm and mesoderm B. Muller’s muscle
54. Sphincter and dilator pupillae muscles are derived C. Superior rectus
embryologically from the: D. Inferior rectus
A. Surface ectoderm 63. Most sensitive part of eye is:
B. Mesoderm A. Fovea centralis
C. Neuroectoderm B. Macula lutea
D. All of the above C. Blind spot
D. Temporal retina
55. All of the following ocular structures are derived
embryologically from the neuroectoderm except: 64. Volume of the vitreous is:
A. Epithelial layers of ciliary body and iris A. 2 mL
B. Sphincter and dilator pupillae muscles B. 3 mL
C. Optic nerve C. 4 mL
D. Optic nerve sheaths D. 7 mL
56. Normal A: V ratio of retinal blood vessels is: 65. Avascular coat in eye is:
A. 1 : 2 A. Sclera
B. 2 : 3 B. Cornea
C. 3 : 2 C. Retina
D. 3 : 4 D. Choroid

48 : A 49 : B 50 : A 51 : B 52 : A 53 : A 57 : C 58 : B 59 : B 60 : A 61 : C 62 : B
54 : C 55 : D 56 : B 63 : A 64 : C 65 : B
Anatomy, Development and Physiology of Eye 11
66. Which continues to grow in the lifetime: EXTRA EDGE QUESTIONS
A. Cornea
B. Iris 69. Corneal endothelial cell count is done by:
C. Lens A. Specular microscopy
D. Retina B. Keratometry
67. Which part of orbicularis oculi is known as Horner’s C. Gonioscopy
muscle: D. Slit lamp
A. Orbital
70. Anterior chamber depth:
B. Lacrimal
C. Temporal A. Increases with age
D. Muller’s muscle B. Is lesser in women
C. Is lesser in myopes
68. All visual reflexes are developed by: D. Has hardly any effect on anterior chamber
A. 1 year volume
B. 2 year
C. 5 year
D. 10 year

66 : C 67 : B 68 : A 69 : A 70 : B
CHAPTER

2
Clinical Methods in Ophthalmology

Quick Text Review


COMMON OCULAR SYMPTOMS AND THEIR Transient loss of vision (amaurosis fugax)
CAUSES • Carotid artery disease
• Papilloedema
Sudden painless loss of vision
• Giant cell arteritis
• Central retinal artery occlusion • Migraine
• Massive vitreous haemorrhage • Raynaud’s disease
• Retinal detachment involving macular area • Severe hypertension
• Ischaemic central retinal vein occlusion. • Prodromal symptom of CRAO.
Sudden painless defective vision
Night blindness (Nyctalopia)
• Central serous retinopathy
• Vitamin A deficiency
• Optic neuritis
• Retinitis pigmentosa and other tapetoretinal
• Methyl alcohol amblyopia
degenerations
• Non-ischaemic central retinal vein occlusion.
• Congenital stationary night blindness
Sudden painful loss of vision • Pathological myopia
• Acute congestive glaucoma • Peripheral cortical cataract
• Acute iridocyclitis • Advanced case of POAG.
• Chemical injuries to the eyeball Day blindness (Hamarlopia)
• Mechanical injuries to the eyeball.
• Central nuclear or polar cataracts
Gradual painless loss of vision • Central corneal opacity
• Progressive pterygium involving pupillary area • Central vitreous opacity
• Corneal degenerations • Congenital deficiency of cones (rarely).
• Corneal dystrophies Defective vision for near only
• Developmental cataract
• Senile cataract • Presbyopia
• Optic atrophy • Cycloplegia
• Chorioretinal degenerations • Internal or total ophthalmoplegia
• Age-related macular degeneration • Insufficiency of accommodation.
• Diabetic retinopathy Black spots in front of the eyes
• Refractive errors. • Vitreous haemorrhage
Gradual painful loss of vision • Vitreous degeneration, e.g. senile, pathological
• Chronic iridocyclitis myopia
• Corneal ulceration • Lenticular opacity
• Chronic glaucomas. • Exudates in vitreous.
Clinical Methods in Ophthalmology 13
Flashes of light in front of the eyes (Photopsia) CORNEA
• Prodromal symptom of retinal detachment Decreased corneal sensations
• Vitreous traction on retina • Herpes simplex keratitis
• Retinitis. • Neuroparalytic keratitis
Micropsia (small size of objects), macropsia (large size • Leprosy
of objects) and metamorphopsia (distorted shape of • Herpes-zoster ophthalmicus
objects). • Absolute glaucoma
• Central chorioretinitis. • Acoustic neuroma.

Coloured halos Superficial corneal vascularization

• Acute congestive glaucoma • Trachoma


• Early stages of cataract • Phlyctenular keratoconjunctivitis
• Mucopurulent conjunctivitis. • Rosacea keratitis
• Superficial corneal ulcers.
Diplopia
Deep corneal vascularization
a. Uniocular diplopia
• Interstitial keratitis
• Subluxated lens
• Deep corneal ulcers
• Double pupil
• Chemical burns
• Incipient cataract
• Sclerosing keratitis
• Keratoconus.
• After keratoplasty.
b. Binocular diplopia
Increased corneal thickness
• Paralytic squint (paralysis of third, fourth or sixth (corneal thickness is best measured by pachymeter)
cranial nerve)
• Corneal oedema.
• Myasthenia gravis
• Diabetes Abnormal corneal surface (Placido’s disc is used to de-
• Thyroid disorders tect smoothness or irregularities of corneal surface)
• Blow-out fracture of floor of the orbit • Corneal abrasion
• Anisometropic glass (e.g. uniocular aphakic glass) • Corneal ulcer
• After squint correction in the presence of abnormal • Keratoconus.
retinal correspondence (paradoxical diplopia).
ANTERIOR CHAMBER
COMMON OCULAR SIGNS AND THEIR CAUSES Shallow anterior chamber
CONJUNCTIVA • Primary angle closure glaucoma
Conjunctival follicles • Hypermetropia
• Malignant glaucoma
• Trachoma
• Postoperative shallow anterior chamber due to
• Acute follicular conjunctivitis
– Leaking wound
• Chronic follicular conjunctivitis
– Ciliochoroidal detachment
• Benign folliculosis.
• Corneal perforation
Conjunctival papillae • Intumescent (swollen cataractous) lens
• Trachoma • Iris bombe formation
• Spring catarrh • Adherent leucoma.
• Allergic conjunctivitis Deep anterior chamber
• Giant papillary conjunctivitis.
• Aphakia
Concretions • Total posterior synechiae
• Trachoma • Myopia
• Degenerative conditions • Keratoglobus
• Idiopathic. • Keratoconus
14 Review of OPHTHALMOLOGY

• Anterior dislocation of lens in the anterior chamber • Head injury (pontine haemorrhage)
• Posterior perforation of the globe • Senile rigid miotic pupil
• Buphthalmos. • During sleep
Hyphaema
• Argyll-Robertson pupil
• Poisonings
• Ocular injuries – Alcohol
• Post-operative – Barbiturates
• Herpes-zoster iritis – Organophosphorus compounds
• Gonococcal iritis – Morphine
• Intraocular tumour – Carbolic acid
• Spontaneous (from rubeosis iridis). • Hyperpyrexia.
Hypopyon Mydriasis
• Corneal ulcer • Topical sympathomimetic drugs such as adrenaline
• Iridocyclitis and phenylephrine
• Retinoblastoma (pseudohypopyon) • Topical parasympatholytic drugs such as atropine,
• Endophthalmitis homatropine, cyclopentolate, tropicamide
• Panophthalmitis. • Acute congestive glaucoma (vertically oval, large,
IRIS immobile pupil)
• Absolute glaucoma
Nodules on the iris surface
• Optic atrophy
• Granulomatous uveitis (Koeppe’s and Busacca’s • Retinal detachment
nodules) • Internal ophthalmoplegia
• Melanoma of the iris • Third nerve paralysis
• Tuberculoma • Belladona poisoning
• Gumma. • Coma
Rubeosis iridis (Neovascularization of iris) • Sympathetic stimulation
• Diabetes mellitus – Aortic aneurysm
• Central retinal vein occlusion – Cervical rib
• Chronic iridocyclitis – Irritative lesions in neck
• Sickle-cell retinopathy – Mediastinal sarcoma, lymphosarcoma,
• Retinoblastoma. Hodgkin’s disease and pulmonary carcinoma
– Emotional excitement
Iridodonesis • Severe anaemia
• Dislocation of lens • Adie’s tonic pupil is larger than its fellow.
• Aphakia Leukocoria (white reflex in pupillary area)
• Hypermature shrunken cataract
• Buphthalmos. • Congenital cataract
• Retinoblastoma
PUPIL • Persistent hyperplastic primary vitreous
• Retrolental fibroplasia
Normal pupil • Toxocara endophthalmitis
• Diameter 3 to 4 mm • Coat’s disease.
• In infancy pupil is smaller than at birth
Marcus Gunn Pupil
• Myopes have larger pupil than hypermetropes.
(In swinging flash light test, the pupil on the diseased
Miosis side dilates on transferring light to it)
• Effect of miotic drugs (Parasympathomimetic • Optic neuritis
drugs, e.g. pilocarpine) • Optic atrophy
• Effect of systemic morphine • Retinal detachment
• Iridocyclitis (narrow, irregular non-reacting pupil) • Central retinal artery occlusion
• Horner’s syndrome • Ischemic central retinal vein occlusion.
Clinical Methods in Ophthalmology 15
LENS • Eale’s disease
• Old CRVO
Subluxation of lens
• Trauma Sub-retinal neovascular membrane
• Marfan’s syndrome (superotemporal) • Wet ARMD
• Homocystinuria (inferonasal) • Presumed ocular histoplasmosis syndrome
• Weil-Marchesani syndrome. (POHS)
• Angiod streaks
RETINA • Choroidal naevus
Cherry red spot • Choroidal rupture
• Central retinal artery occlusion • High myopia
• Commotio retinae (Berlin’s oedema) • Inappropriate photocoagulation
• Taysach’s disease • Optic disc drusen.
• Niemann-Pick’s disease Bull’s eye maculopathy
• Gaucher’s disease • Chloroquine toxicity
• Sialidosis • Cone dystrophy
• Gangliosidosis • Benign concentric annular macular dystrophy
• Metachromatic leukodystrophy
• Batten’s disease
• Multiple sulphates deficiency
• Bardet-Biedl syndrome
• Rarely in krabbe’s disease
• Occasionally Leber’s Amaurosis.
Macular oedema Neovascularization of retina
• Trauma • Diabetic retinopathy
• Intraocular operations • Eale’s disease.
• Uveitis • Sickle-cell retinopathy
• Diabetic maculopathy. • Central retinal vein occlusion.
Superficial retinal haemorrhages Proliferative retinopathy
• Hypertensive retinopathy • Proliferative diabetic retinopathy
• Diabetic retinopathy • Sickle-cell retinopathy
• Central retinal vein occlusion • Eale’s disease
• Anaemic retinopathy • Ocular trauma.
• Leukaemic retinopathy
Salt and pepper appearance of fundus
• Retinopathy of AIDS.
• Prenatal rubella
Soft exudates (Cotton wool spots) on the retina • Prenatal influenza
They are due to disturbance in axoplasmic flow • Varicella
• Hypertensive retinopathy • Mumps
• Retinopathy of toxaemia of pregnancy • Congenital syphilis.
• Diabetic retinopathy Arterial pulsations at the disc
• Anaemic retinopathy
• Visible arterial pulsations are always pathological
• DLE, PAN and Scleroderma, SLE
• True pulse waves are seen in:
• Leukaemic retinopathy
– Aortic regurgitation
• Retinopathy of AIDS
– Aneurysm
• Eale’s disease
– Exophthalmic goitre
• Rarely CMV retinitis in AIDS.
• Pressure pulse is seen in:
Hard exudates on the retina – Glaucoma
They are lipid deposits in retina – Orbital tumours.
• Diabetic retinopathy Venous pulsations at the disc
• Hypertensive retinopathy
• Visible in 10-20% of normal people
• Coat’s disease
• Absent in papilledema.
• Circinate retinopathy.
16 Review of OPHTHALMOLOGY

Capillary pulsations Altitudinal hemianopia


• Are seen in aortic regurgitation as a systolic • Loss of upper or more rarely lower halves of field
reddening and diastolic paling of the disc. from pressure upon the chiasma
• Early loss in upper half of field—intra or extrasellar
VISUAL FIELDS
tumours
Enlargement of blindspot • Early loss in lower half of field—suprasellar
• Primary open-angle glaucoma. tumours.
• Papilloedema
Quadrantic hemianopia
• Medullated nerve fibres
• Drusen of the optic nerve • Homonymous upper quadrantinopia (pie in the
• Juxtapapillary choroiditis. sky)—temporal lobe lesions involving lower fibres
• Progressive myopia with a temporal crescent. of optic radiations.
• Homonymous lower quadrantinopia (pie on the
Peripheral fieled contraction floor)—anterior parietal lobe lesions involving
• Papillitis upper fibres of optic radiations.
• Papilloedema • Quadrantic hemianopia also occurs due to lesions
• Optic atrophy in the occipital cortex involving the calcarine
• Glaucoma fissure.
• Peripheral retinochoroiditis
• Retinitis pigmentosa OCULAR EXAMINATION TECHNIQUES AND
• Quinine or salicylate poisoning. DIAGNOSTIC TESTS
Tubular vision Loupe and lens examination
• Terminal stage of advanced glaucomatous field • Power of the corneal loupe is +41 DS
defect • Magnification of image with corneal loupe is 10 X.
• Advanced stage of retinitis pigmentosa.
Slit-lamp examination
Ring scotoma • Slit-lamp was invented by Gullstrand in 1911
• Glaucoma • Aqueous flare and keratic precipitates are best
• Retinitis pigmentosa. demonstrated by slit-lamp examination.
Central scotoma Testing of visual acuity
• Optic neuritis • Visual acuity in a child below 2 years of age can be
• Tobacco amblyopia tested by:
• Macular hole, cyst, degeneration. – Opto-kinetic nystagmus (OKN)
Bitemporal hemianopia – Preferential looking test
– Pattern visual evoked potential
• Central lesions of chiasma
(STYCAR: Sight Testing for Young Children and
• Pituitary tumours (common)
Retard).
• Suprasellar aneurysms
• Two distant points can be visible as separate only
• Craniopharyngioma
when they subtend an angle of one minute at the
• Glioma of third ventricle
nodal point.
• Meningiomas at tuberculum sellae.
• Each individual letter of Snellen’s test type
Homonymous hemianopia subtends an angle of 5 minutes at the nodal
• Optic tract lesions point. Whereas, each component part of the letter
• Lateral geniculate body lesions subtends an angle of 1 minute.
• Lesions involving total fibres of optic radiations
Tonometry (Intraocular pressure measurement)
• Visual cortex lesions (usually sparing of macula).
• Schiotz tonometer is an indentation tonometer
Binasal hemianopia • Concept of applanation was introduced by
• Lateral chiasmal lesions Goldmann in 1954. It is based on Imbert-Fich law
• Distension of third ventricle • Applanation tonometers are: Goldmann tonometer,
• Atheroma of posterior communicating arteries. Perkin’s tonometer and Pneumatic tonometer
Clinical Methods in Ophthalmology 17
• Applanation tonometer is more accurate than the • Magnification of the image does not depend
Schiotz tonometer because factor of scleral rigidity upon the refractive error of the observer’s eye but
is not involved in the former depends upon:
• Central corneal thickness influences applanation – Power of convex lens used
tonometry (false high reading in thick cornea – Refractive error of the observed eye and
>600 mm and false low reading in thin cornea – Position of the convex lens in relation to the
<550 mm) eye.
• Normal range of intraocular pressure is 10–21 mm Biomicroscopic examination of the fundus
Hg
• Best accuracy with Schiotz tonometer is attained Biomicroscopic examination of the fundus is
with that weight with which a scale reading of 3–4 performed with the help of any of the following:
is achieved. • –58.6 D Hruby lens
• +78 D or +90 D small diameter lens (Best method
Gonioscopy for clinical examination of macular lesions)
• Angle structures cannot be examined directly since • Posterior fundus contact lens
the light emitted from here undergoes total internal • Goldmann’s three mirror contact lens
reflection at the anterior surface of the cornea Amsler grid test
• Goldmann and Zeiss gonioscopes are indirect
Amsler grid is test for macular degeneration and
goniolenses and provide a mirror image of the
other visual problems.
opposite angle
• Koeppe goniolens provides a direct view of the Perimetry
angle. • Normal extent of field of vision for white colour
Direct ophthalmoscopy is:
– Superior : 60°
• It was invented by Babbage in 1848 and reinvented
– Nasal : 60°
and popularized by von Helmholtz in 1850
– Inferior : 70°
• It should be performed from as close to the
– Temporal : 90°
patient’s eye as possible (ideally 15.4 mm)
• Field of vision for blue and yellow colour is roughly
• Image formed is erect, virtual and 15 times
10° less and that for red and green colour is about
magnified in emmetropes (more in myopes and
20° less than that for white (smallest is with green
less in hypermetropes).
colour)
Distant direct ophthalmoscopy • Red colour perimetry is particularly useful in the
• It is performed from a distance of 20-25 cm diagnosis of bitemporal hemianopia with chiasmal
• It is useful in detecting opacities in the media of compression and of central scotoma of retrobulbar
the eye, a hole in the iris, a detached retina and a neuritis
subluxated lens • Kinetic perimetry can be performed with Lister’s
• The black shadow produced by an opacity in the perimeter, Goldmann perimeter and tangent
pupillary plane remains stationary, that in front of screen
the pupillary plane moves in the direction of the • Static perimetry is performed with adapted
movement of the eyeball and that behind it will Goldmann perimeter, Friedmann perimeter and
move in opposite direction. automated perimeter
• Campimetry (scotometry) is performed to evaluate
Indirect ophthalmoscopy
the central and paracentral area (30°) of the visual
• It was invented by Nagel in 1864 field on Bjerrum’s screen.
• In it the examining eye is made myopic by placing
a strong convex lens in front of patient’s eye Fundus fluorescein angiography
• It is performed from a distance of an arm’s length • 5 ml of 10 percent solution of sterile sodium
(60-75 cms) fluorescein dye is injected in the antecubital vein
• Image in indirect ophthalmoscopy is formed in the • In the blood fluorescein is readily bound to the
air between the convex lens and examiner’s eye albumin
and it is real, inverted and about 5 times magnified • Blue light (420-490 nm) is used for exciting the
with +13D lens fluorescein present in the blood vessels
18 Review of OPHTHALMOLOGY

• Yellow-green filter is used to receive the fluorescent • VER assesses the functional state of the visual
light (510-530 nm) for photography. system beyond the retinal ganglion cells
• Hyperfluorescence is seen in: • Flash VER is based on light perception while
– Atrophy of retinal pigment epithelium (RPE) pattern reversal VER is based on form sense and
– Detachment of RPE thus gives a rough estimate of the visual acuity
– Central serous retinopathy • Clinically VER is used to:
– Cystoid macular oedema – Assess visual acuity in infants and mentally
– Leakage of dye from neovascularization retarded individuals
– Drusens – Confirm malingering
– Papilloedema – Confirm optic nerve diseases like retrobulbar
• Hypofluorescence is seen in: neuritis.
– Retinal haemorrhages
– Choroidremia Ultrasonography
– Central retinal artery occlusion. • Ophthalmic ultrasound is based upon pulse-echo
Electroretinography (ERG) and technique empolying frequencies in the range of
electro-oculography (EOG) 10 MHz
• Normal ERG consists of: • A-Scan (Time amplitude) produces un­idimensional
– a-wave is a negative wave possibly arising from image echoes plotted as spikes. The distance
the rods and cones between the two echo spikes provides an indirect
– b-wave is a large positive wave which is generated measurement of tissue such as eyeball length,
by Muller’s cells, but represents the activity of anterior chamber depth and lens thickness
bipolar cells • B-scan (intensity modulation) produces two-
– c-wave is a positive wave representing metabolic dimensional dotted section of the eyeball
activity of pigment epithelium. • Uses of ultrasound are:
ERG: 3 waves (Pneumonic: RMP) – Biometric studies using A-scan to calculate
R: Rods and cones power of the intraocular lens to be implanted
M: Mullers cells (glial cell) – Assessment of the posterior segment in the
P: Retinal Pigment presence of opaque media
• ERG is abnormal in patients with: – Study of intraocular and intraorbital tumours
– Retinitis pigmentosa and other tapetoretinal – Ultrasonographic pachymetry (measurement
degenerations of corneal thickness).
– Central retinal artery occlusion
– Total retinal detachment Optical Coherence Tomography (OCT)
• ERG is normal in diseases involving ganglion cells • OCT gives cross-sectional images of biological
and higher visual pathways, such as optic atrophy tissues within less than 10µ axial resolution and
• EOG is based on the resting potential of the eye transverse resolution in 20µ
which exists between the cornea (+ve) and back • Specially useful in retinal disorders, where it is akin
of the eye (-ve) to in vivo histopathology of retina
• EOG is abnormal in diseases such as retinitis
• Based on interferometry and low-coherence light
pigmentosa, vitamin A deficiency, retinal
in near infrared range
detachment
• Colour coding: Red yellow colours represent areas
• EOG is more sensitive than ERG in diagnosis of
of maximal optical reflection and back scattering.
retinitis pigmentosa
• The ratio the voltage (i.e. light peak divided by dark Blue-black colours represent area of minimal
trough) is known as the Arden ratio. In practice, signals
the measurement is similar to the eye movement • Clinical applications include: Posterior segment
recordings. OCT-macular disorder, optic disc disorders
especially glaucomatous damage, anterior
Visually evoked response (VER) segment OCT-angle evaluation, pachymetry, pre
• VER refers to electroencephalography (EEG) and post LASIK information, IOL and implant
recorded at the occipital cortex imaging.
Clinical Methods in Ophthalmology 19

Multiple Choice Questions (MCQs)

1. All of the following are causes of sudden painless B. Pattern visual evoked potential
loss of vision except: C. Preferential looking behaviour
A. Central retinal artery occlusion D. Optokinetic nystagmus
B. Optic atrophy
C. Optic neuritis 7. In microcornea, diameter of cornea is less than:
D. Retinal detachment A. 9 mm
B. 10 mm
2. Sudden, transient and painless loss of vision may C. 11 mm
be complained by the patients with all of the D. 8 mm
following diseases except:
8. Corneal diameter is increased in:
A. Carotid transient ischaemic attacks
B. Papilloedema A. Megalocornea
C. Papillitis B. Keratoglobus
D. Migraine C. Keratoconus
D. All of the above
3. All of the following may be associated with night-
blindness except: 9. Diseased and devitalized epithelial cells of the
conjunctiva and cornea are stained with:
A. Pathological myopia
B. Retinitis pigmentosa A. Fluorescein dye
C. Retinitis punctata albescens B. Rose Bengal dye
D. Retinitis proliferans C. Alcian blue dye
D. All of the above
4. Flashes of light before the eyes (photopsia) is a
feature of: 10. Corneal sensations are decreased in all of the
following conditions except:
A. Impending retinal detachment
B. Vitreous traction of the retina A. Recurrent corneal erosion syndrome
C. Retinitis B. Herpetic keratitis
D. All of the above C. Neuroparalytic keratitis
D. Leprosy
5. Snellen’s test types are based on the fact that
two distant points can be visible as separate only 11. The aqueous flare is best demonstrated by:
when they subtend at the nodal point of the eye A. Biomicroscope
an angle of: B. Keratoscope
A. 1 minute C. Pentoscope
B. 3 minute D. Ophthalmoscope
C. 5 minute 12. Photostress test is positive in:
D. 2 minute A. Central serous retinopathy
6. Assessment of the visual acuity in children below B. Optic neuritis
2 years of age can be made by the following tests, C. Ethambutol toxicity
except: D. Central retinal artery occlusion
A. Landolt’s charts E. All of the above

1 : B 2 : C 3 : D 4 : D 5 : A 6:A 7 : B 8 : D 9 : B 10 : A 11 : A 12 : A
20 Review of OPHTHALMOLOGY

13. Indentation tonometer is based on the funda­ 21. The normal extent of temporal field of vision for
mental fact that a plunger with indent: white colour is about:
A. More in soft eye A. 60°
B. More in hard eye B. 80°
C. Equal in soft and hard eye C. 90°
D. None of the above D. 110°

14. All of the following conditions can be diagnosed 22. In colour perimetry the smallest field of vision is
on distant direct ophthalmoscopy except: with:
A. Opacities in the refractive media A. Green object
B. A hole in the iris B. Blue object
C. A detached retina C. Yellow object
D. A hole in the macula D. Red object
23. Campimetry (Scotometry) is performed in the
15. All of the following are characteristics of the image
visual field area of:
formed on direct ophthalmoscopy, except that it
A. 50°
is: B. 30°
A. Erect C. 60°
B. Real D. 90°
C. Fifteen times magnified in emmetropes
D. More magnified in myopes than emmetropes 24. Dark adaptation is delayed in all of the following
conditions except:
16. In indirect ophthalmoscopy the examining eye is A. Pigmentary retinal dystrophy
made: B. Vitamin A deficiency
A. Myopic C. Primary open-angle glaucoma
B. Hypermetropic D. Heredomacular degeneration
C. Emmetropic 25. In electroretinogram (ERG) the b-wave represents
D. None of the above the activity of:
A. Rods and cones
17. All of the following are characteristics of the image
B. Bipolar cells
formed on indirect ophthalmoscopy except that C. Ganglion cells
it is: D. All of the above
A. Virtual
B. Inverted 26. In diagnosis of retinitis pigmentosa ERG is:
C. Magnified A. More sensitive than the EOG
D. Formed between the convex lens and the B. Less sensitive than the EOG
observer C. Equally sensitive as EOG
D. None of the above
18. Biomicroscopic examination of the fundus is
performed with the help of: 27. Visually-evoked response (VER) is useful in the
diagnosis of all of the following except:
A. –58.6 D Hruby lens
A. Retinitis pigmentosa
B. +78 D small diameter lens
B. Retrobulbar neuritis
C. Both of the above
C. Visual potential in eyes with opaque media
D. None of the above
D. Optic atrophy
19. Small opacities in the media of the eye are best 28. For biometric studies of the eyeball best pulse
detected by: echo technique on ultrasonography is:
A. Distant direct ophthalmoscopy A. A Scan
B. Direct ophthalmoscopy B. B Scan
C. Indirect ophthalmoscopy C. C Scan
D. All of the above D. All of the above
20. Diameter of the optic disc is: 29. Kayser-Fleischer ring is pathognomonic of:
A. 1.5 mm A. Keratoconus
B. 5.5 mm B. Wilson’s disease
C. 2.5 mm C. Lowe’s syndrome
D. 3.5 mm D. All of the above

13 : A 14 : D 15 : B 16 : A 17 : A 18 : C 21 : C 22 : A 23 : B 24 : D 25 : B 26 : B
19 : A 20 : A 27 : A 28 : A 29 : B
Clinical Methods in Ophthalmology 21
30. Roth’s spots are seen in: C. Glaucoma
A. Diabetic retinopathy D. Retrolental fibroplasia
B Chorioretinitis
C. Subacute bacterial endocarditis 40. Angiography is the investigation of choice in:
D. Retinoblastoma A. Posterior vitreous detachment
B. Rhegmatogenous retinal detachment
31. Amaurotic cat’s eye reflex is seen in: C. Retinoschisis
A. Papilloedema D. Central serous retinopathy
B. Papillitis
C. Retinoblastoma 41. Periphery of retina is best visualised with:
D. Retinitis pigmentosa A. Retinoscopy
B. USG
32. Thickness of cornea is best measured by: C. Direct ophthalmoscopy
A. Keratometer D. Indirect ophthalmoscopy
B. Pachymeter
C. Optometer 42. Campimetry is used to measure:
D. Pentoscope A. Squint
B. Angle of deviation
33. Landolt’s broken ring test is used for testing: C. Pattern of retina
A. Form sense D. Field charting
B. Contrast sense
C. Central field 43. The electroretinogram may assist in the diagnosis
D. Scotopic vision of all of the following, except:
A. Bilateral disease
34. White pupillary reflex is seen in:
B. Progression of retinal disease
A. Retinoblastoma
C. Clinically unsuspected disease in familial
B. Complete retinal detachment
degenerations
C. Endophthalmitis D. Complications of glaucoma
D. All of the above E. Differentiation between peripheral and central
35. The image in indirect ophthalmoscopy is: retinal disease
A. Erect, virtual, magnified 44. Anomaloscope is used to detect:
B. Erect, real, magnified A. Squint
C. Inverted, real, magnified B. Retinopathy
D. Inverted, virtual, normal C. Congenital glaucoma
36. Examination of vitreous is best done by: D. Colour blindness
A. Direct ophthalmoscope 45. Best diagnostic test for Best disease is:
B. Indirect ophthalmoscope A. Dark adaptation
C. Slit-lamp with contact lens B. ERG
D. Oblique illumination C. EOG
D. Gonioscopy
37. In retinal artery angiogram, the dye is injected
through the: 46. Snellen’s chart is used to test:
A. Peripheral veins A. Vision
B. Aorta B. Refraction
C. Retinal artery C. Presbyopia
D. Colour blindness
D. Retinal vein
47. Retinoscopy is used for visualising the:
38. The distance used in distant direct ophthalmoscopy A. Whole retina
is: B. Only the peripheral part of the retina
A. 10 cm C. Detecting errors of refraction
B. 15 cm D. None
C. 25 cm
48. Magnification of direct ophthalmoscopy is:
D. 100 cm
A. Five
39. Amaurotic cat’s eye is not seen in: B. Ten
A. Cataract C. Fifteen
B. Cyclitic membrane D. Thirty

30 : C 31 : C 32 : B 33 : A 34 : D 35 : C 40 : D 41 : D 42 : D 43 : D 44 : D 45 : C
36 : B 37 : A 38 : C 39 : C 46 : A 47 : C 48 : C
22 Review of OPHTHALMOLOGY

49. Amsler’s grid is used in: 58. Axial resolution in optical coherence tomography
A. Detecting colour vision is about:
B. Retinal function test A. 10 µ
C. Maculopathy B. 30 µ
D. Refractive errors C. 100 µ
D. 300 µ
50. Not true about macular function test is:
A. Two-point discrimination 59. Optical coherence tomography is most useful in
B. Maddox rod disorders of:
C. Laser interferometry A. Macula
D. Retinogram B. Crystalline lens
C. Refractive errors
51. All are seen in increased intracranial tension
D. Intraocular tumours
except:
A. Macular oedema 60. Area of fundus seen with direct ophthalmo­scope
B. Papilloedema is
C. Normal vision A. 1 DD
D. Afferent pupillary defect B. 2 DD
C. 3 DD
52. Which of the following is used for treatment of
D. 4 DD
myopia:
A. Nd: YAG laser
B. Excimer laser EXTRA EDGE QUESTIONS
C. Argon laser
D. Holmium laser 61. Indocyanine green angiography (ICG Angiography)
is most useful in detecting:
53. Visual acuity test is a test of: A. Occult choroidal neovascularization (Occult
A. Light sense CNV)
B. Colour sense B. Classic choroidal neovascularization (Classic
C. Contrast sense CNV)
D. Form sense C. Angioid streaks with choroidal neo­vascularization
54. Hutchinson’s pupils: (CNV)
A. Seen in syphilis D. Polypoidal choroidal vasculopathy
B. Unilateral constricted pupil
C. Irregular pupil 62. A wave in ERG is due to activity of:
D. None of the above A. Pigmented epithelium
B. Rods and cones
55. In retinoscopy for refractive error at 1m we add C. Ganglion cell
–1D. If done at a distance of 66 cm, the addition D. Bipolar cell
factor will be:
A. –2D 63. Arden index is related to:
B. –1.5 D A. ERG (Electroretinogram)
C. –0.5 D B. EOG (Electroculogram)
D. –5 D C. VER (Visual evoked response)
D. Perimetry
56. Shallow anterior chamber is seen in all except:
64. Perimetry is a test to assess the:
A. Old age A. Visual acuity
B. Steroid induced cataract B. Intraocular pressure
C. Hypermetropia C. Visual field
D. Angle-closure glaucoma D. Depth of the anterior chamber
57. Optical coherence tomography is akin to in 65. Sudden painful loss of vision is seen in:
vivo: A. Angle closure glaucoma
A. Histopathology B. Central retinal artery occlusion
B. Ultrasonic biomicroscopy C. Acute uveitis
C. Confocal microscopy D. Endopthalmitis
D. Roentgen examination E. Retinal detachment

49 : C 50 : D 51 : D 52 : B 53 : D 54 : D 58 : A 59 : A 60. B 61 : A 62 : B 63 : B
55 : B 56 : B 57 : A 64 : C 65 : A, C and D
CHAPTER

3
Optics and Refraction

Quick Text Review


ELEMENTRY AND PHYSIOLOGICAL OPTICS Angles of the eye
1. Angle alpha. It is formed between the optic axis
LIGHT AND GEOMETRICAL OPTICS and visual axis at the nodal point.
• Light is the visible portion (400-700 nm) of the 2. Angle gamma. It is formed between the optical
electromagnetic spectrum axis and fixation axis at the centre of rotation of
• Cornea absorbs rays shorter than 295 nm. Lens the eyeball.
absorbs rays shorter than 350 nm 3. Angle kappa. It is formed between the visual axis
• Critical angle refers to the angle of incidence in the and central pupillary line. A positive angle kappa
denser medium, corresponding to which angle of results in pseudoexotropia and a negative angle
refraction in the rarer medium is 90° kappa in pseudoesotropia.
• Image formed by a prism is virtual, erect and 4. Visual angle is the angle subtended by an object at
displaced toward its apex the nodal point.
• Strum’s conoid refers to the configuration of rays Refractive power of the eye
refracted through a toric surface.
• Total : +60 D
OPTICS OF THE EYE • Cornea : +44 D
• Total dioptric power is about +60 D • Lens : +16 D
• Principal point lies 1.5 mm behind the anterior Refractive indices of the media of the eye
surface of cornea
• Cornea : 1.37
• Nodal point is situated 7.2 mm behind the anterior
• Aqueous humour : 1.33
surface of cornea
• Crystalline lens : 1.39
• Anterior focal point is 15.7 mm in front of the
• Vitreous humour : 1.33
anterior surface of cornea
• Posterior focal point (on the retina) is 24.4 mm Refractive indices of the other media
behind the anterior surface of cornea
• Air : 1
• Anterior focal length is 17.2 mm (15.7 + 1.5)
• Water : 1.33
• Posterior focal length is 22.9 mm (24.4 – 1.5).
• Tear fluid : 1.338
Axis of the eye • HEMA : 1.43
1. Optic axis. It is the line that passes through centre of • PMMA : 1.496
the cornea, centre of the lens and meets the retina • Spectacle, crown glass : 1.52
on the nasal side of the fovea. • Flint glass : 1.61
In practice it is not possible to measure optic • Diamond : 2.52
axis. Therefore, it is substituted by pupillary line,
which is perpendicular to the cornea at center of ERRORS OF REFRACTION
pupil.
2. Visual axis. It is the line joining the fixation point, EMMETROPIA
nodal point and the fovea. • An emmetropic (optically normal) eye is slightly
3. Fixation axis. It is the line joining the fixation point hypermetropic for red rays and myopic for green
and the centre of rotation. rays
24 Review of OPHTHALMOLOGY

• At birth, eye is hypermetropic by +2 to +3 D and • All disadvantages of aphakic glasses are eliminated
usually becomes emmetropic by the age of 5 to 7 by intraocular lens use
years. • Average weight of an IOL in air is 15 mg and in
aqueous humour is 5 mg
HYPERMETROPIA • Power of an IOL in air is more (+ 60D) than that in
Hypermetropia (long sightedness) is of following the aqueous humour (+20D)
types: • Equivalent power of an anterior chamber IOL is
• Axial hypermetropia due to decreased axial length about +3D less than that of the posterior chamber
of the eye ball is the most common cause. One mm IOL
decrease in axial length causes three dioptres of • Further details of IOL (See page 42).
hypermetropia.
Surgical treatment of hypermetropia:
• Curvatural hypermetropia occurs due to flattening
of the cornea. One mm increase in the radius • Hyperopic LASIK is effective in correcting
of curvature of cornea causes six dioptres of hypermetropia up to +4D
hypermetropia. • Other corneal procedures includes hyperopic
• Index hypermetropia occurs due to increase in the PRK, Holmium laser thermoplasty and conductive
refractive index of the cortex of the crystalline lens keratoplasty (CK)
as seen in early cortical cataract and in diabetics • Lens based procedures include phakic refractive
• Positional hypermetropia results due to backward lenses (PRL) or implantable collamer lenses (ICL)
displacement of the lens. for hyperopia of between >4D to 10D and refractive
lens exchange (RLE) for high hyper-metropia
Components of hypermetropia
especially in the presbyopic age.
• Latent hypermetropia. It is corrected by the ciliary
tone. It is detected when refraction is carried out MYOPIA
after abolishing the ciliary tone with atropine.
• Axial myopia. One mm increase in axial length
• Manifest hypermetropia. It consists of two
produces myopia of 3 D.
components.
• Curvatural myopia. One mm decrease in radius of
– Facultative hypermetropia. It can be overcome
by an effort of accommodation. curvature of cornea produces myopia of 6 D.
– Absolute hypermetropia. It cannot be overcome • Congenital myopia. Present since birth, usually
by an effort of accommodation. unilateral, usually the error is of about 8–10 D
• Fundus changes in hypermetropes, in hyper­ which mostly remains constant.
metropia pseudopapilledema may be seen. • Simple or developmental myopia. It is the
commonest type of myopia. It does not progress
Optical changes in aphakia after adolescence and the error usually does not
• Eye becomes highly hypermetropic exceed -6 to -8 D.
• Total power of eye is reduced from +60 D to +44 D • Pathological (degenerative) myopia. It results
• Anterior focal point becomes 23.2 mm in front of from a rapid axial growth of eyeball (posterior
the cornea to equator) which is strongly linked with
• Posterior focal point is 31 mm behind the cornea heredity since it is familial and more common in
• Loss of accommodation. certain races (Chinese, Jews, Japanese). Fundus
Disadvantages of aphakic glasses examination may reveal/patches of chorioretinal
• Produce image magnification by 25 to 30 percent degeneration, and Foster-Fuch’s spot (dark
• Pincushion distortion due to spherical aberration circular area due to intrachoroidal haemorrhage
is the most frequently noticed problem at the macula).
• Limited field of vision Complications of pathological myopia
• Roving ring scotoma (Jack in the box phenomenon)
• Complicated cataract
• Chromatic aberration.
• Choroidal haemorrhage
Intraocular lens in aphakia • Tears and haemorrhages in the retina
• Exact power is calculated by biometry using • Vitreous haemorrhage
keratometer and A-scan ultrasound • Retinal detachment
• Standard power of +20 D of posterior chamber IOL • Diseases associated with myopia are micro­
is equivalent to +10 DS spectacles phthalmos, congenital glaucoma, micro­cornea,
Optics and Refraction 25
retrolental fibroplasia, Marfan’s syndrome, • Against-the-rule astigmatism. It is just reverse
Turner’s syndrome, and Ehlers-Danlos syndrome. of with-the-rule astigmatism, i.e. here in the
Surgical treatment of myopia
horizontal meridian is more curved than the
vertical.
• Radial keratotomy. Multiple radial incisions are • Oblique astigmatism. Here in the two principal
given in the periphery of cornea (leaving central meridia are at right angle to each other but these
4 mm optical zone) in order to flatten the curvature are not the horizontal and vertical, e.g. these may
of cornea (Not done nowadays). be 45° and 135°.
• Photorefractive keratectomy (PRK). In it reshaping • Bioblique astigmatism. In this condition the two
of the cornea is done with excimer laser. (Not principal meridia are not at right angle to each
preferred presently except in patients with thin other e.g. these may be 30° and 100°.
corneas).
• Laser assisted in situ keratomileusis (LASIK) Depending upon the type of refractive error
is presently the preferred surgical technique • Simple astigmatism. Here in the rays of light are
for correcting myopia of up to -8D. In it, the focused on the retina in one meridian and either
midstromal tissue is ablated with excimer laser in front (simple myopic astigmatism) or behind
after raising a 130-160 micron thick flap of anterior (simple hypermetropic astigmatism) the retina in
corneal tissue. the other meridian.
Customized (C) LASIK based on the wavefront • Compound astigmatism. In this type light rays are
technology is useful in correcting the aberrations. focused in both the principal meridia either in
Epi (E) LASIK is done without stromal flap. front (compound myopic astigmatism) or behind
• Femto-LASIK, also known as ‘All Laser LASIK’ or (compound hypermetropic astigmatism) the
‘ No blade LASIK’, involves making of corneal flap retina.
with the help of femtosecond laser. • Mixed astigmatism. Here the light rays are focused
• Refractive Lenticule Extracion (ReLEx) is a in front of the retina in one meridian and behind
technique for myopia correction in which a the retina in the other meridian. Thus, eye is
lenticule of corneal stroma is extracted with the myopic in one meridian and hypermetropic in the
help of femtosecond laser. The technique is now other. Such patients have comparatively less visual
named SMILE (samll incision lenticule extraction) symptoms as circle of least diffusion is formed on
• Phakic intraocular lenses (PRL) also knows
the retina.
as implantable collamer lenses (ICL) is being
performed in myopia of > -8D. Irregular astigmatism
• Refractive lens exchange (RLE) i.e. removal of • It is seen in patients with irregular corneal scars
clear crystalline lens by extracapsular cataract and keratoconus. In it there are multiple meridia
extraction (preferably by phacoemulsification) which admit no geometrical analysis.
with IOL implantation of appropriate power is • It can be best treated by rigid gas permeable
being recommended for surgical treatment of contact lens which replaces the anterior surface
myopia of >12D. of cornea for refraction.
• Orthokeratology a non-surgical reversible method • Penetrating keratoplasty is indicated in extensive
of molding the cornea with overnight wear unique corneal scarring.
rigid gas permeable contact lenses, is also being
considered for correction of myopia upto -5D. It can ANISOMETROPIA
be used even in the patients below 18 year of age. In this refractive condition, degree of error is
unequal in two eyes.
ASTIGMATISM • Simple anisometropia. One eye is emmetropic and
Depending upon the axis and the angle between the the other either myopic or hypermetropic.
two principal meridia • Compound anisometropia. Both eyes are either
• With-the-rule astigmatism, wherein the vertical hypermetropic (compound hypermetropic
meridian is more curved than the horizontal anisometropia) or myopic (compound myopic
and the two principal meridia are at right angle anisometropia) but error in one eye is higher than
to each other. Thus, correction of with-the-rule the other.
astigmatism will require a concave cylinder at 180 • Mixed anisometropia. One eye is myopic and the
± 20° or a convex cylinder at 90 ± 20°. other hypermetropic.
26 Review of OPHTHALMOLOGY

• Simple astigmatic anisometropia. One eye Surgical treatment of presbyopia


emmetropic and the other has either simple The techniques still under trial are:
myopic or simple hypermetropic astigmatism. • Monovision LASIK
• Presbyopic bifocal LASIK (LASIK - PRAM)
ANISEIKONIA • Anterior ciliary sclerotomy (ACL) with tissue
In this condition the images projected on to the barrier
visual cortex from the two retinae are abnormally • Bifocal or multifocal or accommoding IOLs
unequal in size and/or shape. • Conductive keratoplasty (CK).
• Optical aniseikonia occurs in high anisometropia.
• Retinal aniseikonia may develop due to stretching DETERMINATION AND CORRECTION OF
or oedema of the retina in macular area. REFRACTIVE ERRORS
• Cortical aniseikonia refers to asymmetrical Retinoscopy (Skiascopy or shadow test)
simultaneous perception inspite of equal size of
When a plane mirror retinoscope is used at a
the images formed on the two retinae.
distance of 1 m, depending upon the movement of
the red reflex (shadow) the results are interpreted
ACCOMMODATION AND ITS ANOMALIES
as below:
• No movement : Myopia of 1 D
ACCOMMODATION • With movement : E i t h e r e m m e t ro p i a
• Near point or punctum proximum. It is the nearest or hypermetropia or
point at which small objects can be seen clearly. myopia less than 1 D
It varies with age, being about 7 cms at the age of • Against movement : Myopia more than 1 D
10 years, 25 cms at 40 years and 33 cms at 45 years.
Auto-refractometry
• Far point (punctum remotum). It is the farthest
point from where objects can be seen clearly. • Auto-refractometry is an objective method
In an emmetropic eye far point is infinity, in a of finding out the error of refraction by using
hypermetropic eye it is virtual and lies behind the computerized autorefractometers
eye, and in myopia it is real and lies in front of the eye. • These are based on the principle of indirect
• Range of accommodation. It is the distance ophthalmoscopy
between the near point and the far point of an eye. • Autorefractometer quickly gives information about
• Amplitude of accommodation. It is the difference the refractive error in terms of sphere, cylinder with
between the dioptric power needed to focus at near axis and inter-pupillary distance
point and far point. It varies with age, being about • It is a good alternative to retinoscopy for busy
14D at 10 years, 4D at 40 years and 2D at 50 years practice, mass screening and epidemiological
of age. studies
• Subjective refraction is must even after auto­
PRESBYOPIA refractometry.
It is not an error of accommodation but a condition of Tests for confirmation of subjective refraction
age related physiological insufficiency of accommo­ • Duochrome test. It is based on the principle of
dation, leading to failing vision for near (usually chromatic aberration. When red letters are more
after the age of 40 years). clear than the green it indicates that patient is
Causes of presbyopia slightly myopic.
• Decrease in the elasticity and plasticity (hardening) • Jackson’s cross-cylinder test. It is used to verify the
of lens with age. strength and axis of the cylinder prescribed. The
• Age-related decrease in the power of ciliary muscle. cross cylinder is a combination of two cylinders
of equal strength but with opposite sign, placed
Causes of premature presbyopia with their axis at right angles to each other. It is
• Hypermetropia formed by a spherical lens (e.g., +0.5 DS) with a
• Primary open-angle glaucoma cylinder of the opposite sign and double power
• General debility, causing presenile weakness of (e.g., -ID C).
ciliary muscle • Astigmatic fan test. It is used to confirm the cylind­
• Premature sclerosis of the crystalline Jens rical correction. In the presence of astigmatism,
• Excessive close work. some lines will be seen more sharply defined.
Optics and Refraction 27
• Pin-hole test. An improvement in the visual acuity Advantages are O2 permeability and their ability
while looking through a pin hole indicates that to correct astigmatism.
optical correction in the trial frame is incorrect. Disadvantages include difficult fitting technique
Spectacles
and tolerance problems in early stages.
• Crown glass of refractive index 1.5223 is most SOME SALIENT POINTS
commonly used for making spectacles.
• Resin lenses are made of ‘allyl diglycol carbonate’. • Astigmatism is the most common refractive error.
These are light weight, unbreakable and scratch • Refractive errors are the most common cause of
resistant. defective visual acuity.
• Meniscus lenses are used to make spectacles of • Commonest problem associated with aphakic
small to moderate power. Periscopic lenses have glasses is pin-cushion distortion.
a concave posterior surface of —1.2 D. Deep • Commonest type of astigmatism encountered in
meniscus type lenses have a concave posterior general population is against-the-rule astigmatism.
surface of - 6D. Spherical correction added to the • Curvatural myopia and hypermetropia occur
anterior surface of the meniscus lenses. commonly as a factor of astigmatism.
• Lenticular form lenses are used for high plus and • The retinal image of the aphakic eye is about a
high minus lenses. quarter larger than the emmetropic retinal image.
• Aspheric lenses are also used to make high power • Pathological curvatural myopia is seen typically in
(e.g. aphakic) lenses. conical cornea.
Contact lenses • Visual angle is the angle subtended by the object
at the nodal point.
• Hard contact lenses are made up of PMMA
• Basic principle of stenopaeic slit is pin-hole
(Polymethyl methacrylate) which is a light weight
phenomenon.
and non-toxic but hydrophobic material, durable,
• All accommodation is lost in aphakia.
cheap and of high optical quality. Disadvantages:
• S, G, H, and B are the easiest letters to recognize
can cause corneal hypoxia and corneal abrasions
(not used). on the Snellen chart, whereas L, T, U, V and C are
• Soft contact lenses are made up of presently HEMA the five most difficult ones.
(hydroxy ethyl methacrylate) which is hydrophilic. • The pigment epithelium on the back of the iris and
Advantages: Being soft and oxygen permeable, the retinal pigmentary epithelium at the back of
they are most comfortable and so well tolerated. the eye absorb radiation of all wavelengths.
Disadvantages: These include problems of • The eye is normally myopic for blue and green rays
proteinaceous deposits, getting cracked, limited and hypermetropic for red rays.
life, inferior optical quality, more chances of • Distant vision is often found to be surprisingly
corneal infections and can not correct astigmatism good with mixed astigmatism due to circle of least
of more than 1 D. diffusions.
• Rigid gas permeable (RGP) contact lenses are • Asthenopia is worse in lower degree of astig-
classically made up of copolymer of PMMA matism than the higher degrees of astigmatism
and silicone containing vinyl monomer and due to circle of least diffusion.
fluoropolymers (which is permeable to O 2 ). • Image jump occurs with the use of bifocal lenses.
28 Review of OPHTHALMOLOGY

Multiple Choice Questions (MCQs)

1. Crystalline lens absorbs light rays shorter than: 7. The posterior focal point of the reduced eye
A. 295 nm situated on the retina is how many millimetre
B. 350 nm behind the anterior surface of cornea:
C. 390 nm A. 22.9 mm
D. 490 nm B. 24.4 mm
2. The prism produces displacement of the objects C. 23 mm
seen through it towards the: D. 21 mm
A. Apex 8. The optical axis of the eye meets the retina at a
B. Base point which:
C. Sideways A. Coincides with the fovea centralis
D. None of the above B. Is nasal to the fovea centralis
3. The critical angle refers to the angle of incidence C. Is temporal to the fovea centralis
in the denser medium, corresponding to which D. Is nasal to optic disc
angle of refraction in the rarer medium is: 9. Visual axis of the eye meets the retina at a point
A. Less than 90° which:
B. 90° A. Coincides with the fovea centralis
C. More than 90° B. Is nasal to fovea centralis
D. Equal to angle of incidence C. Is nasal to optic disc
4. The refractive power of an emmetropic eye is D. Is temporal to fovea centralis
about: 10. At birth eye is usually:
A. +50D A. Hypermetropic
B. +55D B. Myopic
C. +60D C. Emmetropic
D. +65D D. Aniseikonic
5. In the media of the eye, highest refractive index 11. The most common type of refractive error is:
is of the: A. Hypermetropia
A. Cornea B. Myopia
B. Aqueous humour C. Astigmatism
C. Lens D. None of the above
D. Vitreous humour
12. One millimeter decrease in axial length of the
6. The anterior focal length of the schematic eye is: eyeball leads to hypermetropia of:
A. 15.7 mm A. 6 dioptres
B. 17.2 mm B. 2 dioptres
C. 13 mm C. 3 dioptres
D. None of the above D. 4 dioptres

1 : B 2 : A 3 : B 4 : C 5 : C 6:B 7 : B 8 : B 9 : A 10 : A 11 : C 12 : C
Optics and Refraction 29
13. One millimeter increase in the radius of curvature 21. In Against-the-rule astigmatism:
of cornea leads to hypermetropia of: A. Vertical meridian is more curved than the
A. 3 dioptres horizontal
B. 4 dioptres B. Horizontal meridian is more curved than the
C. 5 dioptres vertical
D. 6 dioptres C. Both meridia are equally curved
14. In an aphakic eye the anterior focal point from the D. None of the above
anterior surface of cornea is about: 22. Radial keratotomy corrects myopia by causing:
A. 15 mm A. Steepening of cornea
B. 21 mm B. Flattening of cornea
C. 23 mm
C. Shortening of cornea
D. 31 mm
D. Pin-hole effect
15. In an aphakic eye posterior focal point from the
23. Range of accommodation is the distance between:
back of cornea is about:
A. Far point and near point of the eye
A. 23 mm
B. 25 mm B. The eyes and the near point
C. 31 mm C. The eyes and the far point
D. 21 mm D. The retina and the near point

16. The standard power of an intraocular lens 24. Causes of premature presbyopia include all of the
implanted in the posterior chamber is: following except:
A. 18 dioptres A. General debility
B. 20 dioptres B. Primary open-angle glaucoma
C. 23 dioptres C. Uncorrected myopia
D. 25 dioptres D. Premature sclerosis of the lens
17. An aphake wearing aphakic glasses will most 25. During accommodation, there occurs decrease in
commonly notice: the radius of curvature of the:
A. Pincushion distortion A. Anterior surface of the lens
B. Spherical aberration B. Posterior surface of the lens
C. Barrel distortion C. Both surfaces of the lens
D. Chromatic aberration D. None of the above
18. Sturm’s conoid refers to configuration of the rays 26. Near point of the eye varies with:
refracted through: A. The age of the patient
A. Concave spherical surface B. The static refraction of the eye
B. Convex spherical surface C. Both of the above
C. Toric surface D. None of the above
D. Irregular surface
27. Far point of the eye varies with:
19. Because of circle of least diffusion, the distant
A. The age of the patient
vision is comparatively good in:
B. The static refraction of the eye
A. Simple myopic astigmatism
C. Both of the above
B. Compound myopic astigmatism
D. None of the above
C. Mixed astigmatism
D. Compound hypermetropic astigmatism 28. Bilateral paralysis of accommodation can occur in
20. A difference in the size of two retinal images which patients with:
can be well tolerated is: A. Diabetes
A. 2 percent B. Syphilis
B. 3 percent C. Diphtheria
C. 5 percent D. All of the above
D. 10 percent E. None of the above

13 : D 14 : C 15 :C 16 : B 17 : A 18 : C 21 : B 22 : B 23 : A 24 : C 25 : C 26 : C
19 : C 20 : C 27 : B 28 : D
30 Review of OPHTHALMOLOGY

29. During retinoscopy with a plane mirror from a 37. Soft contact lenses are made of:
distance of 1 M, no movement of pupillary red A. Polymethyl methacrylate
reflex with the movement of the mirror indicates: B. Hydroxymethyl methacrylate
A. Emmetropia C. Glass
B. Myopia of 1 D D. Silicone
C. Hypermetropia of less than 1 D E. All of the above
D. All of the above
38. Consistency of the gas permeable contact lenses
30. The end point of streak retinoscopy is: is:
A. Neurtalization of the red reflex A. Hard
B. Streak disappears and the pupil appears B. Soft
completely light or dark C. Semisoft
C. Just reversal of the red reflex D. None of the above
D. All of the above
39. Contact lens is best used in:
31. The fastest acting cycloplegic drug is: A. High myopia
A. Atropine B. Irregular astigmatism
B. Tropicamide C. Aphakia
C. Cyclopentolate D. Regular astigmatism
D. Homatropine
40. Excessive accommodation causes:
32. All of the following are true about auto­
A. Hypermetropia
refractometry except:
A. Quick procedure B. Myopia
B. Gives information about spherical and cylindrical C. Pseudomyopia
error D. Pseudohypermetropia
C. Measures interpupillary distance 41. Haemorrhage at macular spot in high myopia is
D. Subjective verification of refraction is not called:
required A. Lacquer’s lines
33. While performing duochrome test, if the patient B. Foster-Fuchs fleck
reports that he sees red letters more clear than C. Dalen-Fuchs nodules
green, it indicates that he is slightly: D. Berlin’s oedema
A. Myopic
42. Over correction is preferable in:
B. Hypermetropic
A. Myopia
C. Presbyopic
B. Presbyopia
D. None of the above
C. Hypermetropia
34. While performing subjective verification of D. Aphakia
refraction, the cross cylinder is used to check:
A. Axis of the cylinder to be prescribed 43. Kappa angle is the angle between the:
B. Power of the cylinder to be prescribed A. Pupillary axis and visual axis
C. Both of the above B. Visual axis and optical axis
D. None of the above C. Centre of eyeball rotation and line of fixation
D. None of the above is correct
35. The principle of the stenopaeic slit test is based
on: 44. Alpha angle is the angle between the:
A. Astigmatic fan A. Pupillary axis and optical axis
B. The circle of least diffusion B. Visual axis and optical axis
C. Pin-hole phenomenon C. Centre of eyeball rotation and line of fixation
D. Sturm’s conoid D. None of the above is correct

36. Preferably presbyopia should be: 45. Unit of light emitted from a surface is:
A. Fully corrected A. Lambert
B. Under corrected B. Candella
C. Over corrected C. Lux
D. None of the above D. Lumen

29 : B 30 : B 31 : B 32 : D 33 : A 34 : C 37 : B 38 : A 39 : B 40 : C 41 : B 42 : C
35 : C 36 : B 43 : A 44 : A 45 : D
Optics and Refraction 31
46. Astigmatism is considered to be: 55. The power of reduced eye is:
A. Spherical abberation A. 17 D
B. Curvatural ametropia B. 45 D
C. Axial ametropia C. 59 D
D. Index ametropia D. 66 D

47. Presbyopia is due to: 56. Aniseikonia refers to:


A. Loss of elasticity of lens capsule A. Difference in the corneal diameter
B. Weakness of ciliary muscle B. Difference in the image size (retinal image)
C. Weakness of suspensory ligament C. Difference in the refractive power
D. All of the above D. Difference in image colour

48. The term anisometropia indicates: 57. A female patient wants LASIK surgery for her eye.
A. Refractive error She asks for your opinion. All the following things
B. Long vision are suitable for performing LASIK surgery except:
C. Short vision A. Myopia of 4 diopters
D. Ageing process B. Age of 15 years
C. Stable refraction for 1 year
49. Facultative hypermetropes manage to see because D. Corneal thickness of 600 microns
of:
58. Accommodation is maximum at the age of:
A. Wrinkling of eye
A. 25 years
B. Ciliary muscle contraction
B. 5 years
C. Accommodation
C. 14 years
D. Use of cycloplegics
D. 30 years
50. Constantly changing refractive error is seen in:
59. Image formed by a prism is:
A. Traumatic cataract A. Virtual, erect and displaced towards its apex
B. Diabetic cataract B. Real, erect and displaced towards its base
C. Morgagnian cataract C. Real, inverted and displaced towards its apex
D. Intumescent cataract D. Virtual, inverted and displaced towards its base
51. Maximum refractive power is due to the: 60. Presently surgical treatment of choise in a 22 year
A. Anterior surface of lens male with - 10D myopia is:
B. Posterior surface of lens A. Phakic refractive lens
C. Anterior surface of cornea B. Refractive lens exchange
D. Posterior surface of cornea C. Wavefront guided LASIK
D. LASIK with femtosecond laser
52. The most accepted method for treatment of a
myopic with refractive error of 2D is: 61. Visual axis is:
A. Spectacles A. Center of cornea to retina
B. Contact lens B. Object to fovea
C. Radial keratotomy C. Center of lens to cornea
D. Excimer laser D. None
53. Which is the most common complication of high 62. Foster’s fusch’s spots are seen in:
myopia: A. Hypermetropiea
A. Glaucoma B. Myopia
B. Cataract C. Astigmatism
C. Haemorrhage D. None
D. Retinal detachment
63. Jack in box scotoma is seen after correction of
54. Lattice degeneration is seen in: Aphakia by:
A. Myopia A. IOL
B. Hypermetropia B. Spectacles
C. Aphakia C. Contact Lens
D. Presbyopia D. None

46 : A 47 : D 48 : A 49 : C 50 : D 51 : C 55 : C 56 : B 57 : B 58 : B 59 : A 60 : A
52 : A 53 : D 54 : A 61 : B 62 : B 63 : B
32 Review of OPHTHALMOLOGY

64. Accommodation is due to: 68. On performing refraction using a plane mirror on
A. Relaxation of ciliary muscles a patient who has a refractive error of -3D sphere
B. Contraction of ciliary muscles with -2D cylinder at 90 degree from a distance of
C. Contraction of dilator pupillae 1 metre under no cycloplegia, the reflex would be
D. None seen to move:
A. With the movement in the horizontal axis and
65. Objective assessment of the refractive state of the
against the movement in the vertical axis
eye is termed:
B. With the movement in both the axes
A. Retinoscopy
C. Against the movement in both the axes
B. Gonioscopy
D. With the movement in the vertical axis and
C. Ophthalmoscopy
against the movement in horizontal axis.
D. Keratoscopy
69. A lady want LASIK surgery for her daughter. She
EXTRA EDGE QUESTIONS asks for your opinion. All the following things are
suitable for performing LASIK surgery except:
66. Which component of the eye has maximum A Myopia of 4 Diopters
refractive Index: B. Age of 15 years
A. Anterior surface of the lens C. Stable refraction for 1 year
B. Posterior surface of the lens D. Corneal thickness of 600 microns
C. Centre of the lens
D. Cornea 70. Best corrected Visual Acuity is 6/18 of a patient
with a corneal scar which improves with pin hole
67. A 55-years-old male with a limbal scar presents to to 6/9. Best explanation is:
the ophthalmology clinic with markedly defective A. Spherical aberration
vision for near and far. Clinical examination B. Myopic astigmatism
reveals a wide and deep anterior chamber, C. Irregular astigmatism
iridodonesis and a dark pupillary reflex. A vision D. Cataract
of 6/6 is achieved with correcting lens of +11D.
Which of the following is the most likely diagnosis: 71. Maximum refraction takes place between:
A. Aphakia A. Air tear film
B. Pseudophakia B. Tear film and cornea
C. Hypermetropia C. Cornea and aqueous
D. Posterior dislocation of lens D. Aqueous lens

64 : B 65 : A 66 : C 67 : A 68 : C 69 : B 70 : C 71 : A
CHAPTER

4
Diseases of Conjunctiva

Quick Text Review


CONJUNCTIVITIS BACTERIAL CONJUNCTIVITIS
Normal flora of the conjunctiva include coagulase­ • Staphylococcus aureus is the most common
negative staphylococcus and diphtheroids. cause of bacterial conjunctivitis and blepharo-
conjunctivitis.
Common causative organisms for different types • Pneumococcal conjunctivitis is usually associated
ofconjunctivitis are: with petechial subconjunctival haemorrhages.
• Purulent (hyperacute : Gonococci, Neisseria • Streptococcus pyogenes usually produces pseudo­
conjunctivitis) meningitides membranous conjunctivitis.
• Angular conjunctivitis : Moraxella-Axenfeld • Haemophilus influenzae (H. aegyptius or Koch-
(Haemophilus Weeks Bacillus) classically causes epidemics of
lacunatum) mucopurulent conjunctivitis (red-eye).
• Swimming pool : Chlamydia trachomatis • Moraxella-Axenfeld bacillus is most common cause
conjunctivitis serotypes D to K of blepharoconjunctivitis.
(inclusion • Neisseria gonorrhoeae produce acute purulent
conjunctivitis) conjunctivitis.
• Epidemic kerato- : Adenovirus type 8,19 • Corynebacterium diphtheriae causes acute
conjunctivitis membranous conjunctivitis.
• Acute haemorrhagic : Enterovirus type 70 Acute mucopurulent conjunctivitis
conjunctivitis (Apollo
conjunctivitis) • Most common type of acute bacterial conjunctivitis.
• Beal’s conjunctivitis : Adenovirus type 3 &7 • Common causative organisms: Staphylococcus
(Pharyngoconjunctival aureus (commonest), H. aegyptius (Koch-Weeks
fever) bacillus), pneumococcus and streptococcus.
• Egyptian ophthalmia : Chlamydia tracho- Acute purulent conjunctivitis
(Trachoma) matis serotype A, B, • Commonest causative organism is gonococcus,
Ba, C rarely it may be Staphylococcus aureus or pneumo­
• Acute membranous : Corynebacterium coccus.
conjunctivitis diphtheriae and • May be associated with urethritis and arthritis.
streptococcus • In gonococcal conjunctivitis, when cornea is not
haemolyticus involved, a single IM injection of ceftriaxone 1 gm
• Ophthalmia : Gonococci, Chlamydia is effective.
neonatorum trachomatis (D to K), • When cornea is involved a 5 days course of
staphylococci, herpes ceftriaxone 1-2 gm daily is needed.
simplex
• Acute follicular : Adenovirus, Chlamydia Acute membranous conjunctivitis
conjunctivitis oculogenitalis, herpes • A rare disease, typically caused by corynebacterium
simplex diphtheriae and occasionally by virulent type of
• Ophthalmia nodosa : Caterpillar hair. streptococcus haemolyticus.
34 Review of OPHTHALMOLOGY

• Corneal ulceration is frequent complication. • Incubation period varies from 5 to 21 days.


The bacteria may even invade the intact corneal • Conjunctival reaction in trachoma is both follicular
epithelium. (conjunctival follicles) and papillary (papillary
• Other Complications are: Symblepharon, trichiasis, hyperplasia).
entropion, and conjunctival xerosis. • Herbert follicles refer to typical trachoma follicles
present in the limbal area (pathognomic).
Treatment: Topical penicillin and antidiphtheric
• Pathognomic features of trachoma follicles are
serum (ADS) and I/M injections of penicillin and
presence of Leber’s cells and areas of necrosis.
ADS.
• Trachomatous pannus initially is seen in the upper
Pseudomembranous conjunctivitis part. In progressive pannus infiltration is ahead of
Acute conjunctivitis with pseudomembrane forma­ vascularization while in regressive pannus it stops
tion, does not bleed on peeling. short of limbus.
Cause: Severe Adenoviral infection, Ligneous Herbert’s pits: Cicratized follicles at the superior
conjunctivitis, Gonococcal conjunctivitis, Auto­ limbus.
immune conjunctivitis, Streptococcus haemolyticus,
Note: Arlt’s line: It is a thick band of scar tissue in the
Staphlococci, and low virulent diphtheria infection.
palpebral conjunctiva of the eye, near the lid margin
Treatment: Topical antibiotics and anti- inflamma­ seen in cicatrizing trachoma.
tory drugs. McCallan classification of trachoma
Angular conjunctivitis • Stage I (Incipient trachoma)—Hyperaemia of
Caused by Moraxella-Axenfeld (MA) diplococci, so upper palpebral conjunctiva with immature
also called as diplobacillary conjunctivitis. follicles
• Source of infection is usually nasal cavity. • Stage II (Established trachoma)—Mature follicles,
• M.A. bacillus produces proteolytic enzyme which papillae and progressive pannus
macerates epithelium of the conjunctiva, lid • Stage III (Cicatrising trachoma)—Conjunctival
margin and the skin surrounding angles of the eye. scarring, Herbert’s pit
• Stage IV—Stage of sequelae.
CHLAMYDIAL CONJUNCTIVITIS
WHO classification (FISTO)
Chlamydia lie midway between bacteria and
• Trachomatous inflammation follicular (TF): Five
viruses. Like viruses, they are obligate intracellular
or more follicles (each bigger than 0.5 mm) on the
and filterable, whereas like bacteria they contain
upper tarsal conjunctiva.
both DNA and RNA, divide by binary fission and are
• Trachomatous inflammation intense (TI):
sensitive to antibiotics.
Inflammatory thickening of the upper tarsal
Trachoma conjunctiva obscures more than half of the normal
• Also called Egyptian ophthalmia is caused by deep tarsal vessels.
Chlamydia trachomatis. Serotypes, A,B, Ba and • Trachomatous scarring (TS): Scarring on the tarsal
C are associated with hyperendemic (blinding) conjunctiva.
trachoma. • Trachomatous trichiasis (TT): When at least one
• Chlamydia trachomatis is epitheliotropic and eye lash rubs the eye.
produces intracytoplasmic inclusion bodies called • Trachomatous corneal opacity (CO): Easily visible
H.P. (Halberstaedter Prowazeke) bodies. corneal opacity present over the pupil.
• Most common mode of trachoma spread is Sequelae of trachoma
through fomites and flies. Reservoir of infection
• Lids : Trichiasis, entropion, tylosis, ptosis,
are children with active disease
madarosis, ankyloblepharon.
• Prevalence of trachoma is worldwide (affecting
• Conjunctiva: Concretions, pseudocyst, xerosis,
about 500 million people), but is highly prevalent
symblepharon.
in North-Africa, Middle-East and certain regions
• Cornea: Opacity, xerosis, total corneal pannus.
of South-East Asia.
• It is responsible for 15-20 percent of the world’s WHO criteria for trachoma elimination
blindness. Leading cause of preventable blindness • TF prevalence 5% in 1–9 years children
in the world. • TI prevalence 1% per 1000 in total population.
Diseases of Conjunctiva 35
Complication: The only complication of trachoma • Staphylococcal/streptococcal infection
is corneal ulceration leading to corneal opacity. • Chemical conjunctivitis induced by silver nitrate
or antibiotics used as prophylaxis
Treatment: Topical tetracycline or systemic
• Herpes simplex infection by type II virus is a rare
azithromycin, constitute treatment of choice.
cause.
• Other drugs: Topical erythromycin eye ointment
o r topical sulphonamides eye drops or ointment. Incubation period
Systemic tetracycline or erythromycin can also be Causative agent Incubation
given if needed. period
Adult inclusion conjunctivitis • Chemicals 4-6 hours
• Gonococcus 2-4 days
• Caused by serotypes D to K of Chlamydia
• Other bacterial 4-5 days
trachomatis
• Neonatal inclusion conjunctivitis 5-14 days
• Source of infection is urethritis in males and
• Herpes simplex 5-7 days
cervicitis in females
• Mode of infection is through contaminated fingers Drug prophylaxis
or more commonly through contaminated water of Drug prophylaxis for ophthalmia neonatorum
swimming pool (hence the name swimming pool includes use of Providine-iodine 2.5% solution,
conjunctivitis). tetracycline eye ointment, erythromycin eye oint­
ment or i.e 1 percent silver nitrate eye drops Crede’s
VIRAL CONJUNCTIVITIS method (not used now).
Acute haemorrhagic conjunctivitis
ALLERGIC CONJUNCTIVITIS
• Caused by picorna viruses (enterovirus type 70)
• Also known as ‘Apollo conjunctivitis’ or ‘epidemic Spring catarrh (Vernal keratoconjunctivitis)
haemorrhagic conjunctivitis (EHC). • A hypersensitivity reaction to exogenous allergens
• It is thought to be an atopic allergic disorder in
Epidemic keratoconjunctivitis (EKC)
many cases, in which there occurs Th2 lymphocyte
• Caused by adenovirus 8 and sometimes by 19 alteration and secondarily the IgE mediated
• Acute follicular conjunctivitis associated with mechanisms play important role
preauricular lymphadenopathy • More common in boys than girls between 4 and 20
• Superficial punctate keratitis which appears after years of age
one week of onset is a distinctive feature of EKC. • Intense itching is a characteristic symptom
Pharyngoconjunctival fever (Beat’s conjunctivitis) • Cobble stone arrangement of papillae on the upper
• Caused by adenovirus 3 and 7 tarsal conjunctiva, Homer Tranta’s spots and ropy
• Primarily affects children and appears as epidemics discharge are pathognomic.
• Acute follicular conjunctivits associated with • Palpebral form is more common than bulbar and
pharyngitis, fever and pre-auricular lympha­ mixed types.
• Vernal keratopathy occurs in following forms:
denopathy.
– Punctate epithelial keratitis involving upper
OPHTHALMIA NEONATORUM cornea
– Ulcerative vernal keratitis (shield ulcer)
• Bilateral inflammation of conjunctiva occuring
– Vernal corneal plaques
during first 30 days of life
– Subepithelial scarring
• Any discharge or watering from the eyes in the first
– Pseudogerontoxon (Cupids bow)
week of life should arouse its suspicion since tears
• Keratoconus may be associated with spring
are not formed till then.
catarrh.
Etiology Atopic keratoconjunctivitis (AKC)
• Gonococcal infection is most common cause in • It can be thought of as an adult equivalent of vernal
developing countries keratoconjunctivitis
• Neonatal inclusion conjunctivitis caused by sero­ • Often associated with atopic dermatitis
types D to K of chlamydia trachomatis is emerging • Associations may be keratoconus and atopic
as the most important cause cataract (posterior subcapsular).
36 Review of OPHTHALMOLOGY

Giant papillary conjunctivitis (GPC) scratch disease, tuberculosis, syphilis, lympho­


• A localised allergic response to a physically granuloma venereum, sarcoidosis and tularemia.
rough or deposited surface such as contact lens,
prosthesis, leftout nylon sutures DEGENERATIONS AND XEROSIS
• Papillae are 1 mm or more in diameter. DEGENERATIVE CONDITIONS
Phlyctenular keratoconjunctivitis Pterygium
• Phlycten is a pinkish white nodule (1 to 3 mm in • It is a combined elastotic degeneration with
size), surrounded by hyperaemia on the bulbar hyperplasia of the subconjunctival tissue in the
conjunctiva, usually near the limbus form of vascularised granulation tissue.
• It is believed to be a delayed hypersenstivity (type IV- • Exposure to ultraviolet rays of sunlight is implicated
cell mediated) response to endogenous microbial in the etiology.
proteins such as: tubercular, staphylococcal (most • Fully developed pterygium has three parts—head,
common), moraxella bacillus, and parasites (worm neck and body.
infestation) • Deposition of iron seen sometimes in corneal
• More common in girls than boys between 3 and 15 epithelium anterior to advancing head is called
years of age stocker’s line.
• Corneal involvement (phlyctenular keratitis) may • Recurrence after surgical removal is 30-50 percent.
be in the form of scrofulous ulcer, fascicular ulcer • Measures to reduce recurrence include: trans­
and miliary ulcer. plantation in lower fornix (McReynold’s operation)
Contact dermoconjunctivitis old technique not used now, post­operative beta
A delayed hypersensitivity (type IV) response to irradiation, postoperative use of antimitotic
prolonged contact with chemicals and drugs, such drops (mitomycin-C or thioTEPA), and mucous
as, atropine, penicillin, neomycin, soframycin, and membrane grafts. Conjunctival limbal autograft
gentamicin. (CLAU) is the best and preferred method.
• Lamellar keratectomy and lamellar keratoplasty is
MISCELLANEOUS CONJUNCTIVITIS indicated in recurrent recalcitrant cases.
Acute follicular conjunctivitis is a feature of: Concretions
• Adult inclusion conjunctivitis • These are formed due to accumulation of inspissated
• Epidemic keratoconjunctivitis (EKC) mucos and dead epithelial cell debris into the
• Pharyngoconjunctival fever (PCF) conjunctival depressions called loops of Henle.
• New-castle conjunctivitis • These are not calcareous deposits.
• Acute herpetic conjunctivitis. • Causes are trachoma, age-related degeneration
Pseudomembranous conjunctivitis is a feature of: and idiopathic.
• Severe adenoviral infection
• Ligneous conjunctivitis CONJUNCTIVAL XEROSIS
• Gonococcal conjunctivitis Parenchymatous xerosis occurs following cicatricial
• Autoimmune conjunctivis. disorganization of the conjunctiva due to local
causes, such as:
Conjunctivitis associated with skin diseases such
• Trachoma
as acne rosacea, Stevens-Johnson syndrome,
• Diphtheric membranous conjunctivitis
epidermolysis bullosa, and pemphigoid. Ligneous
• Stevens-Johnson syndrome
conjunctivitis is a cicatrizing conjunctivitis in which
• Pemphigus
membrane is cast off and recurs again and again.
• Pemphigoid
Ophthalmia nodosa is a granulomatous inflamma­ • Thermal, chemical or radiational burns of
tion of the conjunctiva in response to irritation conjunctiva
caused by the retained hair of caterpillar. • Prolonged exposure due to lagophthalmos.
Parinaud’s oculoglandidar syndrome is a group Epithelial xerosis occurs due to hypovitaminosis-A.
of conditions characterised by unilateral granulo­ It typically occurs in children and is characterised
matous conjunctivitis, pre-auricular lympha­ by varying degree of conjunctival thickening,
denopathy, and fever. Its common causes are cat wrinkling and pigmentation.
Diseases of Conjunctiva 37
CYSTS AND TUMOURS • Topical steroids are contraindicated in acute
bacterial conjunctivitis
• Commonest cysts of the conjunctiva are lymphatic • Hyperaemia is the most conspicuous clinical sign
cysts and implantation cysts of acute conjunctivitis
• Commonest congenital tumour of the conjunctiva • Unilateral chronic conjunctivitis should suggest the
is epibulbar dermoid presence of a foreign body retained in the fornix,
• In Goldenhar’s syndrome epibulbar dermoid is trichiasis or inflammation of the lacrimal sac
associated with preauricular skin tag, hemifacial • Trachoma and other conjunctival inflammation in
hypoplasia and vertebral anomalies the newborn cannot produce a follicular reaction;
• Epithelioma (epidermoid squamous cell carci­ because the adenoid layer is devoid of lymphoid
noma) of the conjunctiva usually occurs at limbus tissue until 2-3 months postnatally
• Premalignant conditions of conjunctiva are • Preauricular lymphadenopathy is a feature of
intraepithelial epithelioma (Bowen’s disease), viral and chlamydial conjunctivitis which is rarely
superficial spreading melanoma, lentigomaligna present in bacterial conjunctivitis, but seldom in
(Hutchinson’s freckle) allergic conjunctivitis
• Naevus of Ota refers to oculodermal melanosis. • Conjunctival ulceration should always suggest
either the presence of an embedded foreign body
SOME SALIENT POINTS or a tuberculous or syphilitic lesion.
• Bandage should not be applied in acute catarrhal • Epidemic keratoconjunctivitis is the only serious eye
or mucopurulent conjunctivitis disease known to be transmissible by tonometry.
38 Review of OPHTHALMOLOGY

Multiple Choice Questions (MCQs)

1. Epidemics of conjunctivitis are known to occur 7. Trachoma inclusion bodies in conjunctival smear
with: are detected by:
A. Bacterial infections A. Giemsa stain
B. Viral infections B. Iodine stain
C. None of the above C. Immunofluorescent staining
D. Both of the above D. All of the above

2. M o s t co m m o n b a c te r i a a s s o c i ate d w i t h 8. Swimming pool conjunctivitis is caused by:


conjunctivitis is: A. Chlamydia trachomatis
A. Staphylococcus aureus B. Adenovirus type 8
B. Streptococcus pneumoniae C. Picorna virus
C. Haemophilus influenzae D. Gonococcus
D. Neisseria gonorrhoea
9. Acute haemorrhagic conjunctivitis is caused by:
3. Preauricular lymph nodes may be enlarged in all
A. Entero virus
except:
B. Adeno virus
A. Bacterial conjunctivitis
C. Pseudomonas
B. Viral conjunctivitis
D. Streptococcus haemolyticus
C. Allergic conjunctivitis
D. Chlamydial conjunctivitis 10. Occur in epidemics:

4. Commonest causative organism for angular A. Entero virus conjunctivitis


conjunctivitis is: B. Staphylococcal conjunctivitis
A. Moraxella axenfield C. Adeno virus conjunctivitis
B. Klebsiella pneumoniae D. All of the above
C. Haemophilus influenzae E. None of the above
D. None of the above
11. Ophthalmia neonatorum is:
5. Form the normal bacterial flora of the A. Inflammation of the conjunctiva occurring in an
conjunctiva: infant less than 30 days old
A. Corynebacterium xerosis B. Any discharge/watering from the eye in first week
B. E. Coli of life
C. Streptococci C. Always caused by gonococci
D. All of the above D. All of the above

6. Pathognomic features of trachoma follicle are: 12. Incubation period of gonococcal ophthalmia
A. Presence of Leber’s cells neonatorum is:
B. Areas of necrosis A. 24 hours
C. Both of the above B. 5-7 days
D. None of the above C. 7-10 days
D. None of the above

1 : D 2 : A 3 : C 4 : A 5 : A 6 : C 7 : D 8 : A 9 : A 10 : D 11 : A 12 : D
Diseases of Conjunctiva 39
13. Intense itching is pathognomic feature of: 22. Follicle formation may be seen in all of the
A. Spring catarrh following except:
B. Trachoma A. Trachoma
C. Follicular conjunctivitis B. Vernal keratoconjunctivitis
D. Angular conjunctivitis C. Inclusion conjunctivitis
E. All of the above D. Epidemic keratoconjunctivitis
14. Spring catarrh may be associated with: 23. Acute haemorrhagic conjunctivitis is seen with:
A. Anterior subcapsular cataract
A. Adenovirus
B. Keratoconus
B. Staphylococcus
C. Interstitial keratitis
D. All of the above C. Pneumococcus
D. Haemophilus
15. Associations of atopic keratoconjunctivitis include
all except: 24. Pseudomembranous conjunctivitis is caused by:
A. Keratoconus A. Gonococcus
B. Atopic cataract B. Staphylococcus
C. Atopic dermatitis C. Streptococcus
D. Interstitial keratitis D. Keratoconjunctivitis sicca
16. Giant papillary conjunctivitis occurs as an allergic 25. Conjunctivitis in newborn is commonly caused by:
response to all except: A. Streptococcus
A. Contact lens B. Gonococcus
B. Intraocular lens C. Pseudomonas
C. Prosthesis
D. Chlamydia
D. Nylon sutures
26. Unilateral conjunctivitis is commonly seen in:
17. All are true about phlyctenular conjunctivitis
A. Blepharitis
except:
A. It is type-IV cell mediated hypersensitivity B. Vernal conjunctivitis
B. Allergens are endogenous as well as exogenous C. Dacryocystitis
C. Incidence is higher in girls than boys D. Trachoma
D. Nodular lesion usually occurs near the limbus
27. Ligneous conjunctivitis is caused by:
18. Ophthalmia nodosa occurs due to: A. Purulent conjunctivitis
A. Leprotic conjunctivitis B. Membranous conjunctivitis
B. Syphilitic conjunctivitis C. Angular conjunctivitis
C. Sarcoidosis conjunctivitis D. Phlyctenular conjunctivitis
D. Irritation by hair of caterpillar E. Any of the above
E. All of the above
28. Horner Tranta’s spots are seen in:
19. All are known to cause conjunctival xerosis except: A. Vernal conjunctivitis
A. Trachoma
B. Phylectenular conjunctivitis
B. Membranous conjunctivitis
C. Angular conjunctivitis
C. Angular conjunctivitis
D. Ocular pemphigoid D. Follicular conjunctivitis

20. All are known to produce parenchymatous 29. H.P. inclusion bodies in trachoma are seen to
conjunctival xerosis except: be:
A. Vitamin A deficiency A. Extracellular
B. Diphtheric membranous conjunctivitis B. Intracytoplasmic
C. Trachoma C. Intranuclear
D. Stevens-Johnson syndrome D. None
21. Goldenhar syndrome is associated with which
prominent ocular manifestation: 30. Pathognomonic feature of trachoma is:
A. Microcornea A. Bulbar papillae
B. Megalocornea B. Palpebral papillae
C. Sclerocornea C. Bulbar follicles
D. Epibulbar dermoids D. Palpebral follicles
13 : E 14 : B 15 : D 16 : B 17 : B 18 : D 22 : B 23 : A 24 : C 25 : D 26 : C 27 : B
19 : C 20 : A 21 : D 28 : A 29 : B 30 : C
40 Review of OPHTHALMOLOGY

31. “Safe strategy” has been developed for the control 40. Follicles of a diameter of 5 mm are typically seen
of: in:
A. Conjunctivitis A. Pharyngoconjunctival fever
B. Trachoma B. Trachoma
C. Refractive error C. Drug induced follicular conjunctivitis
D. Ocular trauma D. Ophthalmia neonatorum

32. Subconjunctival haemorrhage can occur in all 41. Tetracycline ointment for mass prophylaxis:
conditions except: A. 0.1%
B. 0.5%
A. Passive venous congestion
C. 1%
B. Pertusis
D. 5%
C. Trauma
D. High intraocular tension 42. Spring catarrah is:
A. Type I hypersensitivity reaction
33. Unilateral chronic conjunctivitis may be associated B. Type II
with C. Type III
A. Habit of smoking D. Type IV
B. Use of uniocular microscope
C. Foreign body retained is the fornix 43. Complication of vernal kerato conjunctivitis:
D. Unilateral aphakia A. Cataract
B. Keratoconus
34. Trachoma in a newborn cannot produce follicular C. Retinal detachment
reaction because: D. Vitreous hemorrhage
A. Antibodies are transfered from mother 44. Treatment of vernal keratoconjunctivitis includes
B. Adenoid layer is devoid of lymphoid tissue all except:
C. Immunity is not developed A. Steroids
D. Incubation period is one year B. Chromoglycate
C. Olopatadine
35. Conjunctival ulceration may suggest:
D. Antibiotics
A. Embedded foreign body
B. Tuberculosis 45. The histology of pterygium includes:
C. Syphillis A. Elastotic degeneration
D. Any of the above. B. Epithelial inclusion bodies
C. Precancerous changes
36. Commonest congenital tumour of conjunctiva is: D. Squamous metaplasia of the epithelium
A. Epibulbar dermoid
B. Benign melenoma 46. Subconjunctival cyst is seen in:
C. Papilloma A. Toxoplasmosis
D. Capillary haemangioma B. Cysticercosis
C. Leishmaniasis
37. Inclusion body conjunctivitis true is all except: D. Chaga’s disease.
A. Self limiting
B. Present only in infants EXTRA EDGE QUESTIONS
C. Occurs while passage from birth canal
D. Caused by chlamydia 47. In the grading of trachoma, trachomatous
inflammation-follicular is defined as the presence
38. Pathognomonic of trachoma is: of:
A. Bulbar papillae A. 5 or more follicles in the lower tarsal conjunctiva
B. Palphebral papillae B. 3 or more follicles in the lower tarsal conjunctiva
C. Bulbar follicles C. 5 or more follicles in the upper tarsal conjunctiva
D. Palpebral follicles D. 3 or more follicles in the upper tarsal conjunctiva
39. All are seen in stage III trachoma except: 48. Inclusion conjunctivitis is caused by:
A. Tarsal epitheliofibrosis A. Chlamydia trachomatis
B. Trachomatous pannus B. Chlamydia psittaci
C. Herbert’s pits C. Herpes
D. Disappearance of Bowman’s membrane D. Gonorrhea
31 : B 32 : D 33 : C 34 : B 35 : D 36 : A 40 : B 41 : A 42 :A 43 : B 44 : D 45 : A
37 : B 38 : C 39 : B 46 : B 47 : C 48 : A
Diseases of Conjunctiva 41
49. Features of vernal conjunctivitis are: D. Allergic conjunctivitis
A. Shield ulcer E. Molluscus contagiosum
B. Horner-Tranta’s spots
54. Pterygium all are true except:
C. Papillary hypertrophy
A. Arise from any part of conjunctiva
D. Herbert pits
B. Can cause astigmatism
E. Pannus
C. Surgery is treatment of choice
50. Features of vernal keratoconjunctivitis are: D. UV exposure is risk factor
A. Papillary hypertrophy E. Stromal defect seen
B. Shield’s ulcer F. A connective tissue degeneration
C. Herbert’s pits 55. Stocker’s line is seen in:
D. Trantra’s spot A. Pinguencula
E. Ropy discharge B. Pterygium
51. Topical sodium cromoglycate is used in treatment C. Congenital Ocular Melanosis
of: D. Conjunctival epithelial melanosis
A. Phlyctenular conjunctivitis 56. Neonatal conjunctivitis is caused by all except:
B. Vernal catarrh A. Gonococcus
C. Subconjunctival haemorrhage B. Chlamydia
D. Trachoma C. Aspergillus
52. A Malnourished child from a poor socioeconomic D. Pseudomonas
status, residing in over­crowded and dirty areas 57. A patient complains of pain in both eys with
present with a nodule around the limbus with congestion. Blurring of vision, photophobia and
hyperemia of surrounding conjunctiva in his mucopurulent discharge since one day. Many cases
left eye. He is also observed to have axillary and have been reported from the same community. The
cervical lymphadenopathy. Which of the following causative agent is probably:
is the most likely diagnosis: A. Adenovirus
A. Phlyctenular conjunctivitis B. Enterovirus 70
B. Foreign body granuloma C. Herpes simplex
C. Vernal keratoconjunctivitis D. Hemophilus
D. Episcleritis
58. All are features of Trachoma stage III, except:
53. Follicular conjunctivitis are found in all except: A. Herbert’s pits
A. Herpes simplex conjunctivitis B. Pannus
B. Drug induced C. Necrosis in scar
C. Adult inclusion conjunctivitis D. Scar on tarsal conjunctiva

49 : A, B and C 50: A, B, D and E 51 : B 52 : A 54 : A 55 : B 56 : C 57 : B 58 : B


53 : None
CHAPTER

5
Diseases of Cornea

Quick Text Review


CONGENITAL ANOMALIES Pathogens which can invade the normal intact
corneal epithelium are: Neisseria gonorrhoea,
Megalocornea Neisseria meningitis, Corynebacterium diphtheriae,
• The horizontal diameter of cornea at birth is Listeria and Haemophillus.
10 mm. The adult size of 11.7 is attained by the
age of 2 years. Commonest organisms/etiological agents
• Megalocornea is labelled when the horizontal responsible for different corneal ulcers are:
diameter of cornea is of adult size at birth or • Hypopyon corneal ulcer : Pneumococcus
13 mm or greater after the age of 2 years. Cornea • Mycotic corneal ulcer : Aspergillus fumigatus
is clear with normal thickness and normal vision. • Marginal corneal : Koch-Weeks bacillus
Megalocornea is often associated with Marfan’s ulcer (Catarrhal (Haemophilus
syndrome. corneal ulcer) aegyptius)
Microcornea • Typical dendritic : Herpes simplex virus
• In microcornea, the horizontal diameter is less corneal ulcer
than 10 mm since birth. • Very rapidly
• The condition may occur as an isolated anomaly progressive sloughing
(rarely) or in association with nanophthalmos corneal ulcer : Pseudomonas
(normal small eyeball) or microphthalmos • Ulcus serpens : Pneumococcus
(abnormal small eyeball). Eye is usually hyper­ • Fascicular ulcer : Phlyctenulosis
metropic. • Atheromatous ulcer : Degenerative change
Congenital cloudy cornea in old leucoma
The conditions responsible can be denoted by the
Hypopyon corneal ulcer
eponym STUMPED as follows:
• Sclerocornea • Staphylococci, Streptococci, Gonococci and Mora­
• Tears in descemet’s membrane xella may produce hypopyon; but by far the most
• Ulcer dangerous are Pseudomonas pyocyanea and the
• Metabolic conditions Pneumococcus
• Posterior corneal defect • Characteristic hypopyon ulcer caused by Pneumo­
• Endothelial dystrophy coccus is called ulcus serpens which has a tendency
• Dermoid. to creep over the cornea
• Commonest cause of failure in treatment is
KERATITIS secondary glaucoma.
CORNEAL ULCER (ULCERATIVE KERATITIS)
Mycotic corneal ulcer
Bacterial corneal ulcer • Common causative fungi are Aspergillus (most
Commonest causative organisms are pneumo­ common), Candida and Fusarium
coccus, others are Staphylococcus, pseudomonas • Common mode of infection is injury by vegetative
and gono­coccus. material, e.g. thorn, branch of a tree, etc.
Diseases of Cornea 43
• Injudicious use of antibiotic and steroids increase • Herpes zoster virus is the most common virus
the incidence of fungal infections causing the corneal ulcer
• Typical fungal ulcer is dry looking, greyish white • Lesions are strictly limited to one side of the midline
with finger-like extensions into the surrounding of the head
stroma under the intact epithelium. A sterile • Frontal nerve is more frequently affected than the
immune ring (Immune ring of Wessely) and lacrimal and nasociliary
multiple satellite lesions are characteristic. Usually • Cutaneous lesions in the area of distribution of the
a big hypopyon (pseudohypopyon) which is non- involved nerve appear after 3–4 days of onset of
sterile, infective presents even if the ulcer is very disease
small. Corneal vascularization is conspicuously • Ocular lesions occur in 50 percent cases
absent. • Hutchinson’s rule which implies that ocular
• Drug of choice for filamentous infections is nata­ involvement is frequent if the side or tip of the nose
mycin and for Candida is Amphoterecin B (0.15%). presents vesicles is useful but not infallible
Herpes simplex corneal ulcer
• Superficial punctate epithelial keratitis is the most
common feature
Herpes simplex virus (HSV) is a DNA virus. Ocular • Nummular keratitis may occur as multiple tiny
infection is more common with HSV-1 and rare with granular deposits surrounded by a halo of stromal
HSV type II haze
Primary ocular herpes occurs in non-immune • Iridocyclitis occurs in 50 percent cases of Zoster
children. Lesions are vesicles of skin of the lids, acute keratitis
follicular conjunctivitis. Fine or coarse punctate • Neuroparalytic keratitis occurs as a complication
epithelial keratitis occurs in 50 percent cases of Gasserian ganglion destruction
• Episcleritis and scleritis occur in 50 percent cases
Recurrent ocular herpes is caused by periodic
• Palsy of 3rd, 6th and 7th cranial nerve may occur
reactivation of virus (which lies dormant in the
in severe infection
trigeminal ganglion) by following precipitating
• Optic neuritis occurs in 1 percent cases
stimuli: Fever such as malaria or flu, general ill
• Post-herpetic neuralgia may persist for months or
health, exposure to ultraviolet rays (sunlight),
years
immunocompromised patients, e.g. in AIDS, and
• Oral acyclovir 800 mg 5 times a day × 10 days is
excessive use of topical or systemic steroids
effective
Lesions of recurrent ocular herpes are: • Systemic steroids are indicated in cranial nerve
• Punctate epithelial keratitis is initial lesion palsies and optic neuritis.
• Dendritic ulcer is pathognomonic lesion
• Geographical (amoeboid) ulcer is formed when Acanthamoeba keratitis
branches of dendritic ulcer enlarge and coalesce • Acanthamoeba is found in soil, fresh water, well
• Disciform keratitis is a delayed hypersensitivity water, sea water, sewage and air.
reaction to HSV antigen, characterised by a focal • Occurs both in immunocompetent and immuno­
disc-shaped patch of stromal oedema without suppressed
necrosis • Keratitis is more common in contact lens wearers
• Metaherpetic ulcer is a sterile trophic ulceration using home-made saline
which occurs due to healing defect at the site of • Severe pain, out of proportion to the degree of
previous herpetic ulcer. inflammation is typical feature
Antiviral drugs effective against HSV are: • Typical lesions are central or paracentral ring
• 5-iodo-2 deoxyuridine (IDU) - 1 percent drops 1 shaped stromal infiltrates with overlying epithelial
hourly during day and 2 hourly at night defects with associated radial keratoneuritis
• Trifluorothymidine (TFT) - 1 percent drops 2 hourly • KOH and lactophenol cotton blue stained film may
• Acyclovir - 3 percent ointment 5 times a day. It is demonstrate cysts
the only drug effective in stromal keratitis. • Culture medium: Non-nutrient agar with E.coli
and does not depend on host for completion of
Herpes zoster ophthalmicus life cycle.
• Varicella-zoster virus is a neurotropic DNA virus • Treatment consists of 0.1 percent propamidine
which produces acidophilic intranuclear inclusion isethionate (Brolene) and neomycin eye drops.
bodies Keratoplasty is required in resistant cases.
44 Review of OPHTHALMOLOGY

Neuroparalytic keratitis SUPERFICIAL PUNCTATE KERATITIS


• Occurs due to paralysis of sensory nerve supply of Characterised by multiple spotty lesions in
the cornea. Most typically as a result on 5th nerve superficial layers of cornea associated with acute
paralysis and as a result of radical treatment of pain, photophobia and lacrimatoin.
trigeminal neuralgia Etiology of SPK includes: Viral infections, chlamy­
• Typical features are absence of pain, lacrimation dial infections, toxic due to Staphylococcal toxins,
and complete loss of corneal sensations. exposure keratitis and neuroparalytic keratitis,
Exposure keratitis allergic lesions, keratoconjunctivitis sicca, Thygeson’s
SPK, and photophthalmia.
• It occurs in conditions producing lagophthalmos
such as extreme proptosis, facial nerve palsy, coma, Treatment: Topical weak steroids have suppressive
overcorrection of ptosis, etc. effect.
• Initial dessication due to exposure occurs in Photophthalmia
the interpalpebral area leading to fine punctate
Superficial punctate keratitis which occurs due
epithelial keratitis followed by frank ulceration and
to effect of ultraviolet rays (especially 290–311 nm)
vascularization.
associated with exposure to:
Rosacea keratitis • Naked arc light, as in industrial welding and
• Rosacea, a disease of sebaceous glands of the skin, cinema operators
occurs in elderly women as facial eruptions • Bright light of short circuiting
• Ocular lesions include blepharoconjunctivitis, acne • Reflected ultraviolet rays from snow surface (snow
rosacea keratitis, corneal vascularization and iritis blindness)
Characteristic features are severe burning pain,
in severe cases
lacrimation, marked photophobia and blepharo­
• Treatment consists of topical steroids and systemic
spasm occurring 4–5 hours after exposure to the
tetracycline.
ultraviolet rays.
Mooren’s ulcer Superior limbic keratoconjunctivitis
(Chronic serpiginous or rodent ulcer)
• Occurs with greater frequency in female patients
• Exact etiology is not known, probably it is an
with hyperthyroidism.
autoimmune disease
• Conjunctiva in region of superior limbus and
• Typical ulcer is a shallow furrow with whitish over-
adjoining parts of bulbar and tarsal conjunctiva
hanging edge, vascularized base accompanied by
are congested. Cornea shows superficial punctate
severe pain
keratitis which stains both with fluorescein and
• Immunosuppression with cyclosporin is useful.
rose bengal.
Non-healing corneal ulcer Thygeson’s superficial punctate keratitis
Local causes • It is a chronic, recurrent, bilateral, superficial
• Raised intraocular pressure keratitis characterised by coarse punctate epithelial
• Misdirected cilia lesions (snow flake)
• Large concretions • Each lesion is a cluster of heterogenous granular
• Chronic dacryocystitis grey dots.
• Impacted foreign body
Filamentary keratitis
• Lagophthalmos
• Inadequate therapy • Superficial punctate keratitis associated with
• Wrong therapy. formation of epithelial filaments which are freely
movable over the cornea, firmly attached at the base
General causes and stain both with fluorescein and rose bengal
• Diabetes mellitus • Common causes are: Keratoconjunctivitis sicca,
• Severe anaemia superior limbic keratoconjunctivitis, prolonged
• Malnutrition patching, following epithelial erosions as in herpes
• Vitamin-A deficiency simplex, Thygeson’s keratitis, recurrent corneal
• Immunocompromised patients. erosion syndrome and trachoma.
Diseases of Cornea 45
INTERSTITIAL KERATITIS • Presents as band-shaped opacity in the inter-
An inflammation of the corneal stroma without palpebral area
primary involvement of the epithelium or endo­ • Surface of the opaque band is stippled due to holes
thelium. in the calcium plaques in the area of nerve canals
of Bowman’s membrane
Common causes are syphilis, tuberculosis, Cogan’s
• Treatment consists of chemical removal of calcium
syndrome, trypanosomiasis, malaria, leprosy, and
salts with 0.01 molar solution of EDTA. Photo­
sarcoidosis.
therapeutic keratectomy (PTK) with excimer laser
Syphilitic (leutic) interstitial keratitis is very effective in clearing the cornea.
• Ninety percent cases are associated with congenital Salzmann’s nodular degeneration
syphilis
• Occurs in eyes with recurrent attacks of phlyctenular
• In congenital syphilis, manifestations are usually
keratitis, rosacea keratitis or trachoma
bilateral and develop between 5 and 15 years of age
• Cornea shows one to ten bluish white nodules
• In acquired syphilis, manifestations are usually
arranged in a circular fashion, due to deposition
unilateral
of hyaline plaques between epithelium and
• Interstitial keratitis may occur alone or as a
Bowman’s membrane.
part of Hutchinson’s triad which also includes:
Hutchinson’s teeth and vestibular deafness Spheroidal degeneration
• Disease is a result of antigen-antibody reaction (climatic droplet keratopathy)
• Ground glass appearance of cornea and ‘Salmon • Occurs in men who work outdoors, especially in
patches’ are typical features hostile climates. Its occurrence has been related
• After healing, corneal opacities and ghost vessels to exposure to ultraviolet rays and/or ageing.
are left behind. • Cornea shows amber-coloured spheroidal granules
Tubercular interstitial keratitis accumulated at the level of bowman’s membrane
and anterior stroma in the interpalpebral area.
• It is more frequently unilateral and sectorial.
Cogan’s syndrome CORNEAL DYSTROPHIES
It comprises interstitial keratitis of unknown etiology, Classification
acute tinnitus, vertigo and deafness typically occurring
1.Anterior dystrophies (superficial dystrophies),
in middle-aged adults. It is often bilateral.
primarily affecting epithelium and Bowman’s layer:
DEGENERATIONS, DYSTROPHIES AND ECTATIC • Epithelial basement membrane dystrophy
CONDITIONS • Reis–Bückler’s dystrophy
• Meesman’s dystrophy
CORNEAL DEGENERATIONS • Recurrent corneal erosion syndrome
Arcus senilis • Stocker–Halt dystrophy.
• Annular lipid infiltration of corneal periphery 2. Stromal dystrophies
is seen in elderly people. The ring of opacity is • Granular (Groenouw’s type I) dystrophy
separated from the limbus by a clear zone (lucid • Lattice dystrophy
interval of vogt) • Macular (Groenouw’s type II) dystrophy
• Most commonly encountered peripheral corneal • Crystalline (Schnyder’s dystrophy).
opacity. 3. Posterior dystrophies, affecting primarily the
Arcus juvenilis
corneal endothelium and Descemet’s membrane:
• Cornea guttata
• Condition similar to arcus senilis occurring in
• Fuchs’ epithelial endothelial dystrophy (late
young persons (below 40 years of age).
hereditary endothelial dystrophy)
Calcific degeneration (Band keratopathy) • Posterior polymorphous dystrophy of Schlichting
• Characterised by deposition of calcium salts in • Congenital hereditary endothelial dystrophy
Bowman’s membrane and most superficial part (CHED).
of stroma 4. Ectatic dystrophies
• It is associated with chronic uveitis in adults, • Keratoconus (anterior)
children with Still’s disease, phthisis bulbi, chronic • Posterior keratoconus
glaucoma, chronic keratitis, and ocular trauma • Keratoglobus.
46 Review of OPHTHALMOLOGY

Autosomal dominant dystrophies • Hurricane keratopathy: Whorl pattern of SPK due


• Lattice dystrophy is the most common stromal to effect of contact lens
corneal dystrophy (lattice type 1) • Fleischer’s ring seen at the base of cone is perhaps
• Granular dystrophy due to hemosiderin deposition in epithelium
• Fleck dystrophy • Applanation tonometry: Pulsation of mires is
• Meesmann dystrophy visible due to thin cornea
• Reis–Bückler dystrophy • An annular dark shadow is seen on distant direct
• Avellino dystrophy. ophthalmoscopy
Autosomal recessive dystrophies • Scissors reflex on retinoscopy
• Macular dystrophy: Least common stromal • Irregular astigmatism on keratometry
dystrophy • Irregularity of circles on placido disc examination
• Lattice type 3. • Acute hydrops: Sudden hydration of corneal stroma
due to rupture of Desemet’s membrane.
Salient points
• Corneal topography makes early diagnosis.
• Cogan’s microcystic dystrophy is the most common of
Keratoglobus
all corneal dystrophies seen in working-age adults.
• Reis–Bückler’s dystrophy primarily involves • It is a familial and hereditary (autosomal recessive)
Bowman’s membrane. bilateral congenital disorder characterised by
• In macular dystrophy, acid mucopolysaccharides thinning and hemispherical protrusion of the
are deposited in the cornea. entire cornea
• Cornea guttata may occur independently or as a • It should be differentiated from buphthalmos
part of early stage of Fuchs’ dystrophy. where increased corneal size is associated with
• Primary open-angle glaucoma is a common raised intraocular pressure, angle anomaly and
association of Fuchs’ dystrophy. cupping of the disc.
• Epithelial basement membrane dystrophy is the
most common anterior corneal dystrophy MISCELLANEOUS AND SYMPTOMATIC
• Fuch’s endothelial dystrophy is the most common CONDITIONS
posterior endothelial dystrophy.
• Posterior polymorphous dystrophy. It is dominantly MISCELLANEOUS KERATOPATHIES
inherited and is associated with corectopia Vortex keratopathy
(abnormal location of pupil). • Also called cornea verticillata
ECTATIC CONDITIONS OF CORNEA • Characterised by bilateral, symmetrical, grayish or
Keratoconus
golden corneal epithelial deposits
• Occurs in patients taking chloroquine, amiod
Non-inflammatory ectatic condition of cornea
arone, amodiaquine, meperidine, indomethacin,
giving it a conical shape with resultant irregular
chlorpromazine and tamoxifen. Also seen in
myopic astigmatism. It is usually bilateral (85%). It
patients with Fabry’s disease (glycolipidosis).
is more common in girls after puberty.
Classification: Morphologically divided into: Crystalline keratopathy
a. Nipple cone (5 mm) Chrysiasis refers to deposition of gold in corneal
b. Oval cone (5-6 mm) displaced inferotemporally stroma following prolonged administration in
c. Globus cone (>6 mm) may involve over 75% of patients with rheumatoid arthritis.
cornea.
Cystinosis refers to widespread deposition of
Its important signs and symptoms are:
cysteine crystals in conjunctiva, cornea, iris, lens and
• Impaired vision due to progressing myopia and
retina. A hereditary disorder (autosomal recessive)
irregular astigmatism.
leading to defect in its lysosomal transport.
• Munson’s sign i.e., localised bulging of lower lid
when patient looks down, is positive in late stages. Monoclonal gammopathy refer to deposition of
• Vogt’s Striae: Stress lines parallel to sleep axis of crystal in corneal epithelium and stroma seen in
cornea patients with:
• Increased visibility of nerves: It is due to thinning • Multiple myeloma
of the cornea and not due to thickening of nerves • Waldenstrom’s macroglobulinemia
• Reduced corneal sensation • Lymphoma.
Diseases of Cornea 47
SYMPTOMATIC CONDITIONS OF CORNEA • Pseudomonas is the most common organism
Vascularization of cornea causing keratitis/corneal ulcer in a contact lens
wearers
Superficial corneal vascularisation
• Macular dystrophy is the least common corneal
• The vessels are arranged in an arborizing pattern,
dystrophy in clinical practice
present below the epithelial layer and their
• Pseudocornea has only two layers—the connective
continuity can be traced with the conjunctival
tissue lined with epithelium
vessels.
• Poor antigenicity of corneal stroma is due to
• Causes: Trachoma, phlyctenular kerato-conjunc­ avascularity, sparse cell count and separation of
tivitis, superficial corneal ulcers, contact lens user, cells by ground substance
cornea graft rejection and rosacea keratitis. • Lamellar keratoplasty is the treatment of choice
Deep corneal vascularisation for Mooren’s ulcer
• The vessels are derived from the anterior ciliary • Therapeutic keratoplasty is the treatment of choice
arteries and lie in the corneal stroma. These for impending corneal perforation
vessels are usually straight, not anastomosing • In Keratoconus, the cone is situated just below the
and their continuity cannot be traced beyond the centre of the cornea
limbus. • Pachymetry refers to measurement of corneal
• Causes: Interstitial keratitis, disciform keratitis, thickness
deep corneal ulcer, chemical burns, sclerosing • Specular microscopy is useful for study of endo­
keratitis and corneal graft. thelial cells
• Keratometry is measurement of corneal curvature
Corneal Opacity • Corneal topography is the study of corneal shape
• Nebular corneal opacity is a faint opacity resulting • Deep anterior lamellar keratoplasty (DALK) is
from superficial scars involving Bowman’s indicated where endothelium and Descemet’s
layer and superficial stroma. A nebular opacity membrane are normal e.g., keratoconus
covering the pupil produces more discomfort and • Descemet’s striping endothelial keratoplasty (DSEK)
blurring of vision (owing to irregular astigmatism) is indicated where only endothelium is defective
than a leucoma which completely cuts off the light e.g., post-operative endothelial decompensation.
rays. • Enlarged corneal nerves are seen in Herpes simplex
• Adherent leucoma is a leucomatous opacity in keratitis, MEN 2B, keratoconus, neurofibromatosis,
leprosy, advanced age, keratoconjunctivitis sicca,
which iris tissue is incarcerated. It results following
congenital glaucoma and trauma.
corneal perforation.
• Photorefractive keratometry (PRK) causes ablation
• Kerectasia refers to bulged-out thin corneal scar.
of the superficial layers of the cornea resulting in
• Anterior staphyloma is bulging out of ectatic flattening of the anterior cornea to reduce myopia,
opaque cornea (pseudocornea) with iris tissue so it can also be used to remove superficial corneal
plastered behind it. Pseudocornea is a scar formed opacities such as those associated with band
from organized exudates and fibrous tissue keratopathy and treat superficial corneal disease
covered with epithelium. It results following total such as recurrent corneal erosions.
sloughing of the cornea. • Pigment deposition in cornea is of two types:
Superficial (Amiodarone, chloroquine, indo­
SOME SALIENT POINTS
methacin) and stromal (phenothiazine).
• Rheumatoid arthritis is the most common collagen • Cornea Verticillata are asymptomatic, whorl like
vascular disorder to affect the peripheral cornea opacities in the corneal epithelium seen in patients
• The most common ocular complication of SLE is on long term medication such as amiodarone,
punctate epithelial keratopathy chloroquine, indomethacin, phenothiazine and
• Staphylococus aureus is the commonest organism reversible on stopping drugs. It is also seen in
responsible for corneal ulceration disease like Fabry’s disease.
48 Review of OPHTHALMOLOGY

Multiple Choice Questions (MCQs)

1. The organism which can invade the intact corneal 8. Viral infections usually cause:
epithelium and produce purulent corneal ulcer is: A. Conjunctivitis
A. Neisseria meningitidis B. Kerato conjunctivitis
B. Pseudomonas pyocyanea C. Keratitis
C. Pneumococcus D. Blepharo conjunctivitis
D. Streptococcus haemolyticus
9. Is not the clinical presentation of herpes simplex
2. Hypopyon ulcer may be produced by:
keratitis:
A. Pneumococcus
A. Diffuse stromal necrotic keratitis
B. Pseudomonas
C. Gonococcus B. Punctate epithelial keratitis
D. All of the above C. Disciform keratitis
D. Nummular keratitis
3. Ulcus serpens is caused by:
A. Pneumococcus 10. Ocular complications in herpes zoster ophthal­
B. Pseudomonas micus usually appear:
C. Gonococcus A. At the subsidence of skin eruptions
D. All of the above B. Simultaneous with cutaneous lesions
4. All are the causes of a non-healing corneal ulcer C. Two days after the skin eruptions
except: D. During stage of erythematous skin lesions
A. Raised intraocular pressure 11. Secondary glaucoma in early stage of herpes
B. Associated iridocyclitis zoster ophthalmicus occurs due to:
C. Chronic dacryocystitis
A. Trabeculitis
D. Diabetes mellitus
B. Iridocyclitis
5. Marginal catarrhal corneal ulcer is caused by: C. Haemorrhagic hypopyon
A. Staphylococcus D. Hypersecretion of aqueous humour
B. Moraxella E. All of the above
C. Haemophilus
D. All of the above 12. Systemic acyclovir in herpes zoster is useful:
A. When started immediately after the onset of rash
6. Satellite lesions in the cornea may be seen in:
B. For post-herpetic neuralgia
A. Fungal corneal ulcer
C. For ocular lesions only
B. Bacterial corneal ulcer
D. All of the above
C. Viral corneal ulcer
D. None of the above 13. Systemic corticosteroids in herpes zoster ophthal­
7. Immune ring in the cornea is a feature of: micus are indicated when associated with:
A. Bacterial corneal ulcer A. Facial nerve palsy
B. Fungal corneal ulcer B. Optic neuritis
C. Ring ulcer of the cornea C. Post-herpetic neuralgia
D. Disciform keratitis D. All of the above

1 : A 2 :D 3 : A 4 : B 5 : D 6 : A
8 : B 9 : D 10 :A 11 : A 12 : A 13 : B
7:B
Diseases of Cornea 49
14. The nerve which is most frequently involved in 23. Hutchinson’s triad includes all except:
herpes zoster ophthalmicus is: A. Interstitial keratitis
A. Frontal nerve B. Hutchinson’s teeth
B. Nasociliary nerve C. Vestibular deafness
C. Lacrimal nerve D. Flat nose bridge
D. Facial nerve
24. Interstitial keratitis may be associated with:
15. Hutchinson’s rule in relation to herpes zoster A. Congenital syphilis
ophthalmicus: B. Malaria
A. Implies that ocular involvement is infrequent if C. Sarcoidosis
the side or tip of the nose presents vesicles D. All of the above
B. Is based on involvement of nasociliary nerve 25. Commonest causative organism of corneal ulcer
C. Is 100 percent predictor of ocular involve­ment is:
D. All of the above A. Pneumococcus
16. Radial keratoneuritis is a feature of: B. Staphylococcus
A. Acanthamoeba keratitis C. Streptococcus
B. Herpes zoster keratitis D. Fungi
C. Neuroparalytic keratitis 26. Metabolically active corneal layer is:
D. All of the above A. Epithelium
17. Exposure keratitis is not associated with: B. Stroma
A. 7th nerve paralysis C. Descemet’s membrane
B. 5th nerve paralysis D. None of the above
C. Symblepharon 27. Fascicular ulcer is seen in:
D. Ectropion A. Phlyctenular keratitis
18. All are features of rosacea keratitis except: B. Rosacea keratitis
A. Corneal vascularisation C. Riboflavin deficiency
B. Associated blepharoconjunctivitis D. All of the above
C. Central superficial ulcer 28. Corneal dystrophy asso ciated with acid
D. Progresses to involve the whole cornea mucopolysaccharidosis is:
19. All are true for Mooren’s ulcer except: A. Lattice
A. Peripheral ulcerative keratitis B. Granular
C. Macular
B. Advancing edge is undermined
D. Peripheral
C. Perforation is common
D. Floor of ulcer is quickly vascularised 29. All of the following result in loss of corneal
20. Photo-ophthalmia results from exposure to: sensations except:
A. Ultraviolet rays A. Acute congestive glaucoma
B. Infrared rays B. Dendritic ulcer
C. b-irradiation C. Absolute glaucoma
D. All of the above D. Exposure keratitis

21. In photophthalmia site of lesions is: 30. The earliest symptom to occur in corneal ulcer is:
A. Cornea A. Pain
B. Retina B. Photophobia
C. Optic nerve C. Loss of sensation
D. All of the above D. Diminished vision
22. Filamentary keratitis may occur: 31. Corneal reflex is lost in the disease of:
A. In trachoma A. Ophthalmic nerve
B. In kerato conjunctivitis sicca B. Ciliary ganglion
C. Following cataract surgery C. Supra orbital nerve
D. All of the above D. Motor nucleus of 5th cranial nerve

14 : A 15 :B 16 : A 17 : B 18 : C 19 : C 23 : D 24 : D 25 : A 26 : A 27 : A 28 : C
20 : A 21 : A 22 : D 29 : D 30 : A 31 : A
50 Review of OPHTHALMOLOGY

32. The commonest cause of hypopyon is: C. Royal blue


A. Moraxella D. Green
B. Gonococcus
41. Metabolically most active layer of cornea is:
C. Pneumococcus
A. Endothelium
D. Staphylococcus
B. Stroma
E. C. diphtheriae
C. Descemet’s membrane
33. Keratomalacia includes the following except: D. Epithelium
A. Night blindness
42. Earliest symptom in corneal ulcer is:
B. Severe pain in the eye
A. Loss of sensation
C. Xerosis of the cornea
D. Perforation of cornea B. Diminished vision
C. Photophobia
34. Kayser Fleischer ring is found in which layer of D. Pain
cornea ? 43. Munson’s sign is seen in:
A. Bowman’s membrane A. Keratoconus
B. Substantia propria B. Keratoglobus
C. Descemet’s membrane C. Microcornea
D. Endothelium
D. All
35. Corneal transparency is due to all except: 44. “Orange skin” cornea results due to:
A. Normal IOP A. Chalcosis
B. Na+ K+ pump B. Siderosis
C. Hypercellular stroma C. Ammonia burn
D. Peculiar arrangement of stromal lamella D. Mustard gas
36. Condition which is always bilateral: 45. Anterior lenticonus is seen in:
A. Infantile glaucoma A. Marfan’s syndrome
B. Megalocornea B. Ehler Danlos syndrome
C. Acute congestive glaucoma C. Weil-Marchessani syndrome
D. All of the above D. Homocystinuria
E. None of the above
46. For transplantation, cornea is preserved in:
37. Commonest causative organism of corneal ulcer is: A. Modified MK medium
A. Pneumococci B. Glycerine medium
B. Streptococci C. Wet medium
C. Staphylococci D. All of the above
D. Fungal
47. Microscopy of corneal ulcer showed branching
38. Corneal ulceration may be caused by injury to septate hyphae. The probable diagnosis is:
which cranial nerve: A. Candida
A. Third B. Mucormycosis
B. Fifth C. Aspergillus
C. Sixth D. Histoplasma
D. All of the above
48. Corneal epithelium is composed of:
39. What type of corneal dystrophy is associated with A. Stratified keratinized epithelium
acid mucopolysaccharidosis: B. Stratified non-keratinized epithelium
A. Granular C. Columnar epithelium
B. Lattice D. Pseudostratified epithelium
C. Macular E. Transitional epithelium
D. Peripheral
49. Keratomalacia is associated with:
40. The colour of fluorescein staining in corneal ulcer A. Measles
is: B. Mumps
A. Yellow C. Rubella
B. Blue D. Chicken pox

32 : C 33 : B 34 : C 35 : C 36 : B 37 : C 41 : D 42 : D 43 : A 44 : D 45 : A 46 : A
38 : B 39 : C 40 : D 47 : C 48 : B 49 : A
Diseases of Cornea 51
50. Herpes zoster ophthalmicus is a predicator of: C. SLE
A. Leukemia D. DLE
B. Lymphoma
C. HIV 60. Following corneal transplantation, most common
D. All of the above infection occur:
A. Staph epidermidis
51. All are true about keratoconus except: B. Streptococcus
A. Increased curvature of cornea C. Klebsiella
B. Astigmatism D. Pseudomonas
C. K.F. ring cornea
D. Thick cornea 61. Enlarged corneal nerves may be seen in all of the
following except:
52. Thinning of cornea occurs in:
A. Keratoconus
A. Megalocornea
B. Herpes simplex keratitis
B. Bullous keratopathy
C. Leprosy
C. Endothelial dystrophy
D. Neurofibromatosis
D. Keratoconus
53. Treatment of choice for photopthalmia is: 62. A 28-year-old male complains of glare in both
A. Irrigation with antibiotics eyes. The cornea shows whorl like opacities of
B. Irrigation with local anaesthesia the epithelium. He also gave a history of long
C. Irrigation with saline term treatment with amiodarone. The most likely
D. Patching the eye diagnosis is:
A. Terrain’s marginal degeneration
54. Deep anterior lamellar keratoplasty is indicated B. Cornea verticillata
in: C. Band shaped keratophathy
A. Disease of deeper cornea e.g. endothelial damage D. Arcus juvenalis
B. Full thickness corneal opacities
C. Bullous keratopathy 63. Dellen is:
D. Superficial corneal opacities A. Localized thinning of peripheral cornea
B. Raised lesions in corneo limbal junction
55. Recurrent corneal erosion seen in: C. Age related macular degeneration
A. Corneal dystrophy d. Marginal keratitis
B. Keratoglobus
C. Keratoconus
EXTRA EDGE QUESTIONS
D. Peutz-anaomalies
56. Posterior polymorphous dystrophy 64. True about anatomy of adult cornea:
A. Causes corectopia A. Horizontal diameter is 12 mm
B. Is inherited in an autosomal recessive fashion B. Horizontal diameter is 10 mm
C. Causes blindness in over 90% of sufferers C. In megalocornea diameter is >12 mm
D. Can be treated with lamellar corneal grafts D. In microcornea diameter <10 mm
E. Vertical diameter> Horizontal diameter
57. To prevent keratoconus what is used:
A. Antibiotics 65. True about cornea:
B. Cycloplegics A. Power is 43 D
C. Glasses B. Majority of refraction occur at air-tear interface
D. None C. With the rule astigmatism is present because
58. Interstitial keratitis is seen in all except: vertical meridian more sleep than horizontal
A. Syphilis meridian
B. Acanthamoeba D. Spherical structure
C. Chlamydia trachomatis E. Refractive index 1.334.
D. HSV 66. Corneal transparency is maintained by all except:
E. HZV
A. Relative dehydration
59. Band shaped keratopathy seen in: B. Increased mitotic figures in centre of cornea
A. JRA C. Unmyelinated nerve fibers
B. RA D. Uniform spacing of collagen fibrils

50 : D 51 : D 52 : D 53 : D 54 : D 55: A 60 : A 61: B 62 : B 63 : A 64 : A and D


56 : A 57 : D 58 : B 59 : A 65 : A, B and C 66 : B
52 Review of OPHTHALMOLOGY

67. Ionic pump in corneal endothelium is necessary C. Lattice type III


for maintaining deturgescence of the cornea and D. Granular corneal dystrophy
thus transparency. It can be blocked by:
73. Which one of the following stromal dystrophy is
A. Inhibition of anaerobic glycolysis
a recessive condition?
B. Activation of anaerobic glycolysis
A. Lattice dystrophy
C. Inhibition of Kreb’s cycle B. Granular dystrophy
D. Inhibition of HMP pathway C. Macular dystrophy
68. A young man aged 30 years, presents with D. Fleck dystrophy
difficulty in vision in the left eye for the last 10 74. Corneal vascularisation is/are caused by:
days. He is a rural community and gives history A. Graft rejection
of trauma to his left eye with vegetative matter B. Chemical burn
10–15 days back. On examination, there is an C. Contact lens use
ulcerative lesion in the cornea, whose base has D. Vitreous haemorrhage
raised soft creamy infiltrate. Ulcer margin is E. Viral injection
feathery and hyphate. There are a few satellite
lesions also. The most probable etiological agent 75. The central nebular corneal opacity is easily
is: treated with:
A. Acanthamoeha A. Lamellar keratoplasty
B. Corynebacterium diphtheria B. Penetrating keratoplasty
C. Gas permeable contact lens
C. Fusarium
D. Soft contact lens
D. Streptococcus pneumonia
76. Which of the following statements regarding
69. Kallu, a 25 year male patient presented with a corneal transplantation is true:
red eye and complains of pain, photophobia, A. Whole eye needs to be preserved in tissue culture
watering and blurred vision. He gives a history B. Donor not accepted if age >60 years
of trauma to his eye with a vegetable matter. C. Specular microscopy analysis is used to assess
Corneal examination shows a dendritic ulcer. endothelial cell count
A corneal scraping was taken and examined. D. HLA matching is mandatory
Microscopy showed macrophages like cells 77. Pigment deposition on cornea seen in:
on culturing the corneal scrapings over a non- A. Chloroquine
nutrient agar enriched with E.coli, there were B. Digoxin
plaque formations. Which organism is most likely: C. Ranitidine
A. Herpes simplex D. Amiodarone.
B. Acanthameba
C. Candida 78. Corneal transplantation:
D. Adeno virus A. Donor >60 years not allowed
B. Whole eye preserved in culture
70. Corneal dystrophies are usually: C. Specular microscopy done for corneal endothelial
A. Primarily unilateral count
B. Primarily bilateral D. HLA matching required
C. Primarily unilateral without systemic disease
D. Primarily bilateral with systemic disease 79. Corneal tattooing is done by:
A. Iron
71. Corneal dystrophies are:
B. Silver
A. Macular
C. Copper
B. Granular
C. Lattice D. Platinum
D. Moorens
80. Neuroparalytic keratitis is due to which cranial
E. Fuchs nerve:
72. Which of the following is the least common corneal A. 3rd nerve
dystrophy: B. 5th nerve
A. Macular dystrophy C. 6th nerve
B. Lattice type I D. 7th nerve

67 : A 68 : C 69 : B 70 : B 71 : A, B, C and E 73 : C 74 : A, B and C 75 : A 76 : C 77 : A and D


72 : A 78 : C 79 : D 80 : B
CHAPTER

6
Diseases of Sclera

Quick Text Review


INFLAMATORY DISORDERS • Patients of particularly necrotizing scleritis may
have one of the following systemic diseases:
EPISCLERITIS
Rheumatoid arthritis, connective tissue vascular
• Typically affects young adults, being twice as disorders like polyarteritis nodosa, SLE, Wegener’s
common in women than men granulomatosis and miscellaneous diseases
• Etiology in most cases is not known. Occurs in
like relapsing polychondritis, herpes zoster and
association with gout, rosacea, psoriasis and as
surgically induced scleritis.
hypersensitivity reaction to endogenous tubercular
or streptococcal proteins • Scleromalacia perforans refers to anterior
• Types: Diffuse episcleritis, nodular episcleritis necrotizing scleritis without inflammation.
• In nodular episcleritis, a pink or purple flat nodule It is common in women with long-standing
surrounded by congestion is usually situated seropositive rheumatoid arthritis. There is no
2-3 mm away from the limbus effective treatment for this condition
• Episcleritis periodica refers to a fleeting type of • Posterior scleritis is frequently misdiagnosed.
disease Its features are proptosis, limitation of ocular
• Treatment in severe cases is with topical steroids movements, exudative retinal detachment,
and systemic indomethacin 50 mg twice daily. macular oedema
SCLERITIS • Cornea and uveal tract are frequently involved in
scleritis and not in episcleritis.
• Usually occurs in elderly patients (40–70 years),
involving females more than the males. More rare
than episcleritis. MISCELANEOUS CONDITIONS AND SALIENT
• About 50 percent cases are associated with some POINTS
systemic disease including connective tissue
disease. BLUE SCLERA
• About 0.5 percent of patients with seropositive • A typical association of blue sclera is osteogenesis
rheumatoid arthritis develop scleritis. imperfecta
• Causes of non-pyogenic scleritis are syphilis, • Other associations are Marfan’s syndrome, Ehlers-
tuberculosis, and leprosy. Danlos syndrome, pseudoxanthoma elasticum,
• In scleritis pain is moderate to severe, deep and congenital glaucomas, scleritis and Werner’s
boring in character. syndrome.
Classification
STAPHYLOMA
  I. Anterior scleritis
• Non-necrotizing: Diffused or nodular • Staphyloma refers to bulging of the ecstatic
• Necrotizing: With or without inflammation cicatricial outer coat of the eyeball lined by uveal
II. Posterior scleritis tissue
• Anterior staphyloma results after total sloughing
Salient features of the cornea
• Non-necrotizing anterior diffuse scleritis is the • Intercalary staphyloma (ectasia of sclera with
most common clinical variety incarceration of root of iris). It may occur after
54 Review of OPHTHALMOLOGY

scleritis, perforating injury, peripheral corneal CAUSES OF SCLERAL THINNING


ulceration • Uveitis
• Ciliary staphyloma may occur following scleritis, • Scleritis
perforating injury or absolute glaucoma • Tuberculoma
• Equatorial staphyloma commonly occurs at the • Congenital glaucoma.
regions of sclera which are perforated by vortex SOME SALIENT POINTS
veins • Diseases of the sclera are chronic because of relative
• Posterior staphyloma may occur due to patho­ avascularity of sclera.
logical myopia (most common cause), posterior • Sclera is thickest at the posterior pole and thinnest
scleritis, posterior perforating injury. at the lamina cribrosa.
Diseases of Sclera 55

Multiple Choice Questions (MCQs)

1. All of the following are features of episcleritis C. Iridocyclitis


except: D. High myopia
A. Redness
9. Episcleritis periodica refers to:
B. Marked pain
A. Fleeting type of episclerities
C. Photophobia
D. Lacrimation B. Complicated episclertis
C. Intermediate stage between episcleritis and
2. Scleritis is most commonly associated with: scleritis
A. Polyarteritis nodosa D. None of the above
B. Tuberculosis
C. Rheumatoid arthritis 10. Scleritis is associated with some systemic disease
D. Sarcoidosis including connective tissue disorder in:
3. The most common variety of scleritis is: A. 35% of cases
A. Non-necrotizing anterior diffuse B. 50% of cases
B. Non-necrotizing anterior nodular C 66% of cases
C. Anterior necrotizing with inflammation D. 85% of cases
D. Anterior necrotizing without inflammation 11. Staphlymoa involvement:
4. The clinical variety of scleritis associated with A. Iris with conjunctiva
collagen diseases is: B. Conjunctiva with cornea
A. Necrotizing nodular scleritis C. Choroid with retina
B. Non-necrotizing nodular scleritis D. Iris with cornea
C. Scleromalacia perforans
12. Most common cause of posterior staphyloma:
D. All of the above
A. Trauma
5. Ciliary staphyloma occurs due to all of the B. Glaucoma
following except: C. Myopia
A. Scleritis D. Scleritis
B. Perforating injury
C. Absolute glaucoma 13. In scleritis all are true except:
D. Episcleritis A. Scleromalacia perforans are more commonly
associated with systemic diseases than posterior
6. Blue sclera is seen in:
A. Alkaptonuria scleritis
B. Osteogenesis imperfecta B. Pain is not a prominent feature
C. Lowe’s syndrome C. Retinal detachment is a known complication
D. All of the above D. Glaucoma is a known complication.
7. Sclera is thinnest at:
A. Posterior pole EXTRA EDGE QUESTIONS
B. Equator 14. The most common systemic association of scleritis
C. Corneoscleral junction is:
D. Points of muscular attachments A. Ehlers-Danlos syndrome
8. Commonest cause of posterior staphyloma is: B. Disseminated systemic sclerosis
A. Glaucoma C. Rheumatiod arthritis
B. Retinal detachment D. Giant cell arteritis

1 : B 2 : C 3 : A 4 : D 5 : D 6 : B 9 : A 10 : B 11 : D 12 : C 13 : B 14 : C
7 : D 8:D
CHAPTER

7
Diseases of Uveal Tract

Quick Text Review

UVEITIS (INFLAMMATIONS OF UVEA) – Mutton fat KPs are pathognomonic of granulo­


matous iridocyclitis
IRIDOCYCLITIS – Small and medium KPs are seen in nongranulo­
Etiology matous iridocyclitis
• Allergic uveitis is the most common variety – Very fine KPs occur in Fuch’s heterochromic
hypersensitivity to tubercular proteins is an cyclitis
important cause in developing countries (most – KPs are arranged in a triangular fashion due to
common) convection current
• HLA associated uveitis • Aqueous flare is the earliest sign of iridocyclitis.
– HLA-B27: Acute anterior uveitis associated with It is best seen with slit lamp examination. Occurs
ankylosing spondylitis and Reiter’s syndrome due to leakage of proteins and leucocytes into
– HLA-B5: Behcet’s disease the anterior chamber as a result of breakdown of
– HLA-DR4: Vogt-Koyanagi-Harada syndrome
blood-aqueous barrier
• Reiter’s syndrome is being considered to be caused
• Aqueous cells are sign of active inflammation
by chlamydial infection.
• Iris nodules are a feature of granulomatous
• Granulomatous uveitis is seen in:
– Tuberculosis - Syphilis iridocyclitis. Koeppe’s nodules are situated at the
– Sarcoidosis - Toxoplasmosis pupillary border. Bussaca’s nodules are situated
– Sympathetic ophthalmia - Vogt-Koyanagi- near the collarette
Harada syndrome • Small and irregular pupil in iridocyclitis results
• Common systemic disorders associated with acute from sphincter irritation due to toxins and
non-granulomatous anterior uveitis are: sacroilitis, engorgement of the radial blood vessels of the iris
ankylosing spondylitis, Reiter’s syndrome, psoriasis, • Iris bombe results from annular posterior
ulcerative colitis and Crohn’s disease. synechiae. In it, anterior chamber becomes funnel
• Masquerade uveitis is seen in retinal detachment, shaped.
intraocular tumours, CNS lymphomas and
multiple sclerosis. Complications of uveitis
• Fungal uveitis is most commonly due to candida.
Acute anterior uveitis: Complicated cataract,
Clinical features secondary glaucoma and CME.
• Endothelial bedewing of the cornea is the earliest
sign of iritis Chronic anterior uveitis: Rhegmatogenous retinal
• Keratic precipitates (KPs), i.e. proteinaceous detachment, phthisis bulbi (end result of chronic
cellular deposits at the back of the cornea are an uveitis, in which intraocular tension is reduced due
important feature of cyclitis. They may be the only to decreased aqueous formation). Histopathology
sign in insidious cases of cyclitis. KPs are seldom shows marked scleral thickening and intraocular
present in simple iritis ossification.
Diseases of Uveal Tract 57
Granulomatous versus non-granulomatous uveitis • Most common organisms causing endophthal-mitis
Feature Granulomatous Non- are divided into two groups:
granulomatous 1. Surgeon fault: Staphylococcus epidermidis,
Onset Insidious Acute Staphylococcus aureus and pseudomaonas.
Keratic Mutton fat Small 2. Patient fault: Propionbacterium acnes.
precipates • Puff ball opacities in the vitreous are patho­
(KPs) gnomonic of fungal endophthalmitis
Iris nodules Usually present Absent • Intravitreal injection, if antibiotic is treatment of
Posterior Thick and broad Thin and choice. Vitreous samples collected in emergency
synechiae based tenuous should be stored at 4o C.
Fundus Nodular lesions Diffuse • Antibiotics should be given by all the routes viz.
involvement topical, subconjunctival, intravitreal (treatment
Treatment of choice) and intravenously in bacterial endo­
phthalmitis
• Atropine is the most important topical drug (of
• Steroids should be started after 12-24 hours of
choice) for the management of a case of acute
intensive antibiotic therapy
iridocyclitis
• Vitrectomy is the treatment of choice for fungal
• Steroids, topical as well as systemic are effective in
endophthalmitis. In bacterial endophthalmitis, it
most cases
• Immunosuppressive drugs are specially useful should be performed when the condition does not
in severe cases of Behcet’s disease, sympathetic improve with intensive conservative therapy for 48
ophthalmitis, pars planitis, and Vogt-Koyanagi- hours.
Harada syndrome. • On examination—Amaurotic cat’s eye reflex.
SPECIFIC CLINICOETIOLOGICAL VARITIES OF NON-
CHOROIDITIS (POSTERIOR UVEITIS) SUPPURATIVE UVEITIS
• Diffuse choroiditis refers to a large spreading lesion
I. Uveitis in chronic systemic bacterial infections
involving most of the choroidal tissue. It is usually
Tubercular uveitis
tubercular or syphilitic in origin
• Disseminated choroiditis is characterised by • Accounts for 1 percent of uveitis patients in
multiple small areas of inflammation scattered developed countries. However, it is a very common
over the greater part of choroid, usually tubercular cause of uveitis in developing countries
or syphilitic in origin • Most frequent feature is chronic granulomatous
• Central choroiditis involves macular area, common uveitis
causes are: Toxoplasmosis, histo­plasmosis, tuber­ • Isoniazid test (300 mg/day for 3 weeks) for sus­
culosis, syphilis, visceral larva migrans (oncocerca) pected ocular involvement is a useful diagnostic
test
and rubella.
• Systemic treatment consists of a course of isoniazid,
• Juxtacaecal choroiditis involves the area adjoining
pyridoxine and rifampicin for 12 months.
optic disc, typical example is jensen’s choroiditis
• Anterior peripheral choroiditis is usually syphilitic Acquired syphilitic uveitis
in origin • Acute plastic iritis, typically occurs in secondary
• In late stages of choroiditis-negative scotoma syphilis
occurs in field of vision. • Gummatous anterior uveitis occurs in late secondary
• Drug of choice—Systemic steroids. syphilis and is characterised by yellowish red highly
vascularized nodules arranged near the pupillary
ENDOPHTHALMITIS or ciliary border of iris
• Inflammation of inner structure of the eyeball • Chorioretinitis may be peripheral, disseminated or
(which include; uveal tissue, retina, vitreous and diffuse
sclera is spared) with pouring of exudates into the • Neuroretinitis may cause optic atrophy.
anterior and posterior chamber of the eye. Leprotic uveitis
• Exogenous infection is the most common mode for • Uveitis (predominantly anterior) occurs more
purulent endophthalmitis. Most common cause is commonly in lepromatous than the tuberculoid
intraocular surgery. form of leprosy
58 Review of OPHTHALMOLOGY

• Chronic granulomatous iritis is characterised by Reiter’s syndrome


presence of small glistening ‘iris pearls’ near the • Characterised by a triad of urethritis, arthritis and
pupillary margin in necklace form. conjunctivitis
II. Uveitis in non-infectious systemic diseases • Acute non-granulomatous iridocyclitis occurs in
Sarcoid uveitis 20-30 percent cases
• Typically affects young males, 70 percent of whom
Sarcoid uveitis accounts for 2 percent cases of are positive for HLA-B27.
uveitis and may occur as:
• Acute unilateral non-granulomatous anterior Juvenile chronic arthritis (JCA)
uveitis occurs in young patients with acute • Anterior uveitis associated with JCA is chronic
sarcoidosis bilateral non-granulomatous with insidious onset
• Chronic bilateral granulomatous iridocyclitis • Bilateral in 70% cases (white uveitis)
occurs in older patients with chronic lung disease • Uveitis is much more common in pauciarticular
• Uveoparotid fever (Heerfordt’s syndrome): Bilateral JCA than polyarticular JCA
granulomatous panuveitis, painful enlargement of • About half of the cases are positive for HLA-
parotid glands, cranial nerve palsies, skin rashes, DW5 and 75 percent are positive for antinuclear
fever and malaise antibodies (ANA)
• Fundus changes include: • Complications include posterior synechiae,
– Periphlebitis-advanced stage of vascular complicated cataract and band-shaped kerato­
sheathing leads to Candle-wax drippings. pathy (commonest).
– Retinal granulomata
IV. Parasitic uveitis
– Pre-retinal nodules (Lander’s sign)
– Retinal haemorrhages (in acute sarcoid Toxoplasmosis
retinopathy) Congenital toxoplasmosis
– Choroidal granulomata • Fetus gets infestation from the involved mother
– Optic disc may show granuloma, neovasculari- through transplacental route.
zation, papilledema or optic atrophy. • Characteristic triad includes convulsions,
Behcet’s disease
calcification (intracranial), and chorioretinitis.
• Chorioretinitis at birth may be: (1) Inactive
• Typically affects young men who are positive for (more common)—bilateral punched out heavily
HLA-B5 pigmented scar at macula, or (2) Active (rare) -
• Ocular features are bilateral, recurrent, acute necrotic granulomatous retinochoroiditis.
non-granulomatous iridocyclitis associated with
transient hypopyon. Acquired toxoplasmosis
• Very rare
Vogt-Koyanagi-Harada (VKH) syndrome
• Most of the cases are subclinical (asymptomatic).
It is more common in Japanese who are usually
positive for HLA-DR4. Clinical features include: Recurrent toxoplasmic retinochoroiditis
• Cutaneous lesions—alopecia, poliosis, and vitiligo • Most common cause of a focal retinochoroiditis
• Neurological lesions are meningism, encephalo­ • Characterised by a whitish-yellow, slightly raised
pathy, tinnitus, vertigo, and deafness area near the margin of old punched out scar of
• Ocular features are chronic granulomatous anterior healed chorioretinitis
uveitis, posterior uveitis, and exudative retinal • There may be associated non-granulomatous type
detachment. of mild anterior uveitis.

III. Uveitis with arthritis Antitoxoplasmic drugs


Uveitis with ankylosing spondylitis • Clindamycin
• Spiramycin
• Ankylosing spondylitis is a common, chronic, • Pyrimethamine
sero-negative inflammatory arthritis which usually • Sulphadiazine.
involves sacroiliac and posterior intervertebral
joints Toxocariasis
• Typically affects HLA-B27 positive young males Infestation occurs in childhood by accidental
• About 30% cases develop a recurrent, unilateral, ingestion of ova of toxocara canis shed in faeces of
non-granulomatous, acute anterior uveitis. cats.
Diseases of Uveal Tract 59
Visceral larva migrans produces following ocular VII. Idiopathic specific uveitis syndromes
lesions (usually unilateral): Fuch’s uveitis syndrome (Heterochromic iridocyclitis)
• Toxocara chronic endophthalmitis which presents Unilateral mild grade, non-granulomatous, anterior
between 2 and 10 years of age as leukocoria uveitis occurring between 20 and 40 years of age,
• Posterior pole granuloma—between 5 and 15 years characterised by:
of age • Heterochromia of iris due to diffuse stromal atrophy
• Peripheral granuloma—between 6 and 40 years of • Fine KPs
age. • Paint aqueous flare
V. Fungal uveitis • No posterior synechiae
• Neovascularization of the angle
Presumed ocular histoplasmosis syndrome (POHS)
• Early development of the complicated cataract
• Presumed to be caused by histoplasma capsulatum (usually the presenting sign)
• Lesions (usually bilateral) include: • Secondary glaucoma (uncommon).
– Histospots: Atrophic spots scattered in the
Intermediate uveitis (pars planitis)
midretinal periphery
– Neovascular maculopathy ending in disciform • Patients presents with floaters or defective vision
scarring. • Eye usually looks quiet
• ‘Snow ball’ vitreous opacities in the inferior
Candidiasis quadrant, which may coalesce to form ‘snow
An opportunistic infection occuring in: banking’.
• Immuno-compromised patients (e.g., those Glaucomatocyclitis crisis
suffering from AIDS or malignancies) (Posner Schlossman syndrome)
• Patients with long-term indwelling catheter Etiology
• Drug addicts using infected needles. Typically affects young adults, 40 percent of whom
Ocular lesions
are positive for HLA-BW 54.

• Anterior uveitis associated with hypopyon Clinical features


• Multifocal chorioretinitis with Roth’s spots • White eye (no or minimal congestion)
• Endophthalmitis—characterised by ‘puff ball’ or • Cornea usually clear/mild epithelial oedema
‘cotton ball’ colonies, which joined by exudative • Fine KPs but no synechiae
strands form a ‘string of pearls’. • Dilated pupil.
• IOP: Recurrent attacks of acute rise of the intra­
VI. Viral uveitis ocular pressure (40-50 mm Hg) without shallowing
Herpes zoster uveitis of anterior chamber.
• Gonioscopy: open-angle glaucoma
• Unilateral non-granulomatous acute anterior
• Fundus shows no optic nerve cupping
uveitis occurs in about 50 percent cases with
• Visual field is normal.
herpes zoster ophthalmicus
• Complications include: Treatment
– Large segmental iris atrophy in 20 percent • Anti-glaucoma drugs
– Secondary glaucoma due to trabeculitis in 10 • NSAIDs and rarely
percent • Steroids.
– Complicated cataract may occur in late stages. HETEROCHROMIA
Acquired cytomegalovirus (CMV) retinitis Congenital causes
• Occurs in immunocompromised patients (e.g., • Waardenburg’s syndrome
patients with AIDS and those on cytotoxic therapy • Horner’s syndrome
for malignancies etc.) • Naevus of ota
• Ocular lesions includes: Cotton-wool spots, areas • Congenital ocular melanocytosis.
of retinal necrosis, and areas of vasculitis and Acquired causes
haemorrhage (sauce and cheese retinopathy) • Chronic iritis
• Complications are exudative retinal detachment, • Fuch’s heterochromic cyclitis
retinal atrophy and optic atrophy. • Iris naevus or melanoma
60 Review of OPHTHALMOLOGY

• Siderosis Glaucoma may develop with or without uveitis due


• Rubeosis to any of the following causes:
• Topical latanoprost. • Obstruction of vortex veins
• Angle blockage by forward displacement of the
WHITE-DOT SYNDROME lens iris diaphragm
Characterized by multiple white dots in fundus. • Neovascularization of the angle
• Infiltration of the angle by tumour cells.
Inflammatory causes
• Presumed ocular histoplasmosis syndrome (POHS) MISCELLANEOUS POINTS
• Multiple evanescent white dot syndrome (MEWDS)
• Bird-shot retinochoroidopathy • Patients with aniridia have a higher prevalence of
• Multifocal choroiditis Wilms’ tumour
• VKH syndrome • Coloboma of the uveal tract is the commonest
• Sympathetic ophthalmitis congenital anomaly of the eye
• Serpiginous choroidopathy • Typical iris coloboma occurs in the inferonasal
• HIV retinopathy quadrant. It results from failure of closure of the
• Acute posterior multifocal placoid pigment embryonic fissure
epitheliopathy (APMPPE). • Albinism results from tyrosinase deficiency
• Blue iris occurs due to the absence of pigment in
Degenerative/dystrophic causes the iris stroma
• Stargardt’s disease • Sarcoidosis and Behcet’s syndrome show increased
• Retinitis punctata albescens IgA levels
• Cystinosis
• Persistent pupillary membrane is a remnant
• Drusen.
of anterior vascular sheath of the lens. It is
Neoplastic causes characterised by the stellate-shaped shreds of the
• Leukemic retinopathy pigmented tissue attached at the collarette
• Metastatic tumours • Lesions in the choroid are restricted to isolated
• Large-cell lymphoma (Non-Hodgkin’s lymphoma). areas because of segmental blood supply to
choroid
Traumatic causes
• Purtscher’s retinopathy. • Anterior uveitis: Drug of choice is topical steroid
and second drug of choice is mydriatic and
Miscellaneous causes cycloplegic
• Choroquine or tamoxifen toxicity • Intermediate and posterior uveitis: Drug of choice
• Photocoagulation spots. is systemic steroids
• Management of uveitic glaucoma includes: Topical
MALIGNANT MELANOMA OF CHOROID steroids , mydriatics and cycloplegics
• Of the tumours of uveal tract the malignant – Topical beta blockers (first drugs of choice),
melanoma is the most common primary intraocular epinephrine or dipivefrine, apraclonidine
tumour of the adults (alpha 2 agonist),carbonic anhydrase, in­
• Extremely rare in Negroes hibitors, hyperosmotic agents
• Commonly seen between 6th and 9th decade of – Laser iridotomy when medical therapy fails.
life Note: Prostaglandin analogues (latanoprost)
• It arises from the pigment cells derived from the should not be used as they may increase
neural crest (neuroectodermal) cystoids macular edema
• May arise from the pre-existing naevus or de-novo • Juvenile Rheumatoid arthritis (pauciarticular) is
from the mature melanocytes present in the stroma associated with uveitis, cataract, ANA positive,
• Earliest pathognomic sign is appearance of orange rheumatoid factor negative. After cataract
patch due to accumulation of lipofuscin in the operations IOLs is contraindicated in JRA because
retinal pigment epithelium it act as a foreign body. In other type of uveitis, we
• Associated are intraretinal or vitreous haemorr­ put heparin coated IOLs which retard the pigment
hage. from iris.
Diseases of Uveal Tract 61
• Choroidal neovascularisation is associated with • Polypoidal choroidal vasculopathy: It is a peculiar
angioid streaks, choroidal rupture (trauma) and hemorrhagic disorder invoving macula. It is
pathological myopia. characterized by recurrent sub-retinal and sub-
• Choroidal effusion syndrome (ciliochoroidal retinal pigment epithelium bleeding in middle aged
effusion) is characterized by: black women. Indocyanine green angiography and
– Thick sclera (main feature), ciliochoroidal subsequently optical coherent tomography is used
detachment (abnormal accumulation of serous for diagnosis.
fluid in outer layer of ciliary body and choroid. • Choroideremia: Degenerative disease, involving
– Absence of inflammation and neoplastic disease. chorio­c apillaries, retinal pigment epithelium
– Eyes may be nanophalmic or hypermetropic. and photoreceptors. X linked recessive, so,
– Leopards spots i.e mottling of pigment only males are affected. First symptoms is night
epithelium. blindness.
62 Review of OPHTHALMOLOGY

Multiple Choice Questions (MCQs)

1. Most common etiological variety of uveitis is: 7. Busacca’s nodules:


A. Infective A. Are a feature of nongranulomatous uveitis
B. Allergic B. Are situated at the pupillary border
C. Toxic C. Usually initiate posterior synechiae formation
D. Metabolic D. Consists of lymphocytes, plasma cells, epitheloid
2. All of the following HLA-phenotypes are associated and giant cells
with uveitis except:
8. Festooned pupil results from:
A. HLA-B27
B. HLA-B5 A. Irregular dilatation of pupil with atropine in the
C. HLA-BW54 presence of segmental posterior synechiae
D. HLA-10 B. Annular synechiae
C. Occlusion pupillae
3. All of the following are features of granulo­matous
D. All of the above
iridocyclitis except:
A. Minimal ciliary congestion 9. Role of atropine in iridocyclitis includes all of the
B. Mutton fat KPs following except:
C. Marked aqueous flare
A. It dilates the pupil, prevents the formation of
D. Nodules on the iris
synechiae and may break the already formed
4. All of the following are features of acute non- synechiae
granulomatous iridocyclitis except: B. Gives comfort and rest to the eye by relieving
A. Marked ciliary congestion ciliary muscle spasm
B. Numerous small keratic precipitates C. It reduces exudation by decreasing hyperaemia
C. Minimal aqueous flare D. It lowers the intraocular pressure increasing
D. No iris nodules
aqueous outflow facility
5. Aqueous flare seen in anterior chamber is due to:
10. Granulomatous uveitis is seen in all of the
A. Leakage of protein particles into the aqueous
following except:
humour following breakdown of blood aqueous
barrier A. Uveitis with ankylosing spondylitis
B. Leakage of leukocytes into anterior chamber B. Sympathetic ophthalmitis
C. Both of the above C. Tubercular uveitis
D. None of the above D. Uveitis in sarcoidosis

6. Koeppe’s nodules: 11. Dalen Fuch’s nodules are pathognomic of:


A. Are a feature of nongranulomatous iridocyclitis A. Pathological myopia
B. Are situated at the pupillary border B. Sympathetic ophthalmitis
C. Consist of polymorphonuclear cells C. Fuch’s uveitis syndrome
D. Do not initiate posterior synechiae formation D. Sarcoidosis

1 : B 2 : D 3 : C 4 : C 5 : A 6:B 7 : D 8 : A 9 : D 10 : A 11 : B
Diseases of Uveal Tract 63
12. A healed patch of chorioretinitis gives rise to: 21. The following features of panophthalmitis
A. Negative scotoma differentiate it from endophthalmitis except:
B. Positive scotoma A. Presence of pus in the anterior chamber
C. Both of the above B. Proptosis
D. None of the above C. Limited and painful ocular movements
D. Complete loss of vision
13. ‘Snow ball’ opacities near the ora-serrata are
pathognomonic of: 22. In a patient with suspected tubercular anterior
A. Fungal endophthalmitis uveitis the diagnosis is confirmed by:
B. Pars planitis A. Positive Mantoux test
C. Diabetic retinopathy B. Associated findings suggestive of old systemic
D. Anterior choroiditis tuberculosis
C. A positive response to isoniazid test
14. Sign of activity in chronic iridocyclitis is: D. All of the above
A. Aqueous cells
23. ‘Iris-pearls’ are seen in:
B. Aqueous flare
A. Syphilis
C. Pigmented KPs
B. Leprosy
D. All of the above
C. Sarcoidosis
15. The pathognomonic sign of acute iridocyclitis is: D. Tuberculosis
A. Small pupil 24. Heerfordt’s disease is characterised by all of the
B. Aqueous flare following except:
C. Keratic precipitates A. Unilateral non-granulomatous panuveitis
D. All of the above B. Painful enlargement of parotid glands
16. ‘Puff-balls’ opacities in the vitreous are C. Cranial nerve palsies
pathognomonic of: D. Skin rashes, fever and malaise
A. Fungal endophthalmitis 25. Behcet’s disease is characterised by all of the
B. Pars planitis following except:
C. Exudative retinopathy of Coats A. Unilateral granulomatous uveitis
D. Sympathetic ophthalmitis B. Recurrent hypopyon
C. Aphthous ulceration
17. In bacterial endophthalmitis systemic steroids
D. Genital ulcerations
should be
A. Started immediately 26. All of the following are true for Vogt-Koyanagi-
B. Started after 12–24 hours of intensive antibiotic Harada syndrome except:
therapy A. More common in Japanese people, who are
C. Deferred usually positive for HLA-B27
D. Given after 7 days of intensive antibiotic therapy B. Ocular features are; Chronic granulomatous
anterior uveitis, posterior uveitis and exudative
18. The cell type most typically seen in the Keratic retinal detachment
precipitates of non-granulomatous uveitis is: C. Cutaneous lesions are alopecia, poliosis and
A. Polymorphonuclear cells vitiligo
B. Lymphocytes D. Neurological lesions include, meningism,
C. Plasma cells encephalopathy, tinnitis, vertigo and deafness
D. Epitheloid cells
27. Reiter’s syndrome is typically characterised by:
19. Iritis roseata is seen in: A. Urethritis, conjunctivitis and iridocyclitis
A. Leprosy B. Arthritis, conjunctivitis and iridocyclitis
B. Syphilis C. Urethritis, arthritis and conjunctivitis with or
C. Tuberculosis without iridocyclitis
D. Sarcoidosis D. All of the above
20. Heterochromia iridis is a feature of: 28. Recurrent toxoplasmic retinochoroiditis, all are
A. Malignant melanoma of iris true except:
B. Sympathetic paralysis A. Manifests at an average age of 25 years
C. Glaucomatocyclitis crisis B. The infestation is acquired by eating the under-
D. Siderosis bulbi cooked meat of intermediate host containing cyst
E. All of the above of the parasite

12 : A 13 : B 14 : A 15 :C 16 : A 17 : B 21 : A 22 : C 23 : B 24 : A 25 : A 26 : A
18 : B 19 : B 20 : E 27 : C 28 : B
64 Review of OPHTHALMOLOGY

C. Typical lesion is a patch of focal necrotizing 36. Malignant melanoma of the choroid of following
retinochoroiditis adjacent to a pigmented scar histological features has got the best prognosis:
D. There may be associated iritis A. Epitheloid cell melanoma
B. Spindie - A melanoma
29. All of the following are true for the acquired
C. Spindle - B melanoma
cytomegalo inclusion disease except:
D. Mixed cell melanoma
A. It occurs only in immunosuppressed patients
B. The infection is acquired from the infected cervix 37. Malignant melanoma of the choroid with
of the partner during sexual inter­course following histopathological features has got the
C. Typical lesion is acute necrotizing retinitis worst prognosis:
D. Exudative retinal detachment may occur A. Epitheloid cell melanoma
30. Granulomatous uveitis with involvement of B. Spindle - A melanoma
parotid gland is seen in: C. Spindle - B melanoma
A. Tuberculosis D. Mixed cell melanoma
B. Syphilis 38. The most common histopathological type of
C. Mumps malignant melanoma of choroid is:
D. Sarcoidosis A. Spindle - A cell
31. All of the following are true for colloid bodies B. Spindle-B cell
(drusens) except: C. Epitheloid cell
A. Are hyaline excrescences of Bruch’s membrane D. Mixed cell
B. Are secreted by pigment epithelial cells
39. Essential atrophy of the choroid is due to inborn
C. Are usually associated with marked loss of
error of metabolism of which amino acid?
vision
D. Are a percursor of disciform maci degeneration A. Cystine
in some eyes B. Cysteine
C. Arginine
32. In clinical aniridia all of the following are true D. Ornithine
except: E. Lysine
A. Iris is completely absent
40. The earliest symptom of sympathetic ophthalmitis
B. Usually familial condition
is:
C. May be associated with congenital glaucoma
D. Ciliary processes may be visible A. Pain
B. Decreased distant vision
33. Typical coloboma of iris occurs: C. Photophobia
A. Inferonasally D. Diplopia
B. Superonasally
41. The most earliest sign of anterior uveitis is:
C. Inferotemporally
D. Superotemporally A. Aqueous flare
B. Keratic precipitates
34. Malignant melanoma of conjunctiva is usually of: C. Constriction of pupil
A. Spindle A type D. Raised intraocular pressure
B. Spindle B type
C. Mixed cell type 42. Separation of rods and cones due to exudative
D. Epitheloid cell type choroiditis causes:
E. All of the above A. Photopsia
B. Micropsia
35. Malignant change in a choroidal nevus is evidenced C. Macropsia
by: D. Metamorphopsia
A. Increased pigmentation or height of the nevus
B. Appearance of orange patches of lipofusin over 43. Commonest cause of endogenous uveitis in India
the surface is:
C. Appearance of serous detachment in the area of A. Tuberculosis
nevus B. Leprosy
D. All of the above C. Syphilis
E. None of the above D. Diabetes mellitus

29 : B 30 : D 31 : C 32 : A 33 : A 34 : E 36 : B 37 : A 38 : D 39 : D 40 : C 41 : A
35 : D 42 : B 43 : A
Diseases of Uveal Tract 65
44. The most frequent bacterial agent causing non- C. TORCH agent test
granulomatous uveitis is: D. USG abdomen
A. Staphylococcus E. ELISA for HIV
B. Streptococcus 53. Uveitis is caused by all except:
C. Pneumococcus A. T.B.
D. Influenza bacillus B. Staphylococcus
E. None of the above C. Streptococcus
D. Klebsiella
45. Peripheral anterior synechiae after an attack of acute
congestive glaucoma occur earliest in the: 54. Photopsia occurs in:
A. Lower part A. Iritis
B. Upper part B. Choroiditis
C. Lower and inner part C. Cyclitis
D. Lower and outer part D. Scleritis
E. Medial part
55. ‘Snow banking’ is typically seen in:
46. In complete albinism the colour of iris is: A. Pars planitis
A. White B. Endophthalmitis
B. Black C. Coat’s disease
C. Pink D. Eales’ disease
D. Blue
E. Green 56. May masquerade as uveitis:
A. Choroidal detachment
47. Iris bombe occurs is: B. Retinal detachment
A. Ring synechiae
C. Age related macular degeneration
B. Anterior synechiae
D. Central retinal artery occlusion
C. Posterior synechiae
D. All 57. Commonest cause of fungal uveitis is:
A. Candida
48. Drug of choice for acute iridocyclitis is:
A. Steroids B. Aspergillus
B. Acetazolamide C. Fusarium
C. Atropine D. Mucor mycosis
D. Antibiotics 58. All are causes of white-dot syndrome except:
49. The correct statement regarding the duration after A. Fuch’s heterochromic uveitis
which sympathetic ophthalmitis develops is: B. VKH syndrome
A. 3 wks–12 wks after trauma C. HIV retinopathy
B. Within 1 week of trauma D. Sympathetic ophthalmitis
C. After 2 months of trauma 59. Most common cause of anterior uveitis associated
D. Typically at 10 days after trauma with arthritis:
50. One of the most common complication of A . Ankylosing spondylitis
iridocyclitis is: B. Rheumatoid arthritis
A. Scleritis C. Syphilis
B. Secondary glaucoma D. Tuberculosis
C. Band-shaped keratopathy
60. 1st sign of anterior uveitis:
D. Corneal ulcer
A. Keratic precipitate
51. First sign of sympathetic ophthalmitis is: B. Aqueous flare
A. Keratic precipitates C. Hypopyon
B. Miosis D. Miosis
C. Aqueous flare
61. Keratic precipitates are on which layer of
D. Retrolental flare
cornea:
52. The investigations of anterior uveitis for a 25 years A. Epithelium
old boy are all except: B. Endothelium
A. HLA-B27 C. Stroma
B. X-ray sacroiliac joint D. Bowman’s membrane
44 : B 45 : B 46 : C 47 : A 48 : C 49 : A 53 : D 54 : B 55 : A 56 : B 57 : A 58 : D
50 : B 51 : D 52 : D 59 : A 60 : B 61 : B
66 Review of OPHTHALMOLOGY

62. Which of the following indicates activity of ant 70. Signs of uveitis:
uveitis: A. Generalized conjunctival congestion
A. Cells in anterior chamber B. Circumciliary congestion
B. Circumcorneal congestion C. Cells and flare in aqueous
C. Keratic precipitate D. Keratic precipitate
D. Corneal edema
71. A 25 year old lady presents with severe sudden
63. True about heterochromic uveitis: onset of pain, corneal congestion, photophobia
A. Involves posterior surface of iris and deep anterior chamber in the right eye. The
B. Involves anterior part of iris left eye is normal. X-ray pelvis shows sacroilitis.
C. Involves posterior chamber The diagnosis is:
D. Posterior synechiae A. Anterior uveitis
B. Posterior uveitis
64. Vogt-Koyanagi-Harada (VKH) syndrome is: C. Intermediate uveitis
A. Chronic granulomatous uveitis D. Scleritis
B. Chronic non-granulomatous uveitis
C. Acute purulent uveitis 72. In patients with anterior uveitis, decrease in vision
D. None due to posterior segment involvement can occur
because of:
65. Histological changes in lens induced uveitis
A. Visual floaters
include:
B. Inflammatory disc edema
A. Ghost cells
C. Exudative retinal detachment
B. Giant cell reaction D. CME
C. Amyloid in the Cornea
D. Vasculitis 73. Mutton for keratic precipitate and Busacca’s
nodules is seen in:
66. Drug used in LUMINATE program for non- A. Granulomatous uveitis
infectious uveitis is: B. Non-granulomatous uveitis
A. Cyclosporine C. Posterior uveitis
B. Voclosporin D. Choroiditis
C. Methotrexate
D. Infliximab 74. Uveal effusion syndrome may be associated with
all of the following except:
A. Myopia
EXTRA EDGE QUESTIONS
B. Ciliochoroidal detachment
67. Granulomatous uvetis is seen in: C. Structural defect in Sclera
A. Vogt-Koyanagi-Harada disease D. Nanophthalmos
B. Fuch’s disease 75. In heterochromic cyclitis:
C. Bechet’s syndrome A. 60% of patients develop glaucoma
D. Sarcoidosis B. Show a good response when treated with steroids
E. Psoariasis C. Lens implantation following cataract surgery is
contraindicated
68. A 10 year boy present with b/l hronic uveitis. Which
D. Hyphaema during cataract surgery is due to iris
investigation should be ordered:
neovascularization
A. Hemogram
B. X-ray of sacroiliac joint 76. A young patient presents to the ophthalmic
C. HIV rest outpatient department with gradual blurring
D. Mantoux test of vision in the left eye. Slit lamp examination
reveals fine stellate keratatic precipitates and
69. The investigation of anterior uveitis of a young aqueous flare and a typical complicated posterior
boy are: subcapsular cataract. No posterior synechiae were
A. HLA B 27 observed. The most likely diagnosis is:
B. X-ray of sacroiliac joint A. Intermediate Uveitis (Pars plants)
C. TORCH agents B. Heerford’s disease
D. ELISA for HIV C. Heterochromic iridocyclitis of Fuch’s
E. USG abdomen D. Subacute Iridocyclitis

62 : A 63 : B 64 : A 65 : B 66 : B 70 : B, C and D 71 : A 72 : C 73 : A 74 : A
67 : A and D 68 : D 69 : A and B 75 : D 76 : C
Diseases of Uveal Tract 67
77. Which of the following drug is currently used C. Macular degeneration
for the prophylaxis of non infectious uveitis in D. Fuch’s uveitis
LUMINATE program:
A. Cyclosporine 79. The use of highly active anti-retroviral therapy
B. Vaclosporine (HAART) is associated with the development
C. Methotrexate of:
D. Infliximib A. Keratitis
78. Amsler’s sign is seen in: B. Uveititis
A. Posner-Schlossman syndrome C. Retinitis
B. Pars planitis D. Optic neuritis

77 : B 78 : D 79 : D
CHAPTER

8
Diseases of Lens

Quick Text Review


CATARACT 6. Coronary cataract
• Occurs at puberty and thus involves the adolescent
CONGENITAL AND DEVELOPMENTAL CATARACT nucleus or deeper cortex
• Characterised by club-shaped radiating opacities
1. Cataracta centralis pulverulenta
involving the periphery (so vision is usually un­
• It is also known as embryonic nuclear cataract affected).
• Bilateral small rounded opacity with powdery
appearance lying in the centre (embryonic 7. Blue-dot cataract (Cataracta-punctata cerulea)
nucleus) Stationary, rounded, bluish, punctate opacities
• Hereditary with dominant genetic trait. involving adult nucleus or deep cortex. In minute
degrees, it is almost universal in occurrence.
2. Lamellar (Zonular) cataract
Develops in second decade of life. It does not
• Commonest type, accounting for 50 percent of involve vision.
visually significant cases
8. Total cataract
• Usually bilateral, opacity involves a zone of foetal
nucleus around the embryonic nucleus with It may be hereditary or associated with rubella,
peripheral riders (spokes of a wheel). either unilateral or bilateral.
• May be genetic or due to vitamin-D deficiency 9. Congenital membranous cataract
or maternal rubella between 7 and 8 weeks of Occurs due to partial or total absorption of lens
gestation. Hypoparathyroidism during pregnancy matter.
may also cause this type of cataract.
Management of Congenital cataract
3. Sutural cataract
Usually static, bilateral, punctate opacities scattered Surgical treatment in the form of lens aspiration
around the Y-shaped sutures, of different patterns, or lensectomy with posterior capsulotomy and
(floriform, coralliform, spear shaped and anterior anterior vitrectomy is the procedure of choice.
axial embryonic cataract). Timing of surgery
4. Anterior polar cataract • Bilateral cataract, in dense early surgery by 4-6
• It is also known as pyramidal cataract weeks of age and in partial, it may not be required
• Occurs due to delayed formation of anterior until later if at all.
chamber or acquired following corneal perforation • Unilateral cataract. In dense cataract, urgent
in childhood or opthalmia neonatorum. surgery within days is required. Partial cataract
• Morphological types are: Thickened white plaque, can be observed and treated nonsurgically by part
anterior pyramidal cataract and reduplicated time occlusion to prevent amblyopia.
cataract or double cataract (the buried opacity is Visual prognosis is very poor in unilateral advanced
called ‘Imprint’). cataract because of dense stimulus deprivation
5. Posterior polar cataract amblyopia.
Occurs due to persistence of posterior vascular Correction of paediatric aphakia above two
capsule of the lens. years with posterior chamber intraocular lens
Diseases of Lens 69
implantation, and below two years with extended may be prevented by early diagnosis and elimination
wear contact lens, or spectacles in bilateral cases. of milk from the diet.

ACQUIRED CATARACTS
3. Hypocalcaemic cataract may be associated
with parathyroid tetany.
I. Senile cataract (age-related cataract)
4. Sunflower cataract may be associated with
• It is the most common form of cataract. inborn error of copper metabolism (Wilson’s
• Senile cataract is an age change. Factors affecting its disease).
onset and maturation include: Heredity, exposure
to ultraviolet radiations from sunlight, diet, and 5. Cataract may be seen in Lowe’s (oculocerebral-
dehydrational crisis in childhood. renal) syndrome; an inborn error of amino acid
• The main biochemical features of cortical senile metabolism. Other ocular features are micro­phakia,
cataract are decreased levels of total proteins, posterior lentiglobus and glaucoma.
amino acids and potassium associated with III. Complicated cataract
increased concentration of sodium and marked • It may occur secondary to uveitis, retinitis
hydration of the lens. While nuclear cataract pigmentosa, myopic chorioretinal degeneration
is accompanied by a significant increase in and long standing retinal detachment.
water insoluble proteins which give brown • Posterior subcapsular cataract is typically
colour to nucleus. Intensification of age-related characterised by polychromatic lustre and bread-
nuclear sclerosis associated with dehydration and crumb appearance.
compaction of nucleus resulting in hard cataract.
• In a fully developed cataract, glutathione, ascorbic IV. Toxic cataracts
acid and riboflavin are deficient or absent. These 1. Corticosteroid-induced cataract: Posterior sub­
are the principal agents mediating the auto- capsular opacities may be associated with the use
oxidative system of the lens. of topical as well as systemic steroids.
• Cuneiform cataract accounts for 70 percent cases, 2. Miotics-induced cataract: Anterior subcapsular
cupuliform cataract for 5 percent cases and Nuclear granular cataract may be associated with the use
cataract for 25 percent cases of senile cataract. of long acting miotics such as echothiophate and
• Monoocular diplopia or polyopia (incipient stage) demecarium bromide.
occurs in cuneiform cataract. 3. Other causes of toxic cataracts are chlorpromazine,
• Cupuliform (posterior subcapsular) cataract lies amiodarone, gold (used for treatment of
right in the pathway of axial rays, and thus, causes rheumatoid arthritis, and busulphan (used for
an early loss of vision. treatment of chronic myeloid leucaemia).
• Second sight or myopic shift is seen in nuclear V. Radiational cataract
cataract.
1. Infra-red (heat) cataract. It typically occurs as
II. Metabolic cataracts discoid posterior subcapsular opacities in workers
1. Diabetic cataract: The true diabetic cataract, of glass industry, hence the name ‘glass-blower’s
usually occuring in young adults is characterised by cataract’.
acute onset, often bilateral, ‘snow-flake opacities’. 2. Irradiation cataract: It may follow exposure to
Accumulation of ‘Sorbitol’ due to NADPH+ X-rays, γ-rays or neutron.
dependent aldose reductase pathway is primarily 3. Ultraviolet radiation has been linked with senile
responsible for development of true diabetic cataract.
cataract. Fluctuating refractive error is a feature of • Most common type of radiational cataract is
diabetic cataract. posterior subcapsular cataract (PSC).
2. Galactosaemic cataract (oil droplet cataract) VI. Electric cataract
is associated with inborn error of galactose It may occur following passage of powerful electric
metabolism due to deficiency of galactose-1- current through the body. Punctate subcapsular
phosphate uridyltransferase (GPUT). A related opacities which mature rapidly.
disorder occurs due to deficiency of galactokinase
VII. Syndermatotic cataract
causing lamellar cataract. Accumulation of
‘dulcitol’ is primarily responsible for development It is associated with skin disorders like atopic
of galactosaemic cataract. Development of cataract dermatitis, scleroderma and keratosis.
70 Review of OPHTHALMOLOGY

VIII. Traumatic cataract – Rollable IOLs are ultrathin lenses for implan­
Traumatic cataract is flower rosette shaped cataract. tation through microincision (1 mm) after
phakonit technique.
IX. Miscellaneous cataract
• Primary IOL implantation refers to the use of
• Christmas tree cataract is seen in myotonic IOL during surgery for cataract, while secondary
dystrophy IOL is implanted to correct aphakia in previously
• Shield cataract occurs in atopic kerato­conjunctivitis operated eye.
• Neurofibromatosis (NF2) is associated with • Calculation of IOL power is done by SRK formula:
posterior subcapsular cataract. P = A – 2.5 L – 0.9 K; where P = IOL power in diopters.
Management of cataract in adults A = specific constant of IOL, L = axial length of
Extracapsular cataract extraction (ECCE) with eyeball in mm and K = average keratometric
posterior chamber IOL is the treatment of choice. reading.
• Incidence of post-operative complications such
as endophthalmitis, cystoid macular oedema DISPLACEMENTS OF LENS
and retinal detachment is comparatively low • On distant direct ophthalmoscopy, edge of the clear
with extracapsular cataract extraction (ECCE) as subluxated lens is seen dark due to total internal
compared to intracapsular cataract extraction reflection of the light.
(ICCE). • Anterior dislocation (in anterior chamber): Clear
• Phacoemulsification is a technique of ECCE. lens is seen as an oil drop in the aqueous.
Surgical steps include a 3.2 mm clear corneal • Posterior dislocation (in vitreous humor): lens may
volvular incision, continuous cur vilinear be floating in the vitreous (lens nutans) or fixed to
capsulorrehexis, hydrodissection, emulsification the retina (lens fixata).
and aspiration of nucleous using ultrasonic • Simple ectopia lentis: Displacement is bilaterally
phacoprob e. The phaco ne e dle vibrates
symmetrical and usually upwards. Autosomal
longitudinally at an ultrasonic speed of 40,000
dominant inheritance.
times per second.
• Ectopia lentis et pupillae: Displacement of
• Femtosecond laser assisted cataract surgery
(FLACS) is the most advanced technique of MICS. lens is associated with slit-shaped pupil. Other
• Manual small incision cataract surgery (SICS) associations may be cataract, glaucoma and retinal
is a low-cost alternative to phacoemulsification detachment.
which offers the advantages of sutureless cataract • Ectopia lentis with systemic anomalies include:
surgery with the added advantages of having wider Marfan’s syndrome: Lens is subluxated upward
applicability and an easier learning curve. and temporally in both eyes.
• Homocystinuria: Lens is subluxated downward.
Intraocular lens implantation is the best method
• Well-Marchesani syndrome: Characterised by
for correction of aphakia.
spherophakia and anterior subluxation of lens.
• Anterior chamber IOLs: These lie entirely in front
• Ehlers–Danlos syndrome. Subluxated lens may
of the iris and are supported in the angle. These
are not much popular due to comparatively higher be associated with blue sclera, keratoconus and
incidence of bullous keratopathy. Commonly used angioid streakes.
IOL is Kelman multiflex lens. • Consecutive or spontaneous displacements as seen in
• Iris supported lenses: These are fixed on the iris with hypermature cataract, buphthalmos, high myopia,
the help of sutures, loops or claws. These are also intraocular tumours, and chronic cyclitis.
not popular due to higher rate of complications.
• Posterior chamber lenses: These lie behind the iris CONGENITAL ANOMALIES OF LENS
and may be supported by ciliary sulcus or capsular • Coloboma of the lens: A notch usually seen in the
bag. These are very popular and are available in inferior quadrant of the equator. Occurs due to
modified C-loop and other designs. defective development of the suspensory ligament
Three types of PClOLs available are: in that part.
– Rigid IOLs are made of PMMA • Anterior lenticonus: Cone-shaped anterior axial
– Foldable IOLs are made of silicone, acrylic bulge. May occur in Alport’s syndrome.
or hydrogel for implantation through a small • Posterior lenticonus: Posterior axial bulge may
(3.2 mm) incision. occur in Lowe’s syndrome.
Diseases of Lens 71
• Micro-spherophakia: A small spherical lens may • Zonular or lamellar cataract is the commonest
occur as an isolated finding or as a feature of Well congenital cataract causing visual impairment
Marchesani’s or Marfan’s syndrome. (otherwise blue dot cataract is the commonest
• Microphakia is a small lens which occur in Lowe’s congenital cataract).
syndrome. • Most common postoperative complication of extra­
• L entiglobus : Generalized hemi-spherical capsular cataract extraction is posterior capsule
deformity. thickening.
• The best method to decide about the immaturity
SOME SALIENT POINTS and maturity of senile cataract is distant direct
ophthalmoscopy.
• The lens is incapable of becoming inflamed due to • The visual loss in posterior polar cataract is much
the capsule. more than the anterior polar cataract, because the
• The most common manifestation of develop­ former is close to the nodal point of the eye.
mental cataract is punctate cataract (blue dot • Cryoextraction is the safest method for intra­
cataract). capsular technique in intumescent cataract.
• Presenile cataract occurs in patients with atopic • Cupuliform or posterior cortical cataract seldom
dermatitis (stellate opacities mostly posterior), matures.
dsytrophica myotonia (christmas tree cataract), • Down syndrome. Ocular features include: Kerato­
and GPUT and GK enzyme deficiency. conus, bruschfield spot, and blue dot cataract.
72 Review of OPHTHALMOLOGY

Multiple Choice Questions (MCQs)

1. Is the most common type of congenital cataract: D. Diffuse lens changes


A. Lamellar cataract E. None of the above
B. Cataracta centralis pulverulenta
8. The type of cataract seen in Wilson’s disease is:
C. Coronary cataract
A. Sunflower cataract
D. Coralliform cataract
B. Snowflake cataract
2. Visual prognosis is poor in: C. Posterior subcapsular cataract
A. Bilateral congenital cataract D. Coronary cataract
B. Unilateral congenital cataract
9. All are the causes of complicated cataract except:
C. Zonular cataract
A. Pars planitis
D. Cataract pulverulenta
B. Retinitis pigmentosa
3. The most common type of senile cataract is: C. Retinal detachment
A. Cuneiform cataract D. Posterior vitreous detachment
B. Cupuliform cataract
10. In a patient with senile cataract the macular
C. Nuclear cataract
functions can be tested by all except:
D. None of the above
A. Two-light discrimination test
4. Complete unilateral congenital cataract should B. Swinging flash light test
preferably be operated: C. Maddox rod test
A. Within a few weeks of birth D. Laser interferometry
B. At the age of 6 months
11. Elschnig’s pearls are a sign of:
C. At the age of 2 years
A. Chronic uveitis
D. At the age of 5 years
B. Secondary cataract
5. Immature senile cataract can be best differentiated C. Cystoid macular oedema
from mature senile cataract by: D. All of the above
A. Iris shadow test
12. Lens subluxation occurs in all except:
B. Oblique illumination examination
A. Homocystinuria
C. Visual acuity testing
B. Ehlers-Danlos syndrome
D. Distant direct ophthalmoscopy
C. Congenital syphilis
E. Pin-hole test
D. Congenital rubella
6. A true diabetic cataract is also known as:
A. Sunflower cataract 13. All are true except:
B. Rosette-shaped cataract A. The infantile nucleus is completely formed by
C. Snow-storm cataract one year of age
D. Coronary cataract B. The embryonic nucleus is situated between the
two Y sutures
7. The early changes in corticosteroid-induced C. Congenital blue dot cataracts are associated with
cataract are in the form of: development of senile cataract at an early stage
A. Central posterior subcapsular lens changes D. Zonular cataracts typically affect the outer part
B. Anterior cortical lens changes of the fetal or the inner part of the adult nucleus
C. Nuclear changes

1 : A 2 : B 3 : A 4 : A 5 : D 6 : C 8 : A 9 : D 10 : B 11 : B 12 : D 13 : A
7:A
Diseases of Lens 73
14. Anterior lenticonus may be associated with: 23. Lens capsule is thinnest at the:
A. Alport’s syndrome A. Central anteriorly
B. Lowe’s syndrome B. Laterally
C. Marfan’s syndrome C. Superior pole of the lens
D. Homocystinuria D. Posterior pole of the lens

15. During IOL implantation, corneal endothelial 24. Cataract brunescens result due to deposition of:
damage can be prevented by use of: A. Melanin
A. Methyl cellulose B. Copper
B. Sodium hyaluronate C. Iron
C. Chondroitin sulfate D. Silver
D. All of the above E. Gold

16. Is responsible for presenile cataract: 25. Polyopia is a symptom of:


A. Cortical cataract
A. Atopic dermatitis
B. Cupuliform cataract
B. Blue dot congenital cataract
C. Dystrophica myotonica C. Radiational cataract
D. None of the above D. Electrical cataract
E. All of the above 26. All of the following are features of Ehler-Danlos
syndrome except:
17. Hyperlysinemia may be associated with:
A. Blue sclera
A. Subluxation of the lens B. Long stature
B. Spherophakia
C. Subluxation of lens
C. Strabismus
D. Epicanthal folds
D. All of the above
E. Keratoconus
18. Most common complication following extra­ 27. Ideal site for intraocular lens implantation is:
capsular cataract extraction is: A. Anterior to the pupil
A. After cataract B. Behind the cornea
B. Corneal endothelial decompensation C. In the lens capsule
C. Secondary glaucoma D. Behind the lens capsule
D. Cystoid macular oedema
28. Developmental cataract is seen in:
19. Expulsive choroidal haemorrhage is caused by A. Rubella
rupture of: B. Galactosemia
A. Retinal vessels C. Mongolian idiocy
B. Short posterior ciliary arteries D. Cretinism
C. The choriocapillaris E. All of the above
D. Long posterior ciliary arteries
29. Rosette cataract is seen due to:
20. Cataract in newborn is: A. Trauma
A. Zonular
B. Copper foreign body
B. Coronary
C. Diabetes
C. Snowflake
D. Hyperparathyroidism
D. Cortical
21. During cataractogenesis lens stria appear at first 30. True about zonular cataract is:
in: A. Bilateral
A. Upper nasal quadrant B. Stationary
B. Lower nasal quadrant C. Autosomal dominant
C. Upper temporal quadrant D. Association with hypocalcemia
D. Lower temporal quadrant E. All of the above

22. A mature uniocular cataract in a 3-year-old child: 31. The commonest type of cataract in adults is:
A. Will require refraction A. Nuclear cataract
B. May be absorbed B. Cortical cataract
C. Should be removed C. Morgagnian cataract
D. Should be left as such D. None of the above

14 : A 15 :D 16 : E 17 : D 18 : A 19 : B 23 : D 24 : A 25 : A 26 : B 27 : C 28 : E
20 : A 21 : B 22 : C 29 : A 30 : E 31 : B
74 Review of OPHTHALMOLOGY

32. The commonest side effect of lens implant surgery 41. Christmas tree cataract is seen in:
is: A. Down’s syndrome
A. Vitreous haemorrhage B. Rubella
B. Glaucoma C. Myotonic dystrophy
C. Iridocyclitis D. Diabetes
D. Panophthalmitis 42. All are the advantages of leaving the capsule
behind in cataract surgery except:
33. Most common type of cataract following radiation A. Prevents cystoid macular edema
is: B. Decreases endothelial damage
A. Posterior subcapsular C. Progressively improves vision
B. Anterior subcapsular D. Decreases chance of retinal detachment
C. Tear-drop cataract E. Decreases chance of endophthalmitis
D. Diffuse cataract
43. Cataract is caused by all except:
34. Lens has a respiratory quotient of: A. Ultraviolet radiation
A. 1 B. MRI
B. 0.6 C. Infrared radiation
C. 0.7 D. Microwave radiation
E. Ionizing radiation
D. 0.9
44. In preoperative assessment of cataract patient
35. Causes of early onset of cataract are all except: following is to be done:
A. Diabetes mellitus A. Axial length of cornea
B. Smoking B. Corneal thickness
C. Trauma C. Thickness of the lens
D. Recurrent episodes of diarrhoea D. Corneal curvature
36. Cataract is responsible for what percentage of 45. Modern IQL is not made up of:
blindness in India: A. Acrylic acid
A. 62% B. PMMA
B. 55% C. Silicon
C. 80% D. Glass
D. 75% 46. Second sight phenomenon is seen in:
A. Nuclear cataract
37. Most common complication of extracapsular
B. Cortical cataract
cataract surgery is:
C. Senile cataract
A. Retinal detachment D. Iridocyclitis
B. Opacification of posterior capsule
C. Vitreous haemorrhage 47. A child has got a congenital cataract involving the
D. Bullous keratopathy visual axis which was detected by the parents right
at birth. This child should be operated:
38. Dislocation of lens is seen in: A. Immediately
A. Retinoblastoma B. At 2 months of age
B. Medulloblastoma C. At 1 year of age when the globe becomes normal
C. Neuroblastoma sized
D. None of the above D. After 4 years when entire ocular and orbital
growth becomes normal
39. In a patient, highest visual morbidity is seen in: 48. Best site for IOL implant is:
A. Nuclear cataract A. Endocapsular
B. Intumescent cataract B. Scleral fixation
C. Posterior subcapsular cataract C. Anterior chamber
D. Anterior subcapsular cataract D. Iris claw implant
40. Commonest type of cataract is: 49. Congenital morgagnian cataract is a feature of:
A. Blue dot A. Rubella cataract
B. Zonular B. Lowe’s syndrome
C. Cupuliform C. Hereditory cataract
D. Cuneiform D. Galactosemic cataract

32 : C 33 : A 34 : A 35 : B 36 : A 37 : B 41 : C 42 : C 43 : B 44 : D 45 : D 46 : A
38 : A 39 : C 40 : A 47 : A 48 : A 49 : A
Diseases of Lens 75
50. IOL preferred in children in: 58. Which of the following is the most important
A. Foldable acrylic lens factor in the prevention of the endophthalmitis
B. Foldable silicon lens in cataract surgery?
C. Three piece PMMA lens A. Preoperative preparation with povidone iodine
D. Single piece PMMA lens B. One week antibiotic therapy prior to surgery
C. Trimming of eyelashes
51. The capsule of the crystalline lens is thinnest at: D. Use of intravitreal antibiotics
A. Anterior pole
B. Posterior pole 59. A patient presents to the emergency department
with uniocular diplopia. Examination with
C. Equator
oblique illumination shows golden crescent while
D. None
examination with co-axil illumination show a dark
52. The crystalline lens derives its nourishment from: crescent line. Which of the following is the most
A. Blood vessels likely diagnosis:
B. Connective tissue A. Lenticonus
C. Aqueous and vitreous B. Coloboma
D. Zonules C. Microspherophakia
D. Ectopia lentis
53. The major function of major intrinsic protein-26 60. In Mafan’s syndrome lens dislocation is commonly
(MIP-26) is: seen:
A. Glucose transport in lens A. Upwards
B. Transport of water in lens B. Downwards
C. Diffusion barrier C. Supero-temporally
D. Capsule of lens D. Nasally

54. Most common type of congenital cataract is: 61. Which of the following is the only reversible
cataract:
A. Capsular
A. Senile cataract
B. Zonular B. Cataract in galactosemia
C. Coralliform C. Congenital cataract
D. Blue dot D. None
55. In Cataract, spectacles are advised for following 62. Minimum vision loss with which cataract:
number of weeks after operation: A. Blue dot cataract
A. 6 weeks B. Zonular cataract
B. 10 weeks C. Anterior polar cataract
C. 12 weeks D. Posterior polar cataract
D. 14 weeks 63. In preoperative assessment of cataract patient
following is to be done:
56. Which laser is used in the management of after A. Axial length of eyeball
cataracts: B. Corneal thickness
A. Argon C. Keratometry
B. Krypton D. Thickness of the lens
C. Nd-YAG E. Corneal curvature
D. Excimer
64. Modern IOL is:
A. Acrylic acid
EXTRA EDGE QUESTIONS B. PMMA
C. PML
57. An infant present with bilateral white pupillary D. Silicon
reflex. On slit lamp examination a zone of E. Glass
opacity is observed around the fetal nucleus
with spoke like radial opacities. The most likely 65. Complications of cataract surgery:
diagnosis is: A. Endophthalmitis
A. Cataracto Centralis Pulverulenta B. Optic neuropathy
B. Lamellar cataract C. Retinal detachment
C. Coronary cataract D. Vitreous loss
D. Posterior polar cataract E. Lagophthalmos

50 : A 51 : B 52 : C 53 : B 54 : D 55: A 58 : A 59 : D 60 : C 61 : B 62 : A
56 : C 57 : B 63 : A, C and E 64 : A, B and D 65 : A, C and D
76 Review of OPHTHALMOLOGY

66. Dislocation of lens is seen in all the following B. Cornea


conditions except: C. Retina
A. Congenital rubella D. Optic nerve
B. Marchesani’s syndrome
C. Marfan syndrome 69. Branching posterior spoke like cataracts (Pro­
D. Homocystimuria peller) are seen in:
A. Down’s syndrome
67. Ectopia lentis is seen in: B. Fabry’s disease
A. Marfan’s syndrome C. Neuro fibromatosis
B. Congenital rubella D. Atopic keratoconjunctivitis
C. Homocystinuria
D. Sulfite oxidase deficiency 70. Lens contain which antigen?
E. Myotonic dystrophy A. Sequestered antigens
68. Which ocular structures is the most radio­ B. Cross antigens
sensitive: C. Heterophile antigens
A. Lens D. Isoantigens

66 : A 67 : A, C, D 68 : A>B>C>D 69 : B 70 : A
CHAPTER

9
Glaucoma

Quick Text Review


AQUEOUS HUMOUR Congenital glaucoma with associated systemic
• Normal range of intraocular pressure is 10–21 mm syndromes, e.g. Sturge–Weber syndrome, Von
Hg (mean 16 ± 2.5 mm Hg). Recklinghausen’s neurofibromatosis, and Lowe’s
• Normal amount of aqueous humour present in syndrome.
the anterior chamber is 0.25 mL and in posterior Primary congenital glaucoma
chamber is 0.06 mL (total 0.31mL). • In about 40% cases, it presents since birth (New
• Normal aqueous production rate is about 2.3 µL/
born glaucoma or True congenital glaucoma)
minute.
• In about 50% cases, it manifests prior to the age of
• Aqueous humour is secreted by non-pigmented
2 years (Infantile glaucoma)
epithelium of pars plicata region of the ciliary body.
• In 10% cases, it presents between 2 and 10 years of
• Composition of aqueous is similar to plasma
the age (Juvenile glaucoma).
except that it has high concentration of ascorbate,
• When it occurs before 3 years of age, eyeball usually
pyruvate, and lactate; and low concentration
enlarges, and so the term buphthalmos is used.
of protein, urea and glucose. Concentration of
ascorbate and bicarbonate is high and that of Prevalance and genetics
chloride is low in posterior chamber aqueous as • Autosomal recessive inheritance with incomplete
compared to that in anterior chamber. penetrance
• Site of aqueous production is ciliary processes. • Affects 1 in 10,000 live births
Aqueous humour is primarily derived from the • Male : female ratio: 3:2
plasma. • Bilateral in 75% cases.
• Mechanisms concerned with aqueous production
are diffusion (10%), ultrafiltration (20%), and active Clinical features
secretion (70%). • Lacrimation (first symptom), photophobia,
• Trabecular (conventional) outflow of aqueous blepharospasm and eye rubbing.
humour accounts for 90 percent and uveoscleral • Buphthalmos (occurs with onset before the age of
outflow for 10 percent. 3 years), characterized by enlarged eyeball, corneal
diameter more than 13 mm, corneal oedema (first
PRIMARY GLAUCOMAS sign), Haab’s striae (healed splits in Descemet’s
membrane), deep anterior chamber, raised IOP,
CONGENITAL/DEVELOPMENTAL GLAUCOMA
and variable optic disc cupping. Eye becomes
It may occur in three forms: myopic.
Primary congenital glaucoma occurs due to • Gonioscopic examination may reveal: Meso­dermal
trabeculodysgenesis and is not associated with any membrane (Barkan’s membrane), thickening of
other anomaly. trabecular sheets, hypoplastic iris stroma and
insertion of iris above scleral spur.
Congenital glaucoma with other associated
ocular anomalies, such as posterior embryotoxon, Treatment
Axenfeld–Rieger syndrome, aniridia, congenital • Goniotomy is the surgery of choice (80% success
microcornea, occurs due to iridocorneal dysgenesis. rate).
78 Review of OPHTHALMOLOGY

• Trabeculotomy is required when corneal clouding • Tubular vision with a temporal island of vision
prevents visualization of angle for goniotomy. • Advanced field loss with a temporal island of vision
• Combined trabeculotomy and trabeculectomy (with only
antifibrotic treatment) is nowadays the preferred • Complete loss of vision.
sugery with better results. Ocular associations

PRIMARY OPEN-ANGLE GLAUCOMA (POAG) High myopia, Fuchs’ endothelial dystrophy, retinitis
Predisposing and risk factors
pigmentosa, central retinal vein occlusion and
primary retinal detachment.
1. Heredity: POAG has a polygenic inheritance. The Diagnosis
approximate risk of getting disease in siblings is 10
percent and in offspring is about 4 percent. 1. Established POAG: IOP more than 23 mmHg
associated with definite disc cupping and visual
2. Age: Affects 1 in 100 of population over 40 years
field defects.
of age. Risk increases in 50–70 years of age.
2. Glaucoma suspect or ocular hypertension. IOP
3. High myopes are more predisposed than the
more than 23 mmHg with no disc changes or visual
emmetropes.
field defects. These cases should be treated as cases
4. Other risk factors include thyrotoxicosis, of POAG under following circumstances:
diabetes mellitus, and cigarette smoking. • Significant diurnal variation (more than 8 mmHg)
Signs
• Significantly positive water drinking provocative
test (more than 8 mmHg)
1. IOP changes: Initially there is exaggeration of • More than 0.2 asymmetry of cup-disc ratio in two
the normal diurnal variation. A difference of more eyes
than 6 mmHg is suspicious and over 8 is diagnostic • Splinter haemorrhages over or near the disc
(normal below 5). In later stages IOP is permanently • Family history of glaucoma
raised and ranges between 30 and 45 mmHg. • IOP constantly more than 30 mmHg
2. Optic disc changes: Normal cup/disc ratio is 0.3. • Diabetic and highly myopic patients.
Asymmetry of more than 0.2 between two eyes and/ 3. Low tension glaucoma (LTG) or normal tension
or notching of the neural rim is suspicious. Marked glaucoma: IOP less than 21 mmHg with typical disc
cupping (0.7 to 0.9) may occur in advanced cases. and/or visual field defects.
• Nasal shift of blood vessels at the disc with
Treatment
appearance of being broken at the margin
(Bayoneting sign) I. Medical therapy
• NRR thinning/notching a. Topical treatment is the treatment of choice for
• Laminar dot sign POAG in both eyes
• Increased pallor (area of disc lacking small vessels) • Topical prostaglandin analogues are the first drugs
• Haemorrhages on the disc or disc margin
of choice (increase uveoscleral aqueous outflow)
• Glaucomatous optic atrophy (white and deeply
(latanoprost, bimatoprost, travoprost)
excavated disc) is the end result.
• Topical beta blockers decrease aqueous secretion
3. Visual field defects: These run parallel to the from ciliary epithelium and are of second choice.
optic disc changes and progress in the following • Alpha agonists, non-selective, e.g. epinephrine and
sequence: dipivefrine, increase uveoscleral aqueous outflow.
• Baring of blind spot (earliest field defect) Selective alpha 2 agonists, e.g. apraclonidine and
• Paracentral scotoma between 10 and 20 degrees of brimonidine decrease aqueous outflow.
visual field (Bjerrum’s area). It is the earliest clinical • Carbonic anhydrase inhibitors, e.g. dorzolamide,
significant field defect. acetazolamide and brinzolamide decrease
• Seidel’s sign (sickle-shaped extension of blind spot) aqueous secretion due to lack of HCO3.
• Arcuate or Bjerrum’s scotoma • Cholinomimetics, e.g. pilocarpine, physostigmine,
• Ring or double arcuate scotoma carbachol,echothiophate increase aqueous
• Roenne’s central nasal step outflow by opening of trabecular meshwork due
• Peripheral nasal step of Roenne’s to ciliary muscle contraction.
Glaucoma 79
b. Systemic therapy consists of carbonic anhydrase • Emotional stress
inhibitors (acetazolamide, dichlorphenamide, metha­ • Use of mydriatics (atropine).
zolamide), and hyperosmotic agents, e.g. mannitol
Clinical stages/Classifications
and glycerol which reduce the vitreous volume.
New classification (2006) of PAC disease is as
II. Argon or diode laser trabeculoplasty below:
When treatment with antiglaucoma medications
fails, laser trabeculoplasty is indicated. 1. Primary angle closure suspect (PACS) is
diagnosed when gonioscopy reveals iridotrabecular
III. Surgery (filtration surgery) contact in >270° angle and no peripheral anterior
Most common surgery is trabeculectomy. synechia with normal IOP, optic disc and visual
Note: POAG is a bilateral condition. Treatment of fields. Impression: the angle is at risk.
fellow eye is the same. 2. Primary angle closure (PAC) is characterized by:
• > 270° iridotrabecular contact on gonioscopy
PRIMARY ANGLE-CLOSURE GLAUCOMA (PACG)
• IOP elevated and/or PAS present
Prevalence • Optic disc and visual fields normal.
There is a great ethnic variability in the prevalence Impression: Angle is abnormal either in function
of PACG. The ratio of POAG versus PACG reported (elevated IOP) and/or in structure (PAS +ve)
for different ethnic groups is as below:
Acute primary angle closure: Sudden rise in IOP
Ethnic group POAG : PACG
occurs due to total angle closure.
• European, African and
Hispanics 5 : 1 Surgical treatment is required after medical control
• Urban Chinese 1 : 2 of IOP
• Mongolians 1 : 3 • Peripheral iridectomy/laser iridotomy—when
• Indians 1 : 1 peripheral anterior synechiae (PAS) are formed
in less than 50 percent of the angle of anterior
Clinical features
chamber
A. Symptoms • Filtration surgery (e.g. Trabeculectomy)—when
• Severe pain, PAS are formed in more than 50 percent of the
• Associated with nausea and vomiting and
angle
• Colored halos seen.
• Peripheral iridectomy/laser iridotomy should also
B. Signs
be considered for the fellow eye.
• Pupil semidilated, vertically oval, fixed and non
reactive to bright light and 3. Primary angle closure glaucoma (PACG) is
• IOP increased. labeled when:
Etiology • Iridotrabecular contact is seen in >270° angle on
Predisposing factors gonioscopy
• PAS are formed
• Hypermetropic eyes with shallow anterior chamber
• IOP is elevated
• Eyes in which iris-lens diaphragm is placed
• Optic disc and visual fields show typical glauco­
anteriorly
matous damage.
• Eyes with narrow angle of anterior chamber due
to small eyeball, relative large crystalline lens or PACG may develop as a sequelae to subacute
bigger size of the ciliary body or actue PAC or due to gradual and progressive
• Plateau iris configuration (creeping) synechial angle closure.
• Sex; Male : Female is 1 : 3 Absolute glaucoma—In this end stage, the eye is
• Nervous personality with unstable vasomotor painful, completely blind, and IOP is very high.
system
• Positive family history SECONDARY GLAUCOMAS
• Usually fifth or sixth decade of life.
1. Lens-induced glaucomas
Precipitating factors for acute PAC
• Phacomorphic : IOP is raised due to secondary angle
• Dim illumination closure and/or pupil block by lens intumescence
80 Review of OPHTHALMOLOGY

or, anterior subluxation or dislocation of lens or 5. Glaucoma associated with intraocular tumours
spherophakia. Intraocular tumours such as retinoblastoma and
• Phacolytic: An acute secondary open-angle malignant melanoma may raise IOP by one or more
glaucoma due to clogging of trabecular meshwork of the following mechanisms:
by macrophages laden with lens proteins in a • Trabecular block by tumour cells
patient with hypermature cataract. • Neovascularization of the angle
• Lens particle glaucoma: It occurs due to trabecular • Venous stasis following obstruction to vortex veins.
blockage by the lens particles.
6. Pseudoexfoliative glaucoma
• Phacoanaphylactic: It occurs due to sensitisation of
(Glaucoma capsulare)
eye or its fellow to lens proteins. IOP is raised due
to clogging of trabeculae by inflammatory material. • Pseudoexfoliative syndrome (PES) refers to
amyloid like deposits on pupillary border, anterior
2. Glaucomas due to uveitis lens surface, posterior surface of iris, zonules and
• Non-specific hypertensive uveitis: IOP is raised ciliary processes.
due to clogging by inflammatory material and • 70 percent cases of PES are associated with raised
associated trabeculitis. IOP (secondary open-angle glaucoma).
• Specific hypertensive uveitis syndromes: These • Other features and treatment are similar to POAG.
include: Fuchs’ uveitis syndrome and glauc- 7. Glaucoma in aphakia
omatocyclitic crisis.
It implies association of glaucoma with aphakia.
• Post-inflammatory glaucoma: It may result from
annular synechiae, occlusiopupillae, angle closure Causes
following iris bombe formation or angle closure due • Raised IOP due to postoperative hyphaema,
to organisation of the inflammatory debris. inflammation, vitreous filling the anterior chamber
3. Pigmentary glaucoma • Angle closure due to flat anterior chamber
• Pupil block with or without angle closure
• About 35 percent of patients with pigment dis­ • Undiagnosed pre-existing POAG
persion syndrome develop pigmentary glaucoma. • Steroid-induced glaucoma
• Bilateral condition, typically affecting young • Epithelial ingrowth
myopic males. • Aphakic malignant glaucoma.
• Glaucomatous features are similar to POAG
with associated pigment deposition on corneal 8. Steroid-induced glaucoma
endothelium (Krukenberg’s spindle), trabecular • Roughly, 5 percent of general population is high
meshwork, iris, lens and zonules. steroid responder (develop marked rise of IOP after
about 6 weeks of steroid therapy), 35 percent are
4. Neovascular glaucoma
moderate and 60 percent are non-responders
• Rubeosis iridis • Pathogenesis: Probably mucopolysaccharides are
• It results due to formation of a neovascular deposited in the trabecular meshwork
membrane involving angle of the anterior chamber. • Features are similar to POAG.
• Usually, stimulus to new vessel formation is
Management
retinal ischaemia as seen in diabetic retinopathy,
CRVO, Eales’ disease. Other rare causes are • Can be prevented by judicious use of steroids
chronic uveitis, intraocular tumours, old retinal • IOP may normalise in 98 percent of cases within
detachment, “CRAO”(known as 100 day glaucoma) 10 days to 4 weeks of discontinuation of steroids
and retinopathy of prematurity • Medical therapy with 0.5% timolol maleate is
• Neovascularisation begins at pupil and spreads effective during normalization period
centrifugally • Filtration surgery is required in intractable cases.
9. Ciliary block glaucoma (Malignant glaucoma)
Management includes: Panretinal photo­
coagulation to prevent stimulus to new vessel Also known as ciliovitreal block or aqueous mis­
formation direction syndrome.
• Glaucoma implant (e.g. Molteno tube) opera­tion • It can occur as a complication of any intraocular
• Cyclocryotherapy. operation.
Glaucoma 81
• Classically, it occurs following peripheral iridectomy • Congenital anomaly most commonly associated
or filtration operation for primary narrow-angle with buphthalmos is facial haemangiomas.
glaucoma. • refers to heavy pigment
• Pathogenesis involves ciliolenticular or cilio­vitreal deposition in a line above Schwalbe’s line in the
block. angle of anterior chamber (a feature of exfoliative
Clinical Features glaucoma).
• Vogt’s triad includes glaukomflecken (anterior
• Persistent flat anterior chamber with negative
subcapsular lenticular opacity), patches of iris
Seidel’s test
atrophy and slightly dilated nonreacting pupil
• Markedly raised IOP
• May be phakic, aphakic or pseudophakic (due to sphincter atrophy); seen in the eye
• Vortex veins seen in malignant melanoma. which has suffered an attack of acute congestive
glaucoma.
Management • Pilocarpine and other miotics are contraindicated
1. Medical therapy (is useful in about 50 percent in inflammatory glaucoma, malignant glaucoma
cases): and glaucoma due to spherophakia.
a. Atropine eye drops • Most preferred site for filtering operation is superior
b. Acetazolamide 250 mg TDS nasal quadrant.
c. 0.5% Timolol maleate eye drops BD • Pilocarpine in angle closure glaucoma should be
2. Surgical therapy: Anterior vitrectomy and used after control of IOP by aqueous suppressant
injection of air in the anterior chamber. and hyperosmotic agents.
Note: YAG laser hyaloidotomy is also used as a • Argon laser trabeculoplasty is only the adjuvant to
treatment modalities. medical therapy of primary open-angle glaucoma.
10. Glaucoma associated with intraocular
• Miotics are not useful in a buphthalmos, aphakic
haemorrhage glaucoma, glaucomatocyclitic crisis, glaucoma
inversus and in epidemic dropsy glaucoma.
• Haemolytic glaucoma: Acute open-angle glaucoma
• Secondary glaucoma after perforation of the cornea
which occurs due to obstruction of the trabecular
is due to blockage of the drainage angle by anterior
meshwork by macrophages laden with lysed RBC
synechiae.
debris after hyphaema.
• Ghost cell glaucoma: It occurs in aphakic eyes • In acute-congestive glaucoma, the choice of surgery
with vitreous haemorrhage. RBCs converted into between peripheral iridectomy and filtering
Khaki coloured ghost cells block the trabecular operation is decided by gonioscopic examination.
meshwork. • Single most important test in diagnosing POAG and
• Red cell glaucoma: Caused by blockage of trabecular response to treatment is visual field testing.
meshwork by RBCs following massive hyphaema. • Earliest and most constant symptom in infantile
• Haemosiderotic glaucoma: It occurs due to sclerotic glaucoma is lacrimation.
changes in trabecular meshwork induced by iron • Latanoprost: PGF2alpha.
from the haemoglobin. – Uses: Uveoscleral pathway (only drug)
– DOC of open angle glaucoma.
11. Glaucoma associated with iridocorneal-
– Most potent antiglaucoma.
endothelial (ICE) syndromes
– DOC of low tension glaucoma.
• ICE syndromes include progressive iris atrophy, • Safest antiglaucoma/drug of choice in children is
corneal edema, Chandler’s syndrome and Cogan- dorzolamide.
Reese syndrome.
• Antiglaucoma drugs causing blepherocon­
• IOP is raised due to endothelial membrane lining
junctivitis
the trabecular meshwork.
– Latanoprost (clinically significant and most
Histopathology: Collagen deposit on posterior common)
surface of Descemet’s membrane. – Timolol
– Dipiverfrine
MISCELLANEOUS POINTS • Increased IOP in glaucoma damages retinal
• Commonest hazard following surgery of narrow ganglion cells (RGC).
angle glaucoma is malignant glaucoma. • Nasal field first to be damaged and temporal last.
82 Review of OPHTHALMOLOGY

• Diffused iris melanoma causes intractable • Inverse glaucoma: Pilocarpine causes para­doxical
secondary glaucoma. rise in IOP.
• In epidemic dropsy, wide angle glaucoma is • Well–Marchesani syndrome is associated with
associated with raised levels of histamine , spherophakia
prostaglandin and proteins (hypersecretary
Mnemonic (5S)
glaucoma)
• Fincham’s test is used to differentiate the halos of Short
PACG and immature cataract, in this stenopaeic Stubby finger
slit is passed across the pupil, glaucomatous halos Stupid
remains intact while halos due to cataract are Spherophakia
broken into segments. Subluxation.
Glaucoma 83

Multiple Choice Questions (MCQs)

1. Normal aqueous production rate is about: 7. Retinal nerve fibres most sensitive to glaucomatous
A. 2 l/min damage are:
B. 2.3 µl/min A. Superior and inferior arcuate fibres
C. 2.6 µl/min B. Macular fibres
D. 2.9 µl/min C. Superior radiating fibres
2. Trabecular (conventional) outflow of aqueous D. Inferior radiating fibres
humour accounts for: 8. Incidence of primary open-angle glaucoma in
A. 90 percent
population over 40 years of age is about:
B. 80 percent
A. 1 in 100
C. 70 percent
B. 1 in 200
D. 60 percent
C. 1 in 500
3. Incidence of congenital glaucoma is: D. 1 in 1000
A. 1 in 1000 births
B. 1 in 5000 births 9. Sickle-shaped extension of blind spot is known
C. 1 in 10,000 births as:
D. 1 in 34,000 births A. Bjerrum’s sign
B. Seidel’s sign
4. In normal diurnal variation, intraocular pressure
C. Down’s sign
is:
D. Baring of blind spot
A. Highest on awakening and lowest during evening
B. Lowest during morning and highest during 10. Ocular associations of primary open-angle
evening glaucoma include all except:
C. Highest in the morning and evening
A. Central retinal vein occlusion
D. Lowest in the morning and evening
B. Central retinal artery occlusion
5. Normal diurnal variation of intraocular pressure C. Retinal detachment
is: D. Retinitis pigmentosa
A. 0-2 mm of Hg
11. Vogt’s triad is indicative of:
B. 2-3 mm of Hg
C. 3-6 mm of Hg A. Past attack of herpes zoster ophthalmicus
D. 6-8 mm of Hg B. Past attack of acute-angle closure glaucoma
C. Vogt-Koyanagi-Harada syndrome
6. In indentation tonometry: D. Past attack of acute iridocyclitis
A. Plunger indents a hard eye more than a soft eye
B. Lower scale readings are obtained in high 12. Glaukomflecken is a feature of:
intraocular pressure A. Acute narrow-angle glaucoma
C. Low scleral rigidity gives high reading of B. Pseudoexfoliative glaucoma
intraocular pressure C. Juvenile glaucoma
D. All of the above D. Phacolytic glaucoma

1 : B 2 : A 3 : C 4 : A 5 : C 6 : B 7 : A 8 : A 9 : B 10 : B 11 : B 12 : A
84 Review of OPHTHALMOLOGY

13. In the incidence of primary angle closure 21. The most characteristic visual field change in
glaucoma, male to female ratio is: primary open-angle glaucoma is:
A. 1:1 A. Nerve fibre bundle defect
B. 1:2 B. Enlargement of blind spot
C. 1:3 C. Generalised constriction of field
D. 1:4 D. Sector-shaped defects
22. All of the following are true about pigmentary
14. The earliest clinically significant field defect in glaucoma except:
primary open-angle glaucoma is: A. It occurs more often in young myopic men
A. Paracentral scotoma B. Iris transillumination defects are noted
B. Baring of blind spot C. It is associated with Krukenberg’s spindle
C. Seidel’s scotoma D. The intensity of pigment deposit in the angle is
D. Isopter contraction related to iris colour

15. All are the causes of neovascular glaucoma except: 23. Epinephrine:
A. Intraocular tumour A. Reduces aqueous production
B. Reduces outflow facility
B. Central retinal vein occlusion
C. Reduces aqueous production and increases
C. Diabetic retinopathy
outflow facility
D. Central serous retinopathy D. Increases aqueous production and reduces
16. Krukenberg’s spindle seen in patients with outflow facility
pigmentary glaucoma refers to deposition of 24. Treatment of malignant glaucoma is:
pigment on: A. Pilocarpine
A. Trabecular meshwork (gonioscopic sign) B. Cyclocryotherapy
B. Back of cornea C. Vitreous aspiration
C. Anterior surface of the lens D. Trabeculectomy
D. All of the above 25. Neovascular glaucoma may be associated with all
of the following except:
17. In Fincham’s test:
A. Diabetes
A. Glaucomatous halo remains intact B. Hypertension
B. Halo due to immature cataract does not break C. Central retinal vein occlusion
into segments D. Intraocular tumours
C. Halo due to mucopurulent conjunctivitis is
broken into segments 26. Primary open-angle glaucoma is associated with
D. All of the above all of the following except:
A. Diabetes mellitus
18. All are the features of an acute attack of primary B. Myopia
narrow-angle glaucoma except: C. Hyperthyroidism
D. Pars planitis
A. Intraocular pressure is raised up to 940-70 mm
of Hg 27. Cupping of the disc is not a feature of:
B. Eye is red, painful and tender A. Buphthalmos
C. Disc shows glaucomatous cupping B. Chronic simple glaucoma
D. Fellow eye also shows shallow anterior chamber C. Acute congestive glaucoma
D. Megalocornea
19. In Indian population ratio of POAG : PACG is:
A. 4:1 28. Inverse glaucoma occurs in:
B. 2:1 A. Spherophakia
C. 1:1 B. Lenticonus
C. Subluxated lens
D. 1:2 .
D. All of the above
20. All of the following can precipitate the attack of 29. Lowe’s syndrome is characterized by all except:
narrow angle except: A. Glaucoma
A. Prolonged prone position B. Albuminuria
B. Mydriatics C. Glycosuria
C. Prolonged work in bright light D. Oligoammonuria
D. Emotional upsets E. Retinal detachment

13 : D 14 : A 15 : D 16 : B 17 : A 18 : C 21 : A 22 : D 23 : C 24 : C 25 : B 26 : D
19 : C 20 : C 27 : D 28 : A 29 : E
Glaucoma 85
30. All the following can be seen in Axenfeld’s 39. The most reliable provocative test for angle-
anomaly, except: closure glaucoma is:
A. Posterior embryotoxon A. Homatropine mydriatic test
B. Iris synechiae to Schwalbe’s line B. Mydriatic-miotic test
C. Ectopia of the lens C. Water drinking test
D. Glaucoma D. Dark room test
31. All the following are true concerning Rieger’s 40. Which of the following types of senile cataract is
syndrome except: the most notorious to produce glaucoma:
A. Autosomal recessive inheritance
A. Incipient cataract
B. Glaucoma
B. Lamellar cataract
C. The possible presence of facial, dental and
osseous defects C. Hypermature Morgagnian cataract
D. The spectrum of Axenfeld’s anomaly and marked D. Intumescent cataract
abnormal development of the iris mesoderm. 41. All of the following anatomical changes will
32. In chronic simple glaucoma the most common field predispose to primary-angle-closure glaucoma
defect is: except:
A. Arcuate field defect A. Small cornea
B. Baring of blind spot B. Flat cornea
C. Bjerrum scotoma C. Anterior chamber shallow
D. Siedel’s sign D. Short axial length of eyeball
33. Treatment of malignant glaucoma includes all 42. Argon laser trabeculoplasty is used in:
except: A. Closed-angle glaucoma
A. Topical atropine B. Primary open-angle glaucoma
B. Topical pilocarpine C. Neovascular glaucoma
C. IV mannitol D. Aphakic glaucoma
D. Vitreous aspiration
43. In buphthalmos, lens is:
34. The earliest change in glaucoma is:
A. Anteroposterior flat
A. Papilloedema
B. Small
B. Hazy cornea
C. Large
C. Baring of the blind spot
D. Sickle scotoma D. None of the above

35. Neovascular glaucoma can occur in all except: 44. First sign seen in open-angle glaucoma is:
A. Diabetes mellitus A. Arcuate scotoma
B. Hypertension B. Extension above blind spot
C. CRAO C. Roene’s nasal step
D. CRVO D. Siedel’s scotoma
36. Coloured haloes are found in all except: 45. Congenital glaucoma presents as:
A. Pigmentary glaucoma A. Microphthalmos
B. Acute-angle-closure glaucoma B. Photophobia
C. Cataract C. Leucocoria (white reflex)
D. Uveitis D. Pain
37. Secondary glaucoma following corneal perforation 46. In a hypertensive patient with glaucoma which of
is due to: the following is not used:
A. Central anterior synechiae formation
A. Dipivefrine
B. Peripheral anterior synechiae
B. Alpha blocker
C. Intraocular haemorrhage
D. Angle recession C. Alpha agonist
D. Laser trabeculoplasty
38. In haemolytic glaucoma the mechanisms are all
except: 47. Schwalbe’s ring corresponds to:
A. Siderosis of trabeculae A. Corneal endothelium
B. Deposition of haemosiderin B. Descemet’s membrane
C. RBC clogging the trabeculae C. Schlemm’s canal
D. Inflammation D. Ciliary body

30 : C 31 : A 32 : B 33 : B 34 : C 35 : B 39 : D 40 : D 41 : B 42 : B 43 : A 44 : B
36 : D 37 : B 38 : D 45 : B 46 : A 47 : B
86 Review of OPHTHALMOLOGY

48. A 75-year old patient present with deterioration C. Prodrome


of vision. On examination the pupillary reflex D. Constant instability
is observed to be sluggish and the intraocular
pressure is normal. Optic disc evaluation shows 55. Acute angle closure glaucoma:
a large and deep cup and primarily shows A. Colored halos present
paracentral scotomas. The most likely diagnosis B. Flashes of light seen
is: C. Deep anterior chamber
A. Primary narrow angle glaucoma D. Vertically oval pupil
B. Normal tension glaucoma E. Increased IOP
C. Neovascular glaucoma
56. Drug used in acute congestive glaucoma are all
D. Absolute glaucoma
except:
49. Treatment of primary open angle glaucoma: A. Atropine
A. Timolol maleate B. Pilocarpine
B. Atropine C. Acetozolamide
C. Acetazolamide D. Mannitol
D. Prostaglandin analogue E. Timolol
50. True about primary angle closure glaucoma: 57. Treatment of acute congestive glaucoma includes
A More common in females all exept:
B. Shallow anterior chamber in a tank
A. Sclerectomy
C. Deep anterior chamber is a risk factor
B. Trabeculectomy
D. Shorter diameter of cornea is a predisposing
C. Trabeculoplasy
factor
E. Common in myopes D. Vitrectomy
E. Iridotomy
51. Intractable secondary glaucoma is seen in:
A. Diffuse iris melanoma 58. Iridocorneal endothelial syndrome is associated
B. Nodular iris melanoma with:
C. Melanocytic deposits in anterior part of iris A. Progressive atrophy of iris stroma
D. Melanocyte proliferation in posterior oveal B. Bilateral stromal edema of iris and cornea
tissue. C. Deposition of collagen in Descemet’s membrane
D. Deposition of glycosaminoglycan in Descemet’s
EXTRA EDGE QUESTIONS membrane

52. Which of the following is primary glaucoma: 59. Treatment option for glaucoma includes all except:
A. Juvenile glaucoma A. Trabeculectomy
B. Steriod induced glaucoma B. Trabeculotomy
C. Pigmentary glaucoma C. Vitrectomy
D. Congenital glaucoma D. Viscoanulostomy
E. Infantile glaucoma E. Iridectomy

53. In POAG (primary open angle glaucoma) which is 60. False statement about depth of anterior chamber:
not seen: A. Less in women than men
A. Horizontal cupping B. Correspondent to volume of anterior chamber
B. Bayonetting sign (crossing of vessels) C. Increase with age
C. Macular oedema D. More in myopes
D. Dot sign E. Less in hypermetropes
54. A patient presents with H/o evening halos and
occasional headache for some months. His 61. Glaucomflecken is:
examination shows normal IOP but shallow AC. A. Acute uvetis due to glaucoma
He is in which stage of glaucoma: B. Lens opacity due to glaucoma
A. Acute C. Retinal detachment due to glaucoma
B. Absolute D. Corneal opacity due to glaucoma

48 : B 49 : A 50 : A, B and D 51 : A 52 : A, D and E 55 : A, D and E 56 : A 57 : C and D 58 : A


53 : A and C 54 : C 59 : C 60 : B and C 61 : B
CHAPTER

10
Diseases of Vitreous

Quick Text Review


DISORDERS OF VITREOUS • Vitrectomy is indicated if vitreous haemorrhage
does not absorb in 3 months.
VITREOUS LIQUEFACTION (SYNCHYSIS)
• Most common degenerative change PERSISTENT HYPERPLASTIC PRIMARY VITREOUS
• Causes: Senile, myopic degeneration, post- • Usually unilateral, bilateral cases are rare and may
inflammatory, post-traumatic. be associated with Trisomy13 (patau syndrome)
• Typically presents as a white reflex (congenital
POSTERIOR VITREOUS DETACHMENT (PVD) leukocoria) in pupil, it must be differentiated
• PVD with vitreous liquefaction (synchysis) and from other causes of leukocoria, particularly
collapse (syneresis) is of common occurrence in retinoblastoma. Differentiating features from
majority of the normal subjects above the age of retinoblastoma are:
65 years. – Lack of calcification on CT (calcification is an
• Flashes of light and floaters may be associated important feature of retinoblastoma)
• A ring like opacity (Weiss reflex) representing a ring – Development of cataract (cataract is rare in
of vitreous attachment at the optic disc margin is retinoblastoma)
pathognomic. – Unilaterality (retinoblastoma is bilateral in
• May be complicated by retinal break, retinal and/ about one-fourth of cases)
or vitreous haemorrhage and cystoid maculo­pathy. • Associations are: Long ciliary processes, micro­
phthalmos, cataract, glaucoma, and vitreous
VITREOUS HAEMORRHAGE
detachment.
• Causes are diabetic retinopathy, hypertensive • Visual prognosis is poor despite early intervention.
retinopathy, Coats disease, Eales’ disease trauma, • Seen in Patau sundrome.
retinal breaks and/or posterior vitreous detach­
ment, anaemia, leucaemia and sickle cell MISCELLANEOUS POINTS
retinopathy, central retinal vein occlusion, and • Synchysis (liquefaction) and syneresis (collapse)
bleeding disorders—haemophilia, purpura. are the two most common degenerative changes
• Clinical features: Sudden onset of floaters (black in the vitreous gel.
spots in front of the eye) when the hemorrhage • Optimum time for vitrectomy in a patient of
is small; and sudden painless loss of vision if the bacterial endophthalmitis not responding to
haemorrhage is large. conservative treatment is 24 hours after the
• Examination reveals normal anterior segment, no intravitreal injection of antibiotics.
red glow and non-visualization of fundus in a large • Vitreous gel never regenerates
haemorrhage. • Strongest attachment of the vitreous is to the ora
• B-scan USG is particularly helpful in diagnosing serrata (vitreous base attachment)
vitreous haemorrhage • Most common cause of vitreous haemorrhage is
• Complications include retinitis proliferans, trauma
vitreous liquefaction and degeneration, and ghost • Most common cause of spontaneous vitreous
cell glaucoma in aphakes. haemorrhage is proliferative diabetic retinopathy.
88 Review of OPHTHALMOLOGY

VITRECTOMY II. Pars- plana vitrectomy (PPV) can be:


1. One-port vitrectomy, not a preferred technique
Indications of vitreous surgery now-a-days, can be performed with the help of
• Vitreous loss during cataract surgery a multifunction vitrectomy probe comprising
• Along with lensectomy vitreous infusion, suction, and cutter (VSC)
• Endophthalmitis 2. Three-port-vitrectomy, the preferred technique,
• Vitreous haemorrhage not getting absorbed in comprises three different ports one each for
3 months illumination probe, infusion cannula, and cutter
and suction probe. It includes:
• Proliferative retinopathies
• 20 gauge PPV,
• Dropped nucleus and intraocular lens
• 23 gauge PPV, and
• Intraocular foreign bodies
• 25 gauge PPV.
• Vitreous amyloidosis.
• Aphakic keratoplasty Vitreous Substitute
Vitreous substitute are used in vitreo-retinal surgery to:
Types of vitrectomy • Restore intraocular pressure, and
• Anterior vitrectomy, i.e. removal of anterior • Provide intraocular tamponade.
vitreous. Commonly used vitreous substitutes include:
• Core vitrectomy, i.e. removal of central bulk of 1. Expanding gases e.g:
vitreous. • Sulphur hexafluoride (SF6),
• Subtotal and total vitrectomy, i.e. removal of • Perfluoropropane (C3F8)
almost total vitreous. 2. Perfluoro carbon liquids (PFCLS), e.g:
• Perfluoro-n-octane
Techniques of Vitrectomy
• Perfluro–tributylamine,
I. Open-sky vitrectomy. It is performed through • perfluro–decalin, and
the primary wound to manage disturbed vitreous • perfluro–phenanthrene.
in cataract surgery, aphakic keratoplasty, and 3. Silicone oil. It allows more controlled retinal
globe rupture. manipulation during VR surgery.
Diseases of Vitreous 89

Multiple Choice Questions (MCQs)

1. Synchysis refers to: C. Cataract


A. Liquefaction of the vitreous D. All of the above
B. Black spots in front of the eyes
C. Collapse of the vitreous 8. Commonest cause of vitreous haemorrhage is:
D. Detachment of the vitreous A. Diabetes
B. Hypertension
2. Syneresis refers to: C. Trauma
A. Liquefaction of the vitreous D. Lens extraction
B. Black spots in front of the eye
C. Collapse of the vitreous 9. Vitreous haemorrhage is seen in all except:
D. Detachment of the vitreous A. Coat’s disease
B. Eales’ disease
3. In vitreous base detachment, vitreous body is
C. CRVO
detached from its attachment with the:
D. CRAO
A. Optic disc
B. Ora-Serrata 10. Vitreous haemorrhage is not seen in:
C. Posterior surface of the lens A. Hypertension
D. Fovea centralis B. Eales’ disease
4. All of the following are features of asteroid C. Trauma
hyalosis except: D. Diabetes mellitus
A. Usually bilateral E. Vitreous degeneration
B. Solid vitreous
11. A vitreous aspirate has been collected in an
C. Spherical calcium bodies
emergency at 9 pm. What advice would you like to
D. Usually asymptomatic
give to the staff on duty regarding the overnight
5. All of the following are features of synchysis storage of the sample:
scintillans except: A. The sample should be kept at 4°C
A. Fluid vitreous B. The sample should be incubated at 37°C
B. Spherical calcium bodies C. The sample should be refrigerated in deep freezer
C. Shower of gold rain D. The sample should be refrigerated for the initial
D. Secondary to trauma or inflammations of the eye 3 hours and then kept at 37°C
6. Vitrectomy should be considered if the vitreous 12. Eales disease is:
haemorrhage is not absorbed within: A. Recurrent optic neuritis
A. 1 month B. Recurrent pappilloedema
B. 3 months C. Recurrent periphelbitis retinae
C. 6 months D. None
D. 2 months
13. Eale’s disease is:
7. Presistent hyperplastic primary vitreous may be A. Retinal hemorrhage
associated with: B. Vitreous hemorrhage
A. Long ciliary processes C. Conjunctival hemorrhage
B. Microphthalmos D. Choroidal hemorrhage

1 : A 2 : C 3 : B 4 : A 5 : B 6 : B
8 : C 9 : D 10 : E 11 : A 12 :C 13 : B
7:D
90 Review of OPHTHALMOLOGY

14. A 25-year-old male presents with painless sudden EXTRA EDGE QUESTIONS
loss of vision. Ocular and systemic examination
is not contributory. What is probable diagnosis. 15. Persistent primary hyperplastic vitreous (PHPV)
is associated with:
A. Retinal detachment
A. Patau syndrome
B. Eale’s disease B. Edward syndrome
C. Glaucoma C. Trisomy 14
D. Cataract D. Downs syndrome

14 : B 15 : A
CHAPTER

11
Diseases of Retina

Quick Text Review


CONGENITAL AND DEVELOPMENTAL and is characterised by Roth’s spots (superficial
DISORDERS haemorrhages with a white spot in the centre).
• Cytomegalovirus (CMV) retinitis and herpes
Coloboma of the optic disc
zoster retinitis are more common in patients with
• Results from failure of the embryonic fissure closure. AIDS. CMV retinitis is most common cause of
• Minor defect manifests as inferior crescent, usually chorioretinitis in AIDS, typical fundus changes
associated with hypermetropia or astigmatism. are labeled as crumbled chees and ketchup
Drusen of the optic disc appearance (souse and cheese appearance), pizza
• Lies deep beneath the surface of the disc tissue in pie appearance and brush fire appearance.
childhood (presents as pseudopapilloedema) and Periphlebitis retinae
emerges out by the early teens presenting as waxy
pea-like irregular refractile bodies. • Primary periphlebitis retinae (Eales’ disease) an
idiopathic inflammatory venous occlusion, is
Hypoplasia of the optic disc considered a hypersensitive reaction to tubercular
• An important cause of blindness at birth in proteins.
developed countries (rare in developing countries). • Secondary periphlebitis occurs in patients with
• Associated with maternal alcohol use, diabetes and uveitis.
use of certain drugs in pregnancy. • Eales’ disease is a bilateral condition, affecting
• Bilateral in 60 percent cases. peripheral retina in young adult males. It is
• Small disc with ‘double ring sign’ is pathognomonic. characterised by recurrent primary vitreous
Medullated nerve fibres/Opaque nerve fibres haemorrhages.
• Represent myelination of nerve fibres of the retina • Complications of Eales’ diseases include—
(normally myelination stops at lamina cribrosa). rubeosis iridis, neovascular glaucoma, proliferative
• Appear as whitish patch with feathery margins, retinopathy and fractional retinal detachment.
usually present around the disc and may cause
enlargement of the blind spot. VASCULAR DISORDERS OF RETINA
• Disappear in patients with optic atrophy.
RETINAL ARTERY OCCLUSIONS
Congenital remnants of the hyaloid arterial system
• Bergmester’s papilla—a flake of glial tissue Central retinal artery occlusion (CRAO)
projecting from the disc (commonest anomaly of • It occurs due to obstruction at the level of lamina
the hyaloid system) cribrosa.
• Mittendorf dot—remnant of anterior end of hyaloid • Emboli are the most common causes of CRAO.
artery attached to the posterior lens capsule, • Other causes are gaint cell arteritis, SLE, Wegner’s
usually associated with a posterior polar cataract. granulomatosis, scleroderma, and occasionally
raised IOP.
INFLAMMATIONS OF RETINA
• Causes sudden painless loss of vision (central vision
Retinitis not affected in patients with cilioretinal artery).
• Subacute retinitis of Roth typically occurs in patients • Retina becomes white due to oedema and a
with subacute bacterial endocarditis (SABE) ‘Cherry-red-spot’ is seen at the fovea.
92 Review of OPHTHALMOLOGY

• Arterioles become narrow and blood column Arterial reflex changes. Changes in normal reflex,
within the retinal vessels is segmented (Cattle- which is sharp and thin due to blood column under
track appearance). the transparent wall, include:
• Resuscitation time of the human retina following • Diffuse and loss bright reflex is seen due to
retinal ischaemia is 1½ hours. thickening of vessel wall.
Branch retinal artery occlusion (BRAO) • Copper wiring, i.e. reddish brown reflex occurs due
to progressive sclerosis and hyalinization.
• Occurs following lodgement of embolus at a • Silver wiring, i.e. opaque white reflex, occurs
bifurcation.
ultimately due to continued sclerosis.
• Retina distal to the occlusion becomes oedematous
with narrowing of arterioles. Superficial retinal haemorrhages, flame shaped,
• Later on involved area is atrophied leading to occur scattered, more around posterior pole.
permanent sectorial field defect.
Hard exudates, i.e. yellow waxy spots occur due to
Retinal vein occlusion lipid deposits. They are more seen on posterior pole
• Predisposing factors are hypertension, diabetes and may be arranged as macular fan or star.
mellitus, open-angle glaucoma, polycythemia, and Cotton wool spots, fluffy lesions, represent areas of
periphlebitis. infract in nerve fiber layer.
• Non-ischaemic central retinal vein occlusion (CRVO)
is most common variety (75%), characterised by Acute changes of hypertension
mild to moderate visual loss. About 15% cases of Acute hypertensive retinopathy changes includes
nonischaemic CRVO are converted to ischaemic • Marked arteriolar narrowing due to spasm
CRVO in 4 months and about 30% in 3 years. • Superficial retinal haemorrhages, flame shaped,
• Ischaemic central retinal vein occlusion is arranged in clusters.
characterised by much more marked signs and Keith Wegner grading of hypertensive retinopathy
symptoms than non-ischaemic CRVO.
• Fundus changes include tortuous and engorged Grade I: Mild generalized narrowing of arterioles
veins, scattered retinal haemorrhages, cotton wool Grade II: Marked generalized narrowing plus focal
spots, and unilateral disc changes (Blood and constriction of arterioles with deflection of the vein
thunder fundus appearance) at arteriovenous crossing (Salu’s sign)
• Complications. Rubeosis iridis and neovascular Grade III: Grade II changes plus copper wiring of
glaucoma (NVG) occurs in more than 50 percent arterioles, Bonnet sign, Gunn sign, Salu’s sign along
cases within 3 months, vitreous haemorrhage, and with haemorrhages, cotton wool spots and hard
proliferative retinopathy. exudates.
• Differential diagnosis of CRVO, most important,
Grade IV: Grade III changes plus silver wiring and
is ocular ischemic syndrome. Veins are dilated in
papilledema.
both, but veins are tortuous in CRVO.
• Branch retinal vein occlusion: the superotemporal Retinopathy in toxaemia of pregnancy
branch is the most commonly affected (33%). • Earliest changes consist of narrowing of nasal
arterioles followed by generalised narrowing.
HYPERTENSIVE RETINOPATHY • Advent of hypoxic changes (cotton wool spots,
Chronic hypertensive retinopathy haemorrhages and retinal oedema) should be
Generalized narrowing of arterioles which may be considered an indication for termination of
mild or moderate. pregnancy.
Focal arteriolar narrowing is seen as area of
localized vasoconstriction on the disc or within. DIABETIC RETINOPATHY

Arteriovenous nicking (crossing changes, hallmark Risk factors


of chronic hypertensive retinopathy) include: • Duration of diabetes: Incidence is about:
• Salu’s sign, i.e. deflection of veins at crossing – After 10 years of diabetes 20% of Type I and
• Bonnet sign, i.e. banking of veins distal to crossing 25% of Type II.
• Gunn sign, i.e. tapering of veins on either side of – After 20 years of diabetes 90% of Type I and
crossing. 60% of Type II.
Diseases of Retina 93
– After 30 years of diabetes 95% of both Type I • Fluorescein angiography reveals focal leakage with
and Type II. adequate macular perfusion.
• Heredity: Effect is more on proliferative DR, • Treatment of focal DME not involving centre
transmission is recessive of fovea consists of focal argon laser to micro­
• Sex: Male: Female: 3: 4 aneurysms and centre of the hard exudates ring.
• Pregnancy: Accentuates the changes of DR Diffuse exudative maculopathy
• Hypertension: Accentuates the changes of DR. • Changes of NPDR with very few hard exudates in
Other ocular manifestations of DM the macular area
• Cataract (Snowflake/snow strom) • Diffuse retinal oedema and thickening throughout
• Myopia (when there is sudden increase in blood the posterior pole
sugar level) • Cystoid macular oedema and lamellar hole in long
• Rarely hypermetropia standing cases
• Cranial nerve palsy: 3rd (most common), 4th , 6th • Fluorescein angiography shows diffuse leakage but
and 7th. good macular perfusion.
• Treatment consists of intravitreal anti-VEGFs and
Classification of DR argon laser photocoagulation should be reserved
1. Non-proliferative diabetic retinopathy (NPDR) for recalcitrant cases not responding to anti-VEGFs
Signs and intravitreal triamcelone.
• Microaneurysms (an early sign) Ischaemic maculopathy
• Dot and blot haemorrhages • Marked visual loss
• Hard exudates • Microaneurysms, haemorrhages, mild or no
• Retinal thickening due to diffuse oedema oedema and only a few hard exudates if any
• Venous abnormalities beading, looping and • Fluorescein angiography shows poor macular
dilatation perfusion.
• Cotton-wool spots Mixed maculopathy
• Intraretinal microvascular abnormalities (IRMA) • Features of combined ischaemic and exudative
Treatment is not required, management includes: maculopathy are present.
• Adequate control of diabetes 4. Advanced diabetic eye disease
• Annual fundus examination.
• Persistent vitreous haemorrhage
2. Proliferative diabetic retinopathy • Tractional retinal detachment
• Neovascular glaucoma
Signs of PDR: In chronological order, over and above • Treatment is pars plana vitrectomy with endo­
the signs of NPDR are as follows: photocogulation and management of RD.
• Neovascularization on the disc (NVD) or elsewhere
(NVE) SICKLE-CELL RETINOPATHY
• Incomplete posterior vitreous detachment Retinal changes are caused by hypoxia resulting from
• Elevation of new vessels and vitreous haemorrhage blockage of small blood vessels by the abnormal
• Fibrovascular epiretinal membrane shaped rigid red blood cells.
• Tractional retinal detachment.
Clinical features
Treatment: Panretinal argon photocoagulation
• Stage I: Peripheral arteriolar occlusion
(PRP). Intravitreal injection of anti-VEGF before
• Stage II: Peripheral arteriovenous anastomosis
PRP protects the macula and reduces risk of vitreous
• Stage III: Neovascularization
haemorrhage. Intravitreous triamcinolone may be
• Stage IV: Vitreous haemorrhage
considered as an adjunct to PRP.
• Stage V: Vitreoretinal traction bands and tractional
3. Diabetic maculopathy retinal detachment.
Focal exudative maculopathy Treatment
• Characterized by changes of NPDR and hard • Pan retinal photocoagulation (PRP) is effective in
exudates arranged in a circinate pattern in macular regressing the neovascularization.
area. • Pars plana vitrectomy for stage IV and V.
94 Review of OPHTHALMOLOGY

ANAEMIC RETINOPATHY DYSTROPHIES AND DEGENERATIONS OF THE


• Changes appear when haemoglobin falls below RETINA
5 g% RETINITIS PIGMENTOSA (PIGMENTARY RETINAL
• Pale background and pale arterioles DYSTROPHY)
• Dilated veins
• Superficial haemorrhages and subhyaloid Inheritance
haemorrhage • Autosomal recessive—most common and most
• A few Roth’s spots and cotton wool spots severe
• Autosomal dominant—common and relatively
LEUCAEMIC RETINOPATHY benign
• Pale and orange fundus background • Sex linked—least common, relatively severe.
• Dilated and tortuous veins Incidence
• Superficial haemorrhages, Roth’s spots and
subhyaloid haemorrhage. • 1 person per 5000 of the world population
• Perivascular leucaemic infiltration • Male: female ratio is 3: 2
• Bilateral equal involvement.
RETINOPATHY OF PREMATURITY Clinical features
(RETROLENTAL FIBROPLASIA)
• Night blindness—may occur before the retinal
Occurs in premature infants (weighing less than
changes, appear.
1300 gm) exposed to high concentration of oxygen.
Ii is an important cause of leukocoria in children. Fundus changes
Clinical features • Retinal pigmentary changes occur in the form of
perivascular deposition (jet black spots) of bony
• Stage 1 : Formation of demarcation line.
corpuscle pigments in the equatorial region; which
• Stage 2 : Formation of retinal ridge.
later spread both anteriorly and posteriorly
• Stage 3 : Ridge with extraretinal fibrovascular
• Retinal arterioles are attenuated
proliferation.
• Optic disc becomes pale waxy ending in consecutive
• Stage 4 : Subtotal tractional retinal detachment.
optic atrophy.
• Stage 5 : Total retinal detachment.
Management Visual field changes
• Annular or ring scotoma
• Oxygen concentration in incubator should be kept
• Tubular vision in advanced cases.
less than 30 percent.
• Regular screening by indirect ophthalmoscopy Electrophysiological changes
between 2 and 4 months of life is most important • Appear even before the subjective symptoms and
• Ablation of peripheral avascular retina with signs
cryopexy or photocoagulation when neovas- • ERG is subnormal or abolished
cularization is detected • EOG is extinguished.
• Pars plana vitrectomy for tractional retinal
detachment. Ocular associations
• Myopia (common)
COAT’S DISEASE (EXUADATIVE RETINITIS OR • Primary open-angle glaucoma in 30 percent
EXUDATIVE RETINOPATHY OR RETINAL • Microphthalmos
TELANGIECTASIS) • Keratoconus (rare)
• It is an idiopathic condition with massive subretinal • Subcapsular cataract.
exudates and associated with telengiectatic retinal
Systemic associations
vessels and aneurysms.
• Males are more commonly affected in their first Laurence-Moon-Biedl syndrome (most common)
decade of life. • Retinitis pigmentosa
• No genetic predisposition. • Obesity
• White pupillary reflex (leucocoria) may be present. • Hypogenitalism
• Treatment includes early recognition followed by • Polydactyly
prophylactic laser. • Mental deficiency.
Diseases of Retina 95
Cockayne’s syndrome • Fluorescein angiography may show ‘ink-blot’
• Retinitis pigmentosa or ‘smoke-stack’ sign (Mushroom or umbrella
• Nystagmus configuration)
• Progressive infantile deafness • Self-limiting (90%) but recurrent (40%)
• Dwarfism • Laser photocoagulation is indicated in:
• Mental retardation – Long standing cases (more than 4 months)
• Ataxia. – Patients showing multiple leaks
Refsum’s syndrome – Impaired visual acuity in the fellow eye from
• Retinitis pigmentosa GSR.
• Peripheral neuropathy
CYSTOID MACULAR OEDEMA
• Cerebellar ataxia.
Collection of fluid in the outer plexiform (Henle’s
Usher’s syndrome
layer) and inner nuclear layer of the retina, centred
• Retinit’s pigmentosa
around fovea due to break down of the inner blood-
• Labyrinthine deafness.
retinal barrier.
Hallgren’s syndrome
• Retinitis pigmentosa Causes
• Vestibulocerebellar ataxia • Postoperative complication of cataract and
• Congenital deafness keratoplasty operations.
• Mental deficiency. • Inflammations such as pars planitis, and posterior
uveitis
ATYPICAL FORMS OF RETINITIS PIGMENTOSA • Retinal vascular disorders, diabetic retinopathy,
• Retinitis pigmentosa sine pigmento: All features of and retinal vein occlusion.
retinitis pigmentosa, except no or minimal visible • Retinal dystrophy–Retinitis pigmentosa
pigmentary changes. • Side effect of topical 2% adrenaline in aphakic
• Sectorial retinitis pigmentosa: Only one quadrant patients.
(usually nasal) or one half (usually inferior) is • Secondary to other maculopathies
involved. • Complication of peripheral retinal photo­coagu­
• Pericentric retinitis pigmentosa: Pigmentary lation.
changes are confined to an area immediately Clinical features
around the macula.
• Minimal to moderate loss of vision which may be
• Retinitis punctata albescens: Autosomal dominant,
permanent in persistent cases
innumerable discrete white dots are scattered over
• Typical fundus picture in advanced cases is of
the fundus.
‘Honey-comb appearance’ of macula
• Fluorescein angiography in well established cases
MACULAR DISORDERS
presents a ‘flower-petal appearance’
PHOTORETINITIS • Long standing CME may end in macular hole.
• Damage of the foveolar region caused by ultraviolet
AGE-RELATED (SENILE) MACULAR DEGENERATION
ray and infra-red rays of bright sun-light, commonly
(AMD)
occurring during solar eclipse (Eclipse burn).
• Typical lesion appears as a bean or kidney shaped Non-exudative AMD
pigmented spot with yellowish white centre in the • Also called as dry or geographical AMD
foveal region. • Comparatively common (90% cases) and causes
gradually progressive mild to moderate loss of
CENTRAL SEROUS RETINOPATHY (CSR)
vision
• It refers to spontaneous detachment of neuro­ • Fundus shows: Colloid bodies, pale areas of retinal
sensory retina in the macular region pigment epithelium (RPE) atrophy, and irregular
• Typically affects males between 20 and 40 years or clustered pigmentation.
causing sudden painless mild loss of vision (6/9–
6/12), micropsia, metamorphopsia and relative Exudative AMD
positive scotoma • Also called as wet or neovascular AMD
96 Review of OPHTHALMOLOGY

• Comparatively rare (10% cases) but causes RETINAL DETACHMENT


progressive and marked loss of vision.
Retinal detachment (RD) refers to separation
Clinical course rapidly passes through following of neuro­ensory retina from the retinal pigment
stages: epithelium (RPE).
• Stages of drusen formation
• Stage of detachment of RPE. I. RHEGMATOGENOUS RD
• Stage of subretinal neovascular membrane This is the commonest type of retinal detachment.
(SRNVM)
Predisposing factors
• Stage of haemorrhagic detachment of RPE.
• Stage of haemorrhagic detachment of neurosensory • Age—Most common 40–60 years (no bar)
retina • Sex—Male: Female: 3:2
• Terminal stage of disciform macular scarring • Myopia—40 percent cases
(degeneration). • Aphakia (previous cataract surgery)
• Retinal degenerations such as:
Treatment includes – Lattice degeneration (most common)
• Intravitreal injections of anti-VEGF (e.g. avastin) – Snail track degeneration
are recommended
– White-with-pressure and white-without pressure
• Photodynamic therapy (PDT) is the treatment of
lesions
choice after anti-VEGF
– Diffuse chorioretinal degenerations
• Laser photocoagulation (double frequency YAG
– Acquired (senile) retinoschisis
532) may be considered for an extrafoveal SRNVM
• Trauma
situated 200 µ or more from the centre of fovea.
• Senile posterior vitreous detachment (PVD).
HEREDOMACULAR DEGENERATIONS Clinical features

Best’s disease Prevalence


Affects 1 in 1000 population each year
• Autosomal dominant.
Prodromal symptoms
Clinical picture. More commonly it occurs upto 6
years of age. In it loss of vision is not profound. Five • Photopsia (flashes of light) due to vitreoretinal
stages are: traction
• Pre-vitelliform stage: Normal fundus but EOG is • Dark spots (floaters) in front of the eyes (muscae
abnormal. volitantes)
• Vitelliform stage: Egg yolk lesion at macula. Symptoms of detached retina
• Pseudohypopyon stage: Partially absorbed egg yolk • Localized relative loss in the field of vision of
lesion. detachment retina
• Vitelliruptive stage: A scrambled egg appearance • Loss of vision in detachments involving macular
at macula area.
• Stage of scarring: Hypertrophic, atrophic or
vascularized scar at macula. Signs
Note: ERG is normal but EOG is abnormal. • Detached retina gives grey reflex, is raised, thrown
Stargardt’s disease into folds which oscillate with the movements of
the eye
• Autosomal recessive disease
• Retinal breaks holes (round, horse-shoe-shape
• No family history present
or slit like) look reddish and are most frequently
• Presents with central decreased vision in first or
second decade of life (more commonly less than found in the periphery (commonest in the upper
21 years). temporal quadrant)
• Typical fundus picture is ‘beaten-bronze’ or ‘snail- • Signs of old RD are:
slime reflex’ in the macular area. – THinning of the detached retina
• In this ERG and EOG both are normal – Secondary intraretinal cysts
• On fluorescein angiography dark choroid stars are – Subretinal demarcation lines (high water
seen. marks).
Diseases of Retina 97
Management • Other causes: Choroidal neovascularisation,
haemangioma and metastatic tumour of choroid.
Indications of prophylactic treatment
Clinical features
• Symptomatic retinal breaks associated with PVD
• Large asymptomatic retinal tears Exudative RD can be differentiated from rhegmato­
• Asymptomatic retinal break and/or lattice genous RD by following characteristics:
degeneration in the presence of: aphakia, high • Absence of photopsia
myopia, only eye, RD in fellow eye, strong family • Absence of retinal breaks, folds and undulations
history of RD, and Marfan’s syndrome, Stickler’s • Configuration of the RD is convex
syndrome and Ehlers-Danlos syndrome. • Shifting fluid sign is pathognomic of exudative RD
• On transillumination, rhegmatogenous RD is
Basic principles of treatment
transparent while exudative RD is opaque.
1. Sealing of retinal breaks: It can be performed with
cryopexy or photocoagulation. III. TRACTIONAL RETINAL DETACHMENT
2. Bringing the sclera, choroid and detached retina It occurs due to retina being mechanically pulled
with each other. It is carried out by the procedure away from its bed by the progressive contraction of
of external tamponade (scleral buckling or fibrous or fibrovascular membrane over large areas
encirclage) or internal tamponade (PPV with of vitreo-retinal adhesions (vitreo-retinal traction
intravitreal silicone oil). bands).
Indications for drainage of subretinal fluid (SRF) Causes
• Difficulty in localizing retinal breaks in bullous • Post-traumatic retraction of scar tissue
RD • Proliferative diabetic retinopathy
• Long-standing (old) RD • Sickle-cell retinopathy
• RD with inferior breaks • Retinopathy of prematurity
• RD with immobile retina • Eales’ disease
• RD with advanced glaucoma, thin sclera and recent • Plastic cyclitis
cataract extraction. • Post-vitreous haemorrhage retinitis proliferans.
II. EXUDATIVE OR SOLID RETINAL DETACHMENT Clinical features
Retina is elevated due to accumulation of fluid • Presence of vitreoretinal traction bands with lesions
beneath it, associated with RPE damage: of the causative disease
• Configuration of the detached retina is concave
Causes
• Absent—photopsia, floaters, retinal breaks, shifting
1. Systemic diseases fluid sign, mobility of detached retina.
• Toxaemia of pregnancy
• Renal hypertension RETINOSCHISIS
• Blood dyscrasias
• Characterized by the abnormal splitting of the
• Polyarteritis nodosa.
retina’s neurosensory layer, usually in the outer
2. Ocular diseases
plexiform layer, resulting in a loss of vision in the
• Inflammations
corresponding visual field in some rarer forms.
– Harada’s disease
– Sympathetic ophthalmitis • Usually asymptomatic
– Posterior scleritis • It is classified into degenerative (typical and
– Orbital cellulitis reticular), hereditary (X- linked juvenile retino­
• Vascular diseases schisis, familial foveal retinoschisis), and tractional
– Exudative retinopathy of coats and exudative (secondary to optic disc pit).
– Central serous retinopathy
• Neoplasms TUMOURS OF RETINA
– Retinoblastoma (exophytic) RETINOBLASTOMA
– Malignant melanoma of choroid
• Sudden hypotony following: Incidence
– Globe perforation • Most common intraocular tumour of childhood
– Intraocular operation • Occurs in 1 in 15000–20000 live births.
98 Review of OPHTHALMOLOGY

• Age—Usually presents at 18 months (1–2 years) • Hyphaema (rare)


• Rare in Negroes • Proptosis which may mimic orbital cellulitis (rare).
• 25–30 percent cases are bilateral. Fundus examination in early stage
Heredity (before leukocoria) may show:
• Retinoblastoma (RB) gene is a cancer-suppressor Endophytic retinoblastoma which grows inwards
gene located on long arm of chromosome 13 (13q and is white or pearly pink in colour. Fine blood
14) vessels may be present on its surface. In the presence
• Deletion or inactivation of RB gene by two of calcification, it gives the typical ‘cottage-cheese’
mutations (Knudson’s two-hit hypothesis) results appearance.
in occurrence of retinoblastoma. Exophytic retinoblastoma which grows outwards
• Of all cases 10% are familial (inherited by autosomal and causes exudative retinal detachment.
dominant mode) and 90% are sporadic Investigations
• Of sporadic cases, about two-third (60% of all cases)
• Plain X-rays, orbit— may show calcification in 75%
occur by somatic mutation and one-third (30% of
cases.
all cases) occur by germline mutation.
• Ultrasonography and CT scanning are quite useful.
• Heritable or germline cases (10% familial + 30%
• Lactic dehydrogenase (LDH) levels are raised in
sporadic = 40%) occur by first mutation in RB
aqueous humour.
gene on germ cells (gametes) before fertilization
and second mutation in RB gene on retinal cells. Differential diagnosis
These cases have multifocal, usually bilateral (85%) Differential diagnosis of leukocoria
tumours and can transmit to 50% of offspring. • Congenital cataract
• Non-heritable or somatic cases (60%) occur • Retinopathy of prematurity
sporadically by both hits (mutations) in RB gene of • Persistent hyperplastic primary vitreous
retinal cells in embryo. These cases have unifocal, • Toxocara endophthalmitis
unilateral tumours which cannot be passed on to • Coats’ disease
the off spring. • Coloboma of choroid
• Retinal dysplasia.
Pathology
Differential diagnosis of endophytic retinoblastoma
• Arises from the immature retinal neural cells
discovered on fundus examination:
• Histologically, cells may present as highly
• Patch of exudative choroiditis
undifferentiated or well differentiated tumour
• Astrocytoma
• Features of well-differentiated tumour include:
– Flexner-Wintersteiner rosettes (highly specific Differential diagnosis of exophytic retinoblastoma
for retinoblastoma) causing exudative retinal detachment:
– Homer-Wright rosettes (also seen in neuro­ • From other causes of exudative RD, e.g. Coat’s
blastoma and medulloepithelioma) disease.
– Pseudorosettes International classification of retinoblastoma (ICRB)
– Fleurettes Group A: (very low risk) includes all small tumours
• Other histological features of retinoblastoma are: <3 mm in greatest dimension, confined to retina,
– Areas of necrosis located >3 mm from fovea and > 1.5 mm from the
– Calcification. optic disc.
Clinical features
Group B: (low risk) includes large tumours
>3 mm in dimension, and any size tumours located
Presenting Symptoms <3 mm from fovea, and <1.5 mm from the optic
• Leukocoria (61%) disc margin.
• Squint (22%) Group C: (moderate risk) includes retinoblastoma
• Nystagmus in bilateral cases with focal seeds characterized by subretinal and/or
• Secondary glaucoma and buphthalmos (rare) vitreous seeds <3 mm from the retinoblastoma
• Pseudohypopyon (rare) Group D: (high risk) includes retinoblastoma with
• Anterior chamber inflammation which may mimic diffuse seeds characterized by subretinal and/or
anterior uveitis (rare) vitreous > 3 mm seeds from the retinoblastoma.
Diseases of Retina 99
Group E: (very high risk) includes extensive retino­ • Retinal lesions comprise vascular dilatation,
blastoma characterised by any of the following: tortuosity and formation of aneurysms which
tumour touching the lens, neovascular glaucoma, vary from small miliary to balloon-like angiomas,
tumour anterior to anterior vitreous face involving followed by appearance of haemorrhages and
ciliary body and anterior segment, diffuse infiltrating exudates, resembling exudative retinopathy of
tumour, opaque media with hemorrhage, tumour Coats’. Massive exudation is frequently complicated
necrosis with aseptic orbital cellulitis, invasion of by retinal detachment.
postlaminar optic nerve, choroid, sclera, orbit, and 2. Tuberous sclerosis (Bourneville disease)
anterior chamber, or phthisis bulbi.
Classic diagnostic triad includes:
Treatment • Adenoma sebaceum
Enucleation • Mental retardation
• It is treatment of choice for intraocular tumour of • Epilepsy associated with potato-like hamartomas
more than 10 mm in size or when optic nerve is of the brain, retina and viscera.
involved. 3. Neurofibromatosis (Von-Recklinghausen’s disease)
• The eyeball should be enucleated with maximum
• Multiple tumours of skin, nervous system and other
length of the optic nerve and taking care not to
organs
perforate it.
• Cutaneous manifestations vary from cafe-au-lait
Conservative (tumour destructive) therapy spots to neurofibromata
It is indicated when tumour is less than 10 mm in • Ocular manifestations include: Neurofibromas of
size and optic nerve is not involved. lids and orbit, glioma of optic nerve and congenital
glaucoma.
Tumour destructive therapy consists of chemo­
reductions with systemic carboplatin, vincristine Characterstic features of neurofibromatosis
and etoposide followed by focal therapy with any of (NF-1) and neurofibromatosis (NF-2) are as
following modalities: follows:
• Cobalt plaques: For small peripheral tumour. • NF-1 has bare orbit/orphan Anne sign.
• Cryotherapy: For small peripheral tumour. • NF-2 is associated with bilateral acoustic neuroma,
• Argon, diode or xenon arc photocoagulation: For posterior subcapsular cataract and multiple
small posterior tumours not invading either the schawnoma, meningioma and ependymoma.
macula or the optic nerve. 4. Encephalofacial angiomatosis
Surgical debulking of the orbit with radiotherapy (Sturge-Weber syndrome) Characterised by
(external beam therapy) and chemotherapy angiomatosis in the form of port wine stain (Naevus
(etoposide, carboplatin, vincristine) are used as flammeus) involving one side of the face which may
palliative measures. In patients with either orbital be associated with choroidal haemangiomas, lepto­
extension, or intracranial extension or distant meningeal angioma and congenital glaucoma on the
metastasis. affected side.
Prognosis To remember Mnemonic (STURGE) stands for:
S : Port wine Stain, Sezuires
Fair (survival rate 70–85%) if eyeball is enucleated,
T : Tram track appearance on X-ray skull
before the extraocular extension.
U : Unilateral cortical atrophy
Poor prognostic factors are: R : Retardation (mental)
• Optic nerve involvement beyond the transection G : Glaucoma (congenital)
(65% mortality). E : Encephalofacial angiomatosis
• Undifferentiated tumour cells (40% mortality)
• Massive choroidal invasion. MISCELLANEOUS POINTS
• Superficial retinal haemorrhages are most
PHAKOMATOSES
commonly associated with hypertension.
1. Angiomatosis retinae (von Hippel-Lindau’s disease) • Pseudoxanthoma elasticum is the most common
• Angiomatosis involves retina, brain, spinal cord, systemic disorder associated with angioid streaks;
kidneys and adrenals other causes are: macular degeneration, Paget’s
100 Review of OPHTHALMOLOGY

disease, Gronblad-strandberg syndrome, senile • Retinoblastoma is the most common retinal


elastosis. malignancy.
• Angioid streaks: • Retinoblastoma is the most common primary
– May be mistaken for blood vessels malignant intraocular tumour of childhood and
– Are usually situated near the disc the second most common primary intraocular
– Are situated at a deeper level than retinal vessels malignancy of all age groups (Choroidal melanoma
– Result due to defects in Bruch’s membrane. is more common).
• Rods are primarily affected in retinitis pigmentosa. • Most common mode of metastasis of retinoblastoma
• Lattice degeneration most commonly occurs in the is by direct extension, by continuity to the optic
superotemporal quadrant. nerve.
• Salt and pepper fundus is characteristic of • Most common site of metastasis of retinoblastoma
congenital syphilitic retinal affection. It is also
is brain.
seen in Leber’s amaurosis and congenital rubella.
• Snow ball opacities in vitreous are seen in pars
• Bull’s eye macular lesions are seen in chloroquine
planitits, candidiasis and sarcoidosis.
retinopathy and progressive cone dystrophy.
• Causes of ring scotoma include retinitis pigmen­
• Reversal of diabetic retinopathy can be seen in a
woman with Sheehan’s syndrome (due to lack of tosa, high myopia, primary open angle glaucoma,
growth hormone). aphakic spectacles correction and after pan retinal
• Colour vision is not significantly impaired in eyes photocoagulation (PRP).
with early macular disease, in contrast to eyes with • Causes of tubular vision include retinitis pigmen­
early lesions of optic nerve. tosa, high myopia, primary open angle glaucoma,
• Macular star may occur in hypertensive retino­ and CRAO with sparing cilioretinal artery.
pathy, papilledema and Eales’ disease. • Juxta foveal telangiectasia is a variant of Coat’s
• Flame-shaped superficial haemorrhages occur in disease. It is macular telangiectasia which in
the nerve fibre layer of the retina. structural abnormalities are seen in the vessels.
Diseases of Retina 101

Multiple Choice Questions (MCQs)

1. Congenital anomalies of the optic disc include all 7. Risk factors for development of diabetic retino­
except: pathy include:
A. Coloboma A. Duration of diabetes
B. Drusen B. Heredity
C. Hypoplasia C. Pregnancy
D. Medullated nerve fibres D. All of the above

2. Normally retinal fibres are: 8. Diabetic retinopathy is essentially an angiopathy


A. Medullated affecting retinal:
B. Non-medullated A. Precapillary arterioles
C. Medullated in childhood and become non- B. Capillaries
medullated in old age C. Venules
D. Non-medullated in childhood but become D. All of the above
medullated in old age 9. All of the following are true for anaemic
retinopathy except:
3. All of the following are true for medullated nerve
A. Occurs when haemoglobin level falls below 5 gm
fibres of the retina except:
percent
A. Appear as a whitish patch usually present around
B. Arterioles become pale
the disc C. Veins are pale and narrow
B. Are commonly seen at birth in premature D. Superficial retinal haemorrhages and subhyaloid
children haemorrhage are seen invariably
C. Disappear in patients with optic atrophy
D. May cause enlargement of blind spot 10. Is not true about soft exudates (cotton wool
patches on retina):
4. All of the following are features of central retinal A. Are a sign of retinal hypoxia
artery occlusion except: B. Formed due to swelling of nerve fibre layer
A. Marked narrowing of retinal arterioles C. Frequently change their shape
B. ‘Cherry red spot’ at macula D. In late stages are converted into hard exudates
C. Retinal oedema
D. Multiple superficial haemorrhages 11. Hard exudates are seen in the following except:
A. Hypertensive retinopathy
5. The most common site of obstruction in central B. Diabetic retinopathy
retinal artery is: C. Leukemic retinopathy
A. In front of the lamina cribrosa D. Exudative retinopathy of Coat’s
B. At the lamina cribrosa
C. Behind the lamina cribrosa 12. All of the following are true for retinopathy of
prematurity except:
D. The point where the artery enters the optic nerve
A. Occurs in premature infants due to late crying
6. Dot and blot (round) retinal haemorrhages are B. Due to hypoxia there occurs neovasculari­zation
situated at the level of: followed by fibroproliferation
A. Ganglion cell layer C. End result is bilateral blindness
B. Nerve fibre layer D. Blindness can be prevented by early diagnosis
C. Inner nuclear layer and ablation of vascular premature retina with
D. Outer plexiform layer cryotherapy or photocoagulation

1 : D 2 : B 3 : B 4 : D 5 : B 6:D 7 : D 8 : D 9 : C 10 : D 11 : C 12 : A
102 Review of OPHTHALMOLOGY

13. Most common mode of inheritance for retinitis B. Macular oedema


pigmentosa is: C. Detachment of pigment epithelium
A. Autosomal recessive D. Detachment of choroid
B. Autosomal dominant
22. Macular scar is formed in which of the following
C. Sex linked
diseases:
D. None of the above
A. Papillitis
14. Following are the ocular association of retinitis B. Hypertension
pigmentosa except: C. Neuroretinitis
A. Myopia D. Papilloedema
B. Primary angle closure glaucoma E. All of the above
C. Microphthalmos
23. In cystoid macular edema fluid collects in the
D. Conical cornea
macular region at the level of:
15. Retinitis pigmentosa forms a constituent of all the A. Outer nuclear layer
following syndromes except: B. Outer plexiform layer
A. Lowe’s syndrome C. Inner plexiform layer
B. Refsum’s syndrome D. Between pigment epithelium and neuro­sensory
C. Usher’s syndrome retina
D. Hallgren’s syndrome
24. In cystoid macular edema, basic defect is:
16. All of the following are causes of night blind­ness A. Breakdown of inner blood retinal barrier
except: B. Breakdown of outer blood retinal barrier
A. Retinitis punctate albescens C. Increased permeability of the choriocapillaris
B. Choroideremia D. All of the above
C. Retinitis pigmentosa
25. Retinal degenerations predisposed to retinal
D. Retinitis of Roth
detachment include all of the following except:
17. ‘Bull’s eye’ lesion in macular region is seen in: A. Lattice degeneration
A. Progressive cone dystrophy B. Snail track degeneration
B. Choroquine maculopathy C. Focal pigment clumps
C. Both of the above D. Pavingstone degeneration
D. None of the above
26. All of the following are the causes of exudative
18. Cherry red spot at macula is seen in all of the retinal detachment except:
following except: A. Retinopathy of toxaemia of pregnancy
A. Tay-Sach’s disease B. Retinopathy of prematurity
B. Letterer-Siwe disease C. Exudative retinopathy of Coats
C. Niemann-Pick disease D. Sympathetic ophthalmia
D. Commotio retinae
27. Shifting fluid is pathognomonic of:
19. Photoretinitis results from: A. Solid retinal detachment
A. Infra-red rays of sunlight B. Rhegmatogenous retinal detachment
B. Ultraviolet rays of sunlight C. Tractional retinal detachment
C. Both of the above D. Choroidal detachment
D. None of the above
28. All of the following are true about incidence of
20. Spontaneous regression of proliferative retinoblastoma except:
retinopathy may occur in: A. Is the most common intraocular tumour of
A. Diabetic retinopathy childhood
B. Proliferative sickle retinopathy B. Occurs in 1 in 14000–34000 live births
C. Retinopathy of prematurity C. No sex predisposition
D. All of the above D. More common in negroes than whites
E. None of the above
29. Retinoblastoma arises from:
21. In central serous retinopathy, in the macular A. Layer of rods and cones
region, there occurs: B. Layer of bipolar cells
A. Spontaneous detachment of neuro-sensory C. Ganglion cells layers
retina D. Any nucleated retinal layer
13 : A 14 : B 15 : A 16 : D 17 : C 18 : B 22 : E 23 : B 24 : A 25 : D 26 : B 27 : A
19 : A 20 : D 21 : A 28 : D 29 : D
Diseases of Retina 103
30. In a child with phthisical eye of unknown etiology 39. The true pulse in the retinal arteries is seen in:
following possibility should be kept in mind: A. Aortic aneurysm
A. Birth trauma B. Aortic regurgitation
B. Still’s disease C. Exophthalmic goitre
C. Retinoblastoma D. All of the above
D. Untreated congenital glaucoma E. None of the above
31. All of the following are causes of leukocoria 40. Commonest eye tumour is:
except: A. Melanoma
A. Coloboma of choroid B. Retinoblastoma
B. Coloboma of optic disc C. Carcinoma of eyelid
C. Retinopathy of prematurity D. Carcinoma of lacrimal sac
D. Retinoblastoma
41. Visual loss in diabetic retinopathy is due to all
32. Poor prognostic factor in retinoblastoma is: except:
A. Optic nerve involvement A. Cataract formation
B. Massive choroidal invasion B. Background diabetic retinopathy
C. Undifferentiated tumour cells C. Ischaemic maculopathy
D. All of the above D. Vitreous hemorrhage
33. Which is not seen in retinitis pigmentosa: 42. The earliest change noticed in hypertensive
A. Pale waxy optic disc retinopathy is:
B. Attenuated vessels A. Soft exudate
C. Retinal haemorrhages B. Arteriolar spasm
D. Bone corpuscles C. Venospasm
34. Best diagnostic test for Best’s disease is: D. Hard exudate
A. Dark adaptation
43. Primary retinal detachment is seen in:
B. ERG
C. EOG A. Diabetes
D. Gonioscopy B. High myopia
C. Malignant melanoma
35. All are true about angioid streaks except: D. All of the above
A. They are mistaken for blood vessels
B. Are situated near the disc 44. Premature babies can have the following:
C. Situated at a superficial level than the retinal A. Buphthalmos
vessels B. Retinopathy of prematurity
D. Result due to defects in Bruch’s membrane C. Increased incidence of myopia
E. Are associated with pseudoxanthoma elasticum D. Persistent hyaloid artery
E. All of the above
36. Eales’ disease is best treated with:
A. Antitubercular drugs 45. Earliest change in diabetic retinopathy is:
B. Steroids A. Hard exudate
C. Antibiotic drops B. Soft exudate
D. Antibiotics systemically C. Dot haemorrhage
37. The commonest cause of rubeosis iridis is D. Microaneurysm
A. Diabetes mellitus
B. Central retinal vein occlusion 46. Ring scotoma is seen in:
C. Central retinal artery occlusion A. Glaucoma
D. Carotid stenosis B. Retinitis pigmentosa
C. Sympathetic ophthalmia
38. The retinal capillaries are distinguished by having D. Vitreous haemorrhage
all, except:
A. Widening of the capillary calibre in the periphery 47. Retinal detachment is preceeded by:
B. Capillary free zone around the veins A. Floaters and flashes
C. Superficial network B. Eales’ disease
D. Deep denser network C. Trauma
E. Capillary-free-zone around the arteries D. All of the above

30 : C 31 : B 32 : D 33 : C 34 : C 35 : C 39 : D 40 : B 41 : B 42 : B 43 : B 44 : B
36 : B 37 : A 38 : B 45 : D 46 : B 47 : D
104 Review of OPHTHALMOLOGY

48. Resuscitation time of the human retina following B. CRVO


ischaemia is: C. Proliferative retinopathy
A. 30 minutes D. Coat’s disease
B. 45 minutes E. Episcleritis
C. 1 to 2 hours
57. The superficial retinal haemorrhages are situated
D. 15 to 20 minutes
in:
E. 8 minutes
A. Nerve fibre layer
49. Purtscher’s retinopathy results from: B. Outer nuclear layer
A. Head injuries C. Inner nuclear layer
B. Chest injuries D. Inner plexiform layer
C. Trichiasis
D. All of the above 58. Large haemangioma of lid and cheek along with
glaucoma is seen in:
50. In recently recognised IDDM patient examination A. von Recklinghausen’s disease
of fundus is done at: B. Sturge Weber’s syndrome
A. Immediately C. von Hippel’s disease
B. At 1 year D. Lindau’s disease
C. At 5 year
D. None of the above 59. The following about retinoblastoma is true except:
51. For prevention of retrolental fibroplasia O2 should A. Autosomal dominant
be: B. Treatment is enucleation
A. 30–40% C. Radiotherapy is also given
B. 50–60% D. Evisceration is the treatment
C. 20–30% 60. Retinoblastoma is bilateral in:
D. 70–80%
A. 1% of cases
52. Commonest cause of loss of vision in non- B. 50% of cases
proliferative diabetic retinopathy is: C. 100% of cases
A. Vitreous haemorrhage D. 30% of cases
B. Macular edema
C. Detachment of retina 61. Familial retinoblastoma:
D. Subretinal haemorrhage A. Has autosomal recessive inheritance
B. More commonly bilateral
53. All of the following are differential diagnosis for C. Due to mutation
cotton wool spots in fundus except: D. More common than sporadic retinoblastoma
A. AIDS
B. Diabetic retinopathy 62. Ideal treatment of B/L retinoblastoma:
C. Hypertension A. Enucleation
D. Eales’ disease B. Radiotherapy
C. Chemotherapy
54. Sudden loss of vision in patient with diabetic
retinopathy is due to: D. Phototherapy
A. Cataract E. Cyclophotocoagulation
B. Glaucoma 63. Most common second malignancy in patients with
C. Vitreous defects familial retinoblastoma is:
D. Papilloedema
A. Teratoma
55. Enlarging dot sign in fundus fluorescein scanning B. Medullary carcinoma
is seen in: C. Osteosarcoma
A. Cystoid macular edema D. Malignant melanoma
B. Central serous retinopathy
64. In photocoagulation of retina, which quadrant is
C. Significant macular edema
first coagulated:
D. Coat’s disease
A. Temporal
56. In a young patient presenting with recurrent B. Nasal
vitreous haemorrhage diagnosis is: C. Superior
A. Eales’ disease D. Inferior

48 : C 49 : B 50 : C 51 : A 52 : B 53 : D 57 : A 58 : B 59 : D 60 : D 61 : B 62 : A
54 : C 55 : B 56 : A 63 : C 64 : A
Diseases of Retina 105
65. Pseudo-rosettes are seen in: EXTRA EDGE QUESTIONS
A. Retinoblastoma
B. Ophthalmic nodosa 75. Soft exudates are found in:
C. Phakolytic glaucoma A. DM
D. Trachoma
B. HTN
66. Knudson’s hypothesis is applied for: C. Toxemia
A. Glaucoma D. Coat’s disease
B. Retinoblastoma E. All
C. Cataract
D. Melanoma 76. Hard exudates not seen in:
A. Hypertension
67. True about fovea centralis: B. DM
A. Cones present
C. Toxemia of pregnancy
B. Visual acquity lowest
D. SLE
C. Optic nerve passes
D. Rods present E. Coat’s disease
E. Visual acquity highest 77. Hard exudates are seen in all except:
68. Broadest neuroretinal rim is seen in: A. Diabetic retinopathy
A. Sup role B. Retinitis pigmentosa
B. Inf pole C. Eale’s disease
C. Nasal pole D. Retinal artery macroaneurysm
D. Temporal lobe E. Choroidal neovascularisation
69. Cherry red spot is seen in: 78. Cherry red spot found in:
A. Eale’s disease A. Gangliosidosis
B. Retinitis pigmentosa B. Retinopathy of prematurity
C. Central retinal artery thrombosis
C. Tay–Sach’s disease
D. Central retinal vein occlusion
D. Gaucher’s disease
70. All of the following take part in the pathogenesis E. Retinal detachment
of macular edema in diabetic retinopathy except:
A. Retinal pigment epithelium dysfunction 79. Features of non-proliferative DR is all except:
B. Oxidative stress A. Neovascularisation
C. VEGF B. Soft exudate
D. Increased protein kinase-C C. Hard exudate
D. Vitreous detachment
71. Vitreous haemorrhage in diabetic retinopathy
E. Cotton-wool spot
A. Non-proliferative diabetic retinopathy
B. Prolifertive diabetic retinopathy 80. Diabetic retinopathy is characterized by:
C. Both A. Hard exudates
D. None B. Neovascularisation
72. Grid laser photocoagulation is indicated in: C. Glaucoma
A. Ischaemic maculopathy D. Cataract
B. Clinical significant macular oedema E. Retinal detachment
C. Macular hole
D. Proliferative diabetes retinopathy 81. Treatment of diabetic retinopathy
A. Phocoemulsification
73. Panretinal photocoagulation is indicated in: B. Retina laser photocoagulation
A. Macular edema C. LASIK
B. Retinal breaks D. Pars plana vitrectomy
C. Proliferative diabetic retinopathy
D. Tractional retinal detachment 82. Treatment of diabetic retinopathy neovascula­
74. In retinal detachment: risation is/are:
A. Effusion of fluid into the suprachoroidal space A. Retinal laser photocoagulation
B. Retinoschisis B. Pars plana vitrectomy
C. Separation of sensory retina from pigment C. Phacoemulsion
epithelium D. Anti VEGE ab
D. None of the above E. LASIK

65 : A 66 : B 67 : A and E 68 : B 69 : C 75 : A, B and C 76 : C 77 : B 78 : A, C and D 79 : A and D


70 : A 71 : B 72 : B 73 : C 74 : C 80 : A, B, C, D and E 81 : B and D 82 : A and D
106 Review of OPHTHALMOLOGY

83. Which of the following agents is not used in 91. A young adults presents with night blindness
the treatment of Diabetic Macular Edema/ and tubular vision. On examination, intraocular
Retinopathy: pressure was obser ved to be 18 mm and
A. Ruboxistaurim the anterior segment was unremarkable.
B. Pyridazinones Fundoscopy showed attenuation of arterioles
C. Benfotiamine and waxy pallor of the optic disc with bony
D. Tamoxifen corpuscles like spicules of pigmentation in mid
peripheral retina. Ring scotomas were observed
84. All of the following may be used to differentiate
central retinal venous occlusion (CRVO) from on perimetry. Which of the following is the most
ocular ischemic syndrome due to carotidartery likely diagnosis:
stenosis, except: A. Pigmentary retinal dystrophy
A. Dilated retinal vein B. Primary open angle glaucoma
B. Tortuous retinal vein C. Lattice degeneration of retina
C. Retinal artery pressure D. Diabetic retinopathy.
D. Ophthalmodynamometry 92. A 70-year-old man presents with deterioration
85. Commonest type of retinal detachment: of vision 3 weeks after cataract extraction
A. Rheugmatogenous and IOL implantation. Slit lamp examination
B. Choroidal haemorrhage shows honeycomb maculopathy and fluorescin
C. Exudative a n g i o g r a p h y ( FA ) s h o w s ‘ f l o w e r p e t a l ’
D. Tractional hyperfluorescence. The most likely diagnosis
is:
86. Retinal detachment seen in: A. Age related macular degeneration (ARMD)
A. Myopia B. Central serous retinopathy (CSR)
B. Hypermetropia C. Macular dystrophy
C. Eale’s disease D. Cystoid macular edema
D. Cataract extraction
E. Nd:YAG photocoagulation 93. Bull’s eye lesion seen with:
A. Chloroquine
87. Cause of exudative retinal detachment are: B. Dapsone
A. Scleritis C. Rifampicin
B. Toxaemia of pregnancy D. Ethambutol
C. Dysthyroid eye disease E. Gold
D. Central serous retinopathy
E. Sickle cell retinopathy 94. A patient with clinically significant diabetic
macular edema with non progressive diabetic
88. Causes of exudative retinal detachment: retinopathy was treated with non-progressive
A. Central retinal artery occlusion diabetic retinopathy was treated with Macular
B. Hypertensive retinopathy grid photocoagulation. The patient still has vitreo
C. Harada’s syndrome macular traction. What is the preferred treatment?
D. Coat’s disease A. Intravitreal bevacizumab
89. RD is diagnosed by: B. Pars plana vitrectomy
A. +90 D C. Repeat macular grid photocoagulation
B. Hurby lens D. Augmented macular photocoagulation
C. 3 mirror contact lens
95. Mizuo-phenomenon is seen in:
D. Direct ophthalmoscopy
E. Indirect ophthalmoscopy A. Fundus albipathicus
B. Fundus flavimaculatus
90. A patient presented with sudden onset of floater C. Oguchi’s disease
and perception of falling of a curtain (veil) in front D. Choroideremia
of the eye which one of the following is the most
appropriate diagnosis: 96. Which retinal layer is most radioresistant:
A. Retinal detachment A. RPE
B. Eale’s disease B. Layer of rods and cones
C. Vitreous haemorrhage C. Bipolar cell layer
D. Glaucoma D. Ganglion cell layer

83 : D 84 : A 85 : A 86 : A, C and D 87 : A, B and D
91 : A 92 : D 93 : A 94 : B 95 : C 96 : B
88 : C and D 89 : A, B, C, D and E 90 : A
Diseases of Retina 107
97. ‘Sea-Fan’ retina is seen in: C. Sub-retinal neovascularization
A. CRAO D. Anterior chamber angle malformation
B. SLE
105. Vortex Vein invasion is commonly seen in:
C. Sickle cell disease
A. Retinoblastoma
D. Gaucher’s disease.
B. Malignant melanoma
98. Central retinal artery occlusion is known to be C. Optic nerve glioma
associated with: D. Medullo-epithelioma
A. Panophthalmitis
106. All of them are true except:
B. Diabetic retinopathy
A. Retiniblastomas are hereditry 40%, non-
C. CMV retinitis
D. Orbital mucormycosis. hereditary 60%
B. Retiniblastomas are B/L 30%, U/L 70%
99. Lamina cribrosa is absent in: C. Retiniblastomas are familial 6%, non-familial
A. Morning glory syndrome 94%
B. Nanophthalmia D. All are true.
C. Colobama of retina
D. Optic nerve agenesis 107. Which is the only phacomatosis to be inherited on
an autosomal recessive basis:
100. A young patient presents to the ophthal­mology A. Ataxia-telangiectasia
clinic with loss of central vision. There is no obvious B. Sturge-Weber syndrome
family history. ERG and EOG were observed to be C. von Hippel lindau syndrome
normal which of the following is the most likely D. Neurofibromatosis
diagnosis:
A. Stargardt’s disease 108. Which of these features is a classic radiologic
B. Best’s vitelliform dystrophy feature of optic nerve glioma:
C. Retinitis pigmentosa A. Tram-track appearance of optic nerve
D. Cone-rod dystrophy B. Kingking of optic nerve
C. Multiple cystic spaces in optic nerve
101. A young patient presents to the ophthal­mology D. Adjacent bony erosion
clinic with loss of central vision. ERG is normal but
EOG is abnormal. Which of the following is the 109. Retinitis pigmentosa is a feature of all except:
most likely diagnosis: A. Refsum’s disease
A. Stargardt’s disease B. Hallervorden Spatz Syndrome
B. Best’s vitelliform dystrophy C. NARP
C. Retinitis pigmentosa D. Abetalipoprofeinemia
D. Cone-rod dystrophy
110. Bergmeister papilla are present on the:
102. A young patient present with significant loss A. Anterior corneal surface
of central vision and a normal ERG. There is no B. Near the optic disc
obvious family history of similar presentation. C. Anterior lens surface
The most likely diagnosis: D. Posterior lens surface
A. Best’s disease
B. Stargardt’s disease 111. Which of the following is not an ophthalmic
C. Retinitis pigmentosa emergency:
D. Cone-rod dystrophy A. Macular hole
103. Von Recklinghausen disease is associated with: B. Retinal Detachment
A. Glaucoma C. CRAO
B. Optic nerve glioma D. Acute primary angle closure glaucoma
C. Neurofibroma of the lids 112. Marfan’s syndrome associated with a/e:
D. All of them
A. Retinal detachment
104. Von Recklinghausen disease is associated with: B. Viteroius hemorrhage
A. Glaucoma C. Ectopic lentis
B. Choroidal hemangioma D. Roth spots

97 : C > B 98 : D 99 : A 100 : A 105 : B 106 : D 107 : A 108 : B 109 : B 110 : B


101 : B 102 : B 103 : D 104 : A>D 111 : A 112 : D
CHAPTER

12
Neuro-ophthalmology

Quick Text Review


LESIONS OF VISUAL PATHWAY 2. Marcus Gumt pupil (Relative afferent
Lesions of visual pathway are summarized in Table conduction defect): See Page 11
12.1. 3. Efferent pathway defect: In this situation both
direct and consensual light reflexes are absent on
ABNORMAL PUPILLARY REFLEXES the affected side and present on the normal side.
Causes of efferent pathway defect include: effect
1. Amaurotic (pupil) light reflex (Total afferent of parasympatholytic drugs like atropine and
conduction defect): It refers to absence of direct homatropine, internal ophthalmoplegia and third
light reflex on the affected side and absence of nerve paralysis.
consensual reflex on the normal side. It indicates 4. Wernicke’s hemianopic pupil: It is seen in
blindness due to lesions of the optic nerve or retina. lesions of one optic tract. In this condition ipsilateral
Near reflex is normal and pupils are equal in size in direct and contralateral consensual light reflexes
both eyes. are absent when light is thrown on the temporal half

Table 12.1: Lesions of visual pathway

Site of lesion Salient features Common causes


•  Optic nerve –  Ipsilateral blindness, –  Optic atrophy,
–  Absent ipsilateral direct light reflex, –  Traumatic avulsion of optic nerve
–  Absent contra-lateral consensual light –  Acute optic neuritis
reflex
•  Saggital (central) –  Bitemporal hemianopia –  Supra-sellar aneurysms,
lesions of the –  Pituitary tumours,
chiasma –  Craniophary-ngioma,
–  Glioma of third ventricle
•  Lateral chiasmal –  Binasal hemianopia –  Distention of third ventricle,
lesions –  Atheroma of posterior communicating arteries

•  Optic tract –  Homonymous heminanopia – Syphlitic meningitis


–  Wernick’s hemianopic pupillary response – Tuberculosis and tumours of optic thalamus
– Aneurysms of superior cerebellar or posterior cerebral arteries
•  Lateral geniculate –  Homonymous hemianopia with sparing –  Syphlitic meningitis
body of pupillary reflexes –  Tuberculosis and tumours of optic thalamus
•  Optic radiations –  Homonymous hemianopia (sometimes – Vascular occlusion
  – Total fibres sparing the macula) –  Primary and secondary tumours, and
  – Lower fibres only –  Homonymous upper –  Trauma
  – Upper fibres only –  quadrantinopia (pie-in the sky) – Temporal lobe lesions
–  Homonymous lower quadrantinopia  – Anterior parietal lobe lesion
(Pie-in the floor)
Visual cortex –  Homonymous hemianopia   – Vascular occlusion
(usually sparing the macula) –  Primary and secondary tumour
–  Trauma
Neuro-ophthalmology 109
of the retina on the affected side and nasal half of • Earliest sign of optic neuritis is relative afferent
the opposite side. pupillary defect (RAPD).
5. Argyll Robertson pupil(ARP) • Ophthalmoscopic findings are seen in papillitis,
• Near reflex and accommodation present while fundus is normal in retrobulbar neuritis as
(Mnemonic ARP: Accommodation Reflex Present) the part involved is behind the visible part of the
• Light reflex absent optic disc.
• Pupil: Bilateral symmetric, small and irregular (Iris • Leber’s disease is a type of hereditary optic neuritis
damage). which primarily affects males around the age of 20
• Poor dilation to mydriatics and in darkroom. years and is transmitted by female carriers.
• Causes: Neurosyphilis, diabetes mellitus and
TOXIC AMBLYOPIAS
multiple sclerosis.
• Pseudo Argyll Robertson pupil seen in aberrant • Tobacco-alcohol-amblyopia occurring in heavy
degeneration of 3rd nerve. smokers and drinkers is characterized by foggy
6. Adies tonic pupil (Holmes-Adies pupil) vision associated with bilateral centrocaecal
• Reaction to light absent scotomas.
• Near reflex very slow and tonic • Methyl alcohol amblyopia results from degene­
• Pupil usually unilateral, affected pupil is slightly ration of ganglion cells of the retina, due to effect of
larger (anisocoria) (NO MIOSIS) formic acid and formaldehyde which are metabolic
• It constricts with weak pilocarpine (0.125%), while products of methyl alcohol.
normal pupil does not, (denervated iris sphincter is • Quinine amblyopia may occur even with small
suprasensitive to topical parasympathomimetics). doses in susceptible individuals. Visual loss may
• It may be associated with absent knee jerk and be associated with deafness and tinnitus.
occur more often in young women. • Ethambutol amblyopia may recover completely in
• Causes : Post-ganglionic parasympathetic most of the cases after immediate cessation of the
pupillomotor damage. drug.

ANTERIOR ISCHAEMIC OPTIC NEUROPATHY (AION)


DISEASES OF OPTIC NERVE
• Results from the occlusion of short posterior ciliary
OPTIC NEURITIS arteries
• Occurs in two forms arteretic and nonarteretic
• Optic neuritis includes papillitis and retrobulbar
• Arteretic AION caused by giant cell arteritis
neuritis
accounts for 10% cases
• The most common cause is multiple sclerosis,
• Nonarteretic AION (90% cases)—exact etiology not
vitamin B12 deficiency, vitamin B2 deficiency and
known. Risk factors include: structural crowing of
diabetes.
disc with small cup, cataract surgery, hypertension,
Clinical features diabetes mellitus, smoking, hyperlipidemia and
• The condition is usually unilateral, there is drugs such as sildenafil (viagra).
monocular sudden, progressive and profound • Characterized by inferior altitudinal hemianopic
vision loss. field defects.
• Occasionally patients may observe an altered
perception of moving objects (Pulfrich pheno­ AUTOIMMUNE OPTIC NEUROPATHY
menon) or a worsening of symptoms with exercise • Denotes optic nerve involvement in patients with
or increase in body temperature (Uhthoff sign) systemic lupus erythematosis (SLE) and other
• There is pain behind the eyeball, particularly in systemic collagen vascular disorders.
retrobulbar neuritis in which there is pain on eye • Pathogenesis and features are similar to AION.
movement.
• Visual evoked response (VEP) shows reduced PAPILLOEDEMA (CHOKED DISC)
amplitude and delayed transmission time.
• Papilloedema is non-inflammatory edema of optic
Signs disc
• Reduced visual acuity and the most common • Raised intracranial pressure is the commonest
field defect in optic neuritis is a relative central or cause of bilateral papilloedema (optic nerve
centrocaecal scotoma. oedema).
110 Review of OPHTHALMOLOGY

• Tumour of cerebellum, midbrain and parieto- 2. Consecutive optic atrophy: It occurs following
occipital region produce papilloedema more destruction of the ganglion cells secondary to
rapidly. degenerative or inflammatory lesions of the
• Tumour of medulla rarely produce papilloedema. choroid and retina. Its common causes are retinitis
• Foster kennedy syndrome characterized by pressure pigmentosa, diffuse chorioretinitis, pathological
optic atrophy on the side of lesion and papilloedema myopia and occlusion of central retinal artery.
on the otherside is a feature of olfactory groove Its ophthalmoscopic features include: yellow
meningioma and frontal lobe tumours. waxy colour, not so sharply defined margins and
• Pathogenesis of papilloedema involves attenuation of vessels.
Due to disturbance of pressure gradient across 3. Postneuritic optic atrophy: It develops as a
the lamina cribosa sequel to long-standing papilloedema or papillitis.
↓ The disc looks dirty white in colour, its edges are
Stasis of Axoplamic in prelaminar area blurred, physiological cup is obliterated, lamina
↓ cribrosa is not visible, vessels are attenuated and
Axonal swelling perivascular sheathing is often present.

Venous congestion 4. Glaucomatous optic atrophy: It results due
↓ to the effect of raised intraocular pressure. It is
Extra-cellular edema characterised by deep and wide cupping of the
optic disc and nasal shift of the vessels.
Clinical features
• Vision is normal initially 5. Vascular (ischaemic) optic atrophy: It results
• Typically, there is recurring brief episodes from the conditions producing disc ischaemia
(transient) of visual obscuration (Amaurosis fugax) (other than glaucoma) such as: central retinal artery
• Initially there is enlargement of blind spot and occlusion, giant cell arteritis, severe haemorrhage,
progressive contraction of the visual field. severe anaemia and quinine poisoning. Its
• Thus, characteristically there is gradually ophthalmoscopic features are pallor associated
progressive painless loss of vision. with marked vascular attenuation.
Ophthalmoscopic findings Note:
• Blurring of optic disc margin in papilloedema first • Pallor of the disc in optic atrophy is not due to
of all involves nasal margins, followed by superior atrophy of the nerve fibres but due to loss of
and inferior margin and lastly temporal margin. vascularity.
• Venous engorgement and venous congestion • Pallor of the disc can not be correlated with the
• Loss of venous pulsations amount of visual loss.
• There is filling of physiological cup with gradual
obliteration of physiological cup SYMPTOMATIC DISTURBANCES OF THE VISION
• Paton’s lines: Radial retinal lines cascading from
Night blindness (Nyctalopia)
the optic disc.
Its common causes are: vitamin A deficiency, tape-
OPTIC ATROPHY to-retinal degenerations (e.g. retinitis pigmentosa),
1. Primary optic atrophy: It results from lesions congenital high myopia, and Oguchi’s disease.
proximal to the optic disc without antecedent Day blindness (Hamarlopia)
papilloedema. Its common causes are multiple
sclerosis, idiopathic retrobulbar neuritis, Leber’s Its causes are central corneal and central lenticular
and other hereditary optic atrophy, pituitary opacities and congenital deficiency of cones.
tumours, toxic amblyopia, tabes dorsalis and Colour blindness
trauma to the optic nerve.
Ophthalmoscopically, disc is chalky white in a. Congenital colour blindness:
colour, margins are well defined, lamina cribrosa It is an inherited condition affecting males more
is visible, retinal vessels and surrounding retina are (3-4%) than females (0.4%). It may be of following
normal. types:
Neuro-ophthalmology 111
i. Anomalous trichromatic colour vision: In this • Papilloedema
one of the primary colour i.e., red, green or blue is • Giant cell arteritis
defective and the condition is called protano malous, • Raynaud’s disease
deuteranomalous or tritanomalous, respectively. • Migraine
• Prodromal symptom of central retinal artery or
ii. Dichromatic colour vision: Here faculty to
carotid artery occlusion
percieve one of the three primary colours viz. red,
• Hypertensive retinopathy
green or blue is completely absent and the condition
• Venous stasis retinopathy.
is called protanopia, deuteranopia or tritanopia
respectively.
MISCELLANEOUS POINTS
iii. Blue cone Monochromatism (BCM): Here • Suprageniculate lesions of visual pathway usually
only hues of blue colour can be appreciated. It is produce visual field defects with macular sparing.
very rare condition. • Optic nerve lesions produce negative scotomas
iv. Achromatopsia: It refers to total colour blindness. whereas macular lesions cause positive scotoma.
It is an extremely rare condition occurring due to • Chromophobe adenoma is the most common
congenital absence of the cones. Therefore, it is primary intracranial tumour producing neuro­
associated with day blindness and nystagmus. ophthalmological features.
• Gaze evoked amaurosis is seen in optic nerve
b. Acquired colour blindness sheath meningioma.
It may follow damage to macula or optic nerve, • Horner’s syndrome (lack of sympathetic inner­
where red, green discrimination is particularly vation) is characterized by miosis, mild ptosis,
affected. Acquired blue colour defect may occur in mild enophthalmos, anhydrosis of the face on
old age due to increased sclerosis of the crystalline the affected side. Loss of cilio-spinal reflex and
lens. heterochromia (ipsilateral iris is of light colour).
Tests to confirm diagnosis of Horner’s syndrome
Tests for colour vision
are: dilation lag, and cocaine test (normal pupil
• Pseudoisochromatic charts dilates while Horner’s pupil does not dilate with
– Ishihara plates mainly to screen congenital topical cocaine).
protan and deuterons (i.e., red and green • The swelling of the optic disc in papillitis rarely
defects) exceeds 2-3D.
– Hardy- Rand- Rittler plates. • Scintilating scotoma is a feature of migraine.
• Edridge-Green lantern test • Unilateral central scotoma is the earliest symptom
• City university test. of compression of optic nerve.
• Fansworth Munsell 100 hue test (most sensitive) • Hippus (alternate rhythmatic dilation and constric­
• Fansworth D 15 hue discrimination test tion of pupils) is a feature of multiple sclerosis.
• Nagel’s anomaloscope • Er ythropsia (red coloured vision) may be
• Holmgren’s wool test experienced by some patients after cataract
Note: Currently, there is no treatment for colour extraction.
blindness. • Pupil sparing, third nerve paralysis suggests a
Amaurosis medical cause (diabetes or hypertension). While
in surgical causes (aneurysm, tumour) pupil is also
A complete loss of sight in one or both eyes, in
invovled.
the absence of ophthalmoscopic or other marked
• The two most common ocular signs of myasthenia
objective signs.
gravis are ptosis and extraocular muscle weakness
Amaurosis fugax (paralytic squint).
A sudden, temporary (lasting 2-5 minutes) and • Neuromyelitis optica (Devic’s disease) may be
painless monocular visual loss occurring due to a associated with sudden bilateral blindness.
transient failure of retinal circulation. • Papillitis and retrobulbar neuritis : Painful ocular
movement is more common in retrobulbar neuritis
Common causes than papillitis and fundus is normal in retrobulbar
• Carotid transient ischaemic attacks (TIA) neuritis while papillitis has characteristic fundus
• Embolization of retinal circulation abnormalities.
112 Review of OPHTHALMOLOGY

Multiple Choice Questions (MCQs)

1. Process of dark adaptation involves: 7. Marcus Gunn pupil is a feature of:


A. Rods of retina A. Optic neuritis
B. Cones of retina B. Papilledema
C. Pigment epithelium of retina C. Ciliary ganglion lesions
D. Both rods and cones D. Lesion of Edinger-Westphal nucleus
8. Argyll Robertson Pupil results from the lesion of:
2. Visual acuity is a measure of: A. Accessory ganglion
A. Light sense B. Ciliary ganglion
B. Form sense C. Tectum region
C. Contrast sense D. Lateral geniculate body
D. All of the above
9. All of the following are true for the Adie’s Tonic
3. Bitemporal hemianopia is associated with lesions Pupil except:
of the: A. Light reflex is present
B. Near reflex is very slow and tonic
A. Optic tract
C. It is usually unilateral
B. Central chiasma
D. The affected pupil is larger
C. Lateral parts of chiasma
D. Optic radiations 10. Pupil is small in all of the following except:
A. During sleep
4. Homonymous hemianopia with sparing of B. Adie’s pupil
pupillary reflexes is a feature of lesions of: C. Argyll Robertson pupil
A. Optic radiations D. Pontine haemorrhage
B. Visual cortex 11. Pupil is spared in:
C. Geniculate body A. Riley-Day syndrome
D. All of the above B. Disseminated sclerosis
C. Myasthenia gravis
5. Homonymous hemianopia with usually sparing
D. Horner’s syndrome
of the macula is seen in lesions of:
A. Geniculate body 12. The most common field defect in papillitis is:
B. Optic radiations A. A relative central or centrocaecal scotoma
C. Visual cortex B. Enlargement of the blind spot
D. All of the above C. Tubular vision
D. Multiple small peripheral scotomas
6. Wernicke’s heminopic pupillary reaction indicates
13. Bilateral centrocaecal scotoma more marked with
lesions at the level of:
red than white colour is a feature of:
A. Optic tract A. Tobacco amblyopia
B. Distal part of optic nerve B. Leber’s disease
C. Optic chiasma C. Papilledema
D. Optic radiations D. Quinine amblyopiafs

1 : D 2 : B 3 : B 4 : D 5 : C 6:A 7 : A 8 : C 9 : A 10 : B 11 : C 12 : A
13 : A
Neuro-ophthalmology 113
14. Basic lesion in tobacco amblyopia is: C. Olfactory groove
A. Degeneration of ganglion cells especially of D. Orbital surface of frontal lobe
macular region
22. In optic atrophy pallor of the disc is an index of:
B. Degeneration of foveal cones
A. Degeneration of optic nerve fibres
C. Ischaemia of optic nerve fibres
B. Loss of vascularty of the disc
D. All of the above
C. Demyelination of the optic nerve fibres
15. Typical field defect observed in anterior ischaemic D. All of the above
optic neuropathy is: 23. Congenital colour blindness is transmitted as:
A. Altitudinal hemianopia A. Recessive disorder
B. Paracentral scotoma B. Autosomal dominant
C. Homonymous hemianopia C. Sex-linked disorder
D. Baring of the blind spot D. All of the above
16. Tumours most common to cause early papilledema 24. Acquired blue blindness is a feature of:
arise from: A. Increased sclerosis of the crystalline lens
A. Cerebellum B. Disease of optic nerve
B. Mid brain C. Disease of macula
C. Parieto-occipital region D. All of the above
D. All of the above
25. Amaurosis fugax may occur in all of the following
17. Occlusion of short posterior ciliary arteries may conditions except:
cause: A. Papilloedema
A. Ischaemic optic neuropathy B. Papillitis
B. Posterior segment ischaemia C. Giant cell arteritis
D. Raynaud’s disease
C. Anterior segment ischaemia
D. All of the above 26. Spiral field defect is a feature of:
A. Amblyopia exanopsia
18. Pseudo-Foster-Kennedy syndrome is characterised
B. Hysterical amblyopia
by all except:
C. Malingering
A. It is associated with raised intracranial pressure
D. Toxic amblyopia
B. The culprit tumour causes optic atrophy on one
side and papilloedema on the other side 27. In uraemic amaurosis the pupils are:
C. It may be associated with pseudotumour cerebri A. Constricted
D. It needs to be differentiated from tumours arising B. Dilated and don’t react to light
from the orbital surface of frontal lobe C. Normal
D. Dilated but react to light
19. All of the following signs may be appreciated in a
patient with early papilloedema except: 28. In optic neuritis the best investigation to be done
A. Obscuration of superior, inferior and nasal disc includes:
A. Goldman perimetery
margins
B. Keratoscopy
B. Absence of spontaneous venous pulsation
C. Ophthalmoscopy
C. Mild hyperaemia of the disc
D. Opthalmodynamometery
D. Obliteration of physiological cup of the optic disc
29. Normal visual acuity of 6/6 is achieved by the age
20. The most important factor on which development of:
of papilloedema depends in intracranial tumours A. 2 years
is: B. 3 years
A. Site of tumour C. 5 years
B. Size of tumour D. 7 years
C. Nature of tumour
D. Rate of growth of tumour 30. In fundus, first sign of raised intracranial pressure
is:
21. Tumours arising from all of the following structures A. Filling of physiological cup
can induce papilloedema except: B. Blurring of nasal disc
A. Medulla oblongata C. Dilatation of vessels
B. Cerebrum D. None of the above

14 : A 15 : A 16 : D 17 : A 18 : B 19 : D 22 : B 23 : C 24 : A 25 : B 26 : B 27 : D
20 : A 21 : A 28 : A 29 : A 30 : C
114 Review of OPHTHALMOLOGY

31. M o n o c u l a r d i p l o p i a w i t h h o m o ny m o u s C. Ptosis
hemianopia originates in the: D. Loss of light reflex
A. Geniculate body
B. Calcarine cortex 40. Bitemporal hemianopia can be due to:
C. Parietal lobe A. Third ventricle tumour
D. Temporal lobe B. Meningioma of sella diaphragmatica
E. Tip of occipital lobe C. Calcarine cortex infarction
D. Aneurysm of basilar artery
32. Pupillary reflex is lost in:
A. Optic neuritis 41. Homonymous hemianopia is seen in:
B. Glaucoma A. Pituitary adenoma
C. Iridocyclitis B. Optic nerve damage
D. Adie’s pupil C. Post-chiasmic damage
D. All of the above
33. Features of papilloedema include all except:
A. May be due to intracranial haemorrhage 42. Pituitary tumour causes:
B. Disc becomes elevated A. Binasal hemianopia
C. Cotton wool spots may be seen B. Homonymous hemianopia
D. Vision is impaired C. Monocular blindness
D. Bitemporal hemianopia
34. Which is incorrect of papilloedema:
A. Disc edema 43. Psychogenic complaints are all except:
B. Transient blurring of vision A. Haloes around light
C. Sudden painless loss of vision B. Tired eyes
D. Vascular engorgement C. Periodic bluring of vision
35. Functional assessment of optic nerve is by: D. Constant blinking
A. Angiography 44. Most common type of colour blindness is:
B. Fundoscopy A. Protanopes
C. Perimetry B. Deuteranopes
D. CT Scan C. Tritanopes
36. Defect in amblyopia lies in: D. None
A. Lateral geniculate body
45. In pupillary reflex nerve tested is:
B. Afferent pupillary reflex
C. Rods and cones A. 2nd
D. Retina B. 3rd
C. Both 2nd and 3rd
37. Pupil that responds to convergence but light reflex D. 4th
is absent:
46. In optic atrophy, the optic disc appears to pale is
A. Adies pupil
index of:
B. Argyl Robertson pupil
C. Hutchison pupil A. Atrophy of the nerve fibre
D. Wernicke’s pupil B. Loss of vasculature
C. Gliosis
38. Macula is D. All of the above
A. Optic nerve
B. Optic tract 47. In case of anisocoria when 1% pilocarpine is
C. Periodic bluring of vision instilled into the eye with abnormally dilated
D. Constant blinking pupil, pupil remains dilated. Cause of anisocoria
may be:
39. An optic nerve injury may result in all of the A. Adies pupil
following except: B. Pharmacological blockage
A. Loss of vision in that eye C. Uncal herniation
B. Dilatation of pupil D. Diabetic III cranial nerve palsy

31 : B 32 : D 33 : D 34 : C 35 : C 36 : A 40 : B 41 : C 42 : D 43 : A 44 : A 45 : C
37 : B 38 : D 39 : C 46: B 47: B
Neuro-ophthalmology 115
EXTRA EDGE QUESTIONS C. Stasis of axoplasmic transport
D. Swelling of the axon
48. Headache with bitemporal hemianopia with 6/6
50. A young man with blurring of vision in right eye,
vision is seen in:
followed by left eye after 3 months, showing disc
A. Optic neuritis
hyperemia, edema, circumpapillary telangiectasia
B. Trauma
with normal pupillary response with centrocecal
C. Chiasmal lesion
scotoma on perimetry, the cause is:
D. Bilateral cavernous lesion
A. Typical optic neuritis
49. All statements are true about papilloedema except: B. Acute papilledema
A. Collection of extra-cellular fluid C. Toxic optic neuropathy
B. Disruption of neurofilament D. Leber’s hereditary optic neuropathy

48 : C 49 : B 50 : D
CHAPTER

13
Disorders of Ocular Motility

Quick Text Review


ANATOMY AND PHYSIOLOGY OF OCULAR 2. Antagonists: These have opposite action in the
MOTILITY SYSTEM same eye, e.g. medial and lateral recti-muscles.
3. Yoke muscles (contralateral synergists): It refers
PRINCIPLE MUSCLES OF ROTATION to a pair of muscles (one from each eye) which act
simultaneously in conjugate movements of the two
• Adduction: Medial rectus; also superior and
eyes. Six pairs of yoke muscles are as follows:
inferior recti.
• Abduction: Lateral rectus; also superior and Movement Yoke muscles
inferior obliques. Dextroversion Right lateral rectus and left
• Elevation: Superior rectus and inferior oblique medial rectus
• Depression: Inferior rectus and superior oblique Levoversion Left lateral rectus and right
• Intorsion: Superior oblique and superior medial rectus
rectus. Dextroelevation Right superior rectus and
• Extorsion: Inferior oblique and inferior rectus. left inferior oblique
Actions of extraocular muscles Levoelevation Left superior rectus and
Muscle Primary action Subsidiary action
right inferior oblique
Superior rectus Elevation Adduction,
Dextrodepression Right inferior rectus and
Intorsion left superior oblique
Inferior rectus Depression Adduction, Levodepression Left inferior rectus and
Extorsion right superior oblique
Medial rectus Adduction —
Lateral rectus Abduction —
TYPE OF FIXATION MOVEMENT

Superior oblique Intorsion Depression, I. Versions: Eye movements in which both eyes
Abduction move in the same direction are called versions.
Inferior oblique Extorsion Elevation, These include:
Abduction • Saccades are rapid conjugate eye movements.
Note: Mnemonic to remember actions of muscles: These are of two types, voluntary saccades and
RAD (Recti are Adductors except lateral rectus) reflex saccades (involuntary or reflective)
• Smooth pursuit (following eye movements): These
Nerve supply of extraocular muscles are voluntary and slow conjugate eye movements.
All EOMs are supplied by 3rd nerve, except the • Vestibulocular reflex (VOR)
lateral rectus by 6th nerve and superior oblique by • Optokinetic movement, these are slow reflexive
4th nerve. (involuntary) conjugate movements.
Synergists, antagonists and yoke muscles II. Vergences: These are slow and disconjugate eye
1. Synergists: These muscles have the similar movements in which the eye moves in opposite
primary action in the same eye, e.g. superior directions. These includes:
rectus and inferior oblique of the same eye act as • Convergence
synergistic elevators. • Divergence
Disorders of Ocular Motility 117
Note: Mnemonic to remember: Grade III—Stereopsis: It consists of the ability to
perceive the third dimension (depth perception).
For saccades is FIF
F—Fast eye movement SQUINT
I—Involuntary
F—Frontal lobe (opposite) Definition and classification
For pursuit is SVP A misalignment of the visual axes of the two eyes
S—Slow eye movements is called squint or strabismus. Broadly it can be
V—Voluntary classified as below:
P—Parito-occipital lobe (same) I. Apparent squint or pseudostrabismus
Laws governing ocular movements II. Latent squint (Heterophoria)
III. Manifest squint (Heterotropia)
1. Hering’s law of equal innervation: According
1. Concomitant squint
to it, an equal and simultaneous innervation flows
2. Incomitant squint
from the brain to a pair of yoke muscles during
conjugate movements. For example, during Estimation of angle of deviation
dextroversion right lateral rectus and left medial • Hirschberg corneal reflex test. Roughly, the angle
rectus receive equal innervation. of squint is 15° and 45° when the corneal light
2. Sherrington’s law of reciprocal innervation: reflex falls on the border of pupil and limbus,
According to it, during eye movements an increased respectively.
flow of innervation to the contracting muscle is • The prism and cover test (Prism bar cover test, i.e.
accompanied by a decreased flow of innervation PBCT): This will measure the amount of deviation
to the antagonist muscle. For example, during in prism dioptres. Both heterophoria as well as
dextroversion an increased flow to right LR and the heterotropia can be measured by this test.
left MR is accompanied by decreased flow to right • Krimsky corneal reflex test: The power of prism
MR and left LR. required to centre the light reflex in the squinting
eye equals the amount of squint in prism
BINOCULAR SINGLE VISION dioptres.
It is a conditioned reflex which is not present since • Maddox-rod test: The Maddox rod converts the
birth but is acquired during first 6 months and is point light image into a line. Thus, the patient will
completed during first few years. see a point light with one eye and a red line with the
other. Due to disimilar images of the eyes, fusion
Prerequisites for development of
binocular single vision
is broken and heterophoria becomes manifest.
The number on Maddox tangent scale where the
1. Straight eyes starting from the neonatal period red line falls will be the amount of heterophoria in
with precise co-ordination for all directions of gaze degrees.
(motor mechanism). • Maddox wing test: By it the amount of phoria for
2. Reasonably clear vision in both eyes so that similar near (at a distance of 33 cm) can be measured. It
images are presented to each retina (sensory is also based on the basic principle of dissociation
mechanism). of fusion by dissimilar objects.
3. Ability of visual cortex to promote binocular single • Measurement of deviation with synoptophore: Both
vision (mental process). objective and subjective angle of squint can be
Grades of binocular single vision measured accurately with synoptophore.
Grade I—Simultaneous perception: It is the power
to see two dissimilar objects simultaneously. For PSEUDOSTRABISMUS
example, when a picture of a bird is projected on 1. Pseudoesotropia or apparent convergent squint
to the right eye and that of a cage on to the left may be associated with a prominent epicanthal
eye; an individual with presence of simultaneous fold and negative angle kappa.
perception will see the bird in the cage. 2. Pseudoexotropia or apparent divergent squint may
Grade II—Fusion: It consists of the power to be associated with hypertelorism, (a condition of
superimpose two incomplete but similar images to wide separation of the two eyes) and positive angle
form one complete image. kappa.
118 Review of OPHTHALMOLOGY

ABNORMALITIES OF BINOCULAR VASION Concomitant squint


Suppression. It is a temporary active cortical It is a type of manifest squint, in which the amount
inhibition of the image of an object formed at the of deviation remains constant (unaltered) in all
retina of the squinting eye. This phenomenon occurs the directions of gaze; and there is no associated
only during binocular vision (with both eyes open). limitation of ocular movements.
Amblyopia. It is a uniocular impairment of vision in Concomitant esotropia
the absence of any organic disease of ocular media Infantile esotropia usually presents within the
and visual pathway. Strabismic amblyopia results first 6 months of life. It is characterised by fairly
from prolonged uniocular suppression. large angle of squint (>30°), alternate fixation in
Abnormal retinal correspondence (ARC). If fovea primary gaze and crossed fixation in lateral gaze.
of the normal eye and an extrafoveal point on the Binocular vision (both eye fixing simulatneously)
retina of the squinting eye acquire a common visual does not develop. Inferior oblique over action may
direction (become corresponding points). be present initially or develop later and dissociated
vertical deviation develop in 80% by age of 3 years.
HETEROPHORIA
Treatment: Surgery is the treatment of choice. The
It is also known as ‘latent strabismus’. It is a condition
usual recommended time is between 6 months-2
in which the tendency of the eyes to deviate is kept
years of age (and preferably before 1 year of age)
latent by fusion. Therefore, when the influence of
fusion is removed, the visual axis of one eye deviates Refractive accommodative esotropia (Normal
away. Factors predisposing to decompensation: AC/A ratio): It usually develops at the age of 2–3
(i) inadequacy of fusional reserve, (ii) general years and is associated with high hypermetropia
debility and lowered vitality, (iii) psychosis, (+4 to +7D). Mostly it is fully correctable by use of
neurosis, and mental stress, (iv) precision of job; spectacles.
and (v) advancing age. Non-refractive accommodative esotropia: It is
Types of heterophoria
caused by abnormally high AC/A (accommodative
convergence/accommodation) ratio. Esotropia
Esophoria: It is the tendency of the eyes to rotate is greater for near than the distance (minimal or
inwards towards nose. no deviation for distance). It is fully corrected by
Exophoria: It is a tendency of the eyes to rotate out adding +3D S for near vision.
towards temple. Sensory deprivation (secondary) esotropia
Hyperphoria: It is a tendency to deviate upwards, results from monocular lesions (in childhood)
while hypophoria is a tendency to deviate down­ which either prevent the development of normal
wards. However, in practice it is customary to use binocular vision or interfere with its maintenance.
the term right or left hyperphoria depending on the Examples of such lesions are cataract, severe
eye which remains up as compared to the other. congenital ptosis, aphakia, anisometropia, optic
Cyclophoria: It is a tendency to rotate around atrophy, retinoblastoma, central chorioretinitis and
the anteroposterior axis. When the 12 O’clock so on.
meridian of cornea rotates nasally, it is called Consecutive esotropia results from surgical
incyclophoria and when it rotates temporally it is overcorrection of exotropia.
called excyclophoria. Concomitant exotropia
Symptoms Congenital exotropia is rare and almost always
Decompensated heterophoria is associated with present at birth. It is characterised by a fairly
multiple symptoms. Cyclophoria is the rarest type large angle of squint, usually alternating with
of heterophoria but gives greatest discomfort. homonymous fixation in lateral gaze, and no
amblyopia.
HETEROTROPIA Primary exotropia usually starts at the age of
It refers to manifest deviation of the visual axis 2 years as intermittent exotropia. It is associated
of the eye under binocular conditions. It is with normal fusion and no amblyopia. If not treated
of two main types, namely, concomitant and in time, it decompensates to become constant
incomitant. Sensory adaptations in patients with exotropia. It is usually alternating (ADS) with no
heterotropia. binocular single vision.
Disorders of Ocular Motility 119
• Secondary (sensory deprivation) exotropia is a 2. Confusion occurs due to formation of image of two
constant unilateral deviation which results from different objects on the corresponding points of
long-standing monocular lesions (in adults), two retinae.
associated with low vision in the affected eye. 3. Nausea and vertigo result from diplopia and
• Consecutive exotropia is a constant unilateral confusion.
exotropia which results either due to surgical over- 4. Ocular deviation is of sudden onset.
correction of esotropia, or spontaneous conversion
Signs
of small degree esotropia with amblyopia into
exotropia. 1. Primary deviation: It is deviation of the affected
eye and is away from the action of paralysed
Incomitant squint
muscle, e.g. if lateral rectus is paralysed the eye is
It is a type of heterotropia (manifest squint) in converged.
which the amount of deviation varies in different 2. Secondary deviation: It is deviation of the normal
directions of gaze. eye seen under cover; when the patient is made
‘A’ and ‘V’ pattern heterotropias to fix with the squinting eye. It is greater than the
The term ‘A’ or ‘ V’ pattern squint is labelled when primary deviation.
the amount of deviation in squinting eye varies 3. Restriction of ocular movements: It occurs in the
by more than 10° in upward or downward gaze in direction of the action of paralysed muscles.
comparison to that in primary position. 4. Compensatory head posture: It is adopted to avoid
diplopia and confusion. Head is turned towards
‘A’ and ‘V’ Esotropia: In ‘A’ esotropia the amount of
the direction of action of the paralysed muscle.
deviation increases in upward gaze and decreases in
5. False projection or orientation: It can be demon­
downward gaze. The reverse occurs in ‘V’ esotropia.
strated by asking the patient to close the sound
‘A’ and ‘V’ Exotropia: In ‘A’ exotropia the amount eye and then to fix an object placed on the side of
of deviation decreases in upward gaze and paralysed muscle. Patient will locate it to further
increases in downward gaze. The reverse occurs in away in the same direction.
‘V’ exotropia. 6. There is no amblyopia and visual acuity is normal.
Special ocular motility defects Pathological sequelae of an extraocular muscle palsy
1. Duane’s retraction syndrome. It is a congenital 1. Overaction of the contralateral synergistic muscle
ocular motility defect occurring due to fibrous 2. Contracture of the direct antagonist muscle
tightening of lateral or medial or both rectus 3. Secondary inhibitional palsy of the contralateral
muscles. Its features are: limitation of abduction or antagonist muscle.
adduction or both, and retraction of the globe and
Clinical varieties of ocular palsies
narrowing of the palpebral fissure on attempted
adduction. Eye in the primary position may be 1. Isolated muscle paralysis: Lateral rectus and
orthotropic, esotropic or exotropic. superior oblique are the most common muscles
to be paralysed singly, as they have separate nerve
2. Brown’s superior oblique tendon sheath supply. Isolated paralysis of the remaining four
syndrome. It is a congenital ocular motility defect muscles is less common, except in congenital
due to fibrous tightening of the superior oblique lesions.
tendon. It is characterized by: limitation of elevation
of the eye in adduction (normal elevation in 2. Paralysis of the third cranial nerve can be
abduction), usually straight eyes in primary position congenital or acquired,
and positive forced duction test on attempts to Characteristic features are:
elevate eye in adduction. • Ptosis due to paralysis of LPS
• Down and out deviation of eyeball due to
Paralytic squint
unopposed action of lateral rectus and superior
Paralytic squint is the most common type of squint oblique
in adults. • Ocular movements are restricted in all directions
Symptoms except outward
1. Diplopia occurs due to formation of image on • Fixed and dilated pupil due to paralysis of sphincter
dissimilar points of the two retinae. pupillae
120 Review of OPHTHALMOLOGY

• Accommodation lost, due to paralysis of ciliary NYSTAGMUS


muscle. It is the repetitive, involuntary to and fro oscillation
3. Total ophthalmoplegia: In this condition all of the eyes.
extraocular muscles including LPS and intraocular
muscles are paralysed. It results from combined Clinical classification
paralysis of third, fourth and sixth cranial nerves. It 1. Pendular nystagmus: Velocity is equal in each
is a common feature of orbital apex syndrome and direction.
cavernous sinus thrombosis. 2. Jerk nystagmus: Has slow drift and a fast phase.
3. Mixed nystagmus: Pendular in primary position
4. External ophthalmoplegia: In this condition
all extraocular muscles are paralysed, sparing the and jerk in lateral gaze.
intraocular muscles. It results from lesions at the
Etiological calcification
level of motor nuclei sparing the Edinger-Westphal
nucleus.   I. Physiological nystagmus:
Optokinetic nystagmus(OKN). It constitutes of
5. Internuclear ophthalmoplegia: In this
saccadic and pursuit movements. Defect in OKN
condition there is lesion of the medial longitudinal
indicates parietal lobe lesion.
bundle. Internuclear ophthalmoplegia (INO) is
Note: The center of Optokinetic nystagmus is
characterised by defective action of medial rectus
parietal lobe.
on the side of lesion, horizontal nystagmus of
the opposite eye and defective convergence.The II. Motor imbalance nystagmus:
most common cause of unilateral INO is vascular 1. Congenital nystagmus
occlusion associated with diabetes mellitus and 2. Spasmus nutans: It is associated with head
that of bilateral INO is multiple sclerosis. nodding
3. Latent Nystagmus: In infantile esotropia
6. Conjugate paresis which affects both eyes
4. Ataxic Nystagmus in internuclear ophthal­
equally is produced by lesions of cerebral cortex and
mology
supranuclear pathway. In it though, position and
5. See-saw nystagmus is seen in patients with
movements of the eyes are abnormal, they maintain
their relative coordination and thus produce no bitemporal hemianopia.
diplopia. Vestibulo-ocular refexes are normal. III.Ocular nystagmus: Due to sensory deprivation.
Conjugate palsies are common in Niemann-Pick Note: Downbeat nystagmus found in lithium
disease. toxicity.
Disorders of Ocular Motility 121

Multiple Choice Questions (MCQs)

1. The muscle which makes an angle of about 51° C. Measurement of fusional reserve
with the optical axis is: D. Measurement of near point of convergence
A. Superior rectus
8. In heterophoria prism is used:
B. Superior oblique
A. With apex towards the direction of phoria in
C. Inferior rectus
glasses for treatment
D. Lateral rectus
B. With apex towards the direction of phoria for
2. The muscle which makes an angle of about 23° exercises only
with the optical axis is: C. For exercises mainly in hyperphoria
A. Superior oblique D . To detect grade III of binocular vision
B. Superior rectus 9. Conjugate fixation reflex is established by the age of:
C. Inferior oblique A. 3 weeks
D. Medial rectus B. 6 weeks
3. Which of the following is not true for binocular C. 3 months
single vision ? D. 6 months
A. Provides stereoscopic vision 10. An object located in the Panum’s area stimulates:
B. Is present since birth A. Fusion
C. Is the cause of diplopia in paralytic squint B. Confusion
D. Fusion is its second grade C. Diplopia
4. Pseudoesotropia is associated with: D. Stereopsis
A. Hypertelorism 11. Strabismic amblyopia is more common in patients
B. Positive angle kappa with:
C. Negative angle kappa A. Intermittent squint
D. None of the above B. Alternate squint
C. Constant squint
5. Pseudoexotropia is associated with:
D. Latent squint
A. Prominent epicanthal fold
B. Positive angle kappa 12. Which of the following tests helps to differentiate
C. Negative angle kappa between concomitant squint and paralytic squint?
D. None of the above A. Cover-uncover test
B. Direct cover test
6. Asthenopic symptoms are most marked with:
C. Alternate cover test
A. Cyclophoria
D. None of the above
B. Hyperphoria
C. Esophoria 13. All of the following lesions causing paralysis of
D. Exophoria extraocular muscles produce diplopia except:
A. Nuclear lesions
7. All of the following are employed to evaluate a B. Lesions of nerve trunks
case of heterophoria except: C. Lesions of neuromuscular junction
A. Maddox-rod test D. Lesions of supranuclear pathways
B. Alternate cover test E. None of the above

1 : B 2 : B 3 : B 4 : C 5 : B 6 : A 8 : A 9 : B 10 : D 11 : C 12 : C 13 : D
7:B
122 Review of OPHTHALMOLOGY

14. In paralytic convergent squint diplopia is: 23. In ‘A’-Esotropia the amount of deviation:
A. Homonymous A. Increases in upward gaze and decreases in down
B. Heteronymous gaze
C. Both of the above B. Decreases in upward gaze and increases in down
D. None of the above gaze
C. Increases in upward as well as downward gaze
15. The commonest cause of unilateral inter­nuclear
D. Decreases in upward as well as downward gaze
ophthalmoplegia is:
A. Diabetes mellitus 24. In ‘V’-Exotropia amount of deviation:
B. Hypertension A. Increases in upward gaze and decreases in down
C. Multiple sclerosis gaze
D. Lead toxicity B. Decreases in upward gaze and increases in down
gaze
16. The common cause of bilateral internuclear
C. Increases in upward as well as downward gaze
ophthalmoplegia is:
D. Decreases in upward as well as downward gaze
A. Multiple sclerosis
B. Lead toxicity 25. In caloric test left jerk nystagmus occurs when:
C. Diphtheria A. Cold water is poured in right ear
D. Diabetes mellitus B. Hot water is poured in right ear
C. Cold water is poured in left ear
17. Onset of stereopsis occurs at the age of:
D. All of the above
A. 3 to 5 months
B. 1 to 2 years 26. Secondary deviation of the eye is based on the
C. 5 years following law:
D. 7 years A. Hering’s
B. Listing’s
18. Uniocular diplopia occurs in all of the following
C. Donder’s
except:
D. Sherrington’s
A. Paralysis of inferior oblique
B. Keratoconus 27. Uncrossed diplopia is seen with:
C. Iridodialysis A. Esotropia
D. Incipient cataract B. Exotropia
C. Exophoria
19. A point that falls on horopter excites:
D. Esophoria
A. Corresponding retinal points
B. Crossed diplopia 28. All are features of paralytic squint except:
C. Confusion A. Unequal fixation
D. Stereopsis B. Vertigo
C. Amblyopia
20. Convergence insufficiency is associated with all of
D. Abnormal head position
the following except:
A. Increase in accommodation 29. Amplitude of accommodative convergence is:
B. General debility A. Altered by weak cycloplegics
C. Refractive errors B. Altered by lenses and prisms
D. Wide interpupillary distance C. Decreased in older people
D. Altered by orthoptics
21. All of the following may be present in Duane’s
retraction syndrome except: 30. Large angle Kappa gives rise to:
A. Limitation of abduction A. Pseudo squint
B. Narrowing of palpebral aperture in abduction B. Manifest squint
C. Suppression C. Latent squint
D. Head turn D. Periodic squint
22. Brown’s syndrome simulates paresis of: 31. Most uncommon type of latent strabismus is:
A. Inferior oblique A. Esophoria
B. Superior oblique B. Exophoria
C. Superior rectus C. Hyperphoria
D. Inferior rectus D. Cyclophoria

14 : B 15 : A 16 : A 17 : A 18 : A 19 : A 23 : A 24 : A 25 : A 26 : A 27 : A 28 : C
20 : A 21 : B 22 : A 29 : A 30 : A 31 : D
Disorders of Ocular Motility 123
32. In unilateral past pointing nystagmus the site of C. Rotatory
lesion is: D. Can be of any type
A. Cerebellar hemisphere
B. Lateral semicircular canal 40. Down beat nystagmus could be due to:
C. Flocculo-nodular lobe A. Cerebellar lesion
D. Superior semicircular canal B. Arnold-Chiari malformation
C. All of the above
33. Secondary deviation of the eye is an example of D. None of the above
the following law:
A. Herring’s 41. Downward and lateral gaze is action of:
B. Listing’s A. Inferior oblique
C. Sherrington’s B. Medial rectus
D. Donder’s C. Superior oblique
34. Action of right superior oblique muscle is: D. Lateral rectus
A. Dextrodepression
42. Elevators of eye:
B. Dextroelevation
A. SR and IO
C. Laevoelevation
D. Laevodepression B. IO and SO
C. IR and S
35. The best treatment for correction of 15° non- D. SO SR
accommodative esotropia in a 5-year-old child is:
A. Order refraction testing under atropine and full 43. Yolk muscle pair is:
correction A. Rt MR and Rt LR
B. Synoptophore exercise B. Rt MR and Lt LR
C. Surgical alignment C. Rt SO and Lt IO
D. Occlusion patching D. Rt SR and Lt SR

36. In right-sided sixth nerve palsy all of the following 44. Child with mild squint. Intrauterine, birth history,
are seen except: development history till date all normal. Corneal
A. Convergent squint reflex normal. All other eye parameters normal
B. Right-sided abduction loss except exaggerated epicanthal fold. Diagnosis:
C. Diplopia on dextroversion A. Pseudostrabismus
D. Head turned to left B. Accommodative squint
C. Exophoria
37. For a new-born baby with squint surgery should D. Esophoria
be done at:
A. 3-4 years 45. Which of the following best defines the “Saccade”:
B. Immediately A. Voluntary slow eye movements
C. 10-12 years B. Involuntary slow eye movement
D. 18-21 years C. Abrupt, involuntary slow eye movements
D. Abrupt, involuntary rapid eye movements
38. Hirschberg test is used to detect:
46. A patient has a right homonymous hemianopia
A. Squint
with saccadic pursuit movements and defective
B. Field defects
C. Glaucoma optokinetic nystagmus. The lesion is most likely
D. Optic atrophy to be in the :
A. Frontial lobe
39. Miners nystagmus is of which type: B. Occipital lobe
A. Lateral C. Parietal lobe
B. Vertical D. Temporal lobe

32 : A 33 : A 34 : A 35 : C 36 : D 37 : A 40 : C 41 : C 42 : A 43 : B 44 : A 45 : D
38 : A 39 : C 46 : C
124 Review of OPHTHALMOLOGY

EXTRA EDGE QUESTIONS

47. Weakness of both Adduction and Abduction to B. Left inferior oblique


seen in: C. Right superior oblique
A. Duane’s Retraction Syndrome Type 1 D. Right inferior oblique
B. Duane’s Retraction Syndrome Type 2
C. Duane’s Retraction Syndrome Type 3 51. True about infantile esotropia:
D. All A. Presented from birth
B. Large angle of deviation
48. Dilator pupillae is supplied by: C. Inferior oblique muscle hyperactivity
A. Post-ganglionic parasympathetic fibers from D. Variable angle of deviation
Edinger Westphal nucleus
E. A/W refractive error
B. Post-ganglionic sympathetic fibers from cervical
sympathetic chain 52. A patient presents with diplopia with limitation of
C. IIIrd nerve adduction in the left eye and abducting saccade
D. Sympathetic fibers from fronto-orbital branch of in the right eye.Convergence is preserved. Most
V nerve likely etiology is:
49. A 30 year old man came at the outpatient A. Partial 3rd nerve palsy
department because he had suddenly developed B. Internuclear ophthalmoplegia
double vision. On examination it was found that C. Duane’s reactionary syndrome
his right eye, when at rest was turned medially. The D. Absence of medial rectus muscle
most likely anatomical structures involved are:
53. A 26 years old male with restriction of eye
A. Medial rectus and superior division of oculomotor
movements in all directions and moderate ptosis
nerve
but with no diplopia or squint. Diagnosis is:
B. Inferior oblique and inferior division of
oculomotor nerve A. Thyroid ophthalmopathy
C. Lateral rectus and abducent nerve B. Chronic progressive external ophthal­moplegia
D. Superior rectus and trochlear nerve C. Myasthenia gravis
D. Multiple cranial nerve palsies
50. A patient presented with his head tilted towards
left. On examination, he was having left 54. Final centre for horizontal movement of eye is:
hypertropia which increased on looking towards A. Abducent nucleus
right or medially. The muscle which is most likely B. Trochlear nucleus
paralyzed is: C. Oculomotor nucleus
A. Left superior oblique D. Vestibular nucleus

47: C 48 : B 49:C 50 : A 51 : B and C 52 : B 53 : B 54 : A


CHAPTER

14
Diseases of Eyelids

Quick Text Review


INFLAMMATORY DISORDERS CHALAZION (MEIBOMIAN CYST/ TARSAL CYST)
• A chronic non-infective, non-suppurative lipo-
BLEPHARITIS granulomatous inflammation of the meibomian
gland with blockage of duct and collection of
Squamous blepharitis sebaceous material.
• It is usually associated with seborrhoea of scalp • Characterised by painless tarsal swelling which
(dandruff ). In it, glands of Zeis secrete abnormal often may be multiple (most common lid
excessive neutral lipids which are splitted by swelling).
corynebacterium acne into irritating free fatty • Treatment: Incision and curettage of chalazion
acids. (most common mode of therapy) is done from
• Characterised by accumulation of whitish soft the conjunctival side by a vertical incision.
scales along the lid margin. Intralesional injection of steroid may be effective
in soft, small and recurrent chalazion.
Bacterial blepharitis (Ulcerative blepharitis)
• Recurrence may occur in seborrhoeic dermatitis,
• Chronic staphylococcal infection of the lid margin. acne rosacea and malignant changes.
• Characterised by yellow crusts at the root of cilia. • Complications include secondary malignancy
Small ulcers, which bleed easily are seen on (Meibomian gland adenocarcinoma or sebaceous
removing the crusts. Anterior lid margin shows cell carcinoma).
dilated blood vessels (rosettes).
• Complications and sequelae are: HORDEOLUM INTERNUM
– Recurrent styes (very common) • An acute primary staphylococcal infection of
– Recurrent conjunctivitis (common) meibomian gland or secondary infection in a
– Marginal keratitis (common) chalazion.
– Madarosis, poliosis, tylosis and eversion of • Symptoms are similar to stye except that pain is
punctum. more intense, due to the swelling being embedded
Posterior blepharitis (Meibomitis) deeply in the dense fibrous tissue.
• A meibomian gland dysfunction, seen more • It can be differentiated from hordeolum externum
commonly in patients with acne rosacea and by the facts that in it, the point of maximum
seborrhoeic dermatitis. tenderness and swelling is away from the lid margin
• Foam in the tears (meibomian seborrhoea) is and that pus usually points on the conjunctival side
characteristic feature. and not on the root of cilia.

STYE (HORDEOLUM EXTERNUM) MOLLUSCUM CONTAGIOSUM


• An acute suppurative staphylococcal infection of • A viral infection (large pox virus) commonly
gland of Zeis or gland of moll and lash follicle. affecting children with small, pale umbilicated lid
• Recurrent styes are more common in patients nodules.
with asthenopia, diabetes mellitus and chronic • Complications: Ipsilateral chronic follicular
blepharitis. conjunctivitis and epithelial keratitis.
126 Review of OPHTHALMOLOGY

DISORDERS OF EYE LASHES AND LID MARGIN 4. Spastic entropion


Occurs following spasm of orbicularis due to chronic
TRICHIASIS ocular irritation or following tight bandaging.
An inward misdirection of eye lashes with normal More common in elderly and frequently involves
position of lid margin. lower lid.
Causes 5. Mechanical entropion
Trachoma (commonest), ulcerative blepha­ritis, Occurs due to lack of support provided by the globe
healed membranous conjunctivitis, hordeolum to lids as in phthisis bulbi, enophthalmos, after
externum, mechanical injuries, burns and operative enucleation or evisceration operation.
scar on the lid margin.
Treatment ECTROPION

• Epilation is easy but temporary method, as • Out rolling or outward turning of the lid margin.
recurrences occur within 3–6 weeks. • Epiphora is the main symptom in ectropion.
• Electrolysis: A current of 2 mA is passed for 10 1. Congenital ectropion is very rare, but may be
seconds into the lash root. Recurrences are not rare. seen in Down’s syndrome and blepharophimosis
• Cryoepilation: The cryoprobe with –20°C is syndrome.
applied for 20–25 seconds. It is the best method
for segmental trichiasis. Its main disadvantage is 2. Involutional (senile) ectropion
depigmentation of the skin. • Most common variety, involves the lower lid.
• Argon laser destruction is effective but may need • Occurs due to following age-related changes:
to be repeated. – Senile laxity of the lid tissues
– Loss of the tone of orbicularis muscle
ENTROPION – Weakness of medial and lateral canthal tendons.
An inward turning of lid margin. 3. Cicatricial ectropion. Occurs due to skin
1. Congenital entropion scarring as in thermal burns, chemical burns,
A rare condition seen since birth. Lower lid lacerating injuries and skin ulcers.
retractors not well developed. 4. Paralytic ectropion. Occurs in facial nerve palsy
2. Cicatricial entropion in lower lid.
Common variety, more frequently involves the SYMBLEPHARON
upper lid
• Causes—scarring of palpebral conjunctiva • Adhesions of the lids with the eyeball
in trachoma (most common), membranous • Caus es: Chemical burns, thermal burns,
conjunctivitis, chemical burns, pemphigus and membranous conjunctivitis, ocular pemphigus,
Stevens-Johnson syndrome. Stevens-Johnson syndrome
• Types: Anterior, posterior and total.
3. Involutional (senile) entropion
Common variety, affects the lower lid in elderly. ANKYLOBLEPHARON
Etiological factors • Refers to adhesions between upper and lower lid
• Weakening or dehiscence of capsulopalpebral margins.
fascia (lower lid retractors). • Usually associated with symblepharon.
• Degeneration of palpebral connective tissue
allowing preseptal fibres to override the pretarsal LAGOPHTHALMOS
fibres. • Inability to voluntarily close the eye-lids
• Horizontal laxity of the lid. • Causes: Orbicularis paralysis, severe ectropion,
Management symblepharon, marked proptosis, overcorrection
• Tucking or plication of inferior lid retractors (Jones, of ptosis and comatosed patient.
Reech and Wobing operation)
• Modified Wheeler’s operation BLEPHAROSPASM
• Weiss operation • Essential (spontaneous) blepharospasm is a rare
• Bick’s procedure idiopathic condition occurring in elderly patients
Diseases of Eyelids 127
• Reflex blepharospasm occurs in conditions of Causes of bilateral ptosis
chronic ocular irritations • Myasthenia gravis
• Treatment: Eliminate the cause of reflex blepharo­ • Myotonic dystrophy
spasm • Kearns-sayre syndrome.
– Botulinum toxin injection into the orbicularis
Causes of pseudoptosis
muscle.
– Facial denervation in severe cases. • Microphthalmos
• Anophthalmos
PTOSIS • Enophthalmos
• Phthisis bulbi.
Drooping of the upper lid more than the normal
(2 mm). Evaluation of ptosis
I. Degree of ptosis
Congenital myogenic ptosis
• Mild: 2 mm
• Congenital weakness of levator palpebrae supe­ • Moderate: 3 mm
rioris (LPS) muscle. • Severe: 4 mm or more.
• It is the most common congenital anomaly of the
II. Levator function by Burke’s method
lid.
• Normal: 15 mm
• It is characterized by dropping of one or both
• Good: 8 mm or more
lids at birth, with diminished or absent lid crease
• Fair: 5–7 mm
and lid lag on downgaze due to tethering effect of
• Poor: 4 mm or less.
abnormal muscle. It may occur in following forms:
– Simple congenital ptosis: Not associated with III. MRD 1
other anomaly Marginal reflex distance-b/w upper lid margin and
– Congenital ptosis with superior rectus weakness. light reflex in primary gaze, normal is 4- 4.5 mm.
– Blepharohimosis syndrome: Congenital ptosis, IV. Vertical fissure height:
blepharophimosis, telecanthus and epicanthus Male-: 7-10 mm
inversus. Female-: 8-12mm
– Congenital synkinetic ptosis (Marcus Gunn-
winking Ptosis): Retraction of ptotic lid with jaw V. Special tests
movement like chewing, i.e., with stimulation of • Tensilon (edrophonium) test for suspected
ipsilateral pterygoid muscle. myasthenia
• Phenylepherine test for suspected Horner’s
Note: Marcus-Gunn Jaw-Winking syndrome occurs
syndrome.
due to nuclear or infra-nuclear connection between
IIIrd and Vth nerve. Treatment

Acquired ptosis 1. Fasanella—Servat operation (Excision of upper


border of tarsus together with lower border of
1. Neurogenic ptosis. Third nerve palsy, ophthal­ muller muscle and overlying conjunctiva)–for mild
moplegic migraine, multiple sclerosis. Horner’s ptosis with good levator action. It is also indicated
syndrome occurs in: cervical adenitis, thyroid in Horner’s syndrome.
operation, aortic aneurysm, syringomyelia, Pancoast’s 2. Levator resection—for moderate and severe ptosis.
syndrome, trauma, cervical cord tumours and • Blaskovics’ operation (conjunctival approach)
multiple sclerosis. • Everbusch’s operation (skin approach)
2. Myogenic ptosis. Myasthenia gravis, dystrophica 3. Frontalis sling operation (Brow suspension)
myotonica, ocular myopathy, oculopharyngeal • For severe ptosis with poor levator function
muscular dystrophy, trauma to LPS muscle, • Materials used—autogenous fascia lata (best),
thyrotoxicosis, Lambert-Eaton myasthenia syndrome. supramid, prolene, silicone.
3. Aponeurotic ptosis. Involutional (senile) TUMOURS OF EYE LIDS
ptosis, post-cataract operation, blepharochalasis,
traumatic dehiscence or disinsertion of the A. BENIGN TUMOURS
aponeurosis. 1. Simple papilloma
4. Mechanical ptosis. Lid tumours, multiple • Most common tumour
chalazia, lid oedema. • Usually occurs at the lid margin.
128 Review of OPHTHALMOLOGY

2. Xanthelasma • Incidence is much less than the basal cell carcinoma


• Creamy-yellow plaque like lesion • Common site—lid margin
• Occurs on upper and lower lids near the inner • Presentations
canthus – An ulcerated growth with elevated and indurated
• More common in middle aged women, diabetics margins (common)
and those with high cholesterol level. – Fungating or polypoidal growth (rare)
• Metastasis: Preauricular and submandibular
3. Capillary haemangioma
lymph nodes
• A childhood tumour, may occur pari-passi or as a • Characteristic histological feature is whorled
part of Sturge-Weber syndrome arrangement forming horn pearls which may
• In many cases self-resolution may occur by the age contain laminated keratin in the centre.
of 5 years
3. Sebaceous gland carcinoma
4. Neurofibroma
• It is a very rare tumour according to western
• Lids and orbits commonly affected reports, but Indian literature reports it as the most
• Solitary neurofibroma or as a part of neuro­ common lid malignancy.
fibromatosis.
• Arises from meibomian glands (most common),
sebaceous glands of eyebrows and caruncle.
B. PRECANCEROUS CONDITIONS
• May be mistaken for chalazion.
• Solar keratosis
• Carcinoma-in-situ 4. Malignant melanoma (melanocarcinoma)
• Xeroderma pigmentosa. • May arise from a pre-exiting naevus, but usually
arises de-novo from the melanocytes present in
C. MALIGNANT TUMOURS the skin.
1. Basal cell carcinoma • Metastasis: Locally, lymphatics and blood stream.
• Radioresistent, so surgery is treatment of choice.
• Commonest malignant tumour of the lids, seen in
elderly people (western reports).
MISCELLANEOUS POINTS
• Locally malignant.
• Sites: Lower lid–50% (most common) • Blepharophimosis may be associated with Down’s
– Medial can thus: 25% syndrome, microphthalmos, Edward’s syndrome
– Upper lid: 10–15% and Waardenburg’s syndrome.
– Outer canthus: 5%. • Ptosis associated with lower lid of the affected side
• Presentations being at a higher level than the lower lid of the
– Noduloulcerative (most common) normal side is seen in Horner’s syndrome.
– Nonulcerative nodular • Epicanthus may be associated with Down’s
– Sclerosing type syndrome.
– Pigmented basal cell carcinoma. • Blepharitis acaria is caused by Demodex folli­
• Most common histological pattern is solid basal cularium.
cell carcinoma with characteristic peripheral • Cutaneous horn of eye lids is frequently associated
palisaded appearance with an underlying dysplastic (actinic keratosis)
• Treatment: of choice is local excision or neoplastic (squamous cell carcinoma)
– Radiotherapy only in unoperable cases. change.
• Ptosis associated with lid lag in down gaze is feature
2. Squamous cell carcinoma of congenital ptosis.
• Second commonest malignant tumour of the eye • Peculiarities of the skin of the eyelids are loose
lids attachment, extreme thinness, and absence of hair.
Diseases of Eyelids 129

Multiple Choice Questions (MCQs)

1. Distichiasis is characterised by all of the following 7. Pseudotrichiasis is seen in:


except: A. Entropion
A. An extra row of cilia is present B. Ectropion
B. Normal row of cilia is present anterior to the C. Distichiasis
openings of the meibomian glands D. Healed membranous conjunctivitis
C. Extra row of cilia occupies a position posterior to
the openings of the meibomian glands 8. Simple and effective method of treatment for
D. Cilia of the extra row may rub the cornea segmental trichiasis is:
A. Epilation
2. Rosettes found at the lid margin are a feature of:
B. Electrolysis
A. Squamous blepharitis
B. Ulcerative blepharitis C. Cryolysis
C. Both of the above D. Surgical correction
D. None of the above
9. All of the following operations are employed to
3. Stye is an acute suppurative inflammation of: correct senile entropion except:
A. Gland of Zeis A. Modified Wheeler’s operation
B. Gland of Moll B. Modified Ketssey’s operation
C. Meibomian gland C. Weiss operation
D. All of the above D. Bick’s procedure with Reeh’s modification

4. Hordeolum internum is a suppurative inflammation 10. Which of the following types of entropion is not
of the: known:
A. Gland of Zeis A. Spastic entropion
B. Gland of Moll B. Paralytic entropion
C. Meibomian gland
C. Cicatricial entropion
D. All of the above
D. Involutional entropion
5. Of the following conditions most painful is: 11. Senile ptosis is:
A. Stye A. Neurogenic
B. Hordeolum internum
B. Myogenic
C. Ulcerative blepharitis
C. Aponeurotic
D. Inflammation of gland of Moll
D. Mechanical
6. In a chalazion with multiple recurrences at the
same site the incision and currettage should be: 12. Severe ptosis in a new born should be operated
A. Followed by cauterisation with carbolic acid at the age of:
B. Preceded by cryotherapy A. 1 year
C. Preceded by intralesional injection of triam­ B. 3 years
cinolone C. 5 years
D. Replaced by excision biopsy D. Earliest possible

1 : C 2 : B 3 : A 4 : C 5 : B 6:D 7 : A 8 : C 9 : B 10 : B 11 : C 12 : D
130 Review of OPHTHALMOLOGY

13. All of the following are pre-cancerous conditions C. Weak Muller’s muscle
of the lids except: D. Multiple failed surgeries
A. Naevi
23. All are complications of chronic staphylo-coccal
B. Solar keratosis
blepharoconjunctivitis except:
C. Xeroderma pigmentosa
A. Chalazion
D. Carcinoma-in-situ
B. Marginal conjunctivitis
14. The commonest malignant tumour of the lids is: C. Follicular conjunctivitis
A. Basal-cell carcinoma D. Phylectenular conjunctivitis
B. Squamous cell carcinoma 24. Fasanella Servat operation is specifically indicated
C. Adenocarcinoma in:
D. Melanocarcinoma A. Congenital ptosis
15. The most common site of basal cell carcinoma of B. Steroid induced ptosis
the lids is: C. Myasthenia gravis
A. Upper lid D. Horner’s syndrome
B. Lower lid 25. The operation of plication of inferior lid retractors
C. Medial canthus is indicated in:
D. Lateral canthus A. Senile ectropion
16. Sebaceous cell carcinoma of the lids arises from: B. Senile entropion
A. Meibomian gland C. Cicatricial entropion
B. Sebaceous gland of eyebrows D. Paralytic entropion
C. Caruncle
D. All of the above EXTRA EDGE QUESTIONS
17. Epiphora in a patient with seventh nerve palsy is 26. A recurrent chalazion should be subjected
most likely due to: to histopathologic evaluation to exclude the
A. Eversion of lower punctum possibility of:
B. Lateral lower lid ectropion A. Squamous cell carcinoma
C. Failure of lacrimal pump system B. Sebaceous cell carcinoma
D. All of the above
C. Malignant melanoma
18. Ptosis with lid lag is seen in: D. Basal cell carcinoma
A. Traumatic ptosis
27. Fusion of palpebral and bulbar conjunctiva is:
B. Myogenic ptosis
A. Symblepharon
C. Synkinesis
D. Congenital ptosis B. Trichiasis
C. Ectropion
19. Tylosis refers to: D. Tylosis
A. Hypertrophy and drooping of eyelid
B. Inversion of eyelid 28. Eyelid drooping in/are seen in:
C. Senile eversion of eyelid A. Damage to edinger-westphal nucleus
D. Distortion of cilia B. Damage to motor part of facial nerve
C. Damage to sympathetic nerve supply
20. The commonest fungal lesion of the eyelid is: D. Damage to motor root of occulomotor nerve
A. Candida
E. Damage to lacrimal nerve
B. Aspergillosis
C. Sporothrix 29. A patient with ptosis presents with retraction of
D. None ptotic eye lid on chewing. This represents:
A. Marcus gunn Jaw winking syndrome
21. Epilation is not indicated in:
A. Ulcerative blepharitis B. Third nerve misdirection syndrome
B. Phthiriasis C. Abducent palsy
C. Trichiasis D. Occulomotor palsy
D. Madarosis 30. S shaped eyelid is seen in:
22. Sling surgery should be avoided in cases of ptosis A. Plexiform neurofibroma
with: B. Hemangioma
A. Very poor levator action C. Arterio venious fistula
B. Poor Bell’s phenomenon D. Varix
13 : A 14 : A 15 : B 16 : D 17 : C 18 : D 23 : A 24 : D 25 : B 26 : B 27 : A
19 : A 20 : A 21 : D 22 : B 28 : C and D 29 : A 30 : A
CHAPTER

15
Diseases of Lacrimal Apparatus

Quick Text Review


TEAR FILM AND ITS DISORDERS i. Sjogren’s syndrome
• Pure KCS: In it only lacrimal gland is damaged
TEAR FILM by infiltration with mononuclear cells
Structure • Primary Sjogren’s syndrome (sicca complex)
Tear film consist of three layers: consists of KCS and a dry mouth (xerostomia)
• Secondary Sjogren’s syndrome consists of sicca
1. Mucus layer complex and a connective tissue disease (usually
• Innermost and thinnest layer (0.02-0.05 mm) rheumatoid arthritis).
• Consists of mucin secreted by goblet cells and ii. Non-Sjogren’s syndrome. It includes:
glands of Manz • Congenital alacrimia (rare)
• Converts hydrophobic corneal surface into • Riley-Day syndrome
hydrophilic • Idiopathic hyposecretion.
2. Aqueous layer 2. Evaporative dry eye. It is caused by the
• Thickest layer (6.5–7.5 mm) and consists of tears conditions which decrease tear film stability and
secreted by main and accessory lacrimal glands thus increase evaporation. It includes:
• Alkaline and saltish in taste i. Mucin deficiency dry eye
• Contains antibacterial substances—lysozyme, • Hypovitaminosis - A
betalysin, lactoferrin, immunoglobulin-A. • Severe conjunctival scarring (trachoma, Stevens-
Johnson syndrome, chemical burns, radiations
3. Lipid or oily layer and ocular pemphigoid).
• Outermost layer, about 0.1 µm thick ii. Lipid deficiency and abnormalities
• Consists of secretions of Meibomian, Zeis and Moll • Congenital absence of meibomian glands along
glands with anhydrotic ectodermal dysplasia (rare)
• Prevents the overflow of tears, retards their • Chronic blepharitis and meibomitis (common).
evaporation and lubricates the eyelids as they slide iii. Impaired eyelid functions
over the surface of the globe. • Bell’s palsy
Functions
• Dellen
• Symblepharon
• Keeps conjunctiva and cornea moist • Lagophthalmos
• Provides oxygen to corneal epithelium • Ectropion.
• Washes away debris and noxious irritants iv. Corneal epitheliopathies.
• Antibacterial activity
• Facilitates movements of the lids over the globe. Tear film tests

DRY EYE 1. Tear film break-up-time (BUT)


• It is the interval between a complete blink and
Etiology
appearance of first randomly distributed dry spot
1. Aqueous tear deficiency—Kerato-conjunctivitis on the cornea
sicca (KCS) • Normal values: 15–35 seconds
132 Review of OPHTHALMOLOGY

• Value less than 10 seconds imply an unstable tear • Wrong technique of performing regurgitation test
film • Encysted mucocele.
• Low values indicate mucin deficiency/disturbance. • Crocodile tears are seen in abnormal VII nerve
2. Schirmer-I-test regeneration.
• It measures total tear secretion (production)
• It is performed with 5 × 35 mm strip of Whatman DISORDERS OF LACRIMAL SAC AND LACRIMAL
41 filter paper GLAND
• Normal value: 15 mm or above/5 minutes
• Values between 5 and 10 mm: Mild to moderate DACRYOCYSTITIS
dry eye Congenital dacryocystitis
• Value less than 5 mm: Severe dry eye
As many as 30 percent of newborn infants are
• Low values are suggestive of aqueous deficiency
believed to have closure of nasolacrimal duct at birth;
dry eye (kerato-conjunctivitis sicca).
mostly due to membraneous occlusion at its lower
3. Rose Bengal staining end, near the valve of Hasner. Watering develops
• Stains devitalized cells after 7 days of birth followed by mucopurulent
• Irritating–so use xylocaine discharge. Regurgitation test becomes positive.
• Three staining patterns are described:
– C-pattern: Fine punctate stain in interpalpebral Treatment
area (suggests mild dry eye). • Massage over the lacrimal sac area and topical
– B-pattern: Extensive staining (moderate dry eye) antibiotics constitute the treatment of congenital
– A pattern: Confluent staining of conjunctiva and NLD block, up to 6 months of age. This conservative
cornea (severe dry eye). treatment cures obstruction in about 90 percent of
THE WATERING EYE the infants.
1. Hyperlacrimation, i.e. more formation of tears • Probing of NLD with Bowman’s probe should be
is of two types: performed, in case the condition is not cured by
• Primary hyperlacrimation occurs due to direct the age of 6 months (some surgeon wait up to 9
stimulation of the lacrimal gland as in early stages months). In most instances a single probing will
of lacrimal gland tumours and cysts and due to the relieve the obstruction. In case of failure, it may
effect of strong parasympathomimetic drugs. be repeated after an interval of 3–4 weeks.
• Reflex hyperlacrimation results from stimulation • Balloon catheter dilation may be carried out where
of sensory branches of fifth nerve due to irritation repeated probing is a failure.
of cornea or conjunctiva. • Intubation with silicone tube may be tried when
a. Physiological cause of epiphora is lacrimal above treatment is a failure.
pump failure due to lower lid laxity or weakness • Dacryocystorhinostomy (DCR) operation: Optimum
of orbicularis muscle. age is 4 years.
b. Mechanical obstruction in lacrimal passages.
Adult dacryocystitis (Acquired dacrocystitis)
2. Epiphora refers to watering due to decreased
drainage of tears either due to blockage of lacrimal Chronic dacryocystitis
passage or due to lacrimal pump failure (atonia of • It is more common than the acute dacryocystitis.
sac). • The disease is predominantly seen in females
• The commonest cause of epiphora in a newborn (80%).
is membrane at the lower end of NLD • Dacryocystorhinostomy (DCR) operation is the
• The commonest site of obstruction in epiphora treatment of choice.
occurring in an adult is junction of the sac and the • Dacryocystectomy (DCT) is indicated when DCR
NLD is contraindicated.
• For evaluation of functional (physiological) obstruc­
tion due to lacrimal pump failure dacryoscinti­graphy SWELLINGS OF THE LACRIMAL GLAND
or radionucleotide dacryocystography is used and
it is the most sensitive method for evaluation. Mickulicz’s syndrome
Causes of false negative regurgitation test It is characterised by bilaterally symmetrical
• Internal fistula enlargement of the lacrimal and salivary glands
Diseases of Lacrimal Apparatus 133
associated with a variety of systemic diseases. Benign mixed tumour
These include: leukaemias, lymphosarcomas, It is also known as pleomorphic adenoma and
benign lymphoid hyperplasia, Hodgkin’s disease, occurs predominantly in young adult males. It is the
sarcoidosis and tuberculosis. commonest tumour of the lacrimal gland. Clinically
• Dacryops. It is a cystic swelling, which occurs due to it presents as a slowly progressive painless swelling
retention of lacrimal secretions following blockage in the upper-outer quadrant of the orbit. The eyeball
of the lacrimal ducts. may be proptosed with a down and in displacement.
• Lymphoid tumours and inflammatory pseudo­ It is locally invasive and may infiltrate its own
tumors constitute approximately 50% of cases. pseudocapsule to involve the adjacent periosteum.
• Benign epithelial tumours include benign mixed Histologically, it is characterised by presence of
tumours which account for 25 percent cases. pleomorphic myxomatous tissue, just like benign
• Malignant epithelial tumours constitute 25% of mixed tumour of salivary gland. Its treatment
cases and include: Malignant mixed tumour, consists of complete surgical removal with the
adenoid cystic carcinoma, mucoepidermoid capsule. Recurrences are very common following
carcinoma and adenocarcinoma. incomplete removal.
134 Review of OPHTHALMOLOGY

Multiple Choice Questions (MCQs)

1. Innermost stratum of tear film is: 8. All of the following are features of Sjogren’s
A. Mucus layer syndrome except:
B. Aqueous layer A. It is an autoimmune chronic inflammatory
C. Oily layer disease
D. None of the above B. Typically occurs in women after the menopause
2. Thickest layer of tear film is: C. In primar y Sjogren’s syndrome, kerato­
A. Mucus layer conjunctivitis sicca is associated with rheumatoid
B. Aqueous layer arthritis
C. Oily layer D. In secondary Sjogren’s syndrome, dry eye and/
D. None of the above or xerostomia (dry mouth) is associated with
rheumatoid arthritis
3. The antibacterial substance present in the tears
is: 9. Tear film break-up-time is the time:
A. Lysozyme A. Between the last blink and the appearance of first
B. Betalysin dry spot on the cornea
C. Lactoferrin
B. Taken for appearance of dry spot between two
D. All of the above
blinks
4. Keratoconjunctivitis sicca refers to: C. Taken by successive blinks to break the tear film
A. Aqueous tear deficiency dry eye D. Taken for disappearance of dry spots on the
B. Mucin deficiency dry eye cornea
C. Lipid deficiency dry eye
D. All types of dry eye 10. Tear lysozyme levels are decreased in:
A. Keratoconjunctivitis sicca
5. Normal values of tear film break up time range
B. Stevens Johnson syndrome
from:
C. Ocular pemphigoid
A. 5 to 10 seconds
D. All of the above
B. 10 to 15 seconds
C. 15 to 20 seconds 11. Positive fluorescein dye disappearance test
D. 15 to 30 seconds indicates that watering eye is due to:
6. Schirmer-I test measures: A. Atonia of the sac
A. Total tear secretions B. Mechanical obstruction in the lacrimal passages
B. Basal tear secretions C. Both of the above
C. Reflex tear secretions D. None of the above
D. All of the above 12. Most common site of congenital blockage in the
7. Normal values of Schirmer-I test are: nasolacrimal duct is:
A. 5 mm A. At the upper end
B. 10 mm B. In the middle
C. 15 mm C. At the lower end
D. 15 mm or above D. Whole of the duct

1 : A 2 : B 3 : D 4 : A 5 : D 6 : A 8 : C 9 : A 10 : A 11 : C 12 : C
7:D
Diseases of Lacrimal Apparatus 135
13. Optimum age for performing dacryocystor­ 20. Crocodile tears are seen in:
hinostomy operation in a child with congenital A. Frey’s syndrome
dacryocystitis is: B. Conjunctivitis
A. 2 years C. Lacrimal tumour
B. 4 years D. Abnormal VIIth nerve regeneration
C. 6 years
D. 8 years
EXTRA EDGE QUESTIONS
14. In dacr yoc ystorhinostomy operation the
communication of the sac is established with: 21. Treatment of nasolacrimal duct obstruction:
A. Middle meatus A. Syringing
B. Superior meatus B. Probing
C. Inferior meatus C. DCR
D. All of the above D. Dacryocystectomy
15. Functional efficiency of lacrimal drainage system E. Antibiotic
may be assessed by:
22. Initial treatment of congenital dacryocystitis:
A. Lacrimal syringing
A. Massaging
B. Subtraction macrodacryocystography
B. Probing
C. Radionucleotide dacryocystography
C. DCR
D. All of the above
D. Ointment
16. Most common site of obstruction in chronic E. No treatment required
dacryocystitis in adults is:
A. At upper end of nasolacrimal duct 23. A 60 year old man presented with watering
B. At lower end of nasolacrimal duct from his left eye since 1 year. Syringing revealed
C. In the lacrimal sac a patent drainage system. Rest of the ocular
D. In the common canaliculus examination was normal. A provisional diagnosis
of lacrimal pump failure was made. Confirmations
17. Normal pH of tears is:
of the diagnosis would be by:
A. 5.7
A. Dacryoscintigraphy
B. 6.7
B. Dacryocystography
C. 7.5
C. Pressure syringing
D. 7.9
D. Canaliculus irrigation test
18. Tears are produced in the new born after:
A. 1 Week 24. Unilateral lacrimal gland destruction may be
B. 2 Weeks caused by:
C. 3 Weeks A. Inferior orbital fissure fracture
D. 4 Weeks B. Fracture of roof of orbit
C. Fracture of lateral wall
19. A 3 month old infant was brought with complaints D. Fracture of sphenoid
of profuse lacrimation. On pressure pus exudes
from the puncta. The best line of management is: 25. Length of naso lacrimal duct is:
A. Dacryocystorhinostomy A. 10 mm
B. Syringing B. 11 mm
C. Massaging of sac with antibiotics C. 12 mm
D. Syringing and probing D. 9 mm

13 : B 14 : A 15 : C 16 : A 17 : C 18 : A 20 : D 21 : A, B, C, D and E 22 : A 23 : A 24 : B
19 : D 25 : C
CHAPTER

16
Diseases of Orbit

Quick Text Review


PROPTOSIS • Inflammatory conditions: Mikulicz’s syndrome and
late stage of cavernous sinus thrombosis.
DEFINITION • Endocrinal exophthalmos: It may be thyrotoxic or
Proptosis refers to bulging of the eyeball (forward thyrotropic.
protrusion) beyond the orbital margins. Most • Tumours: Symmetrical lymphoma or lympho­
sources define exophthalmos/proptosis as protru­ sarcoma, secondaries from neuroblastoma,
sion of globe greater than 18 mm. nephroblastoma, Ewing’s sarcoma and leukaemic
infiltration.
Exophthalmos though synonymous with proptosis • Systemic diseases: Histiocytosis, systemic amyloi­
is the term often used when describing proptosis dosis, xanthomatosis and Wegner’s granulo­matosis.
associated with Graves’s disease.
Causes of specific type of proptosis
CAUSES Causes of acute proptosis are orbital emphysema
Causes of unilateral proptosis following fracture of the medial orbital wall, orbital
haemorrhage and rupture of ethmoidal mucocele.
• Congenital conditions. Dermoid cysts, congenital
cystic eyeball, and orbital teratomas. Causes of intermittent proptosis (proptosis
developing intermittently and rapidly in one
• Traumatic lesions: Orbital haemorrhage, retained
when venous stasis is induced by forward bending
intraorbital foreign body, traumatic aneurysm and
or lowering the head) are orbital varix (most
emphysema of the orbit.
common), periodic orbital oedema, recurrent
• Inflammatory lesions: Acute inflammations are
orbital haemorrhage and highly vascular tumours.
orbital cellulitis, abscess, thrombophlebitis,
panophthalmitis and cavernous sinus thrombosis Causes of pulsating proptosis are carotico caver­
(proptosis is initially unilateral but ultimately nous fistula (most common), saccular aneurysm
of ophthalmic artery, congenital meningocele or
becomes bilateral) . Chronic inflammatory lesions
meningoencephlocele, neurofibromatosis and
include: pseudotumours, tuberculoma, gumma
traumatic or operative hiatus in the roof of the orbit.
and sarcoidosis.
• Circulatory disturbances and vascular lesions: Causes of proptosis in infants. Craniostenosis,
Angio­n eurotic oedema, orbital varix and meningoencephalocele, microphthalmia with
aneurysms. teratoma, retinoblastoma, capillary haemangioma,
• Cysts of orbit: Haematic cysts, implantation cysts juvenile xanthogranuloma and metastatic neuro­
and parasitic cysts (hydatid cyst and cysticercus blastoma.
cellulosae) . Causes of proptosis beginning in childhood:
• Tumours of the orbit: These can be primary, Dermoid cyst, lymphangioma, cavernous hae­
secondary or metastatic. (see page 81) mangioma, orbital varices, neurofibroma, rhabdo­
myosarcoma, optic nerve glioma, intraorbital
Causes of bilateral proptosis meningioma, orbital cellulitis leukaemic infiltration,
• Developmental anomalies of the skull: Craniofacial granulocytic sarcoma, Burkitt’s lymphoma, eosino­
dysostosis, e.g, oxycephaly (tower skull) . phillic granuloma, Hand-Schuller Christian
• Osteopathies: Osteitis deformans, rickets and disease, sinus histiocytosis, hydatid cyst and fibrous
acromegaly. dysplasia.
Diseases of Orbit 137
Causes of proptosis beginning in adulthood. Causes of intraorbital calcification
Graves’ ophthalmopathy (commonest cause), • Retinoblastoma
lymphoproliferative disorders, fibrous histocytoma, • Optic nerve sheath meningioma
haemangiopericytoma, cavernous haemangioma, • Phlebolith in orbital varix
secondary orbital meningioma, osteoma, mucocele, • Phthisical eye.
orbital varices, lesions of lacrimal gland, secondary Changes in optic canal (optic foramina) in X-rays
tumours of the orbit. • Uniform regular concentric enlargement
Causes of pseudoproptosis/pseudoexophthalmos – optic nerve glioma
• Uniform irregular enlargement
Causes of pseudoproptosis are buphthalmos,
– retinoblastoma
axial high myopia, retraction of upper lid and
– optic nerve sheath meningioma
enophthalmos of the opposite eye.
– orbital neurofibroma
EVALUATION OF PROPTOSIS • Erosion of the upper margin
– raised intracranial pressure
Different views of X-ray orbit
• Erosion of the inferolateral margin
A. Caldwell-Luc view: – infraclinoid aneurysm
• PA view – infraclinoid meningioma.
• Occipito-frontal view
• Structures seen: Superior orbital fissure, greater ENOPHTHALMOS
and lesser wing of sphenoid, ethmoid and • Congenital: Microphthalmos and maxillary
frontal sinus and floor of sella. hypoplasia
B. Towne’s view • Traumatic: Blow-out fractures of floor of the orbit
• AP view • Post-inflammatory: Cicatrization of extraocular
• Frontal occipital muscles as in the pseudotumour syndromes
• Structures seen: Inferior orbital fissure, dorsum • Paralytic enophthalmos: It is seen in Horner’s
sellae of sphenoid bone. syndrome
C. Water’s view: Occipito-mental • Atrophy of orbital contents: Senile atrophy of orbital
• Structures seen-floor of orbit (blow out fat, atrophy due to irradiation of malignant tumour,
fracture) following cicatrizing metastatic carcinoma and due
• Occipitomental, maxillary sinus and floor of
to scleroderma.
orbital
D. Reese view: For optic foramen.
GRAVES’ OPHTHALMOPATHY
Proptosis: Findings in X-ray orbit
Causes of symmetrical enlargement of orbital cavity PATHOLOGY
in X-rays of the orbit The histopathologic reaction of various tissues is
• Optic nerve glioma dominated by a mononuclear cell inflammatory
• Haemangioma reaction. Presence of mucopolysaccharides, pre­
• Neurofibroma dominantly hyaluronic acid, together with
• Retinoblastoma. interstitial oedema and inflammatory cells accounts
Cause of asymmetrical enlargement of orbit for the proptosis. Most data presently support
• Rhabdomyosarcoma the postulate that it is an autoimmune disease
• Dermoid cyst with the orbital fibroblast as the primary target of
• Lacrimal gland tumour. inflammatory attack, and extraocular muscle being
secondarily involved.
Causes of increased bone density (hyperostosis) in
X-rays of orbit CLINICAL FEATURES
• Sphenoidal ridge meningioma
Lid signs
• Chronic periostitis
• Fibrous dysplasia Dalrymple’s sign—Retraction of the upper lids
• Paget’s disease producing the characteristic staring and frightened
• Osteoblastic metastasis. appearance. It is the most common feature.
138 Review of OPHTHALMOLOGY

Lid lag (von Graefe’s sign), i.e. when globe is • Extension of infection from neighbouring structures
moved downward, the upper lid lags behind. (most common-ethmoiditis especially in children)
Enroth’s sign—Fullness of eyelids due to puffy is the commonest mode of orbital infections.
oedematous swelling. • Characteristic features are marked swelling
of lids conjunctival chemosis, proptosis and
Gifford’s sign—Difficulty in eversion of upper lid.
limitation of ocular movements. Important
Stellwag’s sign—Infrequent blinking blinding complications are: central retinal artery
Ocular motility defects occlusion, optic neuritis and corneal ulceration.
• The most common ocular motility defect is • Orbital apex syndrome is characterised by a triad
a unilateral elevator palsy caused by fibrotic of: (i) ophthalmoplegia due to paresis of third,
contraction of the inferior rectus muscle followed fourth and sixth cranial nerves, (ii) anaesthesia
by failure of abduction due to contracture of medial in the region of supply of ophthalmic division of
rectus muscle. Sequence of involvement of muscle fifth nerve and (iii) amaurosis due to involvement
is: IR, MR, SR and LR. of optic nerve.
• Enlargement of extraocular muscles without
enlargement of tendons is the hallmark of thyroid CAVERNOUS SINUS THROMBOSIS
ophthalmopathy (seen on CT scan or MRI orbit) . • Cavernous sinus thrombosis starts initially as
Note: Mnemonic ‘I AM So Lucky’ a unilateral condition, which soon becomes
IR-MR-SR-LR bilateral in more than 50 percent of cases due to
intercavernous communication.
External ophthalmology • Paralysis of opposite lateral rectus muscle is often
Thyroid ophthalmopathy can cause vertical the first sign of involvement of contralateral
diplopia due to paralysis of vertical acting muscles cavernous sinus.
like inferior oblique.
Clinical features
Exophthalmos is the second most common sign, as • Generalized: Abrupt onset of fever with chills and
a rule both eyes are symmetrically affected; but it rigor.
is frequent to find one eye being more prominent
• Lo calized (o cular) features: Chemosis of
than the other. Even unilateral proptosis is not
conjunctiva, ipsilateral ophthalmoplegia ( 3rd-
uncommon.
Ptosis, absent direct and consensual papillary
Werner classification of ocular changes in Graves light refex in ipsilateral side, 4th and 6th nerve
disease palsy), diplopia (lateral gaze is involved earliest)
Six classes depicted by Mnemonic “NO SPECS’’ and corneal anesthesia occurs due to involvement
Class 0: No signs or symptoms of ophthalmic division of 5th cranial nerve- loss of
Class 1: Only signs corneal reflex.
Class 2: Soft tissue involvement (sign and • Oedema in mastoid region is a pathognomonic
symptoms) sign.
Class 3: Proptosis • Vision loss is not an early feature.
Class 4: Extraocular muscle involvement • Magnetic resonance venography (angiography) is
Class 5: Corneal involvement the investigation of choice.
Class 6: Sight loss (optic nerve compression)
• Rapidly developing, acute inflammatory type of
proptosis seen in cavernous sinus thrombosis
ORBITAL CELLULITIS AND INTRAORBITAL
needs to be differentiated from orbital cellulitis
ABSCESS
and panophthalmitis.
• Suppurative inflammation of the fat and cellular
tissues of the orbit.
CAROTID-CAVERNOUS FISTULA
• It may result from penetrating injury especially
when associated with retention of intraorbital Carotid-cavernous fistula is an acquired abnormal
f o re i g n b o d y , f o l l ow i n g o p e rat i o n s l i ke communication between the carotid arterial system
evisceration, enucleation, dacryocystectomy and and the venous cavernous sinus. There are two type
orbitotomy. of carotid-cavernous fistula:
Diseases of Orbit 139
i. High-flow fistula medial displacement of the eyeball. Malignant
These are secondary to trauma or iatrogenic and tumours of lacrimal gland are: adenoid cystic
present acutely with pain, proptosis (often pulsatile carcinoma, pleomorphic adenocarcinoma, and
exophthalmos). Chemosis, congestion, bruit, mucoepidermoid carcinoma.
ophthalmoplegia (III-VI nerve palsies), headache, • Optic nerve glioma may present either as a solitary
and objective pulsatile tinnitus. The classical triad tumour or as a part of von Recklinghausen’s
of presentation is proptosis, chemosis, and bruit. It neurofibromatosis (55%). It is characterised
presents within days or weeks following trauma. by early visual loss associated with a gradual,
painless,unilateral axial proptosis occurring
ii. Low- flow fistula
in a child usually between 4 and 8 years of age
These develop spontaneously in elder women, (female > male) . CT scan typically shows fusiform
during pregnancy or in those with hyperplastic enlargment of optic nerve along with enlargement
tissue. These typically present slowly with less of optic canal.
pronounced symptoms. • Primary intraorbital meningiomas produce
Investigations visual loss associated with limitation of ocular
movements, optic disc oedema or atrophy, and
• Selective carotid angiography using digital
a slowly progressive unilateral proptosis. The
subtraction technique is the investigation of choice
presence of opticociliary shunt is pathognomonic
and essential if the treatment is contemplated.
of an optic nerve sheath meningioma.
• Magnetic resonance angiography (MRA) are also
• Lymphomas. Orbits are involved more commonly
used commonly.
by non-Hodgkin’s lymphomas (Most common
Treatment malignant orbital tumour in adults) .
A carotid-cavernous fistula requires a definitive • Hand-Schuller-Christian disease is characterised
treatment, which currently involves endovascular by a triad of proptosis, diabetes insipidus and bony
technique. The fistula can usually be occluded with defects in the skull.
a balloon or coil without sacrifice of the carotid.
SECONDARY ORBITAL TUMORS

ORBITAL TUMOURS • Tumors from eyeball: Retinoblastoma and malig­


nant melanoma.
PRIMARY ORBITAL TUMOURS • Tumor from eyelids: Squamous cell carcinoma and
• Dermoids are common developmental tumours basal cell carcinoma
in which the cystic component is lined with • Tumors from nose and paranasal sinuses: carci­
keratinizing epithelium and may contain one noma, sarcoma and osteoma
or more dermal adnexal structures such as hair • Tumors from nasopharynx: Nasopharyngeal
follicles and sebaceous glands. carcinoma
• Lipodermoids are solid tumours usually seen • Tumors from cranial cavity: Glioma and menin­
beneath the conjunctiva. gioma
• Teratomas are composed of ectoderm, mesoderm
METASTATIC ORBITAL TUMORS
and endoderm. These may be solid, cystic or a
mixture of both. • In children: Neuroblastoma (most common),
• Capillary haemangioma is commonly seen at birth nephroblastoma, Ewing’s sarcoma, Leukemic
or during the first month. infiltration, testicular embryonal sarcoma and
• Cavernous haemangioma is the commonest ovarian sarcoma.
benign orbital tumour among adults. The tumour • In adults: Carcinoma breast (most common in
is usually located in the.retrobulbar muscle cone female), lungs (most common in males), prostate,
and produces unilateral axial proptosis. thyroid and rectum. Malignant melanoma from
• Rhabdomyosarcoma is the most common primary skin.
orbital tumour among children, usually occurring
below the age of 15 years (90%) . BLOW-OUT FRACTURE OF THE ORBITAL FLOOR
• Lacrimal gland tumours. Pleomorphic adenoma • ‘Blow-out fractures’ mainly involve orbital floor
(mixed cell tumour) is the most common benign (especially in the posteromedial floor) and medial
tumour of lacrimal gland. It causes down and wall.
140 Review of OPHTHALMOLOGY

• Three factors responsible for producing enoph­ • Mucoceles of paranasal sinuses affecting the orbit
thalmos in blow-out fracture are: (a) Escape of most commonly occur in frontal sinus. The most
orbital fats into the maxillary sinus; (b) Backward common cause of intermittent proptosis is orbital
traction on the globe by entrapped inferior rectus varices.
muscle and (c) Enlargement of the orbital cavity • Most common cause of pulsating proptosis is
from displacement of fragments. caroticocavernous fistula.
• Common roentgen findings are: Fragmentation and • Most common primary tumour of the orbital cavity
irregularity of the orbital floor, depression of bony presenting as proptosis is cavernous haeman­gioma.
fragments, proptosis, diplopia (muscle restriction • Rhabdomyosarcoma of the orbit is the commonest
can be confirmed with forced duction test) and primary malignant tumour of childhood.
hanging drop opacity (tear drop sign) of the • Commonest histological type of rhabdomyo-
superior maxillary antrum from orbital contents sarcoma of the orbit is embryonal type.
herniating through the floor. • Paralysis of opposite lateral rectus muscle is often the
Treatment first sign of involvement of contralateral cavernous
Fracture involving half or more orbital floor with sinus.
entrapment of orbital content and persistent • Earliest clinical feature of orbital extension of the
diplopia in the primary position should be repaired basal cell carcinoma of the eyelid is diplopia.
within 2 weeks by using synthetic material such as • Carotid angiography is the investigation of choice
silicon, Teflon or supramid. in caroticocavernous fistula. The first sign of
cavernous sinus thrombosis is restriction of ocular
SOME SALIENT POINTS movements.
• First sign of cavernous sinus thrombosis is restriction
• Most common and most virulent fungal disease
involving the orbit is caused by Mucor (Mucor- of ocular movements.
mycosis) and Rhizopus organism of the • Most common cause of unilateral proptosis in
class Phycomycetes (phycomycosis). Orbital children is orbital cellulitis
mucormycosis occur most often in patients with • Most common cause of unilateral proptosis in adults
diabetic ketoacidosis. is thyroid ophthalmopathy.
Diseases of Orbit 141

Multiple Choice Questions (MCQs)

1. Intermittent proptosis is a sign of: 7. Pseudoproptosis is a feature of:


A. Pseudotumour A. Buphthalmos
B. Carotico-cavernous fistula B. High axial myopia
C. Capillary haemangioma C. Upper lid retraction
D. Orbital varices D. All of the above
2. Marcus Gunn pupil along with proptosis indicates: 8. Opticociliary shunts observed on fundoscopy are
A. Involvement of ciliary ganglion a feature of:
B. Compression of optic nerve A. Meningioma
C Compression of inferior division of 3rd nerve B. Cavernous haemangioma
D. Compression of sympathetic nerves of the eyeball C. Orbital varix
D. All of the above
3. Postural exophthalmometric changes are
diagnostic of: 9. Horner’s syndrome is characterised by all of the
A. Orbital varix following except:
B. Carotico-cavernous aneurysm A. Contralateral enophthalmos
C. Haemangioma B. Ipsilateral miosis
D. Thyroid ophthalmopathy C. Ipsilateral ptosis
E. All of the above D. Anhydrosis of the affected side of face

4. Intraorbital calcification in a patient with proptosis 10. Ocular Graves disease is associated with:
is observed in all except: A. Hyperthyroidism
A. Retinoblastoma B. Hypothyroidism
B. Orbital varix C. Euthyroid state
C. Hydatid cyst D. All of the above
D. Pseudotumour 11. Dalrymple’s sign of ocular Graves’ disease refers
5. Dehiscence of bone can be seen as X-rays findings to:
in a patient with proptosis in all except: A. Retraction of the upper lid
A. Mucocele B. Lid lag
B. Neurofibroma C. Proptosis
C. Lacrimal gland tumour D. All of the above combinedly
D. Rhabdomyosarcoma 12. The most common ocular motility defect noted
6. Orbital lesions of childhood include all of the in ocular Graves’ disease is due to involvement
following except: of:
A. Lymphangioma A. Inferior rectus
B. Secondary orbital meningioma B. Medial rectus
C. Cavernous haemangioma C. Superior oblique
D. Neurofibroma D. Inferior oblique

1 : D 2 : B 3 : D 4 : D 5 : D 6:B 7 : D 8 : A 9 : A 10 : D 11 : A 12 : A
142 Review of OPHTHALMOLOGY

13. The walls of the orbit which are removed in the 21. Cause of lid retraction include:
two wall decompression for proptosis of thyroid A. Graves’ ophthalmopathy
ophthalmopathy include part of: B. Hypokalemic periodic paralysis
A. Orbital floor and medial wall C. Cirrhosis of the liver
B. Orbital floor and lateral wall D. Hydrocephalus
C. Orbital roof and medial wall E. All of the above
D. Medial and lateral walls
22. Secondary tumour may spread to the orbit by all
14. All of the following are the features of orbital apex of the following except:
syndrome except: A. Direct spread from the lids
B. Via blood stream
A. Ophthalmoplegia
C. Via lymphatic channels
B. Enophthalmos
D. Directly from the cranial cavity
C. Ophthalmic nerve anaesthesia
D. Amaurosis 23. The most common tumour that spreads into the
orbit from the intracranial cavity is:
15. Most often the first sign of involvement of A. Astrocytoma
opposite side in cavernous sinus thrombosis is: B. Pituitary adenoma
A. Paralysis of opposite lateral rectus muscle C. Sphenoid wing-meningioma
B. Proptosis of the opposite side D. Neurofibroma
C. Paralysis of opposite 3rd nerve
D. Paralysis of opposite 4th nerve 24. One of the early symptoms of orbital involvement
E. All of the above by basal cell carcinoma of the lid is:
A. Diplopia
16. Simultaneous thrombosis of both the cavernous B. Defective vision
sinuses usually occurs in: C. Proptosis
A. Diseases of the sphenoid sinuses D. Severe pain
B. Septic wounds of the face 25. Intraorbital abscess formation occurs most
C. Pyogenic meningitis commonly in which quadrant of the orbit?
D. Patients with cerebral abscess A. Superotemporal
B. Superonasal
17. The most common mucocele of the paranasal
C. Inferonasal
sinuses involving the orbit arises from:
D. Inferotemporal
A. Frontal sinus
B. Ethmoidal sinus 26. T h e m o s t c o m m o n c a u s e o f p u l s a t i n g
C. Maxillary sinus exophthalmos is:
D. Sphenoidal sinus A. Orbital varices
B. Neurofibromatosis
18. The most commonly seen primary orbital tumour C. Cavernous haemangioma
in children is: D. Caroticocavernous fistula
A. Rhabdomyosarcoma
27. The most common cause of intermittent
B. Glioma of optic nerve
exophthalmos is:
C. Optic nerve sheath meningioma
A. Orbital varices
D. Retinoblastoma
B. Cavernous haemangioma
19. Ophthalmoscopic sign pathognomonic of optic C. Lymphangioma
nerve sheath meningioma is: D. Carotico-cavernous fistula
A. Papilloedema
28. The most common benign tumour of the orbit is:
B. Optic atrophy A. Optic nerve glioma
C. Opticociliary shunt B. Meningioma
D. All of the above C. Benign-mixed tumour
20. The diagnostic triad of exophthalmos, diabetes D. Haemangioma
insipidus and bone lesions is characteristic of: 29. Optical media are clear in:
A. Hand-Schuller Christian disease A. Cavernous sinus thrombosis
B. Letterer-Sieve disease B. Orbital cellulitis
C. Fibrous dysplasia C. Both of the above
D. Osteoporosis D. None of the above
13 : A 14 : B 15 : A 16 : A 17 : A 18 : A 21 : E 22 : C 23 : C 24 : A 25 : B 26 : D
19 : C 20 : A 27 : A 28 : D 29 : C
Diseases of Orbit 143
30. Superior orbital fissure syndrome is frequently 38. Which of the following tumours present with
caused by: proptosis:
A. Carotid aneurysms A. Neuroblastoma
B. Meningioma B. Nephroblastoma
C. Arachnoiditis C. Germ cell tumour
D. All of the above D. Medulloblastoma
E. Meningioma
31. All of the following structures are located in the
lateral wall of the cavernous sinus except: 39. A patient presented with unilateral proptosis,
A. Abducent nerve which was compressible and increases on ending
B. Oculomotor nerve forward. No thrill or bruit was present. MRI shows
C. Trochlear nerve a retro-orbital mass with enhancement. The likely
D. Ophthalmic nerve diagnosis is:
32. Contracted socket occurs because of all the A. AV malformations
following except: B. Orbital encephalocoecle
A. Chronic low grade infection C. Orbital varix
B. Chronic mechanical irritation D. Neurofibromatosis
C. Irradiation 40. Thyroid ophthalmopathy a/w:
D. Loss of fatty tissue during surgery of enucleation A. External ophthalmoplegia
33. A man presents 6 hrs after head injury complaining B. Internal ophthalmoplegia
of mild proptosis and scleral hyperemia: C. Proptosis
A. Pneumo-orbit D. Large extra-ocular muscle
B. Caroticocavernous fistula E. Lid lag
C. Retro-orbital hematoma
41. A 19 years old young girl with previous history of
D. Orbital cellulitis
repeated pain over medial canthus and chronic
34. Blow-out fracture of orbit is characterized by all use of nasal decongestants, presented with abrupt
except: onset of fever with chills and rigor, diplopia on
A. Diplopia lateral gaze, moderate proptosis and chemosis. On
B. ‘Tear drop” sign examination optic disc is congested. Most likely
C. Positive forced duction test diagnosis is:
D. Exophthalmos A. Cavernous sinus thrombosis
B. Orbital cellulitis
35. “Blow-out” fracture of orbit involves:
C. Acute ethmoidal sinusitis
A. Floor
D. Orbital apex syndrome
B. Medial wall
C. All of the above 42. A young man following RTA presented with
D. None of the above proptosis and pain in right eye after four days.
36. All of the following signs could result from On examination there is bruise on forehead and
infection within the right cavernous sinus, except: right eye. What is the diagnosis:
A. Loss of pupillary light reflex A. Cavernous sinus thrombosis
B. Loss of corneal blink reflex B. Internal carotid artery aneurysm
C. Ptosis C. Carotico-cavernous fistula
D. Right ophthalmoplegia D. Fracture of sphenoid
43. A young adult presents with proptosis and pain
EXTRA EDGE QUESTIONS in eye after 4 days of trauma to eye. Chemosis,
conjunctival congestion and extraocular muscle
37. Commonest cause for bilateral proptosis in palsy with inability to move eye are seen.
children is: Investigation of choice:
A. Cavernous hamangioma A. MRI
B. Chloroma B. Digital subtraction angiography
C. Fibrous histiocyloma C. CT
D. Rhabdomyosarcoma D. MR angiography

30 : D 31 : A 32 : D 33 : C 34 : D 35 : C 38 : A, B and E 39 : C 40 : A, C, D and E 41: A


36 : B 37 : B 42 : C 43 : B
144 Review of OPHTHALMOLOGY

44. Blow out fracture of the orbit, most commonly 46. Which of the following signs is classic for CT
leads to fracture of: scanning in Graves ophthalmopathy:
A. Posteromedial floor of orbit A. Nodular muscle enlargement
B. Medial wall of orbit B. Solitary muscle enlargement
C. Fusiform muscle enlargement with sparing of
C. Lateral wall of orbit
tendoms
D. Roof of orbit D. Kinking of extraocular muscles.

45. Most common cause of fracture of roof of orbit: 47. Vascular congestion over insertions of the rectus
muscles (particularly lateral rectus) is seen in:
A. Blow on back of head
A. Lymphoma
B. Blow on parietal bone B. Hemangioma
C. Blow on the forehead C. Graves ophthalmopathy
D. Blow on the upper jaw D. Trauma

44 : A 45 : A 46 : C 47 : C
CHAPTER

17
Ocular Injuries

Quick Text Review


MECHANICAL INJURIES • Iridodialysis (detachment of iris from its root at the
ciliary body): It results in a D-shaped pupil.
OCULAR TRAUMA TERMINOLOGIES • Antiflexion of the iris i.e., rotation of the detached
Birmingham eye trauma terminologies (BETT), as portion of iris in which its posterior surface faces
such adapted by ‘American Ocular Trauma Society’ anteriorly.
(AOTS) terminology given below: • Retroflexion of the iris, i.e. whole of the iris is
doubled back into the ciliary region and becomes
Eyewall: Cornea and Sclera
invisible.
I. Closed—globe injury: No full thickness wound • Vossius’s ring is a circular ring of brown pigment
in eyewall but there is intraocular damage seen on the anterior capsule. It occurs due to
1. Contusion. Closed globe injury with blunt trauma striking of the contracted pupillary margin against
2. Lamellar laceration. Closed globe injury with the crystalline lens.
partial thickness wound of eyewall cause by a sharp • Early rosette traumatic cataract is most typical form
object or blunt trauma. of concussion cataract. It appears as feathery lines
of opacities along the star-shaped suture lines;
II. Open globe injury. Full thickness wound of
usually in the posterior cortex.
eyewall
• Late rosette-shaped traumatic cataract: It develops
1. Rupture, caused by blunt trauma
in the posterior cortex 1–2 years after the injury. Its
2. Laceration, caused by sharp object
sutural extensions are shorter and more compact
• Penetrating injury. One laceration (entry wound) than the early rosette cataract.
• Perforating injury. Two laceration (one entry and • Ruptures of the choroid: These are concentric to the
one exit). optic disc and situated temporal to disc and look
Intraocular foreign body. Technically penetrating like a whitish crescent with fine pigmentation at
injury with retained IOFB. its margins.
EXTRAOCULAR FOREIGN BODIES • Commotio retinae (Berlin’s oedema): The traumatic
macular oedema manifests as milky white
Common sites: On the conjunctiva, it may be
cloudiness involving the posterior pole with a
lodged in the sulcus subtarsalis, fornices or bulbar
‘cherry-red-spot’ in the foveal region.
conjunctiva. In the cornea, it is usually embedded
• Chorio-retinitis sclopeteria, i.e. chorioretinal
in the epithelium, or superficial stroma and rarely
rupture with retinal haemorrhage.
into the deep stroma.
CLOSED GLOBE TRAUMA OPEN GLOBE TRAUMA

• Partial corneal tears Globe rupture


• Blood staining of corneal may occur occasionally Full thickness wound of eyewall (sclera, cornea
from the associated hyphaema and raised or both) caused by blunt trauma. It may occur in
intraocular pressure. Cornea becomes reddish- two ways: direct rupture, at the site of injury and
brown or greenish in colour, which clears very indirect rupture (because of compression force)
slowly from periphery toward centre. Blood at its weakest part, 3 mm away and concentric
staining of cornea simulates dislocation of clear to the limbus in the neighbourhood of canal of
lens in the anterior chamber. schlemm.
146 Review of OPHTHALMOLOGY

Penetrating and perforating injuries Mechanism: Copper ions from the alloy are
Modes of damage include: dissociated electrolytically and deposited under
• Mechanical effects. Uncomplicated/complicated the membranous structures of the eye. Unlike iron
eyewall wound ions these do not enter into a chemical combination
• Intraocular infections. Purulent uveitis/endo­ with the proteins of the cells and thus produce no
pthalmitis and panophthalmitis. degenerative changes.
• Traumatic iridocyclitis
• Sympathetic ophthalmitis. Clinical manifestations include:
1. Kayser-Fleischer ring: It is a golden brown ring
Intraocular foreign bodies
which occurs due to deposition of copper under
Common foreign bodies peripheral parts of the Descemet’s membrane of
Chips of iron and steel (90%), particles of glass, stone, the cornea.
lead pellets, copper percussion caps, aluminium, 2. Sun-flower cataract: It is produced by deposition
plastic and wood. of copper under the posterior capsule of the lens.
It is brilliant golden green in colour and arranged
Reactions of the foreign body
like the petals of a sun-flower.
a. Inorganic foreign body 3. Retina: It may show deposition of golden plaques
1. No reaction: Inert substances, glass, some plastics, at the posterior pole which reflect the light with a
porcelain, gold, silver, platinum, titanium and metallic sheen.
stone.
2. Local irritative reaction leading to encapsulation of b. Organic foreign bodies
the foreign body: Occurs with lead and aluminium The organic foreign bodies such as wood and other
particles. vegetative materials produce a proliferative reaction
3. Suppurative reaction is excited by pure copper, characterised by the formation of giant cells.
zinc, nickel and mercury particles. Caterpillar hair produces ophthalmia nodosum
4. Specific reactions are produced by iron and copper which is characterised by a severe granulomatous
alloys (most toxic) iridocyclitis with nodule formation.
Siderosis bulbi
It refers to the degenerative changes produced SYMPATHETIC OPHTHALMITIS
by an iron foreign body. These usually occur after It is a serious bilateral granulomatous panuveitis,
6 months to 2 years of the injury. which follows a penetrating ocular trauma. The
Mechanism. The iron particle undergoes electrolytic injured eye is called exciting eye and the fellow
dissociation by the current of rest and its ions are normal eye which also develops uveitis is called
disseminated throughout the eye. These ions sympathizing eye.
combine with the intracellular proteins and produce
degenerative changes. In this process, the epithelial Predisposing factors
structures of the eye are most affected. 1. It, almost always, follows a perforating wound.
Clinical manifestations include: 2. Wounds in the ciliary region (the so-called
1. The anterior epithelium and capsule of the lens dangerous zone) are more prone to it.
are involved first of all. Here, the rusty deposits 3. Wounds with incarceration of the iris, ciliary body
are arranged radially in a ring. Eventually, the lens or lens capsule are more vulnerable.
becomes cataractous. 4. It is more common in children than in adults.
2. Iris: It is first stained greenish and later on turns 5. It does not occur when actual suppuration
reddish-brown. develops in the injured eye.
3. Retina develops pigmentary degeneration which
resembles retinitis pigmentosa. Pathology
4. Secondary open-angle type of glaucoma occurs due Dalen-Fuch’s nodules are formed due to proliferation
to degenerative changes in the trabecular mesh- of the pigment epithelium (of the iris, ciliary body
work. and choroid) associated with invasion by the
Chalcosis lymphocytes and epitheloid cells. Retina shows
It refers to the specific changes produced by the perivascular cellular infiltration (sympathetic
alloy of copper in the eye. perivasculitis).
Ocular Injuries 147
Clinical features Radiational injuries
Sympathizing (sound) eye: It is usually involved after 1. Ultraviolet radiations may cause (i) photo- ophthal­
4–8 weeks of injury in the other eye. Sympathetic mia and (ii) may be responsible for senile cataract.
ophthalmitis almost always manifests as acute 2. Infra-red radiations may cause solar macular burns
plastic iritis. Rarely it may manifest as neuroretinitis 3. Ionizing radiational injuries are caused following
or choroiditis. radiotherapy to the tumours in the vicinity of
• Sensitivity to light (photophobia) and transient the eyes. The common ocular lesions include
indistinctness of near objects (due to weakening (i) radiation keratoconjunctivitis; (ii) radiation
of accommodation) are the earliest symptoms. dermatitis of lid and (iii) radiation cataract.
Prophylaxis
MISCELLANEOUS POINTS
Early excision of the injured eye is the best prophy­
laxis when there is no chance of saving useful vision. • Common sites of rupture of the globe are the
Meticulous repair and use of steroids, where eye limbus, the equator and especially under the rectus
can be salvaged. muscles, where the sclera in thinnest.
• Indication of early surgical intervention in
NON-MECHANICAL INSURIES traumatic cataract is secondary glaucoma.
• Prolapsed iris in perforating trauma should
CHEMICAL AND RADIATIONAL INJURIES preferably be abscissed and not reposited because
Alkali burns it will carry intraocular infection.
• In general, alkali burns are much more dangerous • Foreign bodies are the most common cause of eye
than acid burns. injury.
• Alkalies dissociate and saponify fatty acids of • A piece of glass in the anterior chamber is
the cell membrane and, therefore, destroy the exceptionally difficult to see because its trans-
structure of cell membrane of the tissues. parency and refractive index differ little from the
• The strong acids cause instant coagulation of all surrounding media.
the proteins which then act as barrier and prevent • Intraocular foreign body of iron causes maximum
deeper penetration of the acids into the tissues. damage to the eye.
148 Review of OPHTHALMOLOGY

Multiple Choice Questions (MCQs)

1. In a worker engaged in stone breaking with a 7. Traumatic cataract occurs as:


chisel and hammer, the most common foreign A. Rosette-shaped cataract
body which can be lodged in his eye is: B. Zonular cataract
A. Stone particle C. Total cataract
B. Particle from the chisel D. All of the above
C. Piece from the hammer
D. All of the above 8. Blunt trauma to the eye may produce all of the
following changes in the vitreous except:
2. In concussion trauma, damage to ocular structures
is caused by: A. Syneresis
A. Mechanical tearing of the tissues B. Asteroid hyalosis
B. Disruption of physiological activity by damage C. Synchiasis scintillans
to tissue cells D. Liquefaction
C. Vascular damage leading to ischaemia
9. Diameter of the Vossius’s ring is:
D. All of the above
A. Equal to normal pupil
3. After blood staining the cornea may look as: B. Smaller than the normal pupil
A. Reddish brown in colour C. Slightly larger than the normal pupil
B. Greenish in colour D. Much larger than the normal pupil
C. Like a clear lens dislocated into the anterior
chamber 10. Rupture of sclera in ocular contusion is seen most
D. Any of the above commonly in:
4. All of the following are true about blood staining A. Superonasal quadrant
of the cornea except: B. Superotemporal quadrant
A. Its occurrence is hastened by the raised C. Inferonasal quadrant
intraocular pressure D. Inferotemporal quadrant
B. It clears from centre towards periphery
C. Its clearance may take 2 years or more 11. Commonest type of concussion cataract is:
D. Visual prognosis is usually poor in such cases A. Punctate cataract
B. Early rosette-shaped cataract
5. Traumatic iridodialysis may cause: C. Late rosette-shaped cataract
A. Antiflexion of the iris D. Zonular cataract
B. Retroflexion of the iris
C. Iridoplegia 12. Prolapsed iris in perforating trauma should
D. All of the above preferably be abscised and not reposited to
6. Rosette-shaped cataract most commonly involves: prevent chances of:
A. Anterior cortex A. Infection
B. Posterior cortex B. Post-traumatic iridocyclitis
C. Both of the above C. Sympathetic ophthalmitis
D. None of the above D. All of the above

1 : B 2 : D 3 : D 4 : B 5 : A 6:B 7 : D 8 : B 9 : B 10 : A 11 : A 12 : A
Ocular Injuries 149
13. Perforating injuries with retained intraocular 20. All of the following are true about sympathetic
foreign body are more serious than those without ophthalmitis except:
because of: A. It is a bilateral disease
A. More chances of infection B. Pathological features are of non-granulo­matous
B. Deleterious effects of foreign bodies panuveitis
C. More chances of sympathetic ophthalmitis C. Clinically manifests as non-granulomatous
D. All of the above iridocyclitis
D. The non-injured eye developing uveitis is called
14. All of the following intraocular foreign bodies sympathizing eye
produce suppurative reaction except: 21. Regarding occurrence of sympathetic ophthal­
A. Pure zinc mitis, all of the following are true except:
B. Mercury A. It almost always follows a perforating wound
C. Copper alloys B. Wounds in the ciliary region are more prone to it
D. Nickel C. More common in adults than in children
D. Less common when actual suppuration develops
15. The earliest clinical manifestation of siderosis
in the injured eye
bulbi is:
A. Rusty deposits in the anterior subcapsular cells 22. Clinically in the non-injured eye, sympathetic
of the lens ophthalmitis may manifest as:
B. Discolouration of iris A. Acute plastic iridocyclitis
C. Deposits in Descemet’s membrane of cornea B. Neuroretinitis
D. Pigmentary changes in retina C. Choroiditis
D. All of the above
16. In siderosis bulbi the electrolytically dissociated 23. In sympathetic ophthalmitis, Dalen-Fuch’s nodules
iron ions: are formed on the:
A. Are deposited under the membranous structures A. Iris
of the eye B. Ciliary body
B. Combine with intraocular proteins and produce C. Choroid
degenerative changes D. All of the above
C. Produce local irritative reaction at the site where
24. The most serious alkali burns of the eye are
deposited
produced by:
D. All of the above
A. Strong liquid ammonia
17. In chalcosis bulbi the electrolytically dissociated B. Caustic soda
ions of copper are: C. Lime
D. None of the above
A. Deposited under the membraneous structures
of the eye 25. All of the following are true of chemical burns of
B. Combine with intracellular proteins and produce the eye except:
degenerative changes A. Acid burns are more serious than the alkali burns
C. Produce irritative reaction at the local site B. Alkalies combine with lipids of cells to form
D. All of the above soluble compounds, which produce a condition
of softening and gelatinisation
18. Clinical manifestations of chalcosis include all of C. Acids cause instant coagulation of all the proteins
the following except: D. Symblepharon is a distressing sequelae
A. Kayser-Fleischer ring 26. In rosette-shaped cataract the suture act as
B. Sunflower cataract backbone in:
C. Greenish discolouration of iris A. Early-rosette cataract
D. Deposition of golden plaque at the posterior pole B. Late-rosette cataract
of the retina C. Both of the above
19. The most toxic intraocular foreign body is a D. None of the above
particle of: 27. Orange-skin cornea results due to:
A. Copper A. Chalcosis
B. Iron B. Siderosis
C. Tantalum C. Ammonia burn
D. Aluminium D. Mustard gas

13 : B 14 : C 15 : A 16 : B 17 : A 18 : C 20 : B 21 : C 22 : D 23 : C 24 : A 25 : A
19 : A 26 : A 27 : D
150 Review of OPHTHALMOLOGY

28. Most common site for the lodgement of intraocular 35. A 20-year-old man complains of difficulty in
foreign body is: reading the newspaper with his right eye, three
A. Vitreous weeks after sustaining a gun shot injury to his left
B. Posterior chamber eye. The most likely diagnosis is:
C. Lens A. Macular edema
D. Anterior chamber B. Sympathetic ophthalmia
C. Optic nerve avulsion
29. Best method of detection of retained glass D. Delayed vitreous haemorrhage
intraocular foreign body is:
A. CT scan 36. Vossius ring is seen in:
A. Cornea
B. Radiography
B. Lens
C. Ultrasonography
C. Vitreous
D. Tonography
D. Retina
30. Commotio retinae is seen in:
37. Dangerous area of eye:
A. Concussion injury A. Ciliary body
B. Retinopathy of AIDS B. Optic nerve
C. Central vein thrombosis C. Sclera
D. Central artery thrombosis D. Choroid
31. Ruptured globe is suspected if there is:
A. Proptosis
EXTRA EDGE QUESTIONS
B. Subluxation of lens 38. Traumatic eye lesion can causes:
C. Blow-out fracture A. Vitreous haemorrhage
D. Chemosis, haemorrhage, decreased IOP B. Corneal opacity
C. Exudative retinal detachment
32. The incidence of retained intraocular foreign D. Glaucoma
bodies is maximum with injuries due to: E. Cataract
A. Bow and arrow
B. Chisel and hammer 39. A boy gets hit by a tennis ball in the eye following
C. Air-gun pellet which he has complaints of decreased vision.
D. Glass Which of the following tells that blunt injury is
due to the ball?
33. All are seen in blunt injury of the eye except: A. Optic neuritis
A. Hyphema B. Pars planitis
B. Iridocyclitis C. Vitreous base detachment
C. Double perforation D. Equatorial edema
D. Retinal detachment
40. Common feature between sympathetic ophthalmitis
34. Traumatic dislocation of lens is best diagnosed by: and Vogt-Kanayagi-Harada syndrome:
A. Direct ophthalmoscopy A. Autoimmune etiology
B. Indirect ophthalmoscopy B. Injury
C. Distant direct ophthalmoscopy C. Uveitis
D. Slit-lamp examination D. Vitiligo

28 : A 29 : A 30 : A 31 : D 32 : B 33 : C 35 : B 36 : B 37 : A 38 : A, B, C, D and E
34 : D 39 : C 40 : A and C
CHAPTER

18 Ocular Therapeutics, Lasers and


Cryotherapy in Ophthalmology
Quick Text Review
INTRAOCULAR PENETRATION OF DRUGS Cytosine-Arabinoside
Intraocular penetration of topically instilled drugs • Not available commercially.
It is mainly determined by the corneal epithelium Trifluorothymidine
which is lipophilic and is crossed readily by non-
• DNA inhibitor like IDU
polar drugs. Stroma being hydrophilic allows rapid
• Advantages over IDU are: higher solubility, greater
passage of the drug through endothelium into the
potency, lack of toxicity and allergic reactions
anterior chamber. Following features will allow
• Dose: 1% drops 4 hourly for 14 days.
better penetration of the drug through the cornea:
• Solubility both in water and fat Acyclovir
• Pro-drug forms are lipophilic and after absorption • Effective in most forms of herpes simplex and
through epithelium are converted into proper herpes zoster
drugs which can easily pass through the stroma. • Penetrates deep and so is very effective in stromal
• Wetting agents increase the drug absorption. keratitis
The intraocular penetration of systemically adminis- • Dose: (i) Topically 3% ointment is used 5 times a
tered drugs day in patients with herpes zoster ophthalmicus
and recalcitrant cases of herpes simplex.
It mainly depends upon the blood aqueous barrier.
The two characteristics of the drugs which affect their Ganciclovir
passage through the blood aqueous barrier are: • Used for CMV retinitis
• Low molecular weight • Dose: 5 mg/kg body weight every 12 hours for 2–3
• Lipid solubility (e.g. sulphonamides being lipid weeks followed by maintenance dose of 5 mg/kg
soluble are 16 times more permeable than sucrose once daily.
having almost same molecular weight). Foscarnet

COMMON OCULAR THERAPEUTICS • As effective as ganciclovir in treating CMV retinitis


with AIDS.
ANTIVIRAL DRUGS
Zidovudine (Azidothymidine, AZT)
Idoxuridine (IDU, 5 lodo-2 deoxyuridine)
• Combined with immunoglobulins in treating HIV
• Inhibits the synthesis of DNA by substituting for
infection (AIDS).
thymidine and thus prevents replication of virus
• Used as 0.1% eye drops one hourly during day and ANTIFUNGAL DRUGS
0.5% eye ointment at night for 10–21 days I. Polyene antifungals
• Side effects: Follicular conjunctivitis, punctal
• Are isolated from streptomyces
stenosis.
• Act by binding to the sterol group in fungal cell
Adenine arabinoside membranes, rendering them permeable
• Blocks synthesis of nucleic acids 1. Nystatin
• 3% ointment is applied 5 times a day for 14–21 days. • Fungistatic
152 Review of OPHTHALMOLOGY

• Effective against candida and aspergillus Contraindications


• Poor intraocular penetraiton Miotics are not useful in:
• 3.5% eye ointment is used 5 times a day. • Buphthalmos
2. Amphotericin B (Fungizone) • Epidemic dropsy glaucoma
• Effective against candida, histoplasma and • Glaucoma inversus
cryptococcus • Glaucomatocyclitic crisis
• Topically, effective as 0.75% to 0.3% drops in super­ • Aphakic glaucoma
ficial corneal ulcers • Inflammatory glaucoma.
• Intravitreal injection for fungal endophthalmitis II. Sympathomimetic drugs
• Intravenously, it is used as 0.1 mg/ml in 5 percent Mechanisms of action
dextrose for systemic infections.
• Decreased aqueous secretion due to stimulation
3. Natamycin (Pimaricin) of alpha receptors in the ciliary body
• Broad spectrum antifungal (effectiveagainst • Increased aqueous outflow due to stimulation of
candida, aspergillus, fusarium and cephalo­ alpha and beta receptors.
sporium) Indications
• Drugs of choice for fusarium keratitis
• Used as 5% suspension. • POAG—Preferred in the presence of systemic
contraindication to beta-blocker
II. Imidazole antifungal drugs • Secondary glaucomas—Useful in most.
1. Miconazole: Broad spectrum fungicidal topically III. Beta adrenergic blockers
used as 1% solution.
Mechanism of action
2. Clotrimazole: Fungistatic, 1% suspension is
effective against candida and aspergillus keratitis. • Timolol and levobunolol reduce aqueous secretion
3. Econazole: Used as 1% econazole nitrate ointment. by blockade of beta-2 receptors in the ciliary
4. Ketoconazole: Used in single oral dose of 200-400 processes
mg daily in fungal keratitis and endophthalmitis. • Mechanism of action of betaxolol (cardioselective
5. Fluconazole. beta-blocker) is unknown.
III. Pyrimidine group Indications
• Flucytosine used as 1.5% aqueous eye drops one • In POAG and secondary glaucomas, it is the drug of
hourly is very effective against candida and yeast first choice unless contraindicated due to systemic
infections. diseases
• In PACG, it is useful as a temporary adjunct.
IV. Silver compounds
Commonly used preparations
• Silver sulfadiazine eye drops are effective against
aspergillus and fusarium keratitis. Timolol
• Non-selective beta-1 and beta-2 blocker
ANTIGLAUCOMA DRUGS • Available as 0.25 and 0.5 percent eye drops
• Efficacy is very good; however, the phenomenon
I. Miotics
of ‘short-term escape’ (marked initial fall, followed
Mechanisms of action by a transient rise with continued moderate fall in
• In primary open-angle glaucoma, miotics lower IOP) and ‘long term-drift’ (slow rise in IOP after
the IOP by enhancing aqueous outflow facility well controlled with months of therapy) are seen.
by widening inter-trabecular pores due to a pull Betaxolol
exerted on the scleral spur by contraction of the • Relative cardioselective beta-1 blocker (10 times
longitudinal fibres of ciliary muscle. more affinity for beta-1 than beta-2 receptors), so
• In primary angle-closure glaucoma the miotics can be used in patients with bronchial asthma.
open the angle by pulling the iris away from the • Available as 0.25% and 0.5% eye drops.
trabecular meshwork.
• Pilocarpine, once the most commonly used IV. Prostaglandin derivatives
antiglaucoma drugs, is sparingly used presently • Latanoprost (0.005%), an ester analogue of prostag­
because of availability of better drugs. landin F2-α, which increases uveoscleral outflow.
Ocular Therapeutics, Lasers and Cryotherapy in Ophthalmology 153
• Bimatoprost (0.03%), a prostamide, which Preparations
decreases ocular outflow resistance. Glycerol
• Travoprost (0.004%), a synthetic prostaglandin F2
• An oral hyperosmotic agent with sweet and
analogue that increases uveoscleral outflow.
sickening taste
V. Carbonic anhydrase inhibitors (CAI) • Dose: 1–1.5 g/kg body weight, used as 50% solution,
Mechanism of action therefore, dose is 50-80 ml mixed with equal
Reduce aqueous secretion by inhibiting the enzyme amount of lemon juice
carbonic anhydrase. • Action peaks in 1 hour and lasts for 4–6 hours
• Metabolised to glucose and so repeated use in
Indications diabetes is not recommended.
Systemic use (mainly acetazolamide)
• Used as additive therapy for short term in all types Mannitol
of acute glaucomas • Dose: 1–2 g/kg body weight or 5–10 ml/kg body
• Long-term use is reserved for patients with high weight of 20% solution in water
risk of visual loss, not responding to other modes • Should be administered intravenously very rapidly
of therapy. over 20–30 minutes
Topical use. CAIs are used as second line of drug • Action peaks in 30 minutes and lasts for 6 hours.
and also as adjunct therapy. Preparations include: Antiglaucoma drugs: Mechanism of lowering IOP
• Dorzolamide (2%), TDS. at a glance
• Brinzolamide (1%), BD.
Drugs which increase trabecular outflow
VI. Hyperosmotic agents • Miotics (e.g. pilocarpine)
Mechanism of action • Epinephrine, dipivefrine
Increase the osmotic pressure gradient between the • Bimatoprost.
blood and vitreous and thus draw sufficient water Drugs which increase uveoscleral outflow
out of the eye ball, therapy lowering IOR. • Prostaglandins (latanoprost)
Indications • Epinephrine, dipivefrine
• For rapidly lowering of the acutely raised IOP as in • Brimonidine
primary acute congestive glaucomas or secondary • Apraclonidine.
acute glaucomas. Drugs which decrease aqueous production
• As a prophylactic measure prior to intraocular • Carbonic anhydrase inhibitors (e.g. acetazolamide,
surgery.
dorzolamide)
Side effects • Alpha receptor stimulators in ciliary process (e.g.
• Cardiovascular overload epinephrine, dipivefrine, clonidine, brimonidine,
• Headache and backache apraclonidine
• Urinary retention • Beta-blockers (e.g. timolol, betaxolol, levobunolol)
• Nausea • Hyperosmotic agents (e.g. glycerol, mannitol,
• Mental confusion. urea).
154 Review of OPHTHALMOLOGY

LASERS AND CRYOTHERAPY IN OPHTHALMOLOGY


LASERS IN OPHTHALMOLOGY
Types of lasers are summarised in Table 18.1
Table 18.1: Lasers used in ophthalmology

Type of laser Wave lenghth Atomic environment Effects produced Clinical uses
(nanometer) used
Argon 514 Argon gas Photocoagulation Diabetic retinopathy
Krypton 714 Krypton gas Photocoagulation Peripheral retinal vascular
abnormalities such as Eales’
disease, proliferative sickle
cell disease, Coat’s disease and
retinopathy of prematurity
Diode 800 Diode crystal Photocoagulation Intraocular tumours such as
retinoblastoma, malignant
haemangioma
nd-YAG 1064 A liquid dye, or a solid Photodisruption Capsulotomy iridotomy
compound of yttrium-
aluminium garnet and
neodymium
Excimer 193 Helium and flourine gas Photoablation Refractive corneal surgery
(LASIK)
Phototherapeutic
Keratectomy (PTK)
Femtosecond 1053 Neodymium-glass Photodisruption Femto-LASIK
Femto- cataract surgery

CRYOSURGERY IN OPHTHALMOLOGY 3. Lens: Cryoextraction of the lens is the best­


intracapsular technique.
Principle
4. Ciliary body: Cyclocryopexy for absolute glaucoma
Working of cryoprobes is based on the Joule- and neovascular glaucoma.
Thompson principle of cooling. 5. Retina: (i) Cryopexy is widely used for sealing
retinal holes in retinal detachment, (ii) Prophylactic
Cryounit and probe
cryopexy to prevent retinal detachment in
The cryounit uses freon, nitrous oxide or certain prone cases, (iii) Retinal cryopexy for
carbondioxide gas as cooling agent. Temperature neovascularization, (iv) Cryo-treatment of retino­
produced depends upon the size of the cryoprobe blastoma and angioma.
tip, duration of freezing process and the gas used.
• Temperature produced at the tip of cryoprobe MISCELLANEOUS POINTS
– For intracapsular cataract extraction is about–
• Tropicamide is the fastest acting cycloplegic drug.
40°C
• Sulfonamides are most commonly known to cause
– For cyclocryopexy is about –80°C
Stevens-Johnson syndrome.
– For cryoretinopexy in retinal detachment surgery
• Topical steroids are contraindicated in superficial
is about –70°C.
viral keratitis, bacterial corneal ulcer, fungal
Uses corneal ulcer.
1. Lids: (i) Cryolysis for trichiasis, (ii) Cryotherapy • Intraocular permeability of the topically instilled
for warts and molluscum contagiosum, (iii) drugs is mainly determined by the epithelium of
Cryotherapy for basal cell carcinoma and haeman­ cornea.
gioma. • Phenylephrine (10%) drops should not be used in
2. Conjunctiva: Cryotherapy is used for hypertrophied neonates, cardiac patients and patients receiving
papillae of vernal catarrh. antidepressants.
Ocular Therapeutics, Lasers and Cryotherapy in Ophthalmology 155

Multiple Choice Questions (MCQs)

1. Intraocular penetration of topically-instilled 7. Retrobulbar injection of xylocaine blocks all of the


drugs is mainly determined by the corneal: following except:
A. Epithelium A. 3rd cranial nerve
B. Endothelium B. 4th cranial nerve
C. Stroma C. 6th cranial nerve
D. Bowman’s and Descemet’s membrane D. Ciliary nerves and ciliary ganglion
2. The antiviral drug used for treatment of cytomeglo­ 8. Absolute indication of enucleation is:
virus retinitis is:
A. Intraocular retinoblastoma
A. Acyclovir B. Endophthalmitis
B. Ganciclovir
C. Mutilating ocular injury
C. Triflurothymidine
D. All of the above
D. All of the above
9. In the laser machines used in ophthalmology, the
3. In primary open-angle glaucoma pilocarpine eye
atomic environment used consists of:
drops lowers the intraocular pressure by its direct
action on the: A. Crystal rod
A. Trabecular meshwork B. Fluid-filled cavity
B. Ciliary epithelium C. Gas-filled cavity
C. Longitudinal fibres of the ciliary muscle D. Any of the above
D. All of the above
10. Photo disruption is the basic mechanism of action
4. In primary angle-closure glaucoma pilocarpine of:
lowers the intraocular pressure by its direct action A. Argon laser
on the: B. Nd: YAG laser
A. Sphincter pupillae muscle C. Excimer laser
B. Ciliary epithelium D. All of the above
C. Trabecular meshwork
D. All of the above 11. Photoablation is the basic mechanism of action
of:
5. Steroid-induced ocular complication is: A. Argon laser
A. Glaucoma B. Nd:YAG laser
B. Papilledema C. Excimer laser
C. Central retinal vein occlusion
D. All of the above
D. All of the above
6. The technique of blocking the facial nerve at the 12. All of the following are indications of pan retinal
neck of mandible is: photocoagulation except:
A. Van Lint’s block A. Pre-proliferative diabetic retinopathy
B. O’Brien’s block B. Proliferative diabetic retinopathy
C. Nadbath block C. Ischaemic central retinal vein occlusion
D. Atkinson’s block D. Central retinal artery occlusion

1 : A 2 : B 3 : C 4 : A 5 : D 6:B 7 : B 8 : A 9 : D 10 : B 11 : C 12 : D
156 Review of OPHTHALMOLOGY

13. Laser photocoagulation is useful in all of the 20. Main disadvantage of cocaine as local anaesthetic
following macular disorders except: was:
A. Exudative age-related macular degeneration A. Causes dry eyes
B. Central serous retinopathy B. Follicular conjunctivitis
C. Focal type of diabetic maculopathy C. Intraocular penetration
D. Aphakic cystoid macular oedema D. Epithelial erosions

14. Excimer laser is used in the correction of: 21. Combination of pilocarpine and epinephrine use
A. Myopia in glaucoma treatment may inhibit:
B. Hypermetropia A. Pigmented pupillary cyst
C. Astigmatism B. Retinal detachment
D. All of the above C. Vitreous haemorrhage
D. Iridocyclitis
15. Pilocarpine is not used in young adults as it causes:
A. Myopia 22. Type of laser used for capsulotomy is:
B. Cystic blebs of iris A. C02
B. Ruby
C. Fatigue reaction
C. Nd:YAG
D. Allergy with circumcorneal congestion
D. Argon
16. Drugs used in treatment of herpes simplex
23. YAG laser is used in the treatment of:
keratitis include all except:
A. Retinal detachment
A. Idoxuridine 0.1%
B. Diabetic retinopathy
B. 5-Fluorouracil C. Open-angle glaucoma
C. Adenosine arabinoside D. After cataract
D. Cytosine arabinoside
E. Pimarcin 24. Argon laser is used in all except:
A. Retinal detachment
17. In atropine instillation, all are seen except: B. Retinitis pigmentosa
A. Dilated pupil C. Retinal vein occlusion
B. Paralysis of accommodation D. Eales’ disease
C. Increased water content to tear
D. Decreased water content to tear 25. Excimer laser is used in:
A. Glaucoma
18. Near vision is not affected with: B. Cataract
A. Atropine C. Uveitis
B. Adrenaline D. Diabetic retinopathy
C. Homatropine E. None of the above
D. Pilocarpine
EXTRA EDGE QUESTIONS
19. In a patient predisposed to glaucoma, the drug
contraindicated is: 26. Wavelength of ND: Yag laser:
A. Pilocarpine A. 1040 nm
B. Ecothiopate B. 1040 mm
C. Timolol C. 1040 cm
D. None of the above D. 1040 m

13 : D 14 : D 15 : A 16 : E 17 : D 18 : B 20 : D 21 : A 22 : C 23 : D 24 : B 25 : E
19 : D 26 : A
CHAPTER

19
Systemic Ophthalmology

Quick Text Review


OCULAR MANIFESTATIONS OF SYSTEMIC OCULAR MANIFESTATIONS OF IMMUNOLOGICAL
DISEASES DISORDER

OCULAR MANIFESTATIONS OF NUTRITIONAL DEFI- 1. Rheumatoid arthritis


CIENCIES • ‘Dry eye’ episcleritis
Vitamin-A deficiency • Scleritis
• Iridocyclitis
Xerophthalmia is the term now reserved to cover all
• Corneal melting
the ocular manifestations of vitamin A deficiency
• Cataract .
which has been classified by WHO (1982).
2. Systemic lupus erythematosus
Night blindness is the earliest symptom of • Episcleritis
xerophthalmia in children. • Iritis
A revised schedule of vitamin ‘A’ supplements for • Retinopathy.
prophylaxis being followed in India since August 3. Gaint cell arteritis
1992 under the programme, named as ‘child survival • Extraocular muscle palsies
and safe motherhood (CSSM), is as follows: • Anterior ischaemic optic neuropathy.
• First dose (1 lac. I.U.)— at 9 months of age along 4. Sarcoidosis
with measles. • Enlargement of lacrimal gland
• Second dose (2 lac. I.U.)—at 1½ years of age along • Iridocyclitis
with booster dose of DPT/OPV.
• Retinal periphlebitis.
• Third dose (2 lac. I.U.)—at the age of 2 years.
5. Reiter syndrome
Deficiency of vitamin B1 (thiamine) • Conjunctivitis
It can cause corneal anaesthesia, conjunctival and • Uveitis
corneal dystrophy, acute retrobulbar neuritis and • Retinal vasculitis
external ophthalmoplegia. 6. Periarteritis nodosa
• Episcleritis
Deficiency of vitamin B2 (Riboflavin) • Extraocular muscle palsy
It can produce photophobia and burning sensation • Uveitis
in the eyes due to conjunctival irritation and • Retinal haemorrhage
vascularisation of the cornea. • Papilledema.
Deficiency of vitamin C 7. Vogt-Koyanagi-Harada syndrome
• Poliosis
It may be associated with haemorrhages in the
• Uveitis
conjunctiva, lids, anterior chamber, retina and
• Choroiditis
orbit. It also delays wound healing.
• Exudative retinal detachment .
Deficiency of vitamin D 8. Multiple endocrine neoplasia (MENIIb)
It may be associated with zonular cataract, papille­ • Mucosal neuromas of the lid and conjunctiva
dema and increased lacrimation. • Medullated nerve fibres in the cornea.
158 Review of OPHTHALMOLOGY

OCULAR MANIFESTATIONS OF 6. Acquired immune deficiency syndrome


HEMATOLOGICAL DISEASE • Retinal microvasculopathy: It is characterised
1. Lymphocytic leukaemia by multiple cotton wool patches, superficial
• Proptosis and deep haemorrhages. Micro-aneurysms and
• Iris nodules telangiectasia occur rarely.
• Retinal edema • Usual ocular infections which occur with greater
• Haemorrhages frequency and produce more severe infections
• Leukaemic infiltrates in patients with AIDS include: herpes zoster
• Roth spots. ophthalmicus, herpes simplex infections,
toxoplasmosis, syphilis and fungal corneal ulcers.
2. Myeloid leukaemia
• Opportunistic infections of the eye: Cytomegalovirus
• Orbital chloroma
(CMV) retinitis, candida endophthalmitis,
• Haemorrhages
cryptococcal infections and pneumocystis carinii
• Peripheral retinal neovascularisation.
choroiditis.
3. Lymphomas • Unusual neoplasms are Kaposi’s sarcoma of the lids
• Lid/orbital deposits or conjunctiva and Burkitt’s lymphoma of the orbit.
• Uveitis. • Neurophthalmic lesions include isolated or multiple
4. Sickle cell anemia cranial nerve palsies.
• Dilated conjunctival vessels
II. Fungal
• Retinal capillary occlusion
• Neovascularisation 1. Candida and cryptococcus
• Chorioretinal scars. • Conjunctivitis
• Keratitis
OCULAR MANIFESTATIONS OF INFECTIOUS • Retinitis
DISEASES • Endophthalmitis
• Papilledema
I. Viral
• Optic atrophy.
1. Herpes simplex
• Vesicles on the lids III. Bacterial
• Dendritic keratitis 1. Tuberculosis
• Uveitis • Phlyctenular conjunctivitis
• Acute retinal necrosis. • Granulomatous uveitis
2. Measles: • Juxtapapillary choroiditis.
• Catarrhal conjunctivitis 2. Leprosy
• Koplik’s spots on conjunctiva • Facial palsy
• Corneal ulceration • Madarosis
• Optic neuritis • Iritis
• Retinitis. • Secondary glaucoma
3. Mumps: • Cataract.
• Conjunctivitis OCULAR ABNORMALITIES IN TRISOMIES
• Acute dacryoadenitis Trisomy 13 (D Trisomy or Patau Syndrome)
• Keratitis
• Uveitis. • Microphthalmos
• Colobomas (almost 100%)
4. Rubella:
• Retinal dysplasia
• Congenital microphthalmos
• Cataract
• Congenital cataract
• Corneal opacities
• Congenital glaucoma
• Optic nerve hypoplasia
• Chorioretinitis
• Cyclopia
• Optic atrophy.
• Intraocular cartilage.
5 . Whooping cough:
• Subconjunctival haemorrhage Trisomy 18 (E trisomy or Edwards syndrome)
• Rarely orbital haemorrhage leading to • Blepharophimosis
proptosis. • Ptosis
Systemic Ophthalmology 159
• Epicanthal fold Turner syndrome (XO)
• Hypertelorism • Antimongoloid slant
• Microphthalmos • Epicanthus
• Uveal coloboma • Ptosis
• Congenital glaucoma • Strabismus
• Corneal opacities. • Blue sclera
Trisomy 21 (G Trisomy or Down’s syndrome) • Eccentric pupils
• Cataract
• Upward slanting palpebral fissure (Mongoloid slant)
• Colour blindness
• Almond-shaped palpebral fissure
• Pigmentary disturbances of fundus.
• Epicanthus
• Telecanthus
ADVERSE OCULAR EFFECTS OF COMMON
• Narrowed interpupillary distance SYSTEMIC DRUGS
• Esotropia (35% cases)
• High refractive errors CVS drugs
• Cataract • Digitalis: Disturbance of colour vision, scotomas
• Iris hypoplasia • Quinidine: Optic neuritis (rare)
• Keratoconus. • Thiazides: Xanthopsia (yellow vision), Myopia
Ocular abnormalities in chromosomal deletion • Carbonic anhydrase inhibitors: Ocular hypotony,
syndromes Transient myopia
Cri-du-Chat syndrome (5 p.) • Amiodarone: Corneal deposits
• Oxprenolol: Photophobia, Ocular irritation.
• Hypertelorism
• Epicanthus GlT drugs
• Antimongoloid slant • Anticholinergic agents: Risk of angle-closure
• Strabisums. glaucoma due to mydriasis, blurring of vision due
Cri-du-Chat syndrome (11 p.) to cycloplegia (Occasional).
• Aniridia CNS drugs
• Glaucoma • Barbiturates: Extraocular muscle palsies with
• Foveal hypoplasia diplopia, ptosis, cortical blindness
• Nystagmus • Chloral hydrate: Diplopia, Ptosis, Miosis
• Ptosis. • Phenothiazines: Deposits of pigment in conjunctiva,
Cri-du-Chat syndrome (13 q.) cornea, lens and retina, oculogyric crisis
• Retinoblastoma • Amphetamines: Widening of palpebral fissure,
• Hypertelorism dilatation of pupil, paralysis of ciliary muscle with
• Microphthalmos loss of accommodation
• Epicanthus • Monoamine oxidase inhibitors: Nystagmus,
• Ptosis extraocular muscle palsies, optic atrophy
• Coloboma • Tricyclic agents: Pupillary dilatation (glaucoma
• Cataract. risk), cycloplegia
• Phenytoin: Nystagmus, diplopia, ptosis, slight-
De Grouchy syndrome (18 q.)
blurring of vision (rare)
• Hypertelorism • Neostigmine: Nystagmus, miosis
• Epicanthus • Morphine: Miosis
• Ptosis • Haloperidol: Capsular cataract
• Strabismus • Lithium carbonate: exophthalmos, oculogyric crisis
• Myopia • Diazepam: Nystagmus.
• Glaucoma
• Microphthalmos (with or without cyst) Hormones
• Coloboma Female sex hormones
• Optic atrophy • Retinal artery thrombosis
• Corneal opacity. • Retinal vein thrombosis
160 Review of OPHTHALMOLOGY

• Papilloedema Vitamins
• Ocular palsies with diplopia Vitamin A
• Nystagmus • Papilloedema
• Optic neuritis and atrophy • Retinal haemorrhages
• Retinal vasculitis • Loss of eyebrows and eyelashes
• Scotomas • Nystagmus
• Migraine • Diplopia and blurring of vision.
• Mydriasis
• Cyloplegia Vitamin D
• Macular oedema. • Band-shaped keratopathy.
Corticosteroids Antirheumatic agents
• Cataract (posterior subcapsular) • Salicylates: Nystagmus, retinal haemorrhages,
• Local immune suppression causing susceptibility cortical blindness (rare)
to viral (herpes simplex), bacterial and fungal • Indomethacin: Corneal deposits
infection • Phenylbutazone: Retinal haemorrhages.
• Steroid-induced glaucoma.
Antibiotics
MISCELLANEOUS POINTS
• Chloramphenicol: Optic neuritis and optic atrophy • Most common ocular involvement in sarcoidosis
is iritis.
• Streptomycin: Optic neuritis
• The most common ocular finding in AIDS patients
• Tetracycline: Pseudotumor cerebri, transient
is iritis and the most common fundus finding is
myopia.
cotton wool spots.
Antimalarial • The commonest ocular finding in anaemia is pallor
Chloroquine of conjunctiva.
• Diabetes is the most common cause of cotton wool
• Macular changes (Bull’s eye maculopathy)
spots.
• Central scotomas
• Toxoplasmosis is the most common cause of
• Pigmentary degeneration of the retina
posterior uveitis.
• Chloroquine keratopathy
• The major cause of blindness in leprosy is iritis
• Ocular palsies • The most common ocular manifestation of
• Ptosis multiple sclerosis is retrobulbar neuritis.
• Electroretinographic depression. • Ocular signs in SLE are marginal corneal
Amoebicides degeneration, episcleritis, scleritis, retinal
• Diiodohydroxy quinoline: Subacute myelo-optic haemorrhages and cotton-wool retinal exudates.
neuropathy (SMON), optic atrophy. • Pupillary abnormalities in tabes dorsalis include
spinal miosis, anisocoria and argyll-robertson
Chemotherapeutic agents pupil.
• Sulfonamides: Stevens-Johnson syndrome • Most commonly involved cranial nerve in fracture
• Ethambutol: Optic neuritis and atrophy base of the skull is facial nerve.
• Isoniazid: Optic neuritis and optic atrophy. • The commonest ocular sign of hypothyroidism is
cataract.
Heavy metals
• Dalrymple’s sign is the commonest lid sign of
• Gold salts: Deposits in the cornea and conjunctiva Graves’ ophthalmopathy.
• Lead: Optic atrophy, papilledema, ocular palsies. • The commonest feature of Waardenburg’s
Chelating agents syndrome is lateral displacement of both medial
• Penicillamine: Ocular pemphigoid, ocular neuritis, canthi and lacrimal puncta.
ocular myasthenia. • The commonest ocular infection associated with
AIDS is CMV retinitis.
Oral hypoglycemic agents • Ocular complications associated with pregnancy
• Chloropropamide: Transient change in refractive are worsening of diabetic retinopathy, ptosis,
error, diplopia, Stevens-Johnson syndrome. central serous retinopathy, ophthalmoplegia.
Systemic Ophthalmology 161

Multiple Choice Questions (MCQs)

1. Ocular manifestations of vitamin D deficiency C. Cataract


include: D. All of the above
A. Zonular cataract
B. Papilloedema 8. Isolated painful third nerve palsy is a feature of
C. Increased lacrimation aneurysms of:
D. All of the above A. Posterior communicating artery
B. Anterior communicating artery
2. As per WHO classification ‘X2’ stage of xero­ C. Vertebrobasillary artery
phthalmia refers to: D. Ophthalmic artery
A. Conjunctival xerosis
B. Bitot’s spots 9. Which of the following is ocular false localising
C. Corneal xerosis sign of raised intracranial pressure:
D. Corneal ulceration A. Diplopia due to pressure palsy of 6th nerve
B. Sluggish pupillary reactions and unilateral
3. Earliest feature of xerophthalmia is:
A. Night blindness mydriasis
B. Conjunctival xerosis C. Homonymous hemianopia
C. Bitot’s spots D. All of the above
D. Dry eye 10. Temporal lobe tumours may produce:
4. Ocular lesions of rubella include all of the A. Crossed upper quadrantanopia
following except: B. Crossed lower quadrantanopia
A. Congenital megalocornea C. Uncrossed upper quadrantanopia
B. Congenital cataract D. Uncrossed lower quadrantanopia
C. Congenital glaucoma
D. Optic atrophy 11. Parietal lobe tumours may be associated with:
A. Crossed lower homonymous quadranta­nopia
5. All of the following ocular lesions may occur in B. Crossed upper homonymous quadranta­nopia
patients suffering from AIDS except: C. Both of the above
A. Central toxoplasma chorioretinitis D. None of the above
B. Central serous retinopathy
C. CMV retinitis 12. Occipital lobe tumours may produce:
D. Candida endophthalmitis A. Crossed homonymous quadrantanopia
B. Crossed homonymous hemianopia
6. Ocular lesions of gout include all of the following
C. Both of the above
except:
A. Episcleritis D. None of the above
B. Scleritis 13. The commonest cranial nerve palsy in tabes
C. Keratitis dorasalis is:
D. Uveitis A. III nerve
7. Atopic dermatitis may be associated with: B. IV nerve
A. Conjunctivitis C. VI nerve
B. Keratoconus D. Total ophthalmoplegia

1 : D 2 : C 3 : A 4 : A 5 : B 6:C 7 : D 8 : A 9 : D 10 : A 11 : A 12 : C
13 : A
162 Review of OPHTHALMOLOGY

14. Bilateral retrobulbar neuritis occurs in: C. Ring scotoma


A. Multiple sclerosis D. Nystagmus
B. Neuromyelitis optica
C. Both of the above 23. All are seen in albinism except:
D. None of the above A. Glaucoma
B. Photophobia
15. Hutchinson’s pupil is characterised by: C. Refractive error
A. Initial ipsilateral miosis, later followed by D. Nystagmus
dilatation
B. Initial ipsilateral miosis and contralateral 24. Night blindness may be caused by all except:
dilatation A. Vitamin A deficiency
C. Ipsilateral mydriasis and contralateral miosis B. Retinitis pigmentosa
D. None of the above C. Syphilis
16. The commonest ocular infection associated with D. Lattice degeneration
AIDS is: 25. Fundus picture in collagen disease is dominated
A. Herpes zoster by:
B. Cytomegalovirus
A. Multiple cotton wool spots
C. Toxoplasmosis
B. Multiple superficial haemorrhages
D. Tuberculosis
C. Macular edema
17. Ocular lesions of leprosy include all of the D. None of the above
following except:
A. Avascular keratitis 26. The most characteristic eye lesion in diabetes
B. Interstitial keratitis is:
C. Fascicular keratitis A. Flame haemorrhages
D. Neuroparalytic keratitis B. Papilledema
C. Capillary aneurysm
18. Ocular manifestations of Wegener’s granulo­ D. Cataracts
matosis include all of the following except:
A. Proptosis 27. Occurrence of diplopia, dysphagia, dysarthria,
B. Nasolacrimal duct obstruction blurring of vision and muscle weakness could be
C. Necrotizing scleritis due to:
D. Internal ophthalmoplegia A. Diphtheria
19. The most common lid sign associated with Grave’s B. Botulism
ophthalmopathy is: C. Infantile beriberi
A. Von Graefe’s sign D. Myasthenia gravis
B. Dalrymple’s sign 28. Internal ophthalmoplegia is seen in:
C. Stellwag’s sign
A. Migraine
D. Rosenbach’s sign
B. Diabetes
20. Hutchinson’s triad of congenital syphilis includes C. Ethambutol toxicity
all of the following except: D. All of the above
A. Eighth nerve deafness
B. Interstitial keratitis 29. Which of the following is most likely to be the
C. Hutchinson’s teeth cause of painful third nerve palsy of sudden onset
D. Saddle nose in a man of 40?
A. Rupture of an infraclinoid aneurysm
21. Most common ophthalmic affection of diphtheria B. Sphenoidal ridge meningioma
is:
C. Diabetes mellitus neuropathy
A. Ptosis
D. Rupture of a supraclinoid aneurysm
B. Isolated ocular palsies
C. Total ophthalmoplegia 30. Sudden increase in blood sugar in a diabetic
D. Ophthalmoplegia externa causes:
22. Most common adverse effect of oral contra­ceptive A. Myopia
is: B. Hypermetropia
A. Colour blindness C. Presbyopia
B. Optic neuritis D. Anisometropia

14 : C 15 : A 16 : B 17 : C 18 : D 19 : B 23 : A 24 : D 25 : A 26 : C 27 : B 28 : A
20 : D 21 : B 22 : B 29 : D 30 : A
Systemic Ophthalmology 163
31. Basilar insufficiency shows: 40. All are seen in albinism except:
A. Ptosis A. Nystagmus
B. Blurred vision B. Glaucoma
C. Diplopia C. Photophobia
D. VI nerve weakness D. Refractive error

32. In uraemic amaurosis the pupils are: 41. Bowen’s disease is characterized by all except:
A. Constricted A. Predilection for the limbus
B. Dilated and do not react to light B. Poikilocytosis
C. Dilated but react to light C. Presence of monster cell
D. Normal D. Being incapable of metastasizing

33. Diabetes mellitus can cause the following in the 42. Essential atrophy of choroid is due to inborn error
eye except: of metabolism of amino acid:
A. Cystine
A. Cataract
B. Cysteine
B. Retinopathy
C. Arginine
C. Anterior ischemic neuropathy
D. Ornithine E Lysine
D. Neuroparalytic keratitis
43. Which of the following is the most common ocular
34. The presence of Kayser-Fleischer ring is patho­ complication with renal transplantation:
gnomonic of: A. Cataract
A. Keratoconus B. Glaucoma
B. Lowe’s syndrome C. Cytomegalo virus retinitis
C. Wilson’s disease D. Candida keratomycosis
D. Albinism E. Candida endophthalmitis
35. Roth’s spots in the fundus are seen in: 44. Pepper salt fundus is seen in:
A. Diabetes A. CMV retinitis
B. Chorioretinitis B. Toxoplasmosis
C. Bacterial endocarditis C. Rubella
D. Retinoblastoma D. Measles
36. In Weber’s syndrome there is: 45. In mumps the most common eye lesion is:
A. 3rd nerve palsy A. Dacryoadenitis
B. 4th nerve palsy B. Uveitis
C. 5th nerve C. Membranous conjunctivitis
D. 7th nerve D. Chorioretinitis

37. Muscle mostly affected in thyroid ophthal­mopathy 46. Ptosis with orbicularis oculi palsy is seen in:
is: A. Eaton Lambert syndrome
B. Myasthenia gravis
A. Lateral rectus
C. Polymyositis
B. Inferior rectus
D. Motor neuron disease
C. Superior rectus
D. Medial rectus 47. Most common cranial nerve involved in ophthalmo­
plegic migraine is:
38. Dalen Fuch lesion is seen in: A. II nerve
A. Purulent keratitis B. III nerve
B. Epidemic keratoconjunctivitis C. V nerve
C. Retinoblastoma D. VI nerve
D. Sympathetic ophthalmitis
48. In a patient with AIDS chorioretinitis is typically
39. “Rubeosis Iridis” is most commonly seen in: caused by:
A. Diabetes mellitus A. Cytomegalo virus
B. Central retinal vein occlusion B. Toxoplasma gondi
C. Trauma C. Cryptococcus neoformans
D. Central retinal artery occlusion D. All of the above

31 : A 32 : C 33 : D 34 : C 35 : C 36 : A 40 : B 41 : B 42 : D 43 : C 44 : C 45 : A
37 : B 38 : D 39 : B 46 : B 47 : B 48 : A
164 Review of OPHTHALMOLOGY

49. The most common condition of inherited 55. In Waardenburg’s syndrome, following are seen
blindness due to mitochondrial chromosomal except:
anomaly is: A. Widening of the eyebrow
A. Retinopathy of prematurity B. Short palpebral fissure
C. Interstital keratitis
B. Leber’s hereditary optic neuropathy
D. Heterochromia iridis
C. Retinitis pigmentosa
D. Retinal detachment EXTRA EDGE QUESTIONS
50. Toxic amblyopia is produced by:
56. Xerophthalmia is caused by:
A. INH A. Vit C deficiency
B. Rifampicin B. Small bowel resection
C. Ethambutol C. Cystic fibrosis
D. Pyrazinamide D. Chronic alcoholism
E. Glomerulonephritis
51. Contraindications of topical steroids are:
57. Vit A deficiency produces:
A. Dendritic ulcer
A. Bitots spots
B. Disciform keratitis
B. Trantas spots
C. Anterior uveitis C. Keratomalacia
D. Acute angle-closure glaucoma D. Xerophthalmia
E. Color blindness
52. Which drug causes bull’s eye macula:
A. Phenytoin 58. Ocular manifestation of HIV are all except:
B. Chloroquine A. Predispose to viral, bacterial and fungal function
B. Kaposi sarcoma
C. Steroids
C. CMV retinitis
D. Ethambutal D. Cotton wool spot
53. Most common adverse effect on eye in oral E. Intraocular lymphoma
contraceptive usage is: 59. Ocular manifestations in AIDS:
A. Colour blindness A. Kaposi sarcoma
B. Ring scotoma B. Retinitis
C. Optic neuritis C. Lymphoma
D. Nystagmus D. Tuberculosis
E. Herpes
54. Drug not deposited in cornea:
60. Eye involvement is seen in:
A. Gold
A. Seropositive polyarticular JRA late onset
B. Chloroquine B. Seronegative pauciarticular JRA late onset
C. Amiodarone C. Seronegative polyarticular JRA early onset
D. Antimony D. Seronegative pauciarticular JRA early onset

49 : B 50 : B 51 : A 52 : B 53 : C 54 : D 55 : C 56 : B, C and D 57 : A, C and D
58 : None 59 : A, B, C, D and E 60 : D
CHAPTER

20
Community Ophthalmology

Quick Text Review


BLINDNESS glaucoma, diabetes, vascular diseases, cataract,
macular degeneration and hereditary conditions.
DEFINITION OF BLINDNESS
While in developing countries the frequent causes
WHO has defined blindness as “Visual acuity of less are cataract, infectious diseases, xerophthalmia,
than 3/60 (Snellen) or inability to count fingers in injuries and glaucoma.
daylight at a distance of 3 meters”. Causes of blindness in India: The major causes
In the Ninth Revision (1977) of the International of blindness in elderly (70 and above) are senile
classification of diseases (ICD), the visual cataract and age-related macular degeneration. See
impairment (maximum vision less than 6/18 Table 20.2.
Snellen) has been divided into 5 categories. Patients
with the visual fields less than 10° are also labelled Table 20.2: Major causes of blindness in India
as blinds (Table 20.1). RAAB Survey (2006-07)
Table 20.1: Categories of visual impairment (WHO, (1977) Disease condition Percent
blindness
Category of Level of visual acuity (Snellen) impairment
visual Cataract 72.2

Normal vision 1 Less than 6/18 to 6/60 Refractive errors (0.7%) + aphakia (5.6%) 6.3

Low vision 2 Less than 6/60 to 3/60 Glaucoma 4.4

3 Less than 3/60 (FC at 3 m) to 1/60 (FC at Complications of cataract surgery 3.0
1m) or visual field between 5° and 10° Corneal opacity including trachoma 6.5
Blindness 4 Less than 1/60 (FC at 1 m) to light Posterior segment disorders (DR, 0.1% + 3.0
perception or visual field less than 5° ARMD 0.7% + others 2.2%)
5 No light perception NPCB Survey (2001-02)
Cataract 62.6
Avoidable blindness includes both preventable
Refractive errors 19.7
blindness (e.g. xerophthalmia blindness) and
curable blindness (e.g. cataract blindness). Glaucoma 5.8
Posterior segment disorders 4.7
MAGNITUDE OF BLINDNESS Surgical complications 1.2
• The prevalence of blindness in developing Corneal blindness 0.9
countries ranges from 0.5 to 2 percent or higher, Others 5.0
while the representative figures in developed WHO-NPCB Survey (1986-89)
countries are 0.05 to 0.3 per cent. Cataract 80.1
• The prevalence of blindness reported from India Refractive errors 7.35
is 1.49 percent (WHO-NPCB 1986-89) and 1.1%
Glaucoma 1.7
(NPCB, 2001-2002).
Trachoma 0.39
CAUSES OF BLINDNESS Aphakic blindness 4.67

Developed countries versus developing countries. Corneal opacity 1.52


In developed countries, causes include: Accidents, Others 4.25
166 Review of OPHTHALMOLOGY

ICMR Survey (1971-74) VISION FOR THE FUTURE (VFTF)


Cataract 55 The International ophthalmology strategic plan
Malnutrition 2
to restore and preserve vision, directed and
co-ordinated by the International Council of
Glaucoma 0.5
Ophthalmology has been launched as multi year,
Trachoma and associated infections 20 flexible and interactive programme.
Injuries 1.2
Small pox sequele 3 NATIONAL PROGRAMME FOR CONTROL OF
Others 18 BLINDNESS IN INDIA
NPCB in India was launched in the year 1976 with
CHILDHOOD BLINDNESS the objective to reduce prevalence of blindness.
• Approximately 1.5 million children are blind in the Programme organization: NPCB operates through
world (WHO, 1992). activities at central level, state level and district
• Every year, approximately half a million children level.
become blind, i.e. about one every minute (WHO, District blindness control society (DBCS) with
1992) Deputy Commissioner as its chairman who
• Of the children going blind, about 60% die within co-ordinates the activities at district level and
one year (WHO, 1992) ensures participation of the community and
• In about 70% of new cases (of childhood blindness), private sector.
blindness is due to vitamin A deficiency (WHO,
1992). ‘Vision 2020: The Right to Sight, in India has been
adopted under NPCB since August 2002 with
GLOBAL INITIATIVES FOR PREVENTION OF following strategies:
BLINDNESS • Strengthening advocacy,
• Reduction of disease burden,
VISION 2020: THE RIGHT TO SIGHT • Human resource development, and
Objective: It is a global initiative to eliminate • Eye care infrastructure development.
avoidable blindness by the year 2020.
Target diseases identified for intervention in India
Partners of vision 2020 initiative are WHO and task include: Cataract, childhood blindness, refractive
force of international NGOs. errors and low vision, corneal blindness, diabetic
Implementation is being done through four phases retinopathy, glaucoma and trachoma (focal).
each of five year plan, commencing in 2000, 2005,
2010 and 2015, respectively. NPCB DURING 12TH FIVE YEAR PLAN (2012-2017)
Basic strategies include: • NPCB to continue under NCO flexipool as part of
• Disease prevention and control National Health Mission; to be implemented by
• Training of eye health personnel District Health Societies.
• Strengthening of existing eye care infrastructures • Financial contribution: Government of India, 75%
• Use of appropriate and affordable technology, and and state/UT govt,25%
• Mobilization of resources • Cataract surgery target: 33 corer operations with
Global target diseases include: Cataract, child­ above 95% being IOL implantation.
hood, blindness, trachoma, refractive errors, and • Spectacle distribution target for school age group
low vision and onchocerciasis. children: 44 lakhs.
Community Ophthalmology 167

Multiple Choice Questions (MCQs)

1. WHO definition of blindness is a visual acuity in 6. The school health surveys are quite useful in early
the better eye equal to or less than: detection of mainly:
A. 3/60 A. Refractive errors
B. 4/60 B. Amblyopia
C. 5/60 C. Colour vision defects
D. 6/60 D. All of the above
2. In the ninth revision of the international 7. Most common cause of blindness (as per WHO-
classification of diseases, ‘Visual impairment’ is a NPCB Survey, 1989) in India out of the following
visual acuity in the better eye of: is:
A. 6/18 A. Aphakic blindness
B. 6/36 B. Glaucoma
C. 6/60 C. Trachoma
D. 3/60 D. Vitamin A deficiency
3. WHO definition of blindness in addition to visual
8. Commonest cause of blindness in India:
acuity of less than 3/60 (Snellen’s) also includes
a visual field in the better eye equal to or less A. Vitamin A deficiency
than: B. Cataract
A. 5° C. Trauma
B. 10° D. Trachoma
C. 15°
D. 3° 9. Lowest incidence of trachoma is seen in:
A. Punjab
4. As per WHO-NPCB (1986-89) survey, all of the B. Rajasthan
following are true about causes of blindness in C. Uttar Pradesh
India except: D. Orissa
A. Cataract 55%
B. Refractive errors 7.35% 10. Cataract blindness is:
C. Glaucoma 1.7% A. Preventable
D. Trachoma 0.39% B. Curable
5. All of the following are true about prevention of C. Avoidable
vitamin A deficiency by supplements except: D. Curable and avoidable
A. Dose in children over 1 year is 2 lac IU orally
11. Blindness in a child is most commonly due to:
B. Dose in infants less than 6 month (not being
breast fed is 50 thousand IV orally) A. Keratomalacia
C. Dose in first trimester of pregnancy is 1 lac IU B. Congenital cataract
D. Under CSSM Programme first dose (1 lac IU) is C. Glaucoma
given at 9 months of age along with measles D. Injuries

1 : B 2 : C 3 : A 4 : B 5 : C 6 : D 7 : C 8 : A 9 : A 10 : B 11 : A
168 Review of OPHTHALMOLOGY

12. The commonest cause of low vision in India is: B. Epidemic conjunctivitis
A. Uncorrected refractive error C. Cataract
B. Cataract D. Ibcgicercuasus
C. Glaucoma
D. Squint 16. Under the WHO “Vision 2020’ program, the “SAFE”
strategy is adopted for which of the following
13. All of the following are given global prominence
diseases:
in the VISION 2020 goal, expect:
A. Refractive errors A. Trachoma
B. Cataract B. Glaucoma
C. Trachoma C. Diabetic retinopathy
D. Glaucoma D. Onchocerciasis.
EXTRA EDGE QUESTIONS 17. On SAFE strategy false is:
14. Most common cause of blindness in children in A. Screening
India except: B. Antibiotics
A. Malnutrition C. Facial hygiene
B. Ophthalmia neonatorum D. Environmental modification
C. Glaucoma 18. All of the following are causes of night blindness
D. Congenital dacryocystitis
except:
15. All of the following conditions are immediate A. Oguchi’s disease
priorities in the WHO’s “vision -2020: The right to B. Gyrate atrophy
sight” except: C. Choroideremia
A. Trachoma D. Devics disease

12 : C 13 : D 14 : D 15 : B 16 : A 17 : A 18 : D

You might also like